SBAs for the FRCS(Tr&Orth) Examination: A Companion to Postgraduate Orthopaedics Candidate’s Guide [1st ed.] 1108789978, 9781108789974, 1108803644, 9781108803649, 9781108846790

Following the format change to include single best answer questions (SBAs), this book equips candidates with a full rang

231 39 18MB

English Pages 646 Year 2020

Report DMCA / Copyright

DOWNLOAD PDF FILE

Table of contents :
Part I. The FRCS (Tr & Orth) Oral Examination
1. Candidate Guidance for the Part 1 Written SBA Paper Hussuein Nouredfine and Matthew Brown
2. SBA Writing Process Paul Banaszkiewicz
Part II. Adult Elective Orthopaedics and Spine
3. Hip I Structured SBAs Edward Holloway
4. Hip II Structured SBAs James Gill and Majeed Shakokani
5. Knee Structured I SBAs Kiran Singasetti
6. Knee II Structured SBAs Oliver Bailey
7. Foot and Ankle Structured SBAs Lyndon Mason and Gavin Heyes
8. Spine I Structured SBAs Paul Rushton and Niall Eames
9. Spine II Structured SBAs Prasad Karpe
10. Shoulder/Elbow I Structured SBAs Shantanu Shahane and Razvan Tarnu
11. Shoulder/Elbow Structured SBAs Mohan Pullugara and Faizan Jabbar
Part III. Trauma
12. Trauma Structured SBAs I Tim Brock and Rishi Dhir
13. Trauma Structured SBAs II Nayef Aslam-Pervez
Part IV. Children's Orthopaedics/Hand and Upper Limb
14. Hand Structured SBAs I Emma Reay
15. Hand Structured SBAs II Mathew Brown and David Yeoh
16. Children's Orthopaedics Structured SBAs I Sattar Alshriyda and Paul Banaszkiewicz
17. Children's Orthopaedics Structured SBAs II Ling Hong Lee
Part V. Applied Basic Science
18. Anatomy and Surgical Approaches Structured SBAs Chris Ghazala
19. Structure and Fracture of Connective Tissue Structured SBAs Paul Banaszkiewicz
20. Pathology Structured SBAs I Rajesh Kakwani and Rory Morrison
21. Pathology Structured SBAs II Alexander Durst
22. Pathology Structured SBAs III Tom Marjoram
23. Orthopaedic Oncology Structured SBAs Craig Gerrand, Walid A. Elnahal and Hel Havard
24. Prosthetics/Orthotics Structured SBAs Emerson Budhoo
25. Biomechanics Structured SBAs Edward Jeans and Richard Hutchinson
26. Clinical Environment, Evidence Management and Quality Improvement Hussien Nouredfine
27. Evidence Management and Data Analysis SBAs Answers Munier Hossian
28. Evidence Management and Data Analysis SBAs Questions Munier Hossian.
Recommend Papers

SBAs for the FRCS(Tr&Orth) Examination: A Companion to Postgraduate Orthopaedics Candidate’s Guide [1st ed.]
 1108789978, 9781108789974, 1108803644, 9781108803649, 9781108846790

  • Commentary
  • NO BOOKMARKS
  • 0 0 0
  • Like this paper and download? You can publish your own PDF file online for free in a few minutes! Sign Up
File loading please wait...
Citation preview

SBAs for the FRCS(Tr&Orth) Examination A Companion to Postgraduate Orthopaedics Candidate’s Guide Following the format change to include single best answer questions (SBAs), this book equips candidates with a full range of testing examples to develop familiarity with the format and prepare for success in their FRCS (Tr & Orth) examination. Containing over 1,000 level 2 SBA questions, detailed explanations ensure candidates understand the reasoning and evidence-based decision-making behind each answer. Covering the breadth of the orthopaedic syllabus, including more difficult subject areas such as biomechanics, prosthetics/orthotics, anatomy and statistics, this is a crucial resource for all candidates. Encouraging the integration of clinical information with problem solving, this question format help candidates learn and retain the answers more efficiently than with simple factual recall. Written by highly experienced clinicians and examiners, these example questions are essential for preparing for the real examination. Paul A. Banaszkiewicz is Consultant Orthopaedic Surgeon, Queen Elizabeth Hospital and North East Surgery Centre [NENSC], Gateshead and a visiting Professor, Northumbria University, Newcastle upon Tyne, UK. Kiran K. Singisetti is Consultant Orthopaedic Surgeon, Queen Elizabeth Hospital and North East NHS Surgical Centre (NENSC), Gateshead, UK.

SBAs for the FRCS(Tr&Orth) Examination A Companion to Postgraduate Orthopaedics Candidate’s Guide Edited by

Paul A. Banaszkiewicz FRCS (Glas) FRCS (Ed) FRCS (Eng) FRCS (Tr & Orth) MClinEd FAcadMEd FHEA Consultant Orthopaedic Surgeon Queen Elizabeth Hospital and North East NHS Surgical Centre (NENSC), Gateshead, UK Visiting Professor Northumbria University, Newcastle-upon-Tyne, UK

Kiran K. Singisetti MBBS MRCS MS Fellow EBOT FRCS Ed (Tr & Orth) Queen Elizabeth Hospital and North East NHS Surgical Centre (NENSC), Gateshead, UK

University Printing House, Cambridge CB2 8BS, United Kingdom One Liberty Plaza, 20th Floor, New York, NY 10006, USA 477 Williamstown Road, Port Melbourne, VIC 3207, Australia 314–321, 3rd Floor, Plot 3, Splendor Forum, Jasola District Centre, New Delhi – 110025, India 79 Anson Road, #06–04/06, Singapore 079906 Cambridge University Press is part of the University of Cambridge. It furthers the University’s mission by disseminating knowledge in the pursuit of education, learning, and research at the highest international levels of excellence. www.cambridge.org Information on this title: www.cambridge.org/9781108789974 DOI: 10.1017/9781108855235 © Paul A. Banaszkiewicz and Kiran K. Singisetti 2021 This publication is in copyright. Subject to statutory exception and to the provisions of relevant collective licensing agreements, no reproduction of any part may take place without the written permission of Cambridge University Press. First published 2021 Printed in the United Kingdom by TJ International Ltd, Padstow Cornwall A catalogue record for this publication is available from the British Library. ISBN 978-1-108-78997-4 Paperback Cambridge University Press has no responsibility for the persistence or accuracy of URLs for external or third-party internet websites referred to in this publication and does not guarantee that any content on such websites is, or will remain, accurate or appropriate.

.......................................................................................................................................... Every effort has been made in preparing this book to provide accurate and up-to-date information that is in accord with accepted standards and practice at the time of publication. Although case histories are drawn from actual cases, every effort has been made to disguise the identities of the individuals involved. Nevertheless, the authors, editors, and publishers can make no warranties that the information contained herein is totally free from error, not least because clinical standards are constantly changing through research and regulation. The authors, editors, and publishers therefore disclaim all liability for direct or consequential damages resulting from the use of material contained in this book. Readers are strongly advised to pay careful attention to information provided by the manufacturer of any drugs or equipment that they plan to use.

SBAs for the FRCS(Tr&Orth) Examination A Companion to Postgraduate Orthopaedics Candidate’s Guide Following the format change to include single best answer questions (SBAs), this book equips candidates with a full range of testing examples to develop familiarity with the format and prepare for success in their FRCS (Tr & Orth) examination. Containing over 1,000 level 2 SBA questions, detailed explanations ensure candidates understand the reasoning and evidence-based decision-making behind each answer. Covering the breadth of the orthopaedic syllabus, including more difficult subject areas such as biomechanics, prosthetics/orthotics, anatomy and statistics, this is a crucial resource for all candidates. Encouraging the integration of clinical information with problem solving, this question format help candidates learn and retain the answers more efficiently than with simple factual recall. Written by highly experienced clinicians and examiners, these example questions are essential for preparing for the real examination. Paul A. Banaszkiewicz is Consultant Orthopaedic Surgeon, Queen Elizabeth Hospital and North East Surgery Centre [NENSC], Gateshead and a visiting Professor, Northumbria University, Newcastle upon Tyne, UK. Kiran K. Singisetti is Consultant Orthopaedic Surgeon, Queen Elizabeth Hospital and North East NHS Surgical Centre (NENSC), Gateshead, UK.

SBAs for the FRCS(Tr&Orth) Examination A Companion to Postgraduate Orthopaedics Candidate’s Guide Edited by

Paul A. Banaszkiewicz FRCS (Glas) FRCS (Ed) FRCS (Eng) FRCS (Tr & Orth) MClinEd FAcadMEd FHEA Consultant Orthopaedic Surgeon Queen Elizabeth Hospital and North East NHS Surgical Centre (NENSC), Gateshead, UK Visiting Professor Northumbria University, Newcastle-upon-Tyne, UK

Kiran K. Singisetti MBBS MRCS MS Fellow EBOT FRCS Ed (Tr & Orth) Queen Elizabeth Hospital and North East NHS Surgical Centre (NENSC), Gateshead, UK

University Printing House, Cambridge CB2 8BS, United Kingdom One Liberty Plaza, 20th Floor, New York, NY 10006, USA 477 Williamstown Road, Port Melbourne, VIC 3207, Australia 314–321, 3rd Floor, Plot 3, Splendor Forum, Jasola District Centre, New Delhi – 110025, India 79 Anson Road, #06–04/06, Singapore 079906 Cambridge University Press is part of the University of Cambridge. It furthers the University’s mission by disseminating knowledge in the pursuit of education, learning, and research at the highest international levels of excellence. www.cambridge.org Information on this title: www.cambridge.org/9781108789974 DOI: 10.1017/9781108855235 © Paul A. Banaszkiewicz and Kiran K. Singisetti 2021 This publication is in copyright. Subject to statutory exception and to the provisions of relevant collective licensing agreements, no reproduction of any part may take place without the written permission of Cambridge University Press. First published 2021 Printed in the United Kingdom by TJ International Ltd, Padstow Cornwall A catalogue record for this publication is available from the British Library. ISBN 978-1-108-78997-4 Paperback Cambridge University Press has no responsibility for the persistence or accuracy of URLs for external or third-party internet websites referred to in this publication and does not guarantee that any content on such websites is, or will remain, accurate or appropriate.

.......................................................................................................................................... Every effort has been made in preparing this book to provide accurate and up-to-date information that is in accord with accepted standards and practice at the time of publication. Although case histories are drawn from actual cases, every effort has been made to disguise the identities of the individuals involved. Nevertheless, the authors, editors, and publishers can make no warranties that the information contained herein is totally free from error, not least because clinical standards are constantly changing through research and regulation. The authors, editors, and publishers therefore disclaim all liability for direct or consequential damages resulting from the use of material contained in this book. Readers are strongly advised to pay careful attention to information provided by the manufacturer of any drugs or equipment that they plan to use.

SBAs for the FRCS(Tr&Orth) Examination A Companion to Postgraduate Orthopaedics Candidate’s Guide Following the format change to include single best answer questions (SBAs), this book equips candidates with a full range of testing examples to develop familiarity with the format and prepare for success in their FRCS (Tr & Orth) examination. Containing over 1,000 level 2 SBA questions, detailed explanations ensure candidates understand the reasoning and evidence-based decision-making behind each answer. Covering the breadth of the orthopaedic syllabus, including more difficult subject areas such as biomechanics, prosthetics/orthotics, anatomy and statistics, this is a crucial resource for all candidates. Encouraging the integration of clinical information with problem solving, this question format help candidates learn and retain the answers more efficiently than with simple factual recall. Written by highly experienced clinicians and examiners, these example questions are essential for preparing for the real examination. Paul A. Banaszkiewicz is Consultant Orthopaedic Surgeon, Queen Elizabeth Hospital and North East Surgery Centre [NENSC], Gateshead and a visiting Professor, Northumbria University, Newcastle upon Tyne, UK. Kiran K. Singisetti is Consultant Orthopaedic Surgeon, Queen Elizabeth Hospital and North East NHS Surgical Centre (NENSC), Gateshead, UK.

SBAs for the FRCS(Tr&Orth) Examination A Companion to Postgraduate Orthopaedics Candidate’s Guide Edited by

Paul A. Banaszkiewicz FRCS (Glas) FRCS (Ed) FRCS (Eng) FRCS (Tr & Orth) MClinEd FAcadMEd FHEA Consultant Orthopaedic Surgeon Queen Elizabeth Hospital and North East NHS Surgical Centre (NENSC), Gateshead, UK Visiting Professor Northumbria University, Newcastle-upon-Tyne, UK

Kiran K. Singisetti MBBS MRCS MS Fellow EBOT FRCS Ed (Tr & Orth) Queen Elizabeth Hospital and North East NHS Surgical Centre (NENSC), Gateshead, UK

University Printing House, Cambridge CB2 8BS, United Kingdom One Liberty Plaza, 20th Floor, New York, NY 10006, USA 477 Williamstown Road, Port Melbourne, VIC 3207, Australia 314–321, 3rd Floor, Plot 3, Splendor Forum, Jasola District Centre, New Delhi – 110025, India 79 Anson Road, #06–04/06, Singapore 079906 Cambridge University Press is part of the University of Cambridge. It furthers the University’s mission by disseminating knowledge in the pursuit of education, learning, and research at the highest international levels of excellence. www.cambridge.org Information on this title: www.cambridge.org/9781108789974 DOI: 10.1017/9781108855235 © Paul A. Banaszkiewicz and Kiran K. Singisetti 2021 This publication is in copyright. Subject to statutory exception and to the provisions of relevant collective licensing agreements, no reproduction of any part may take place without the written permission of Cambridge University Press. First published 2021 Printed in the United Kingdom by TJ International Ltd, Padstow Cornwall A catalogue record for this publication is available from the British Library. ISBN 978-1-108-78997-4 Paperback Cambridge University Press has no responsibility for the persistence or accuracy of URLs for external or third-party internet websites referred to in this publication and does not guarantee that any content on such websites is, or will remain, accurate or appropriate.

.......................................................................................................................................... Every effort has been made in preparing this book to provide accurate and up-to-date information that is in accord with accepted standards and practice at the time of publication. Although case histories are drawn from actual cases, every effort has been made to disguise the identities of the individuals involved. Nevertheless, the authors, editors, and publishers can make no warranties that the information contained herein is totally free from error, not least because clinical standards are constantly changing through research and regulation. The authors, editors, and publishers therefore disclaim all liability for direct or consequential damages resulting from the use of material contained in this book. Readers are strongly advised to pay careful attention to information provided by the manufacturer of any drugs or equipment that they plan to use.

Contents List of Contributors vii Foreword ix Preface xi Acknowledgements xiii List of Abbreviations xiv Interactive Website xviii

Section 1 General Advice for the FRCS (Tr & Orth) Oral Examination 1

2

Candidate Guidance for the Part 1 Written SBA Paper 1 Hussein Noureddine and Matthew Brown SBA Writing Process Paul Banaszkiewicz

Section 3 Trauma 12 Trauma I Structured SBA 189 Tim Brock and Rishi Dhir 13 Trauma II Structured SBA Nayef Aslam-Pervez

223

9

Section 2 Adult Elective Orthopaedics and Spine 3

Hip I Structured SBA Edward Holloway

4

Hip II Structured SBA 38 James Gill and Majeed Shakokani

5

Knee Structured I SBA Kiran K. Singisetti

59

6

Knee II Structured SBA Oliver Bailey

74

7

Foot and Ankle Structured SBA Gavin Heyes and Lyndon Mason

8

Spine I Structured SBA 105 Paul Rushton and Niall Eames

9

Spine II Structured SBA Prasad Karpe

Section 4 Children’s Orthopaedics / Hand and Upper Limb 14 Hand I Structured SBA Emma Reay

237

15 Hand II Structured SBA 256 Matthew Brown and David Yeoh

27

16 Children’s Orthopaedics I Structured SBA 286 Sattar Alshryda and Paul Banaszkiewicz 17 Children’s Orthopaedics II Structured SBA 306 Ling Hong Lee 84

134

10 Shoulder/Elbow I Structured SBA 157 Razvan Taranu and Shantanu Shahane 11 Shoulder/Elbow II Structured SBA 167 Mohan K. Pullagura and Faizan Jabbar

Section 5 Applied Basic Sciences 18 Anatomy and Surgical Approaches Structured SBA 323 Christopher George Ghazala 19 Structure and Function of Connective Tissue Structured SBA 351 Paul Banaszkiewicz 20 Pathology I Structured SBA 368 Rory Morrison and Rajesh Kakwani

v

Contents

26 Clinical Environment, Evidence Management and Quality Improvement Structured SBA 461 Hussein Noureddine

21 Pathology II Structured SBA 389 Alexander Durst and Majeed Shakokani 22 Pathology III Structured SBA 396 Tom Marjoram and Majeed Shakokani 23 Orthopaedic Oncology Structured SBA 406 Heledd Havard, Walid A. Elnahal and Craig H. Gerrand 24 Prosthetics and Orthotics Structured SBA Emerson Budhoo 25 Biomechanics Structured SBA 439 Edward Jeans and Richard Hutchinson

vi

27 Evidence Management: Data Analysis and Clinical Trials Structured SBA 473 Munier Hossain

428 Index

492

Contributors

Sattar Alshryda MBChB MRCP (UK) MRCS SICOT EBOT FRCS (Tr & Orth) MSc PhD Al Jalila Children’s Specialty Hospital Dubai UAE Nayef Aslam-Pervez MBChB, FRCS (Tr & Orth), Dip Med CAOS Royal Derby Hospital, UK Oliver Bailey MBChB, BSc, PG Dip Sports Med, FRCS (Tr & Orth) Acute Knee Service NHS Lanarkshire UK Paul A. Banaszkiewicz FRCS (Glas) FRCS (Ed) FRCS (Eng) FRCS (Tr & Orth) MClinEd FAcadMEd FHEA Queen Elizabeth Hospital and NENSC Gateshead, UK Northumbria University UK Timothy Brock MBChB BSc FRCS (Tr & Orth) MRCS MSc MRES Newcastle upon Tyne Hospital NHS Foundation Trust UK Matthew Brown MB BS BSc FRCS (Tr & Orth) St John’s Hospital, Livingston West Lothian, UK Emerson Budhoo MBBS, MRCSEd, DM (Ortho), FRCS (Tr & Orth) Caribbean Orthopaedic Foot and Ankle Institute St Clair Medical Centre, Medical Associates St Joseph, Rampersad Medical Centre

Niall Eames MD FRCS (Tr & Orth) Royal Victoria Hospital Belfast, UK Walid A. Elnahal MD FRCS (Eng) FEBOT MSc Royal National Orthopaedic Hospital Stanmore Middlesex UK Lecturer in Orthopaedics, Cairo University Craig H. Gerrand FRCSEd (Tr & Orth), MD, Royal National Orthopaedic Hospital, Stanmore, Middlesex UK Christopher George Ghazala BSc Hons (Dunelm) MB BS MRes (N’cle) MRCS (Eng) PGDipClinEd (Edin), FHEA Newcastle upon Tyne Hospital NHS Foundation Trust UK James Gill MB BChir BSc FRCS (Tr & Orth) West Suffolk Hospital Bury St Edmunds UK Heledd Havard MSc FRCS (Tr & Orth) Royal National Orthopaedic Hospital Stanmore, Middlesex UK Gavin Heyes MBBS Msc FRCS (Tr & Orth) Liverpool University Hospitals NHS Foundation Trust UK Edward Holloway FRCS (Tr UK & Orth) Chesterfield Royal Hospital

Rishi Dhir MBChB BSc (Hons) FRCS (Tr & Orth) Royal National Orthopaedic Hospital Stanmore, UK

Munier Hossain MBBS PGCE MSc (Orth Eng) MSc (Oxon) FRCS (Glas) FRCS (Tr & Orth) FHEA United Lincolnshire Hospitals NHS Trust UK

Alexander Durst MB ChB, BSc (hons), MRCS FRCS (Tr & Orth) Addenbrookes’ Hospital Cambridge UK

Richard Hutchinson MSc (Hons) FRCS (Tr & Orth) MRCS James Cook University Hospital Middlesbrough, UK

vii

List of Contributors

Faizan Jabbar MBChB, BSc, MRCS Queen Elizabeth Hospital and NENSC Gateshead, UK

Hussein Noureddine BSc MRCSEd CHM (Tr & Orth) FRCSEd (Tr & Orth) University Hospital Southampton UK

Edward Jeans BMedSci BMBS MRCS MSc FRCS (Tr & Orth) Honorary research fellow School of Engineering Cardiff University Wrightington Hospital Appley Bridge Wigan UK

Mohan K. Pullagura MS Orth MRCS (Ed) FRCS Ed (Tr & Orth) Whiston Hospital St Helens, UK

Rajesh Kakwani MBBS MRCS MS (Orth) FRCS (Tr & Orth) Northumbria Healthcare NHS Trust UK

Emma Reay MBBS FRCSEd (Tr & Orth) James Cook University Hospital Middlesbrough UK Paul Rushton BMedSci BMBS MSc FRCS (Tr & Orth) Royal Victoria Infirmary, Newcastle Upon Tyne UK

Prasad Karpe MBBS MS (Orth) MRCS (Edin) FRCS (Tr & Orth) Great Ormond Street Hospital for Children London

Shantanu Shahane Ms (Orth) Mch (Orth) FRCS (Tr & Orth) Chesterfield Royal Hospital UK

Ling Hong Lee MD Sunderland Royal Hospital Sunderland

Majeed Shakokani MBBS MRCS FRCS (TR & Orth) West Suffolk Hospital Bury St Edmunds UK

Tom Marjoram MBChB FRCS (Tr & Orth) East Suffolk and North Essex NHS Foundation Trust Ipswich, UK

Kiran K. Singisetti MBBS MRCS MS Fellow EBOT FRCS Ed (Tr & Orth) Queen Elizabeth Hospital and NENSC Gateshead, UK

Lyndon Mason MB BCh MRCS (Eng) FRCS (Tr & Orth) Liverpool University Hospitals NHS Foundation Trust UK

Razvan Taranu MSc, FRCSEd (Tr & Orth) Northumbria Healthcare NHS Trust UK

Rory Morrison BMSc MBChB MRCS Northumbria Healthcare NHS Trust UK

viii

David Yeoh MBBS Msc, FRCS (Tr & Orth) Norfolk and Norwich University Hospital Norwich, Norfolk UK

Foreword

Any fool can know. The point is to understand. – Albert Einstein Once again, the team from Postgraduate Orthopaedics has produced a superbly written guide to the FRCS (Trauma and Orthopaedics) exam. For most orthopaedic trainees, this represents the pinnacle of their training, and passing the exam is validation that they are ready for consultancy. The authors have captured the essence of the exam, which is to demonstrate a deep understanding of the art and science of our speciality and not simply to recite memorised knowledge. The Single Best Answer section is a proven method of testing this, and the book educates the reader as to why it is so important. Questions are supported by

up-to-date evidence as well as defining where there are controversies. Unique in this book are the chapters on tips for the exam as well as the mechanics on how the pass/fail decision is made. This will not only help readers understand the exam process but will also guide their preparation. The Editor, Paul Banaszkiewicz, is internationally recognised for his courses, books and passion for training. He has led much of the progress with education for the British Orthopaedic Association and has assembled an exceptional team of authors. Hiro Tanaka Consultant Orthopaedic Surgeon Chairman of the Education Committee BOA

ix

Preface

The Part 1 or written MCQ paper is often seen as a poor relation to the more intense clinical and viva preparation needed for the Part 2 exam. The belief is that most candidates usually pass the Part 1 without too many difficulties. Occasionally a candidate will have to re-sit if they fall a bit short of the pass mark, but next time it should be plain sailing getting through. There are very few quality Part 1 revision books available. Most have been written without due consideration to the high-stakes MCQ test paper criteria applied when constructing these questions. Single best answers are often confused with single correct answers. It is no great shock, then, that on the real test day some candidates can end up surprised with the different standard, style and type of questions they come across. There has been a move to replace old style level 1 factual recall questions with level 2 higher order thinking SBAs. This is a gradual process as it can sometimes take more than 10 hours of work to produce a good-quality SBA question with additional post-exam scrutiny to make sure the question really is achieving what it was set out to achieve. This book is long overdue for the Postgraduate Orthopaedics book series. We were keen to maintain our own high standards and did not want to end up as some compromised book with patchy, simplistic level 1 recall question setting. The ambition was to include the vast majority of questions as level 2 higher order thinking and also provide an explanation of the correct answer that allowed for a fuller understanding of the topic and for critical thinking and discussion. We also wanted the SBA to be clinically relevant and important. To write consistently good clinically based SBAs from scratch using an evidence-based approach with a quality explanatory answer was extremely difficult.

This was not expected or appreciated until the writing process was attempted. Some evenings no SBAs came forth whatsoever despite intense work. Thank goodness we had a few good evenings and weekends when a few high-quality new questions came together nicely. What we desperately wanted to avoid at all costs was an SBA level 1 factual recall submission. Even more important was to avoid rehashing a previously written level 1 SBA with just one or two words altered or a single option changed. This can happen more often than is acknowledged. A massive additional challenge in writing this book was to maintain a consistently high standard across all the 27 chapters submitted and to deal with unexpected no-show chapters. The EMI questions are being withdrawn, as they were too difficult to write and standardise. Although open to debate, they did not seem to be able to differentiate between good, average and poor candidates particularly well. With the inherent difficulties involved trying to juggle a healthy work life balance for most trainees, Part 1 preparation is about revision in a businesslike manner with both efficiency and effectiveness – most importantly, making maximal use of the revision time available and minimising any unproductive revision sessions. A ready source of realistic, wellexplained SBA questions that stimulate a thoughtprovoking process is surely something that should be aspired to. As with all books in the Postgraduate Orthopaedics series, we make no claim for the originality of the material. We are just trying to distil down key orthopaedic knowledge from a wide variety of sources to better help trainees in their exam preparation. We are not trying to re-invent the wheel. Wherever possible we have attempted to credit our sources used. Please except our sincere apologies if we have missed anyone out.

xi

Preface

We are very grateful to the question review teams who pre-tested some of the SBAs. This was a valuable learning exercise on both sides, with one of the few truly ‘win-win’ situations that we have come across in medicine. If we have written a book that provides a more realistic idea of the type of questions that candidates

xii

will face such that they approach their learning schedule in a more realistic and exam-savvy way, then we will have achieved most of what we set out to do Paul Banaszkiewicz Kiran K. Singisetti

Acknowledgements

Special thanks to all the authors involved with the Postgraduate Orthopaedics book series over the years. Without your input, no books would be possible. As always, thanks to Nicholas Dunton at Cambridge University Press for his help, guidance and strong support over the years. Many thanks to Faizan Jabbar for coordinating our new website structure and Samar Khan for imputing content. Big thanks to our medical artists Mehak Kakwani and Ankita Bhalla who both did a great job of drawing the book illustrations at very short notice. A very special thank-you to our new web designer Farrakh, who has helped us develop our website above and beyond the call of duty. We bombarded him with multiple different design requests and changes to plan and late-at-night WhatsApp queries that needed to be sorted out there and then. As ever, thanks to Jo McStea who keeps the whole PGO setup rolling along.

Specialist registrar London Deanery Ran Wei

Therapists Katrina Fraser, Clinical Specialist Occupational Therapist, Hand Therapy, St John’s Hospital, Livingston, West Lothian. Rebecca Champion, Clinical Specialist Occupational Therapist, Hand Therapy, Norfolk & Norwich Hospital, Norfolk, Norwich

Specialist registrars Northern Deanery Mark Sohatee Becky Critchley Helen Ingoe Nicole Abdul Marina Diament

xiii

Abbreviations

1,2 ICSRA A&E AAI AAOS ABC ABG ABI ABPI AC ACDF ACI ACJ ACL AD ADM ADQ AF AFO AGP AI AIIS AIN AIS AIS AITFL ALIF ALL ALVAL AM AMIC ANOVA Anti-CCP AOFAS AP APB APL ARMD AS ASIA ASIS ATFL ATLS® ATP

xiv

1,2 intercompartmental supraretinacular artery Accident and Emergency atlantoaxial instability American Academy of Orthopaedic Surgeons aneurysmal bone cyst arterial blood gas ankle–brachial index Ankle–Brachial Pressure Index acromioclavicular anterior cervical decompression and fusion autologous chondrocyte implantation acromioclavicular joint anterior cruciate ligament autosomal dominant abductor digiti minimi abductor digiti quinti annulus fibrosis ankle–foot orthosis aerosol-generating procedure acetabular index anterior inferior iliac spine anterior interosseous nerve Abbreviated Injury Scale adolescent idiopathic scoliosis anterior-inferior tibiofibular ligament anterior lumbar interbody fusion anterior longitudinal ligament aseptic lymphocyte-dominated vasculitisassociated lesions anteromedial autologous matrix-induced chondrogenesis analysis of variance anti-cyclic citrullinated peptide American Orthopaedic Foot and Ankle Society anteroposterior abductor pollicis brevis abductor pollicis longus adverse reactions to metal debris ankylosing spondylitis American Spinal Injury Association anterior superior iliac spine anterior talofibular ligament Advanced Trauma Life Support® adenosine triphosphate

AVN BAI BAPRAS BDI BKA BMI BMP BMU BOA BOAST BP BPTB BSCOS BW CC CES CES-C CES-I CES-R CES-S CFL CFU CI CMC CMCJ CML CMT CoC CoCR COPD COR CP CPK CR CROW CRP CT CTEV CTLSO DAA DAIR DASH DASS

avascular necrosis Basion–axial interval British Association of Plastic, Reconstructive and Aesthetic Surgeons Basion–dens interval below-knee amputation body mass index bone morphogenetic protein basic multicellular unit British Orthopaedic Association British Orthopaedic Association Standards for Trauma blood pressure bone–patella–tendon–bone British Society for Children’s Orthopaedic Surgery body weight costoclavicular cauda equina syndrome cauda equina syndrome complete cauda equina syndrome incomplete cauda equina syndrome retention cauda equina syndrome suspected calcaneofibular ligament colony-forming units confidence interval carpometacarpal carpometacarpal joint classic metaphyseal lesion congenital muscular torticollis ceramic on ceramic cobalt chrome chronic obstructive pulmonary disease centre of rotation cerebral palsy creatine phosphokinase cruciate retaining Charcot restraint orthotic walker C-reactive protein computed tomography congenital talipes equinovarus cervical-thoracic-lumbar-sacral orthosis direct anterior approach debridement, antibiotics and implant retention Disabilities of the Arm, Shoulder and Hand Depression Anxiety Stress Scales

List of Abbreviations

DBB DCO DCP DDH DE DEXA DGH DHS DI DIF I DIP DIPJ DISI DRUJ DVT EAC ECRB ECRL ECU EDC EDU EIP EMG EMIs EPB EPL ER ESIN ESR FAI FBC FCR FCU FDG FDL FDP FDS FFP FGF FGF23 FGFR3 FHC FHL FOP FPL FPP GA GCS GCT GLAD GMC GMFCS GPa GT GTN HA HA HAGL

Denis Browne boots and bar damage control orthopaedics dynamic compression plate developmental dysplasia of the hip distractor efficiency dual energy x-ray absorptiometry district general hospital dynamic hip screw discrimination index difficulty index distal interphalangeal distal interphalangeal joint dorsal intercalated segmental instability distal radioulnar joint deep vein thrombosis Early Approprirate Care extensor carpi radialis brevis extensor carpi radialis longus extensor carpi ulnaris extensor digitorum communis extensor carpi ulnaris extensor indicis proprius electromyography extended matching items extensor pollicis brevis extensor pollicis longus external rotation elastic stable intramedullary nailing erythrocyte sedimentation rate femoral acetabular impingement full blood count flexor carpi radialis flexor carpi ulnaris fluorodeoxyglucose flexor digitorum longus flexor digitorum profundus flexor digitorum superficialis fresh frozen plasma fibroblast growth factor fibroblast growth factor 23 fibroblast growth factor receptor gene 3 femoral head coverage flexor hallucis longus fibrodysplasia ossificans progressiva flexor pollicis longus functional pelvic plane general anaesthetic Glasgow Coma Score giant cell tumour glenolabral articular disruption General Medical Council Gross Motor Function Classification System gigaPascal greater trochanter glycerl trinitrate hyaluronic acid hydroxyapatite humeral avulsion of the glenohumeral ligament

HEPA HIV HO HOTS HPAA HS HTO HXLPE ICB ICSRA IFSSH IHDI II IKDC IL IM IOL IOPFF IP IPJ IQR ISS ITB ITOH ITT ITU IV JBJS JCIE KOOS LABC LCH LCL LCP LCPD LIPUS LLD LMWH LOTS LT LUCL MARS MCID MCL MCP MCPJ MCQs MCSF MDT MFCA MFL MHRA MI

high-efficiency particulate air human immunodeficiency virus heterotopic ossification Higher Order Thinking Skills hypothalamic-pituitary-adrenal axis hamstring high tibial osteotomy highly cross-linked polyethylene Intercollegiate Board intercompartmental supraretinacular artery International Federation of Societies for Surgery of the Hand International Hip Dysplasia Institute image intensifier International Knee Documentation Committee interleukin intramedullary interosseous ligament intraoperative periprosthetic femoral fractures interphalangeal interphalangeal joint interquartile range Injury Severity Score iliotibial band idiopathic transient osteoporosis of the hip intention to treat Intensive Care Unit intravenous Journal of Bone and Joint Surgery Joint Committee on Intercollegiate Examinations Knee Injury and Osteoarthritis Outcomes Score lateral antebrachial cutaneous Langerhans cells histiocytosis lateral collateral ligament low compression plates Legg–Calve–Perthes disease low intensity pulsed ultrasound limb length discrepancy low molecular weight heparin Lower Order Thinking Skills lunotriquetral lateral ulnar collateral ligament metal artefact reduction sequence minimal clinically important difference medial collateral ligament metacarpophalangeal metacarpophalangeal joint multiple choice questions macrophage-colony stimulating factor multidisciplinary team medial femoral circumflex artery meniscofemoral ligament Medicines and Healthcare products Regulatory Agency migration index

xv

List of Abbreviations

MIS MMP MOXFQ MPFL MR MRA MRC MRI MSC MTP MTPJ MUA NAI NBME NCT NF NF-1 NF-2 NFD NICE NNH NNT NOF NSAIDs NTN OA OATS OCD OCD ODEP OI OITE OMT ON OPLL OR ORIF PA PADI PCL PDGF PE PE PEP PET PFJ PFVO PGE2 PI PIN PIP PIPJ PITFL PJI PL PL PLC

xvi

minimally invasive surgical metalloproteinase Manchester–Oxford Foot Questionnaire medial patellofemoral ligament magnetic resonance magnetic resonance arthrography Medical Research Council magnetic resonance imaging mesenchymal stem cell metatarsophalangeal metatarsophalangeal joint manipulation under anaesthesia non-accidental injury National Board of Medical Examiners nerve conduction test neurofibromatosis neurofibromatosis type 1 neurofibromatosis type 2 non-functional distractors National Institute for Health and Clinical Excellence number needed to harm number needed to treat neck of femur non-steroidal anti-inflammatory drugs National Training Number osteoarthritis osteochondral autograft transfer system occipitocervical dissociation osteochondritis dessicans Orthopaedic Data Evaluation Panel obturator internus Orthopaedic In-Training Exam Oberg, Manske and Tonkin osteonecrosis ossification of the posterior longitudinal ligament odds ratio open reduction internal fixation posterior anterior posterior atlantodental interval posterior cruciate ligament platelet-derived growth factor polyethylene pulmonary embolism post-exposure prophylaxis positron emission tomography patellofemoral joint proximal femoral varus osteotomy prostaglandin E2 pelvic incidence posterior interosseous nerve proximal interphalangeal proximal interphalangeal joint posterior-inferior tibiofibular ligament periprosthetic joint infection palmaris longus posterolateral posterior ligamentous complex

PLC PLI PLIF PLRI PMC PMMA PMN PMT POSI PPE PROM PRP PS PSA PT PTH PTHrP PTTD PVNS RANK RANKL RCT RF ROC ROM RR RR RSA RTA RVAD SACH SBA SCALE SCD SCFE SCIWORA SCR SD SDR SEM SF-36 SI SIJ SL SLAC SLAP SLE SLL SLR SMA SMC SME SNAC SNAP SONK

posterolateral corner posterolateral instability posterior interbody lumbar fusion posterolateral rotatory instability posteromedial corner polymethylmethacrylate polymorphonuclear neutrophil phosphaturic mesenchymal tumour position of safe immobilisation personal protective equipment patient-reported outcome measure platelet-rich plasma posterior stabilised prostate-specific antigen pronator teres parathyroid hormone parathyroid hormone-related protein posterior tibial tendon dysfunction pigmented villonodular synovitis receptor activator of nuclear factor κβ receptor activator of nuclear factor kB ligand randomised controlled trial rheumatoid factor receiver operating characteristic range of movement relative risk risk ratio reverse shoulder arthroplasty road traffic accident rib vertebral angle difference solid ankle cushioned heel single best answer Selective Control Assessment of the Lower Extremity sickle cell disease slipped capital femoral epiphysis spinal cord injuries without radiographic abnormalities superior capsular reconstruction standard deviation selective dorsal rhizotomy Standard Error of Measurement Short Form (36) Health Survey sacroiliac sacroiliac joint involvement scapholunate scapholunate advanced collapsed superior labrum from anterior to posterior systemic lupus erythematosus scapholunate ligament straight leg raise second moment area selective motor control subject matter expert scaphoid non-union advanced collapsed sensory nerve action potential spontaneous osteonecrosis of the knee

List of Abbreviations

SPECT SPT SRN SS ST3 STT SUFE SUV SVA T&O TB TEG TENS TER TF TFCC TGCT THA THR Ti TIMP TKA TKR TLICS TLSO TLSO TM

single photon emission computed tomography spinopelvic tilt superficial radial nerve sacral slope surgical trainee year 3 scaphotrapeziotrapezoid slipped upper femoral epiphysis standardised uptake value sagittal vertical axis Trauma and Orthopaedics tuberculosis thromboelastography transcutaneous electrical nerve stimulation total elbow replacement transfemoral triangular fibrocartilage complex tenosynovial giant cell tumour total hip arthroplasty total hip replacement titanium tissue inhibitory metalloproteinase total knee arthroplasty total knee replacement Thoracolumbar Injury Classification and Severity thoracolumbar spinal orthosis thoracolumosacral orthosis trabecular metal

TMT TMTJ TNF TNSALP TPD TSA TSR TT TT–TG UCL UHMWPE UKA UKITE UMN US VACTERL VAS VDRO VISA-A VISI VTE WBA WBC WCC XLHR ZPA

tarsometatarsal tarsometatarsal joint tumour necrosis factor tissue-nonspecific isoenzyme of alkaline phosphatase Training Program Director total shoulder arthroplasty total shoulder replacement tibial tubercle tibial tuberosity–trochlear groove ulnar collateral ligament ultra-high-molecular-weight polyethylene unicompartmental knee arthroplasty United Kingdom In-Training Examination upper motor neuron ultrasound vertebral, anorectal, cardiac, tracheal, oesophageal, renal and limb visual analogue scale varus derotation osteotomy of the femur Victorian Institute of Sport AssessmentAchilles Questionnaire volar intercalated segmental instability venous thromboembolism workplace-based assessments white blood cell white blood cell count X-linked hypophosphataemic rickets zone of polarising activity

xvii

Interactive Website

The website to accompany the books www.postgraduateorthopaedics.co.uk This website accompanies the textbook series: Postgraduate Orthopaedics. It includes:  Postgraduate Orthopaedics: The Candidates Guide to the FRCS (Tr & Orth) Examination, third edition  Postgraduate Orthopaedics: Viva Guide for the FRCS (Tr & Orth) Examination, second edition  Postgraduate Paediatric Orthopaedics The aim is to provide additional information and resources in order to maximise the learning potential each book.

xviii

Additional areas of the website provide supplementary orthopaedic material, updates and web links. Meet the editorial team provides a profile of authors who were involved in writing the books. It is very important our readership gives us feedback. Please email us if you have found any errors in the text that we can correct. In addition, please let us know if we haven’t included an area of orthopaedics that you feel we should cover. Likewise, any constructive suggestions for improvement would be most welcome.

Section 1 Chapter

1

General Advice for the FRCS (Tr & Orth) Oral Examination

Candidate Guidance for the Part 1 Written SBA Paper Hussein Noureddine and Matthew Brown

The FRCS (Tr & Orth) exam comprises two parts, and transition to the Part 2 clinical and viva voce exam is dependent upon candidates passing the Part 1 written component.

The Exam Format Part 1 is the written component of the Intercollegiate Examination in Trauma and Orthopaedic Surgery. In 2018 the Joint Committee on Intercollegiate Examinations (JCIE) agreed to phase out extended matching item (EMI) questions. When compared to single best answer (SBA), EMI questions were less able to differentiate candidates and were difficult to construct. Subject to General Medical Council (GMC) approval, EMI questions will not feature in the FRCS (Tr & Orth) examinations from January 2021 onwards. Part 1 exams are currently held at Pearson VUE Test Centres at multiple locations throughout the United Kingdom and Ireland. Candidates can choose their preferred centre during registration. These test centres often host unrelated tests (e.g. driving theory, USMLE) that take place alongside the Part 1 exam. Computer stations are separated by dividers to help minimise visual distraction. Be prepared to focus so as not to be distracted by the movements of others. Some candidates may choose to travel further to utilise quieter test centres. Candidates should bring photographic identification on the day of the exam. This is checked at registration and again before entry into the examination room. Exam conditions are strict. Bags and all but essential items will be stored in the lockers provided at most centres. Unsurprisingly, no mobile devices are permitted in the examination room. Video surveillance of candidates is common. Depending on the location, it is recommended to bring lunch, as some centres do not have local facilities to purchase food. Paper and a pencil are provided for making notes.

The computer-based questions include multimedia images such as radiographs and clinical photographs. Candidates are not permitted to read ahead but will be able to flag difficult or ambiguous questions for later review.

Overview At the time of publication, the Part 1 exam consists of two papers as follows: Paper 1 (2 hours) Single best answer (SBA) – 110 questions Paper 2 (2 hours 30 minutes) Extended matching item (EMI) – 135 questions – 45 themes with option lists – 3 questions using each of these option lists Total 4 hours 30 minutes – 245 questions From January 2021 onwards (subject to GMC approval), the Part 1 exam will consist of two papers as follows: Paper 1 (2 hours 15 minutes) Single best answer (SBA) – 120 questions Paper 2 (2 hours 15 minutes) Single best answer (SBA) – 120 questions Total 4 hours 30 minutes – 240 questions Candidates will have a 2-year period from their first attempt to pass the Part 1 exam, with a maximum of four attempts with no re-entry. Details are available on the JCIE website (www.jcie.org.uk). Candidates with proven dyslexia may be eligible for the Part 1 examination times to be extended and this should be highlighted in advance of the exam.

1

Hussein Noureddine and Matthew Brown

There is no negative marking; therefore, all questions should be attempted. Sample questions can be viewed on the JCIE website. Experienced examiners perform a formal process of standard setting to decide the final pass mark for each paper. The SBA questions are subject to quality assurance procedures, including feedback from both examiners and candidates. Difficulty level, content, discrimination index and internal consistency are analysed. Ambiguous questions or those deemed insufficient to differentiate between candidates are removed through this process. The SBA questions consist of an introductory theme, a question stem and five possible responses (listed A–E), of which one is the most appropriate answer. SBA questions are exactly what the name suggests: candidates choose the best from five possible answers. It is important to note that this is not a ‘single correct answer’ but a ‘single best answer’. Moreover, all five possible answers could be considered correct, but candidates are asked which is best, or most appropriate, given the information provided. As questions are designed to test higher order thinking, this could mean that limited or irrelevant information is provided. Questions require a judgement based on interpretation of the available evidence. Questions that candidates later complain about, for example, ‘there was more than one correct answer’ or that a question was ‘too ambiguous’, can often prove the best performing questions. Although the standard is widely publicised to be set at the level of a day one consultant working in the generality of trauma and orthopaedics, candidates should appreciate that some questions will appear more niche and stretch them more than others. Advantages of SBA questions:  They can assess higher order learning and discriminate between candidates of differing ability.  They can assess a broad sample of the curriculum within a relatively short period of time, which helps to improve reliability and validity.  With all trainees assessed using the same highly standardised questions, they make for a fair assessment.  Automated marking helps to remove examiner subjectivity and reduce costs.

2

 The pitfalls of other question formats (e.g. EMIs, true/false) make SBA questions a popular choice for high-stakes examinations like the FRCS. Disadvantages of SBA questions:  A candidate’s reasoning for selecting a particular answer cannot be assessed.  Despite permitting a broad assessment of the curriculum, there is little opportunity to focus indepth on a particular subject.  They rarely reflect the real-life practices of surgical diagnosis and management, which are varied and nuanced. The five example SBA questions provided on the JCIE website are shown below. 1. A 4-year-old girl has had a swollen, painless left knee joint for 8 weeks. In the last ten days her left ankle has become swollen. It is uncomfortable after she has been sitting for 20 minutes. Examination of her eyes shows an irregular pupil on the right side. What is the most appropriate first step in the management of this patient? A. A painful heel due to enthesopathy is a common associated condition B. Her pupillary abnormality is likely to be due to a dislocated lens C. Synovectomy leads to improved joint function over the short and medium term D. The HLA B27 gene is a strong marker for this condition E. This child is likely to be rheumatoid factor negative 2. A 19-year-old motorcyclist is brought to the Emergency Department following an RTA. He is complaining of difficulty moving his left arm. Clinical examination suggests the latissimus dorsi, subscapularis, pectoralis major and pectoralis minor are functioning, but infraspinatus is not functioning. Which part of the brachial plexus is most likely to have been injured? A. The lateral cord B. The medial cord C. The middle trunk D. The posterior cord E. The upper trunk

Candidate Guidance for the Part 1 Written SBA Paper

3. A 35-year-old man has a combined ACL rupture and posterolateral instability. There is a bony varus knee deformity with lateral thrust in the stance phase of gait. What is the most appropriate treatment for this patient? A. ACL reconstruction alone B. Distal femoral osteotomy C. Reconstruction of the posterolateral corner alone D. Simultaneous reconstruction of the ACL and the posterolateral corner E. Valgus osteotomy and ligament reconstruction simultaneously 4. A 25-year-old man had an acute dislocation of his shoulder, which was reduced. During the followup visit he was found to have wasting of the deltoid and infraspinatus muscles. Injury to which one of the following neural structures accounts for this finding? A. Axillary nerve B. Lateral cord of the brachial plexus C. Posterior cord of the brachial plexus D. Suprascapular nerve E. Upper trunk of the brachial plexus 5. A 20-year-old woman has had low back pain for the past 6 months. She has no fever or constitutional symptoms. Radiographs of her thoracic spine show coarse striations of the vertical trabeculae of the 12th thoracic vertebra. Her ESR is normal. What is the most likely diagnosis? A. Bone cyst B. Fibrous dysplasia C. Haemangioma D. Non-ossifying fibroma E. Osteoblastoma

What is the Relevance of the FRCS (Tr & Orth) Exam? The FRCS (Tr & Orth) exam helps to reassure patients, the GMC and employers that a candidate has reached the necessary standard required for independent practice as a consultant. The reference level is that of a day one consultant working in a district general hospital in the generality of trauma

and orthopaedic surgery. It is important for the public to have confidence in the process. The exam assesses knowledge and judgement, clinical acumen, management and treatment planning, as well as communication skills.

Bloom’s Level 1, 2 and 3 Questions The Part 1 exam is designed to test knowledge from across the Trauma and Orthopaedics (T&O) curriculum by using questions that require higher order thinking. Bloom’s Taxonomy defines six cognitive categories: knowledge, comprehension, application, analysis, synthesis and evaluation. The taxonomy presents a cumulative hierarchy, with categories ordered from simple to complex and concrete to abstract. The five categories that follow knowledge cover skills and abilities, with mastery of simpler levels considered a prerequisite for mastery of the next, more complex level. Rather than assessing factual recall (level 1 questions), the FRCS (Tr & Orth) exam aims to assess each candidate’s ability to apply their knowledge to solve a clinical scenario or problem (level 2 and 3 questions). Higher order questions make up the majority of the question bank, and there is a drive to increase the proportion further. The differences between level 1 and level 2 questions are best illustrated with an example. For a tibial shaft fracture: Level 1: How do you classify (knowledge), how are you going to manage the fracture and what is your operative technique? Level 2: What will be your treatment plan for the featured patient? Higher order thinking is being replaced with the term ‘higher order judgement’. The difference is best illustrated with an example. For a tibial plateau fracture, higher order thinking may test why the fracture is classified as a Schatzker type III rather than type II. With higher order judgement the patient is presented as a 46-year-old man with a tibial plateau fracture that has 4mm of articular depression. This fracture would normally require operative fixation with grafting and elevation of the articular surface. However, the scenario is expanded further to include the patient’s medical history, which includes diabetes, peripheral vascular disease and chronic alcohol abuse. Candidates are expected to make a judgement based on clinical experience. How one manages this patient

3

Hussein Noureddine and Matthew Brown

in reality may differ significantly from what is outlined in the textbooks! The exam is designed to assess real-life decision-making. Multilogical thinking is a new and evolving concept in exam theory that pertains to questions requiring knowledge of more than one fact to logically and systematically apply concepts to solve a problem or clinical scenario. Such questions present multiple viable answers and are highly valued for their capacity to differentiate between candidates.

General Advice The Part 1 exam seeks to test background knowledge and judgement that will have developed during daily clinical work, rather than abstract facts from a book. The exam, and the preceding revision period, will be stressful and exhausting. Candidates should ensure that they are physically and mentally prepared. Eating a well-balanced diet, keeping hydrated, minimising alcohol intake and taking regular breaks during revision are all advised. Engaging with regular physical activity will also help to improve concentration and well-being. It will prove helpful to allocate an evening or afternoon per week to spend with family and friends. Concentrate on sleep patterns in the weeks and months leading up to the exam. Remember that caffeine has a half-life of 4–6 hours, meaning that it will take up to 24 hours for it to be cleared. Additionally, maintain a clear distinction between revision and relaxation by avoiding bedroom-based revision. After completing a bank of questions, it is important to allocate sufficient time to review the answers, which can take far longer than expected. With this in mind, consider supplementing evening revision with early morning revision. Every candidate will have different home circumstances and revision preferences, so it is important to create a personal schedule that suits. Identify areas of knowledge that are lacking and tackle these subjects head-on. Avoid wasting time by reading around answers to questions that were answered correctly and with relative certainty. There is insufficient time for this feel-good approach, given the breadth of the T&O curriculum. It is a valuable revision strategy to review correctly answered questions that were based on a lucky or best guess. Reviewing these questions will help consolidate your knowledge.

4

Many candidates soon realise that practicing questions has to be prioritised over reading or making detailed notes. The following advice may be useful:  Troublesome questions are made more difficult if candidates are underprepared. Understand the breadth of the T&O curriculum and practise the SBA question format to develop the required exam technique.  Confront difficult areas of the curriculum early and avoid burying topics for the few weeks preceding the exam, as this consolidation stage is associated with its own stresses.  One mark can make the difference between a pass and a fail. Practice questions at the correct knowledge level and under timed conditions. Some websites permit a time limit for each question. At the start of your revision, consider allocating 70–80 seconds per question and reduce this to 50–60 seconds as your knowledge and familiarity improve.  Efficient time management during the exam is important. Go fast. Candidates have only one minute to read and interpret each question, consider the options and indicate an answer. Some questions will present cases with a long stem.  Each question carries a 1 in 5 chance of being correct, so be sure to provide an answer for every question. There is no negative marking.  If an answer is not immediately clear, flag the question, mark a best guess and move on promptly with the foresight that returning later may not be possible. Flagging a question electronically will help expedite later review. Marking a best guess will avoid the common error of running out of time and throwing away marks.  Read questions carefully and understand fully what the question stem is asking. All of the options presented may not be ideal, but one must be selected from the options available. Additionally, the answer to questions requesting the ‘most appropriate management’ may differ from those asking the ‘next most appropriate step in management’.  Candidate feedback suggests that around 20% of questions are straightforward. These test standard textbook knowledge, with answers easily narrowed down to perhaps two options (level 1). The remaining questions are less obvious, with

Candidate Guidance for the Part 1 Written SBA Paper

detailed or ambiguous stems and similar answers requiring a more considered judgement (levels 2 and 3).  Do not assume that ambiguous questions will be removed during the final quality assurance process, as very few questions are actually removed.  In recent exams there has been a greater emphasis on higher order anatomy questions. Consider reading an anatomy textbook and practice anatomy questions.

The Build-Up to the Exam Much of the knowledge and judgement required for the FRCS (Tr & Orth) exam will have been acquired during training; however, the T&O syllabus is extensive, and the exam requires lots of preparation. Applications are made through the JCIE website where guidance notes, eligibility criteria and future dates for Parts 1 and 2 (termed Sections 1 and 2) can be reviewed. Candidates should plan their preferred date or ‘diet’ for Part 1 after considering their likely personal and professional circumstances and, for UK trainees with a National Training Number (NTN), the date of their ST6 ARCP. NTN trainees must achieve an Outcome 1 at their ST6 ARCP before they are eligible to apply. Those not in training are required to demonstrate a level of competence and knowledge equivalent to that achieved by NTN trainees at the end of ST6. Be sure to read the JCIE ‘Guidance Notes for Applicants’ document far in advance of making an application. The Part 1 exam is usually timed a few months ahead of the next Part 2 (clinical and viva voce) exam. The preferred exam dates for both parts are requested at the time of initial application; however, the date for Part 2 will only be confirmed after Part 1 is passed. First-time applicants must submit their application with full payment for both Parts 1 and 2 ahead of the published deadlines, which are set approximately 10–12 weeks ahead of each Part 1 exam. The number of candidates permitted to complete Part 2 at each diet is capped and it is widely understood that candidates who apply far in advance of the application deadline are more likely to secure their preferred date for Part 2. The requirement for social distancing in response to the coronavirus pandemic may have implications for those sitting both Parts 1 and 2. Keep an eye on the JCIE website for further guidance.

The following must be satisfied when completing the online application:  Payment in full (covering Parts 1 and 2)  Three completed Structured Reference forms  Curriculum vitae  Summary of Operative Experience  Photographic identification NTN trainees should submit three Structured Reference forms, including one completed by their Training Program Director (TPD) and two by other consultants. Applicants not in training should provide structured references from the head of department (clinical lead) and two other consultants. To avoid unnecessary delays, candidates should contact potential referees far in advance of their planned application date. Plan upcoming clinical rotations to ensure that they will permit the necessary time for revision and cover gaps in clinical knowledge and experience. Such placements should also allow for the clustering of annual and study leave for revision and courses in the weeks leading up to both parts of the exam. It is generally recommended to start revising at least 6 months before the Part 1 exam. Candidates who report preparing for just a few months are either superbly talented, incredibly lucky or inappropriately misleading. Avoid accepting work-related projects, such as research, in the months ahead of your revision. Fee penalties apply to candidates who withdraw from the exam after the closing date, so choose the preferred date carefully after considering all the aforementioned circumstances. It should be noted that candidates who withdraw during an exam will be deemed to have failed and will forfeit one of their four attempts.

Preparation and Revision Resources Approaching the exam as a single assessment consisting of two parts (as opposed to two separate entities) is the preferred revision strategy. The common denominator for Parts 1 and 2 is for candidates to develop knowledge that is of sufficient breadth and depth. Those who begin by revising topics using textbooks and online resources before proceeding to SBA question practice often succeed in passing both parts in successive diets (e.g. February Part 1 followed by April Part 2). In contrast, those who approach Part 1 by concentrating heavily on questions may require a longer interval to convert the SBA ‘best of 5’ skill to that of being able to coherently elaborate and expand

5

Hussein Noureddine and Matthew Brown

on topics during the Part 2 viva voce exam. A combination of both strategies is perhaps the best compromise. Whatever strategy is adopted, be sure to dedicate sufficient time to practice the SBA question format, aiming to complete a few thousand questions as a minimum. Questions will not only reinforce and test knowledge, but also help to gauge the time constraints of the actual exam. Although fundamental to preparing for the Part 2 exam (clinical and viva voce), some candidates find an informal study group helpful during Part 1. Peerto-peer teaching and discussion is a powerful learning tool. Although group learning can take various forms, verbalising knowledge according to a structured revision timetable can prove useful, especially ahead of the Part 2 exam. Revision groups should be limited to a small number of individuals who share similar knowledge levels, plan to sit the exam on the same date and are fully committed to the process. Moreover, it is important to recognise when revision methods are proving ineffectual as evidenced by poor progression or revision group distraction or fatigue. Candidates should be honest with their study partners if this becomes the reality and refocus or redesign their revision strategy at the earliest opportunity. Finally, consistency and continuity are imperative, and candidates should avoid prolonged gaps in their revision. The following revision resources are commonly used by candidates for the Part 1 exam:  Postgraduate Orthopaedics – The Candidate’s Guide (3rd edition) This comprehensive textbook helps to prepare candidates for the clinical and viva voce aspects of the Part 2 exam. It utilises concise prose, graphics, illustrations and case-based examples to consolidate knowledge gained during preparation for Part 1. Cases are designed to reflect those in the exam. Insights from recent candidates help to demonstrate good and bad practice during the viva voce exam. Although this textbook is oriented towards Part 2, the core topic sections will provide a useful revision aid for the questions found in Part 1.  Postgraduate Paediatric Orthopaedics Although oriented towards the Part 2 exam, this textbook is packed with diagnostic and surgical tips that will aid success in both parts. The dysplasias section offers a structured methodology when approaching any skeletal dysplasia, and the

6









1

cerebral palsy section touches on gait analysis with clear graphs of the types that could be asked in both parts of the exam. Miller’s Review of Orthopaedics (7th edition) This comprehensive textbook presents the breadth of T&O surgical practice in one volume, including anatomy and the basic sciences. The book is aligned with the American Board of Orthopaedic Surgery exam but remains very popular for FRCS (Tr & Orth) revision, especially when preparing for Part 1. Recent editions are easier to read and include colour illustrations, clinical photographs and tables. Candidates may choose to focus on specific sections or chapters, read it in its entirety or use it as a reference alongside other resources. Basic Orthopaedic Sciences (2nd edition) This popular textbook for both parts of the exam aims to cover the basic sciences that underpin T&O surgical practice. Topics include biomechanics, biomaterials, immunology, pharmacology, imaging techniques and statistics. Some chapters read more clearly and accurately than others. Orthopaedic Basic Science for the Postgraduate Examination: Practice MCQs and EMQs1 Included here are more than 500 multiple choice and extended matching questions related to orthopaedic basic science. Detailed and insightful explanations are included for each question. The level of knowledge required is perhaps above that expected for the FRCS (Tr & Orth) exam but good to test knowledge on areas of the curriculum that are often neglected in everyday practice. Postgraduate Orthopaedics: MCQs and EMQs for the FRCS (1st edition) The predecessor to this edition was published in 2012. It contains an additional bank of quality SBA and EMQ questions that remain relevant to the FRCS (Tr & Orth) exam. Answer explanations are short but adequate for rapid revision. Candidates may consider using this older textbook

Dawson-Bowling SJ, McNamara IR, Ollivere BJ, et al. Orthopaedic Basic Science for the Postgraduate Examination: Practice MCQs and EMQs. Gloucester: Orthopaedic Research UK Publishing; 2012.

Candidate Guidance for the Part 1 Written SBA Paper











2

3

4

5

6

later in their revision when they are better able to identify information that may be outdated. AAOS Comprehensive Orthopaedic Review (2nd edition)2 This comprehensive and well laid out threevolume text is designed for the American Board exam. The final volume is dedicated to multiple choice question practice. An excellent but expensive resource. Succeeding in the FRCS T&O Part 1 Exam3 This book has received mixed online reviews, with the majority proving highly negative. Questions concentrate on factual recall and explanations can be confusing, unfocused and contradictory. It is perhaps a book to consider borrowing rather than buying and should not form a significant part of one’s Part 1 preparation. Practice Questions in Trauma and Orthopaedics for the FRCS4 The questions in this outdated book more closely reflect the standard of the MRCS exam and are far removed from that of the FRCS (Tr & Orth) Part 1 exam. Poor online reviews reflect the low level of knowledge examined. First Aid for the Orthopaedic Boards (2nd Edition)5 This book is written for the in-service exams (Orthopaedic In-Training Exam [OITE]) of the American Board. It receives mixed reviews, with the question style and depth differing from that observed in the Part 1 exam. Although it is easy to read and may help you score a few extra points, it is expensive for what it provides. Review Questions in Orthopaedics6 Originally written for orthopaedic residents preparing for the in-training (OITE) exams of the Lieberman JR, ed. AAOS Comprehensive Orthopaedic Review. Rosemont, IL: American Academy of Orthopaedic Surgeons; 2009. Gulam Attar F, Ibrahim T. Succeeding in the FRCS T&O. Part 1: Exam. London: BPP Learning Media; 2011. Sharma P. Practice Questions in Trauma and Orthopaedics for the FRCS (Master Pass Series). Milton Keynes: Radcliffe Publishing Ltd; 2007. Mallinzak RA, Albritton MJ, Pickering TR. First Aid for the Orthopaedic Boards. 2nd ed. Bronson, TX: McGrawHill Medical; 2009. Wright JM, Millett PJ, Crockett HC, Craig EV. Review Questions in Orthopaedics. Rosemont, IL: American Academy of Orthopaedic Surgeons; 2001.

American Board, this book (often termed the ‘black book’ by UK candidates) has remained a favourite supplementary question bank for the FRCS (Tr & Orth) Part 1 exam. Despite having been published in 2001, the comprehensive SBA questions and accompanying high-quality explanations have helped maintain its popularity. However, it may be time to re-evaluate, with recent candidates suggesting that the questions are too dissimilar (outdated, difficult and esoteric) when compared to the Part 1 exam.  1000 EMQs in Trauma and Orthopaedic Surgery7 This book does not reliably recreate the questions found in the Part 1 exam and is of limited use. Some trainees have found the questions confusing and overly complicated. Perhaps doubly obsolete when considering that the EMI questions are being phased out. 8  FRCS (Tr & Orth): MCQ and Clinical Cases This book includes around 60 SBA questions and similar number of worked viva voce cases taken from the Bone & Joint Journal (BJJ). Although most SBAs rely on factual recall (level 1), the book’s primary merit lies with the good-quality explanations. Online reviews are mixed, with many preferring to use the book when preparing for the Part 2 exam.

Other Sources

 Orthobullets This website is an essential tool for the Part 1 exam. The generous question bank provides detailed explanations that link to the subject areas or chapters on the website. Candidates can revise topics and then construct sets of questions relevant to the area of focus (i.e. paediatric orthopaedics). The website resembles a virtual textbook with topics generally covered in sufficient detail to guide revision. Topics appear to be loosely based on Miller’s Review of Orthopaedics; however, prose is largely replaced by bullet points, with a focus on American practice and the Board exam. 7

8

Sharma H. 1000 EMQs in Trauma and Orthopaedic Surgery. Glasgow: FRCS Orth Exam Education; 2008. Khanduja V. FRCS (Tr & Orth): MCQs and Clinical Cases. London: JP Medical Ltd; 2014.

7

Hussein Noureddine and Matthew Brown

Orthobullet’s primary advantage is the functionality for candidates to create bespoke SBA test papers. Candidates can set the number and focus of the questions tested (i.e. random or subject-specific) and the time allocated per question. The site also maintains a record of performance, with scores presented according to test date and subject area. The primary disadvantage is that the free questions (more than 2,000) rely heavily on factual recall (level 1) and are generally of a standard below that required for the FRCS (Tr & Orth) exam. The paid-for premium content provides a higher standard but they still more closely resemble the style of the American Board exam.  UKITE The United Kingdom In-Training Examination (UKITE) was established by the British Orthopaedic Association (BOA) in 2007 as a curriculum-based self-assessment tool for the FRCS (Tr & Orth) exam. It has evolved to emulate the Part 1 exam more closely. Although the breadth of the T&O curriculum is sampled, the UKITE assessment relies more heavily on questions testing factual recall. Completing the annual UKITE assessment during orthopaedic

8

training is a useful formative assessment tool for monitoring progression and understanding the breadth of the T&O curriculum.  Postgraduate Orthopaedics The newly revamped website includes SBA questions and case-based discussions that complement core revision material to help reinforce difficult key concepts and develop higher order thinking skills.

The Week of the Exam As the Part 1 exam approaches, consider reducing exam day anxiety by travelling to the test centre in advance. Candidates should consider familiarising themselves with the venue, its surroundings and the available services (i.e. transport, parking, refreshments). For Part 2, consider arriving a day or more in advance of the exam for the reasons outlined above. Get into exam mode by minimising distractions and arranging to meet other candidates for face-to-face practice. Finally, good luck when revising for both parts of the FRCS (Tr & Orth) exam. It is a fair exam that represents the pinnacle of T&O surgical assessment and practice worldwide.

Section 1 Chapter

1

General Advice for the FRCS (Tr & Orth) Oral Examination

Candidate Guidance for the Part 1 Written SBA Paper Hussein Noureddine and Matthew Brown

The FRCS (Tr & Orth) exam comprises two parts, and transition to the Part 2 clinical and viva voce exam is dependent upon candidates passing the Part 1 written component.

The Exam Format Part 1 is the written component of the Intercollegiate Examination in Trauma and Orthopaedic Surgery. In 2018 the Joint Committee on Intercollegiate Examinations (JCIE) agreed to phase out extended matching item (EMI) questions. When compared to single best answer (SBA), EMI questions were less able to differentiate candidates and were difficult to construct. Subject to General Medical Council (GMC) approval, EMI questions will not feature in the FRCS (Tr & Orth) examinations from January 2021 onwards. Part 1 exams are currently held at Pearson VUE Test Centres at multiple locations throughout the United Kingdom and Ireland. Candidates can choose their preferred centre during registration. These test centres often host unrelated tests (e.g. driving theory, USMLE) that take place alongside the Part 1 exam. Computer stations are separated by dividers to help minimise visual distraction. Be prepared to focus so as not to be distracted by the movements of others. Some candidates may choose to travel further to utilise quieter test centres. Candidates should bring photographic identification on the day of the exam. This is checked at registration and again before entry into the examination room. Exam conditions are strict. Bags and all but essential items will be stored in the lockers provided at most centres. Unsurprisingly, no mobile devices are permitted in the examination room. Video surveillance of candidates is common. Depending on the location, it is recommended to bring lunch, as some centres do not have local facilities to purchase food. Paper and a pencil are provided for making notes.

The computer-based questions include multimedia images such as radiographs and clinical photographs. Candidates are not permitted to read ahead but will be able to flag difficult or ambiguous questions for later review.

Overview At the time of publication, the Part 1 exam consists of two papers as follows: Paper 1 (2 hours) Single best answer (SBA) – 110 questions Paper 2 (2 hours 30 minutes) Extended matching item (EMI) – 135 questions – 45 themes with option lists – 3 questions using each of these option lists Total 4 hours 30 minutes – 245 questions From January 2021 onwards (subject to GMC approval), the Part 1 exam will consist of two papers as follows: Paper 1 (2 hours 15 minutes) Single best answer (SBA) – 120 questions Paper 2 (2 hours 15 minutes) Single best answer (SBA) – 120 questions Total 4 hours 30 minutes – 240 questions Candidates will have a 2-year period from their first attempt to pass the Part 1 exam, with a maximum of four attempts with no re-entry. Details are available on the JCIE website (www.jcie.org.uk). Candidates with proven dyslexia may be eligible for the Part 1 examination times to be extended and this should be highlighted in advance of the exam.

1

Hussein Noureddine and Matthew Brown

There is no negative marking; therefore, all questions should be attempted. Sample questions can be viewed on the JCIE website. Experienced examiners perform a formal process of standard setting to decide the final pass mark for each paper. The SBA questions are subject to quality assurance procedures, including feedback from both examiners and candidates. Difficulty level, content, discrimination index and internal consistency are analysed. Ambiguous questions or those deemed insufficient to differentiate between candidates are removed through this process. The SBA questions consist of an introductory theme, a question stem and five possible responses (listed A–E), of which one is the most appropriate answer. SBA questions are exactly what the name suggests: candidates choose the best from five possible answers. It is important to note that this is not a ‘single correct answer’ but a ‘single best answer’. Moreover, all five possible answers could be considered correct, but candidates are asked which is best, or most appropriate, given the information provided. As questions are designed to test higher order thinking, this could mean that limited or irrelevant information is provided. Questions require a judgement based on interpretation of the available evidence. Questions that candidates later complain about, for example, ‘there was more than one correct answer’ or that a question was ‘too ambiguous’, can often prove the best performing questions. Although the standard is widely publicised to be set at the level of a day one consultant working in the generality of trauma and orthopaedics, candidates should appreciate that some questions will appear more niche and stretch them more than others. Advantages of SBA questions:  They can assess higher order learning and discriminate between candidates of differing ability.  They can assess a broad sample of the curriculum within a relatively short period of time, which helps to improve reliability and validity.  With all trainees assessed using the same highly standardised questions, they make for a fair assessment.  Automated marking helps to remove examiner subjectivity and reduce costs.

2

 The pitfalls of other question formats (e.g. EMIs, true/false) make SBA questions a popular choice for high-stakes examinations like the FRCS. Disadvantages of SBA questions:  A candidate’s reasoning for selecting a particular answer cannot be assessed.  Despite permitting a broad assessment of the curriculum, there is little opportunity to focus indepth on a particular subject.  They rarely reflect the real-life practices of surgical diagnosis and management, which are varied and nuanced. The five example SBA questions provided on the JCIE website are shown below. 1. A 4-year-old girl has had a swollen, painless left knee joint for 8 weeks. In the last ten days her left ankle has become swollen. It is uncomfortable after she has been sitting for 20 minutes. Examination of her eyes shows an irregular pupil on the right side. What is the most appropriate first step in the management of this patient? A. A painful heel due to enthesopathy is a common associated condition B. Her pupillary abnormality is likely to be due to a dislocated lens C. Synovectomy leads to improved joint function over the short and medium term D. The HLA B27 gene is a strong marker for this condition E. This child is likely to be rheumatoid factor negative 2. A 19-year-old motorcyclist is brought to the Emergency Department following an RTA. He is complaining of difficulty moving his left arm. Clinical examination suggests the latissimus dorsi, subscapularis, pectoralis major and pectoralis minor are functioning, but infraspinatus is not functioning. Which part of the brachial plexus is most likely to have been injured? A. The lateral cord B. The medial cord C. The middle trunk D. The posterior cord E. The upper trunk

Candidate Guidance for the Part 1 Written SBA Paper

3. A 35-year-old man has a combined ACL rupture and posterolateral instability. There is a bony varus knee deformity with lateral thrust in the stance phase of gait. What is the most appropriate treatment for this patient? A. ACL reconstruction alone B. Distal femoral osteotomy C. Reconstruction of the posterolateral corner alone D. Simultaneous reconstruction of the ACL and the posterolateral corner E. Valgus osteotomy and ligament reconstruction simultaneously 4. A 25-year-old man had an acute dislocation of his shoulder, which was reduced. During the followup visit he was found to have wasting of the deltoid and infraspinatus muscles. Injury to which one of the following neural structures accounts for this finding? A. Axillary nerve B. Lateral cord of the brachial plexus C. Posterior cord of the brachial plexus D. Suprascapular nerve E. Upper trunk of the brachial plexus 5. A 20-year-old woman has had low back pain for the past 6 months. She has no fever or constitutional symptoms. Radiographs of her thoracic spine show coarse striations of the vertical trabeculae of the 12th thoracic vertebra. Her ESR is normal. What is the most likely diagnosis? A. Bone cyst B. Fibrous dysplasia C. Haemangioma D. Non-ossifying fibroma E. Osteoblastoma

What is the Relevance of the FRCS (Tr & Orth) Exam? The FRCS (Tr & Orth) exam helps to reassure patients, the GMC and employers that a candidate has reached the necessary standard required for independent practice as a consultant. The reference level is that of a day one consultant working in a district general hospital in the generality of trauma

and orthopaedic surgery. It is important for the public to have confidence in the process. The exam assesses knowledge and judgement, clinical acumen, management and treatment planning, as well as communication skills.

Bloom’s Level 1, 2 and 3 Questions The Part 1 exam is designed to test knowledge from across the Trauma and Orthopaedics (T&O) curriculum by using questions that require higher order thinking. Bloom’s Taxonomy defines six cognitive categories: knowledge, comprehension, application, analysis, synthesis and evaluation. The taxonomy presents a cumulative hierarchy, with categories ordered from simple to complex and concrete to abstract. The five categories that follow knowledge cover skills and abilities, with mastery of simpler levels considered a prerequisite for mastery of the next, more complex level. Rather than assessing factual recall (level 1 questions), the FRCS (Tr & Orth) exam aims to assess each candidate’s ability to apply their knowledge to solve a clinical scenario or problem (level 2 and 3 questions). Higher order questions make up the majority of the question bank, and there is a drive to increase the proportion further. The differences between level 1 and level 2 questions are best illustrated with an example. For a tibial shaft fracture: Level 1: How do you classify (knowledge), how are you going to manage the fracture and what is your operative technique? Level 2: What will be your treatment plan for the featured patient? Higher order thinking is being replaced with the term ‘higher order judgement’. The difference is best illustrated with an example. For a tibial plateau fracture, higher order thinking may test why the fracture is classified as a Schatzker type III rather than type II. With higher order judgement the patient is presented as a 46-year-old man with a tibial plateau fracture that has 4mm of articular depression. This fracture would normally require operative fixation with grafting and elevation of the articular surface. However, the scenario is expanded further to include the patient’s medical history, which includes diabetes, peripheral vascular disease and chronic alcohol abuse. Candidates are expected to make a judgement based on clinical experience. How one manages this patient

3

Hussein Noureddine and Matthew Brown

in reality may differ significantly from what is outlined in the textbooks! The exam is designed to assess real-life decision-making. Multilogical thinking is a new and evolving concept in exam theory that pertains to questions requiring knowledge of more than one fact to logically and systematically apply concepts to solve a problem or clinical scenario. Such questions present multiple viable answers and are highly valued for their capacity to differentiate between candidates.

General Advice The Part 1 exam seeks to test background knowledge and judgement that will have developed during daily clinical work, rather than abstract facts from a book. The exam, and the preceding revision period, will be stressful and exhausting. Candidates should ensure that they are physically and mentally prepared. Eating a well-balanced diet, keeping hydrated, minimising alcohol intake and taking regular breaks during revision are all advised. Engaging with regular physical activity will also help to improve concentration and well-being. It will prove helpful to allocate an evening or afternoon per week to spend with family and friends. Concentrate on sleep patterns in the weeks and months leading up to the exam. Remember that caffeine has a half-life of 4–6 hours, meaning that it will take up to 24 hours for it to be cleared. Additionally, maintain a clear distinction between revision and relaxation by avoiding bedroom-based revision. After completing a bank of questions, it is important to allocate sufficient time to review the answers, which can take far longer than expected. With this in mind, consider supplementing evening revision with early morning revision. Every candidate will have different home circumstances and revision preferences, so it is important to create a personal schedule that suits. Identify areas of knowledge that are lacking and tackle these subjects head-on. Avoid wasting time by reading around answers to questions that were answered correctly and with relative certainty. There is insufficient time for this feel-good approach, given the breadth of the T&O curriculum. It is a valuable revision strategy to review correctly answered questions that were based on a lucky or best guess. Reviewing these questions will help consolidate your knowledge.

4

Many candidates soon realise that practicing questions has to be prioritised over reading or making detailed notes. The following advice may be useful:  Troublesome questions are made more difficult if candidates are underprepared. Understand the breadth of the T&O curriculum and practise the SBA question format to develop the required exam technique.  Confront difficult areas of the curriculum early and avoid burying topics for the few weeks preceding the exam, as this consolidation stage is associated with its own stresses.  One mark can make the difference between a pass and a fail. Practice questions at the correct knowledge level and under timed conditions. Some websites permit a time limit for each question. At the start of your revision, consider allocating 70–80 seconds per question and reduce this to 50–60 seconds as your knowledge and familiarity improve.  Efficient time management during the exam is important. Go fast. Candidates have only one minute to read and interpret each question, consider the options and indicate an answer. Some questions will present cases with a long stem.  Each question carries a 1 in 5 chance of being correct, so be sure to provide an answer for every question. There is no negative marking.  If an answer is not immediately clear, flag the question, mark a best guess and move on promptly with the foresight that returning later may not be possible. Flagging a question electronically will help expedite later review. Marking a best guess will avoid the common error of running out of time and throwing away marks.  Read questions carefully and understand fully what the question stem is asking. All of the options presented may not be ideal, but one must be selected from the options available. Additionally, the answer to questions requesting the ‘most appropriate management’ may differ from those asking the ‘next most appropriate step in management’.  Candidate feedback suggests that around 20% of questions are straightforward. These test standard textbook knowledge, with answers easily narrowed down to perhaps two options (level 1). The remaining questions are less obvious, with

Candidate Guidance for the Part 1 Written SBA Paper

detailed or ambiguous stems and similar answers requiring a more considered judgement (levels 2 and 3).  Do not assume that ambiguous questions will be removed during the final quality assurance process, as very few questions are actually removed.  In recent exams there has been a greater emphasis on higher order anatomy questions. Consider reading an anatomy textbook and practice anatomy questions.

The Build-Up to the Exam Much of the knowledge and judgement required for the FRCS (Tr & Orth) exam will have been acquired during training; however, the T&O syllabus is extensive, and the exam requires lots of preparation. Applications are made through the JCIE website where guidance notes, eligibility criteria and future dates for Parts 1 and 2 (termed Sections 1 and 2) can be reviewed. Candidates should plan their preferred date or ‘diet’ for Part 1 after considering their likely personal and professional circumstances and, for UK trainees with a National Training Number (NTN), the date of their ST6 ARCP. NTN trainees must achieve an Outcome 1 at their ST6 ARCP before they are eligible to apply. Those not in training are required to demonstrate a level of competence and knowledge equivalent to that achieved by NTN trainees at the end of ST6. Be sure to read the JCIE ‘Guidance Notes for Applicants’ document far in advance of making an application. The Part 1 exam is usually timed a few months ahead of the next Part 2 (clinical and viva voce) exam. The preferred exam dates for both parts are requested at the time of initial application; however, the date for Part 2 will only be confirmed after Part 1 is passed. First-time applicants must submit their application with full payment for both Parts 1 and 2 ahead of the published deadlines, which are set approximately 10–12 weeks ahead of each Part 1 exam. The number of candidates permitted to complete Part 2 at each diet is capped and it is widely understood that candidates who apply far in advance of the application deadline are more likely to secure their preferred date for Part 2. The requirement for social distancing in response to the coronavirus pandemic may have implications for those sitting both Parts 1 and 2. Keep an eye on the JCIE website for further guidance.

The following must be satisfied when completing the online application:  Payment in full (covering Parts 1 and 2)  Three completed Structured Reference forms  Curriculum vitae  Summary of Operative Experience  Photographic identification NTN trainees should submit three Structured Reference forms, including one completed by their Training Program Director (TPD) and two by other consultants. Applicants not in training should provide structured references from the head of department (clinical lead) and two other consultants. To avoid unnecessary delays, candidates should contact potential referees far in advance of their planned application date. Plan upcoming clinical rotations to ensure that they will permit the necessary time for revision and cover gaps in clinical knowledge and experience. Such placements should also allow for the clustering of annual and study leave for revision and courses in the weeks leading up to both parts of the exam. It is generally recommended to start revising at least 6 months before the Part 1 exam. Candidates who report preparing for just a few months are either superbly talented, incredibly lucky or inappropriately misleading. Avoid accepting work-related projects, such as research, in the months ahead of your revision. Fee penalties apply to candidates who withdraw from the exam after the closing date, so choose the preferred date carefully after considering all the aforementioned circumstances. It should be noted that candidates who withdraw during an exam will be deemed to have failed and will forfeit one of their four attempts.

Preparation and Revision Resources Approaching the exam as a single assessment consisting of two parts (as opposed to two separate entities) is the preferred revision strategy. The common denominator for Parts 1 and 2 is for candidates to develop knowledge that is of sufficient breadth and depth. Those who begin by revising topics using textbooks and online resources before proceeding to SBA question practice often succeed in passing both parts in successive diets (e.g. February Part 1 followed by April Part 2). In contrast, those who approach Part 1 by concentrating heavily on questions may require a longer interval to convert the SBA ‘best of 5’ skill to that of being able to coherently elaborate and expand

5

Hussein Noureddine and Matthew Brown

on topics during the Part 2 viva voce exam. A combination of both strategies is perhaps the best compromise. Whatever strategy is adopted, be sure to dedicate sufficient time to practice the SBA question format, aiming to complete a few thousand questions as a minimum. Questions will not only reinforce and test knowledge, but also help to gauge the time constraints of the actual exam. Although fundamental to preparing for the Part 2 exam (clinical and viva voce), some candidates find an informal study group helpful during Part 1. Peerto-peer teaching and discussion is a powerful learning tool. Although group learning can take various forms, verbalising knowledge according to a structured revision timetable can prove useful, especially ahead of the Part 2 exam. Revision groups should be limited to a small number of individuals who share similar knowledge levels, plan to sit the exam on the same date and are fully committed to the process. Moreover, it is important to recognise when revision methods are proving ineffectual as evidenced by poor progression or revision group distraction or fatigue. Candidates should be honest with their study partners if this becomes the reality and refocus or redesign their revision strategy at the earliest opportunity. Finally, consistency and continuity are imperative, and candidates should avoid prolonged gaps in their revision. The following revision resources are commonly used by candidates for the Part 1 exam:  Postgraduate Orthopaedics – The Candidate’s Guide (3rd edition) This comprehensive textbook helps to prepare candidates for the clinical and viva voce aspects of the Part 2 exam. It utilises concise prose, graphics, illustrations and case-based examples to consolidate knowledge gained during preparation for Part 1. Cases are designed to reflect those in the exam. Insights from recent candidates help to demonstrate good and bad practice during the viva voce exam. Although this textbook is oriented towards Part 2, the core topic sections will provide a useful revision aid for the questions found in Part 1.  Postgraduate Paediatric Orthopaedics Although oriented towards the Part 2 exam, this textbook is packed with diagnostic and surgical tips that will aid success in both parts. The dysplasias section offers a structured methodology when approaching any skeletal dysplasia, and the

6









1

cerebral palsy section touches on gait analysis with clear graphs of the types that could be asked in both parts of the exam. Miller’s Review of Orthopaedics (7th edition) This comprehensive textbook presents the breadth of T&O surgical practice in one volume, including anatomy and the basic sciences. The book is aligned with the American Board of Orthopaedic Surgery exam but remains very popular for FRCS (Tr & Orth) revision, especially when preparing for Part 1. Recent editions are easier to read and include colour illustrations, clinical photographs and tables. Candidates may choose to focus on specific sections or chapters, read it in its entirety or use it as a reference alongside other resources. Basic Orthopaedic Sciences (2nd edition) This popular textbook for both parts of the exam aims to cover the basic sciences that underpin T&O surgical practice. Topics include biomechanics, biomaterials, immunology, pharmacology, imaging techniques and statistics. Some chapters read more clearly and accurately than others. Orthopaedic Basic Science for the Postgraduate Examination: Practice MCQs and EMQs1 Included here are more than 500 multiple choice and extended matching questions related to orthopaedic basic science. Detailed and insightful explanations are included for each question. The level of knowledge required is perhaps above that expected for the FRCS (Tr & Orth) exam but good to test knowledge on areas of the curriculum that are often neglected in everyday practice. Postgraduate Orthopaedics: MCQs and EMQs for the FRCS (1st edition) The predecessor to this edition was published in 2012. It contains an additional bank of quality SBA and EMQ questions that remain relevant to the FRCS (Tr & Orth) exam. Answer explanations are short but adequate for rapid revision. Candidates may consider using this older textbook

Dawson-Bowling SJ, McNamara IR, Ollivere BJ, et al. Orthopaedic Basic Science for the Postgraduate Examination: Practice MCQs and EMQs. Gloucester: Orthopaedic Research UK Publishing; 2012.

Candidate Guidance for the Part 1 Written SBA Paper











2

3

4

5

6

later in their revision when they are better able to identify information that may be outdated. AAOS Comprehensive Orthopaedic Review (2nd edition)2 This comprehensive and well laid out threevolume text is designed for the American Board exam. The final volume is dedicated to multiple choice question practice. An excellent but expensive resource. Succeeding in the FRCS T&O Part 1 Exam3 This book has received mixed online reviews, with the majority proving highly negative. Questions concentrate on factual recall and explanations can be confusing, unfocused and contradictory. It is perhaps a book to consider borrowing rather than buying and should not form a significant part of one’s Part 1 preparation. Practice Questions in Trauma and Orthopaedics for the FRCS4 The questions in this outdated book more closely reflect the standard of the MRCS exam and are far removed from that of the FRCS (Tr & Orth) Part 1 exam. Poor online reviews reflect the low level of knowledge examined. First Aid for the Orthopaedic Boards (2nd Edition)5 This book is written for the in-service exams (Orthopaedic In-Training Exam [OITE]) of the American Board. It receives mixed reviews, with the question style and depth differing from that observed in the Part 1 exam. Although it is easy to read and may help you score a few extra points, it is expensive for what it provides. Review Questions in Orthopaedics6 Originally written for orthopaedic residents preparing for the in-training (OITE) exams of the Lieberman JR, ed. AAOS Comprehensive Orthopaedic Review. Rosemont, IL: American Academy of Orthopaedic Surgeons; 2009. Gulam Attar F, Ibrahim T. Succeeding in the FRCS T&O. Part 1: Exam. London: BPP Learning Media; 2011. Sharma P. Practice Questions in Trauma and Orthopaedics for the FRCS (Master Pass Series). Milton Keynes: Radcliffe Publishing Ltd; 2007. Mallinzak RA, Albritton MJ, Pickering TR. First Aid for the Orthopaedic Boards. 2nd ed. Bronson, TX: McGrawHill Medical; 2009. Wright JM, Millett PJ, Crockett HC, Craig EV. Review Questions in Orthopaedics. Rosemont, IL: American Academy of Orthopaedic Surgeons; 2001.

American Board, this book (often termed the ‘black book’ by UK candidates) has remained a favourite supplementary question bank for the FRCS (Tr & Orth) Part 1 exam. Despite having been published in 2001, the comprehensive SBA questions and accompanying high-quality explanations have helped maintain its popularity. However, it may be time to re-evaluate, with recent candidates suggesting that the questions are too dissimilar (outdated, difficult and esoteric) when compared to the Part 1 exam.  1000 EMQs in Trauma and Orthopaedic Surgery7 This book does not reliably recreate the questions found in the Part 1 exam and is of limited use. Some trainees have found the questions confusing and overly complicated. Perhaps doubly obsolete when considering that the EMI questions are being phased out. 8  FRCS (Tr & Orth): MCQ and Clinical Cases This book includes around 60 SBA questions and similar number of worked viva voce cases taken from the Bone & Joint Journal (BJJ). Although most SBAs rely on factual recall (level 1), the book’s primary merit lies with the good-quality explanations. Online reviews are mixed, with many preferring to use the book when preparing for the Part 2 exam.

Other Sources

 Orthobullets This website is an essential tool for the Part 1 exam. The generous question bank provides detailed explanations that link to the subject areas or chapters on the website. Candidates can revise topics and then construct sets of questions relevant to the area of focus (i.e. paediatric orthopaedics). The website resembles a virtual textbook with topics generally covered in sufficient detail to guide revision. Topics appear to be loosely based on Miller’s Review of Orthopaedics; however, prose is largely replaced by bullet points, with a focus on American practice and the Board exam. 7

8

Sharma H. 1000 EMQs in Trauma and Orthopaedic Surgery. Glasgow: FRCS Orth Exam Education; 2008. Khanduja V. FRCS (Tr & Orth): MCQs and Clinical Cases. London: JP Medical Ltd; 2014.

7

Hussein Noureddine and Matthew Brown

Orthobullet’s primary advantage is the functionality for candidates to create bespoke SBA test papers. Candidates can set the number and focus of the questions tested (i.e. random or subject-specific) and the time allocated per question. The site also maintains a record of performance, with scores presented according to test date and subject area. The primary disadvantage is that the free questions (more than 2,000) rely heavily on factual recall (level 1) and are generally of a standard below that required for the FRCS (Tr & Orth) exam. The paid-for premium content provides a higher standard but they still more closely resemble the style of the American Board exam.  UKITE The United Kingdom In-Training Examination (UKITE) was established by the British Orthopaedic Association (BOA) in 2007 as a curriculum-based self-assessment tool for the FRCS (Tr & Orth) exam. It has evolved to emulate the Part 1 exam more closely. Although the breadth of the T&O curriculum is sampled, the UKITE assessment relies more heavily on questions testing factual recall. Completing the annual UKITE assessment during orthopaedic

8

training is a useful formative assessment tool for monitoring progression and understanding the breadth of the T&O curriculum.  Postgraduate Orthopaedics The newly revamped website includes SBA questions and case-based discussions that complement core revision material to help reinforce difficult key concepts and develop higher order thinking skills.

The Week of the Exam As the Part 1 exam approaches, consider reducing exam day anxiety by travelling to the test centre in advance. Candidates should consider familiarising themselves with the venue, its surroundings and the available services (i.e. transport, parking, refreshments). For Part 2, consider arriving a day or more in advance of the exam for the reasons outlined above. Get into exam mode by minimising distractions and arranging to meet other candidates for face-to-face practice. Finally, good luck when revising for both parts of the FRCS (Tr & Orth) exam. It is a fair exam that represents the pinnacle of T&O surgical assessment and practice worldwide.

Section 1 Chapter

2

General Advice for the FRCS (Tr & Orth) Oral Examination

SBA Writing Process Paul Banaszkiewicz

Introduction The National Board of Medical Examiners (NBME) item-writing manual is an excellent starting point for a more detailed analysis of the MCQ writing process.1 It is widely referenced in this chapter, being a mainstay of guidance for question writers aiming to produce high-quality questions. The original ‘red book’ was updated to a 4th edition in 2016,2 continuing to be the gold standard guidance book for improving the quality of multiple choice items. Do candidates really need to know the finer details of how to write good-quality SBAs and the processors involved in constructing the section 1 paper? The answer is definitely yes if you experience any major difficulties with this type of summative high-stakes exam. Some candidates do poorly with MCQ type questions, so any guidance is better than nothing. For most candidates, some general information for the written paper is always useful, especially if it neatly summarises information from a variety of different sources that may be difficult or timeconsuming to find otherwise.

Aims By the end of this chapter, candidates should have a greater appreciation of the complexity of constructing SBAs to ensure a fair, valid and reliable section 1 exam. Going through the process of how SBAs are constructed will provide general guidance to a candidate in their overall preparation for section 1.3 1

2

3

Case, S, Swanson D. Item Writing Manual. 3rd ed. Philadelphia, PA: National Board of Medical Examiners; 2002. (available online). Paniagua MA, Swygert KA. Constructing Written Test Questions for the Basic and Clinical Sciences. Philadelphia, PA: National Board of Medical Examiners; 2016. At the least it is an interesting read.

Investing extra time working through this chapter may score a candidate the extra couple of marks that may pull them over the line as a borderline pass.4 This chapter will make clear why there are so many poor-quality orthopaedic MCQ books out on the market. It is very difficult to construct a goodquality new relevant SBA and much easier to bastardise existing questions already out there or spend an evening producing some poor-quality questions without understanding the sophisticated nuances of SBA construction. Constructing good-quality SBAs needs considerable examiner training and question writers need to initially attend workshops for training and advice in their construction before being allowed to start contributing to the question bank Looking ahead, this chapter may prove useful reading if you end up writing MCQ type questions for exams in the future. For aspiring TPDs or future examiners, it is important to know the intricacies of how to write SBAs and the processors involved in constructing the section 1 paper. This will allow you to give more specific and useful advice to candidates who may be repeated failures on this section of the exam In any detailed lecture on section 1 of the FRCS (Tr & Orth) exam reliability, content validity and educational theory (Miller’s pyramid, Bloom’s Taxonomy) are all discussed. Therefore, it is worth going over these terms as if unfamiliar these concepts can be difficult to grasp. Last, those candidates with an educational slant will find the whole process of constructing the section 1 exam fascinating.

4

That in itself is definitely time well spent rather than having to revise and sit section 1 all over again.

9

Paul Banaszkiewicz

Figure 2.1 Miller’s pyramid. The different layers represent the different components of clinical competency and how they can be assessed. WBA attempt to assess how an individual performs in the workplace, i.e. what they actually do.

Educational Theory Miller in 1990 introduced an important framework that can be presented as four tiers/levels of a pyramid to categorise the different levels at which trainees needed to be assessed. Although SBAs can be used to test application of knowledge and higher order thinking, their construction is difficult and in general they assess the bottom two levels of ‘knows’ and ‘knows how’ in Miller’s pyramid (Figure 2.1).5 Knows – Knowledge or information that the candidate has learned Knows how – Application of knowledge to medically relevant situations Shows how – Simulated demonstration of skills in an examination situation Does – Behaviour in real-life situations Workplace-based assessments (WBA) were introduced into the postgraduate curriculum because there were concerns that high-stakes examinations that used tests such as single best answers or EMI encouraged rote learning. It is also known that performance in a controlled assessment correlates poorly with actual performance in professional practice. In 1956, Bloom et al6 described six levels in the cognitive domain: (1) knowledge recall; (2) 5

6

10

Miller GE. The assessment of clinical skills/competence/ performance, Acad Med. 1990 Sep 1;65(9):S63-S67. Adapted by Drs R Mehay & Burns. Bloom BS. Taxonomy of Educational Objectives. Vol. 1: Cognitive Domain. New York: McKay; 1956:20-24 at 3.

comprehension; (3) application; (4) analysis; (5) evaluation; and (6) synthesis. Over the years Bloom’s Taxonomy has been revised and alternative taxonomies created. A substantial revision occurred in 2001 to a more dynamic classification that uses action verbs to describe the cognitive processes and a rearrangement of the sequence within the taxonomy (Figure 2.2; Table 2.1). More recently, the shape of Bloom’s Taxonomy has been represented not as a pyramid – where there is a large base composed of facts and a tiny peak of creativity (which someone might interpret to mean that we should spend the majority of our time focusing purely on knowledge) to a broad wedge that better highlights the value of creating, evaluating and analysing (Figure 2.3). Remembering: the candidate can remember previously learned material from long-term memory by recalling facts, terms, basic concepts and answers, e.g. List the causes of . . . What are the steps in . . . ? Understanding: the candidate can explain ideas or concepts by organising, translating, interpreting, giving descriptions and stating main ideas, e.g. Discuss the causes of . . . Explain the pathophysiology Applying: the candidate can solve problems by applying acquired knowledge, facts, techniques and rules in a different way, e.g. Provide a differential diagnosis Analysing: the candidate can distinguish between the different parts, how they relate to each other and to the overall structure and purpose. This involves examining

SBA Writing Process

Table 2.1 Bloom’s Taxonomy. Key words to use in questions pitched at each level

Remember

Understand

Apply

Analyze

Evaluate

Create

Who What When Define Identify Describe Label List Name State Match Recognise Select Examine Locate Memorise Quote Recall Retrieve Reproduce Tabulate Copy

Demonstrate Explain Describe Interpret Clarify Classify Categorise Differentiate Discuss Distinguish Infer Predict Identify Report Select Outline Review Express Translate

Solve Illustrate Calculate Execute Carry out Discover Show Examine Choose Schedule Implement Use Make use of Employ Organise

Differentiate Distinguish Analyse Compare Classify Contrast Separate Explain Select Categorise Divide Order Prioritise Divide Inspect Make assumptions Draw conclusions

Check Co-ordinate Reframe Defend Rate Appraise Critique Judge Support Decide Recommend Summarise Assess Choose Defend Estimate Grade Find errors Compare Rate Measure Provide opinion

Design Compose Create Plan Design Formulate Produce Construct Organise Generate Hypothesise Develop Assemble Rearrange Modify Improve Adapt Elaborate

Figure 2.2 Bloom’s Taxonomy

and breaking information into parts by identifying motives or causes, making comparisons and finding evidence to support generalisations, e.g. How will your differential diagnosis be altered in the light of investigation findings?

Evaluating: the candidate makes judgements and justifies decisions about information, presenting and defining opinions by making judgements about information, validity of ideas or quality of work based on a set of criteria e.g.

11

Paul Banaszkiewicz

Figure 2.3 Modification of pyramid shape of Bloom’s Taxonomy into broad wedge to better emphasise the value of creating, evaluating and analysing.

Justify your management of this patient.

Higher Order Thinking

Creating: the candidate puts elements together to form a functional whole, create a new product or point of view, e.g. What will be your plan of management?

This is integration/interpretation (questions which require ‘putting the pieces together’) and problem solving (questions which require ‘clinical judgement’), not simple recall (questions which can be answered with a Google search).

Bloom’s Taxonomy is a hierarchical classification, with the lowest cognitive level being ‘remembering’ and the highest being ‘creating’. The lower three levels can be attained with superficial learning so-called Lower Order Thinking Skills (LOTS) such as memorisation. The upper three levels involve Higher Order Thinking Skills (HOTS) and can only be attained by deep learning. An ongoing development of the examination is the progressive rewriting of questions in the bank that are currently recorded as level 1 questions (factual knowledge) into higher order questions. In constructing multiple choice items to test higher order thinking, it is helpful to design problems that require multilogical thinking, along with designing alternatives that require a high level of discrimination.

12

Multilogical Thinking Multilogical thinking is defined as ‘thinking that requires knowledge of more than one fact to logically and systematically apply concepts to a problem’.7 There has been a conscious move to rewrite the question bank with SBAs that require multilogical thinking to answer.

Highly Discriminating Questions These are questions that provide viable alternatives so that they require a high degree of discrimination to answer. 7

Morrison S, Free K. Writing multiple-choice test items that promote and measure critical thinking, J Nurs Educ. 2001 Jan;40(1):17-24.

SBA Writing Process

SBAs Advantages of SBAs  SBAs can assess a wide sample of curriculum content within a relatively short period of time. This leads to high reliability and improved validity.  They are a highly standardised form of assessment where all the trainees are assessed with the same questions. It is a fair assessment in that all the trainees sit the same exam.  They are easy to administer and mark.  SBA marking is mostly automated and hence examiner subjectivity is removed from the assessment process.

Main Disadvantages of SBAs  The trainee’s reasons for selecting a particular option/response cannot be assessed.  Although a wide sample of assessment material can be assessed, the assessment does not provide an opportunity for an in-depth assessment of the content.  Constructing good SBAs needs considerable examiner training. Exam boards use a utility model to analyse different assessment tools: Utility = (R) 3 (V) 3 (A) 3 (E) 3 (C) 3 (P) R – Reliability. Can the exam results of a given candidate in a given context be reproduced? To what extent can we trust the results? V – Validity. Does the assessment assess what it purports to assess? A – Acceptability. How comfortable are the different stakeholders (candidates, examiners, examination boards, public, National Health Service) with the examination system? E – Educational impact. Does the exam drive the trainees towards educationally and professionally valuable training? C – Cost effectiveness. Is the expenditure– in terms of money, time and manpower– to develop, run and sustain the examination process worthwhile in relation to what is learned about the candidate? P – Practicability. How ‘doable or workable’ is the assessment instrument, given the circumstances? Are there sufficient resources to mount the exam?

Applying the utility model for SBAs we get Reliability: high The SBA results are highly reliable, as almost identical scores can be obtained if a similar candidate with similar ability is given the same set of SBAs, regardless of who marks the questions. Validity: high for knowledge recall An SBA is good at testing factual recall of knowledge. They can also be used to test application of knowledge and higher order thinking, although the construction of such SBAs is difficult and requires training. Acceptability: high SBAs have been used extensively in medical education. Both trainees and examiners have come to accept them. Constructing good SBAs, however, is difficult. Educational impact: (moderately) Properly constructed SBAs will drive the learner towards learning important information. However, SBAs developed to test trivial knowledge will lead to rote learning. Fragmentation of knowledge is another criticism. Cost: moderate The cost of administering an SBA test is low. In contrast, face-to-face peer review meetings of submitted SBAs are expensive to hold, as they involve substantial travel and accommodation costs. However, the quality of scrutiny that can be brought to bear on the question material justifies this outlay and affords considerable confidence in the quality of the product. Practicability: high SBAs are easy to administer as a computer-based assessment.

Item Analysis of SBAs Item analysis output indicates the percentage of candidates in the various subgroups who selected each option of an SBA. Each SBA is analysed as to the percentage of candidates scoring it correctly from each subgroup. The test group is usually divided into fifths, as this allows more detailed analysis around the pass/fail than if quartiles were used. The spread should be like a Gaussian curve. The exam board members are not very interested in

13

Paul Banaszkiewicz

Figure 2.4 Easy SBA

distinguishing the very best or worst candidates. The curve is concentrated in the centre and the exam board members want to spread this middle area out so that one question cannot decide if a candidate passes or fails the exam.

Similar to easy SBAs, difficult SBAs are discarded if they are also of poor quality and require very extensive rewriting.

Easy Questions

These questions (Figure 2.6(a) and (b)) may involve the bottom 20% of candidates getting an SBA mainly correct while the top 20% of candidates scoring mainly incorrectly. It’s a poor SBA, as overall it is not following candidate form. Another example is where there has been a random spread of correct answers between groups. Usually the question is poorly written, the wrong answer has been selected by the examiners or there has been an error of typing. Poorly performing questions are removed. Questions that have more than 90% or 10% failed/ pass are also removed. All questions that score poorly, i.e. where the percentage of correct responses to that alternative is below 30%, are checked. Questions where the top 20% of candidates score significantly lower than average are also reviewed.

With these questions (Figure 2.4), around 90% of candidates get the correct answer. As such, easy questions do not discriminate between the very good or very bad performing candidate. More important, an easy question does not differentiate between candidates around the level of minimal competence required for a pass. When paper 1 analysis flags up these questions, they are either scrapped or have to be extensively reworked.

Difficult Questions These questions (Figure 2.5) are just as useless as an easy question. Again, they do not differentiate between a good or bad candidate or, more important, make a distinction between borderline candidates – those who can be passed and those who must re-sit.

14

Poorly Performing Questions

SBA Writing Process

Figure 2.5 Difficult SBA

Good Performing Questions There is a graduation in candidates obtaining the correct answer from the top one-fifth mainly scoring the question correctly to the bottom onefifth with candidates mainly scoring it incorrectly (Figure 2.7). This question discriminates. There is point by point discrimination; if it is >.3, it is a good question

Ideal SBA These questions (Figure 2.8) discriminate candidates at the pass/fail mark. A good quality question should be answered correctly by 35–85% of just passing candidates (defined as those scoring an overall mark within 10% of the pass mark). There should also be an obvious positive correlation between the performance of the cohort on the individual question and in the examination as a whole (i.e. the question should be answered correctly by appreciably more passing candidates than failing candidates). A reasonable proportion of candidates (especially those who did not pass) should also have chosen each incorrect option.

Item analysis determines difficulty index (DIF I) (p-value), discrimination index (DI) and distractor efficiency (DE)

Difficulty Index (DIF I) DIF I measures the difficulty of an individual item on the test paper. It is calculated by adding the correctly answered item by the upper 33% and lower 33% of candidate’s performance divided by total number of candidates in both the groups: DIF I = [(H+L)/T] 3 100, where H and L are number of correct responses in the high and low achiever groups, and T is the total number of responses in both the groups. Thus, the higher the score is, the easier the item.

Discrimination Index (DI) DI is a measure of how well an individual item can differentiate between the high and low performing students (top and bottom 33%): DI = [(H L)/T] 3 2

15

Paul Banaszkiewicz

Figure 2.6 (a) and (b) Poorly performing SBAs

16

SBA Writing Process

Figure 2.7 Good SBA

Figure 2.8 Ideal SBA

17

Paul Banaszkiewicz

DI > 0.35 is acceptable and the higher the DI, the more differentiating is the item.

It involves a cut-off mark based on the performance of candidates in relation to a defined standard (absolute) as opposed to how they perform in relation to their peers (relative). It involves a judgement being made on exam items (test-centred) as opposed to exam candidates (examinee-centred) and is widely used to standard set high stakes examinations. It is most reliable when supported by another standard setting method.

Distractor Efficiency (DE) Plausible distractors are defined as the distractors selected by >5% of candidates. Non-functional distractors (NFD) are the options selected by 0.75. What is the most appropriate management? A. Two-hole DHS B. Cemented dual mobility THA C. Cemented THA D. Uncemented dual mobility THA E. Uncemented THA 16. After thorough discussion, it is decided that the best option for a 20-year-old manual labourer with post-traumatic hip OA is a fusion. What is the most appropriate position of fusion? A. 0° external rotation, 0° adduction, 5° flexion B. 5° external rotation, 5° adduction, 25° flexion C. 10° external rotation, 15° abduction, 25° flexion D. 15° external rotation, 0° abduction, 5° flexion E. 15° external rotation, 15° abduction, 5° flexion 17. A patient underwent a THR for a NOF fracture while abroad on holiday. The patient was given strict rehabilitation instructions to avoid flexion beyond 90° and extreme internal rotation. What structure was most likely to have been injured during the patient’s surgery? A. Pudendal nerve B. Femoral nerve C. Superior gluteal nerve D. Lateral cutaneous nerve E. Sciatic nerve 18. A 77-year-old man presents with an insidious onset of hip and anterior thigh pain 15 years after a total hip arthroplasty. He denies fever or systemic upset,

and initial bloods show normal inflammatory markers. Anteroposterior (AP) radiograph left hip are shown in the radiograph (Figure 3.1). Figure 3.1 Anteroposterior (AP) radiograph left hip

You advise the patient that he should undergo revision surgery as he is at risk of what complication? A. Infection B. Dislocation C. Periprosthetic acetabular fracture D. Periprosthetic femur fracture E. Pseudotumour formation 19. You review a patient following hip arthroscopy and extensive debridement of a labral cyst. He has weakness of ankle dorsiflexion. Injury has most likely occurred as a result of what? A. Excessive traction at groin post B. Placement of anterior portal C. Placement of anterolateral portal D. Placement of distal anterolateral portal E. Placement of posterolateral portal 20. During intraoperative testing for stability of a THR you have implanted through a Modified Hardinge approach you see the hip is unstable in extension and external rotation. What change will increase the primary arc of hip motion? A. 20° versus 0° liner B. 36mm versus 32mm head C. High versus standard offset stem. D. Lateralised versus standard liner E. Skirted head

29

Edward Holloway

21. A 70-year-old woman presents with a red, swollen and painful area around the incision site of a THR performed 8 weeks earlier. She has a CRP of 78, WCC of 16 and temperature of 37.4° C. Her other observations are unremarkable. She did not attend 6-week follow-up and says the wound has not stopped leaking since she left hospital. What is the most appropriate management? A. Aspiration and culture-specific long-term antibiotic suppression B. Debridement, antibiotics, implant retention with exchange of modular components C. Debridement, antibiotics, implant retention with retention of modular components D. Empiric antibiotics E. Revision arthroplasty 22. A 53-year-old man with severe ulcerative colitis is referred for a THA. His pelvic x-ray shows Brooker grade 3 changes on the ipsilateral side replaced 4 years earlier. Which of the following prophylactic treatments is most appropriate? A. Diphosphonate 20mg/kg for 21 days B. Ibuprofen 400mg TDS for 4 weeks C. Indomethacin 25mg TDS for 6 weeks D. Single 2 500cGy radiotherapy dose E. Three 1 500cGy radiotherapy doses

A. B. C. D. E.

Alpha angle Lateral centre edge angle Sharp’s angle Tonnis angle Vertical centre edge angle

24. A young patient with cerebral palsy is referred with hip pain. You want to examine for contracture in a muscle with an origin at the inferior pubic symphysis and inferior pubic rami, which attaches to a point just posterior to the attachment of sartorius. What is the most appropriate test? A. Ely’s test B. Ober’s test C. Phelp’s test D. Thomas’ test E. Trendelenburg test 25. A 24-year-old woman falls while mountain biking and sustains the injury shown in this radiograph (Figure 3.3). In theatre, you struggle to reduce the fracture with in-line traction and rotation. Figure 3.3 Anteroposterior (AP) radiograph left hip

23. A 23-year-old woman has been referred by her physiotherapist with symptoms of hip impingement. A radiograph of her left hip is shown here (Figure 3.2). What is marked on her radiograph? Figure 3.2 Anteroposterior (AP) radiograph left hip

What is the most appropriate next step? A. Total hip arthroplasty B. Fixation in best position achieved C. Flexion to 90°, adduction, in-line traction, then internal rotation D. Extension, abduction, in-line traction, then external rotation E. Open reduction using a modified SmithPetersen approach

30

Hip I Structured SBA

26. A 30-year-old woman is referred for consideration of THA. Her radiographs have been classified as a Type B using the Hartofilakidis classification. What anatomical characteristic of the femur is most commonly associated with this? A. Excessive anteversion B. Excessive femoral bow C. Excessive retroversion D. Excessive valgus E. Excessive varus 27. The diagram shown here represents the scratch profile of a material used in THA femoral heads (Figure 3.4). Figure 3.4 Scratch profile

If such a component fractures, what bearing couple should be used during revision surgery? A. Ceramic on ceramic B. Ceramic on poly C. Metal on metal D. Metal on poly E. Oxinium on poly 28. A 45-year-old presents with symptoms of hip impingement and radiographs show the femoral head to be medial to Kohler’s line. What is the most likely diagnosis?

A. B. C. D. E.

Otto pelvis Coxa magna Coxa vara Coxa valga Acetabular retroversion

29. A 60kg woman develops progressive pain and numbness in the lateral calf with weakness of ankle dorsiflexion 48 hours after THA. Which of the following is the most appropriate action? A. Book for emergency surgery B. Nurse prone and review the following morning C. Nurse with knee in flexion and review the following morning D. Urgent MRI scan E. Withhold prophylactic LMWH 30. A surgeon admits a mistake was made during a THA. The patient complains that their operated leg feels short. What is most likely to have resulted in this? A. +8mm versus 0mm femoral head B. 125° versus 135° neck–shaft angle implant C. High versus standard offset stem D. Incomplete insertion of an uncemented stem E. Size 4 rather than size 3 implant inserted

31

Edward Holloway

HIP I STRUCTURED SBA ANSWERS 1. Answer E. Zone 7 Iliopsoas originates from the anterior and inferior aspects of the transverse processes of L1–L5 and the bodies and discs of T12–L5. It inserts into the lesser trochanters and is innervated by direct fibres from the lumbar plexus (L1/L2/L3). It acts as a flexor of the hip. Gruen zones are used to describe areas of loosening around a femoral stem. Zone 1 relates to the greater trochanter, zone 7 the lesser trochanter and zones 6 and 5, and zones 2 and 3 on the medial and lateral aspects of the femur, respectively. Zone 4 is at tip of the stem (Figure 3.5). Figure 3.5 Gruen zones

2. Answer A. Harris Hip Score All of the outcome measures apart from the Harris Hip Score are completed by the patient. The Harris Hip Score gives a score of 0–100, with 100 being the best possible outcome. The doctor-completed score consists of the four subscales of pain severity (1 item, 0–44), function (7 items, 0–47 points), absence of deformity (1 item, 0–4 points) and range of motion (2 items, 0–5 points).

32

3. Answer C. Ischiofemoral Impingement test Ischiofemoral impingement occurs due to an abnormal contact between the ischium and the lesser trochanter, most commonly following total hip arthroplasty or proximal femoral intertrochanteric osteotomy. This type of impingement can still develop in non-operative patients such as those with severe osteoarthrosis causing protrusio acetabuli or fracture malunion of the proximal femur. The impingement happens due to narrowed distance between the ischial tuberosity and the lesser trochanter, leading to compression of the quadratus femoris muscle, which becomes oedematous in the early stages but subsequently undergoes fatty replacement. Patients present with hip, groin and gluteal pain which may radiate to the lower leg due to sciatic nerve irritation. In certain cases, bursal inflammatory fluid formation may happen around the iliopsoas tendon at the lesser trochanter. The diagnosis of ischiofemoral impingement can be challenging, as clinical tests may be non-specific, and therefore imaging is crucial. The treatment of this condition varies from conservative measures such as CT-guided cortisone injection to surgical excision of the quadratus femoris. 4. Answer C. Image-guided injection of iliopsoas tendon sheath This case describes iliopsoas impingement syndrome. This is an underdiagnosed cause of groin pain following THR. The pain may be accompanied by limp, tenderness to palpation in the groin and, more rarely, a snapping sensation. Simple provocation tests are straight leg raising or resisted flexion in the seated position. Risk factors for iliopsoas impingement syndrome are acetabular component malposition (more likely in smaller patients), leg length discrepancy, excessively long acetabular screws and retained cement. An image-guided injection of the tendon sheath has diagnostic and therapeutic benefits. Definite options for management are revision of a malpositioned acetabular component, correction of leg length deformity and resection or

Hip I Structured SBA

tenotomy of the tendon depending upon the underlying cause. A radioisotope bone scan is unlikely to be useful 6 months post-surgery and the symptoms are not in keeping with loosening or infection. A revision may be indicated in the future but not as an initial management. Henderson RA, Lachiewicz PF. Groin pain after replacement of the hip: aetiology, evaluation and treatment. J Bone Joint Surg Br. 2012 Feb;94(2):145-151. 5. Answer E. Protected weight bearing Idiopathic transient osteoporosis of the hip (ITOH) is a atraumatic, progressive cause of hip and groin pain presenting more commonly in women (3:1 female:male) and in two distinct groups: (1) women in the third trimester of pregnancy and (2) middle aged men. ITOH is a diagnosis of exclusion with characteristic appearances on plain films and MRI. Radiographic features lag behind clinical symptoms by 4–6 weeks and include subchondral cortical loss, diffuse osteopaenia and joint effusion, but always a preserved joint space. MRI findings include T1: decreased signal and loss of fatty marrow, and T2: effusion and high signal and marrow oedema. Symptoms usually resolve spontaneously in 6–8 months, but weight bearing should be protected to reduce the risk of stress fractures. 6. Answer C. Large male under 65 years An important indication of hip resurfacing is patients with pre-existing metalwork in the medullary canal of the femur which precludes a metaphyseal stem. Advantages of a hip resurfacing over a THA include increased stability due to a larger head size, preservation of femoral head and neck bone stock, simpler femoral revision, lower mortality and the potential of a more normal gait pattern and participation in highdemand activities. After high-profile failures including certain implants the most suitable patient group to benefit from resurfacing is being re-defined, but it is generally considered to be most suitable for younger, larger male patients. Logishetty K, Muirhead-Allwood SK, Cobb JP. Hip resurfacing – what is its role in

modern orthopaedics? Bone & Joint 360. 2020 Feb;9(1):4-9. 7. Answer E. Stand on one leg The superior gluteal nerve is at risk during deep dissection in the direct lateral approach to the hip. The fibres of gluteus medius are split from the middle of the greater trochanter proximally. If the split is continued more than 3–5cm proximal to the trochanter there is risk of nerve damage. The superior gluteal nerve originates from the lumbosacral plexus with contribution from nerve roots L4–S1 and innervates gluteus medius, gluteus minimus, and tensor fascia lata. Weakness will manifest clinically during Trendelenburg test or hip abduction. 8. Answer E. First year, once at 7 years and three yearly thereafter ODEP 10A or 10A* rated hip resurfacing devices are the MatOrtho Adept Resurfacing Head (48–58mm), and Smith & Nephew Birmingham Hip Resurfacing Head (48–62mm). It is recommended that they be followed up during the first year, once at 7 years and three yearly thereafter. Patients at risk of adverse reaction to metal debris (ARMD) (female patients, males with femoral components smaller than 48mm and those with a DePuy ASR implant) should be reviewed annually for as long as the device is implanted. Those not at risk but symptomatic should also be seen annually. Other implants in asymptomatic patients not at risk should be seen annually for the first 5 years, two yearly to 10 years, and then three yearly thereafter. Logishetty K, Muirhead-Allwood SK, Cobb JP. Hip resurfacing – what is its role in modern orthopaedics? Bone & Joint 360. 2020 Feb;9(1):4-9. 9. Answer C. Avascular necrosis Garden classified femoral neck fractures into four types depending on the degree of displacement seen on an anteroposterior radiograph. Type I are incomplete fractures, type II are complete but non-displaced fractures, Type III are complete and partially displaced fractures and Type IV are complete and fully displaced. Young patients with Garden II neck of femur fractures would most commonly be managed

33

Edward Holloway

with closed reduction and fixation with either cannulated hip screws or an alternate device, unless the patient had significant comorbidities making an arthroplasty with a lower risk of reoperation a better option. Leg length discrepancy, sciatic nerve palsy injury and Trendelenburg gait would all be complications of THR. Parker MJ, Gurusamy KS. Internal fixation versus arthroplasty for intracapsular proximal femoral fractures in adults. Cochrane Database Syst Rev. 2006;(4):CD001708. doi:10.1002/ 14651858.CD001708. 10. Answer B. Sartorius The direct anterior approach (DAA) to the hip is performed with the aim of reducing muscle damage, length of stay, pain and complication rate. A fracture table with specific attachments is used by some to assist in femoral exposure. There is a risk of damage to the lateral cutaneous nerve of the thigh. This nerve arises from the lumbar plexus, or more rarely the femoral nerve itself, and travels through the pelvis on the iliacus muscle. It enters the thigh under the inguinal ligament at a point anywhere between the anterior superior iliac spine (ASIS) and the midinguinal point. The nerve then pierces the fascia lata medial and inferior to the ASIS. From here, the nerve takes a variable course but most commonly the medial border of sartorius. Meermans G, Konan S, Das R, Volpin A, Haddad FS. The direct anterior approach in total hip arthroplasty: a systematic review of the literature. Bone Joint J. 2017 Jun;99-B(6):732-740. 11. Answer A. External iliac vessels When placing screws to secure an uncemented acetabular component, quadrants are defined by drawing a line from the ASIS to the centre of the acetabulum and then a second line perpendicular to this. The posterosuperior quadrant is the safest for screw placement. The posteroinferior quadrant should be used with caution, as screws longer than approximately 20mm may put the sciatic nerve, internal pudendal nerve and vessels, and the inferior gluteal nerve and vessels at risk. The anteroinferior quadrant (zone of danger) puts the obturator nerve and vessels at risk. The

34

anteroinferior quadrant (zone of death) puts the external iliac vessels at risk. 12. Answer B. Cementless stem Intraoperative Vancouver A2 fractures are nondisplaced fractures of the proximal metaphysis. All answers are associated with an increased relative risk of calcar fracture, but cementless stems give the greatest relative risk (RR) (RR = 3.8). Age 11 – 49 RR = 1.5, female sex RR = 1.9, paediatric disease RR = 2.6, previous trauma RR = 3.6. Lamb JN, Matharu GS, Redmond A, Judge A, West RM, Pandit HG. Risk factors for intraoperative periprosthetic femoral fractures during primary total hip arthroplasty. An analysis from the National Joint Registry for England and Wales and the Isle of Man. J Arthroplasty. 2019;34(12):3065-3073.e1 13. Answer D. Total hip replacement Avascular necrosis of the hip is predominantly idiopathic but may be associated with alcohol abuse, steroid use, hypercoagulability, Caisson’s disease and sickle cell disease. Treatment options depend upon the clinical and radiographic stage (Ficat, Steinberg) of the disease and age/comorbidities of the patient. The relatively conservative options listed would be options in younger patients or those with earlier stages of disease. Once any significant amount of collapse has occurred in a patient over 40, the most likely option is an arthroplasty procedure. Petek D, Hannouche D, Suva D. Osteonecrosis of the femoral head: pathophysiology and current concepts of treatment. EFORT Open Rev. 2019;4(3):85-97. 14. Answer D. SPT (spinopelvic tilt) Increasingly, the relationship between the spine, hip and knee is assessed and considered in how it may affect the functionality and stability of a THR as the patient moves from lying to standing and from sitting to standing. All options can be measured radiographically to this end. APPt (anterior pelvic plane tilt) refers to the rotation of the pelvis in the sagittal plane as measured by the angle formed between the coronal plane and a line from the anterior superior iliac spine (ASIS) to pubic symphysis.

Hip I Structured SBA

The APP (anterior or anatomical pelvic plane) is defined by the pubic symphysis and the two anterior superior iliac spines. The FPP (functional pelvic plane) rotates this plane according to an individual’s pelvic tilt. It is considered a more accurate reference plane for assessing acetabular component position. PI (pelvic incidence) is the angle between two lines: one from the centre of the femoral head to the centre of the S1 end plate, and a second perpendicular to a line across the S1 end plate, intersecting the centre of the end plate. SPT (spinopelvic tilt) is the angle between a line from the centre of the S1 end plate and the centre of the segment between the two femoral heads (the bicoxofemoral axis), and the vertical. SS (sacral slope) is the angle between two lines: one parallel to the S1 end plate, and a second along a horizontal reference plane. Ike H, Dorr LD, Trasolini N, Stefl M, McKnight B, Heckmann N. Spine-pelvis-hip relationship in the functioning of a total hip replacement. J Bone Joint Surg Am. 2018 Sep 19;100(18):1606-1615. 15. Answer B. Cemented dual mobility THA The Dorr classification depends upon the ratio between the inner canal diameter at the level of the midpoint of the lesser trochanter and a point 10cm below that. Dorr C femurs as defined as a ratio >0.75 are most suitable for a cemented prosthesis. A patient with significant risk of dislocation and a grossly displaced NOF fracture would be a good candidate for a dual-mobility acetabular component. 16. Answer B. 5° external rotation, 5° adduction, 25° flexion This is the most appropriate position of hip fusion. 17. Answer E. Sciatic nerve The position of greatest risk of dislocation for a posterior approach THA is flexion and internal rotation. The structure most at risk during this approach is the sciatic nerve. 18. Answer D. Periprosthetic femur fracture The radiograph shows a loose Charnely femoral stem which is close to fracturing through the

posterior cortex of the femur (Figure 3.6). The THA may be infected but the fact that the acetabulum does not look loose so goes against this. Pseudotumour formation is associated with

Figure 3.6 Lateral radiograph of left hip

metal-on-metal bearing couples. 19. Answer E. Placement of posterolateral portal By far the most common direct nerve injury resulting from hip arthroscopy portal placement is of the lateral femoral cutaneous nerve when placing or working through the anterior portal. Sciatic nerve injury causing a foot drop is a rare complication of posterolateral portal placement, made more likely by placing the leg in external rotation. Papavasiliou AV, Bardakos NV. Complications of arthroscopic surgery of the hip. Bone Joint Res. 2012;1(7):131-144. 20. Answer B. 36mm versus 32mm head The primary arc of hip motion is the range that the hip can move before the neck impinges upon the acetabulum and is an important determinant of THA stability. A lipped or angled liner may improve stability by increasing coverage but actually decreases the primary arc of motion and, in this instance, may make anterior dislocation more likely by causing posterior impingement levering the hip out of joint. A high offset stem or lateralised liner will not alter the primary arc of movement but may improve stability by moving the femoral neck away from impinging soft tissues or osteophytes (potentially to the detriment of proper restoration of hip biomechanics). A skirted head (as found on the longer lengths) decreases the primary arc. 21. Answer E. Revision arthroplasty All the options are valid in managing the infected joint replacement. Patients not sufficiently fit to do well with further significant or potentially multiple procedures can be managed

35

Edward Holloway

with culture-specific, long-term suppressive antibiotics or an excision arthroplasty, if the infection is not able to be suppressed or the patient is unable to take long-term antibiotics. The success rates of DAIR (debridement, antibiotics, and implant retention) procedures vary enormously in the literature, from 11–100%. The success rate is correlated with type of organism (Streptococcus species tend to do better), exchange of modular components and time from procedure in early infection, or time from first presentation of symptoms in late infection. An older patient with long duration of symptoms is less likely to have successful infection eradication with a DAIR so may be best served with a one- or two-stage revision procedure depending upon the infecting organism and the surgeon’s preference. Grammatopoulos G, Kendrick B, McNally M, et al. Outcome following debridement, antibiotics, and implant retention in hip periprosthetic joint infection – an 18-year experience. J Arthroplasty. 2017 Jul;32(7):2248-2255. Kunutsor SK, Beswick AD, Whitehouse MR, Wylde V, Blom AW. Debridement, antibiotics and implant retention for periprosthetic joint infections: A systematic review and metaanalysis of treatment outcomes. J Infect. 2018 Dec;77(6):479-488. 22. Answer E. Three 1 500cGy radiotherapy doses Heterotopic ossification (HO) following THR occurs with an incidence of 5–90% and is associated with the risk factors of male sex, history of previous HO, older age, previous hip fusion, hypertrophic OA, ankylosing spondylitis, posttraumatic OA, Paget disease, osteonecrosis and rheumatoid arthritis. It is classified according to the Brooker classification according to the extent of heterotopic bone formation on an AP radiograph. Prophylaxis can be considered for patients at high risk of HO and the main forms are radiotherapy and non-steroidal anti-inflammatory drugs (NSAIDs). Diphosphonates have not been shown to be effective. NSAIDs would be contraindicated in a patient with severe inflammatory bowel disease. Multiple doses of radiation

36

therapy have been shown to be more effective than single doses. Board TN, Karva A, Board RE, Gambhir AK, Porter ML. The prophylaxis and treatment of heterotopic ossification following lower limb arthroplasty. J Bone Joint Surg Br. 2007 Apr;89 (4):434-440. 23. Answer D. Tonnis angle The following are measured during the radiographic assessment of patients with symptoms of femoroacetabular impingement: The Tonnis angle (also called the acetabular index or acetabular roof angle) is the angle between the horizontal and a tangential line from the medial to the lateral sourcil (weight bearing zone of the acetabulum). Alpha angle is measured on a Dunn view by drawing a line from the centre of the narrowest point of the femoral neck to the centre of the femoral head using a best-fit circle. The angle is between the line down the axis of the femoral neck and a line drawn to the location where the femoral head becomes ‘out of round’. The lateral centre edge angle (of Wiberg) is the angle between a line drawn from the centre of the femoral head to the lateral edge of the acetabulum and a second line that is parallel to the longitudinal pelvic axis. Sharp’s angle, or acetabular angle, is the angle between a horizontal line drawn through the teardrops and a line drawn from the teardrop to the lateral acetabular roof. The vertical centre edge angle is measured from a false profile radiograph and is between a vertical line through the centre of the femoral head and a line from the centre of the femoral head to the anterior most point on the acetabulum. Mannava S, Geeslin AG, Frangiamore SJ, et al. Comprehensive clinical evaluation of femoroacetabular impingement: part 2, plain radiography. Arthrosc Tech. 2017;6(5):e2003-e2009. 24. Answer C. Phelp’s test Phelp’s test assesses gracilis tightness. Ely’s test assesses rectus femoris tightness. Ober’s test assesses tightness of the tensor fascia lata. Thomas’ test assesses fixed flexion of the hip. Trendelenburg test assesses hip abductor dysfunction.

Hip I Structured SBA

25. Answer C. Flexion to 90°, adduction, in-line traction, then internal rotation Leadbetter described a technique in 1939 to aid in the reduction of intracapsular neck of femur fractures. The limb is flexed to relax the musculature around the hip and then internal rotation functions to relax the ligamentous structures. From the above position, further flexion and adduction open the fracture, allowing reduction to be achieved by maintaining traction while bringing the limb into extension with slight abduction and continued internal rotation.

28. Answer A. Otto pelvis Otto pelvis (arthrokatadysis or protrusio acetabuli) is a rare condition associated with hip osteoarthritis. Protrusio acetabuli is defined radiographically when the medial aspect of the femoral head projects beyond the ilioischial (Kohler’s) line. Though sometimes idiopathic, acetabular protrusion is more usually associated with inflammatory arthropathies, osteoporosis, osteomalacia and Paget disease. When performing THR, care must be taken to lateralise the medialised hip centre.

26. Answer A. Excessive anteversion The Hartofilakidis classification recognises three types of congenital hip disease in adults: type A – dysplasia, type B – low dislocation, and type C – high dislocation. In type A disease, the femoral head remains within the original acetabulum. In type B, it articulates with a false acetabulum which partially covers the original acetabulum. In type C, the femoral head has migrated superiorly and posteriorly to the hypoplastic original acetabulum. Adult hip dysplasia is most commonly associated with excessive femoral anteversion.

29. Answer A. Book for emergency surgery Nerve dysfunction following THA may be a result of intraoperative compression from retractors, traction injury secondary to manipulation or lengthening. If noted in the immediate postoperative period, tension can be taken off the sciatic nerve by nursing the patient with the knee in flexion. Progressive, painful sciatic nerve dysfunction should raise the suspicion of compression from haematoma. This is associated with patients of lower mass, and with anticoagulant use. Improved outcomes are associated with prompt exploration and evacuation of haematoma. Su EP. Post-operative neuropathy after total hip arthroplasty. Bone Joint J. 2017 Jan;99-B(1 Suppl):46-49. Butt AJ, McCarthy T, Kelly IP, Glynn T, McCoy G. Sciatic nerve palsy secondary to postoperative haematoma in primary total hip replacement. J Bone Joint Surg Br. 2005 Nov;87 (11):1465-1467.

27. Answer A. Ceramic on ceramic Figure 3.4 (see Questions section) represents the scratch profile of ceramic. Revision surgery for fractured or damaged ceramic components requires thorough debridement and removal of ceramic fragments. Revision with metal heads should be avoided and ceramic components used wherever possible. Trunnions that are damaged need to be replaced, but if only minimal damage exists; then a ceramic head with a titanium liner or trunnion adapter can be used. Rambani R, Kepecs DM, Mäkinen TJ, Safir OA, Gross AE, Kuzyk PR. Revision total hip arthroplasty for fractured ceramic bearings: a review of best practices for revision cases. J Arthroplasty. 2017 Jun;32(6):1959-1964.

30. Answer B. 125° versus 135° neck–shaft angle implant A longer femoral head length and incomplete insertion of an uncemented femoral stem will lengthen the patient. A larger sized femoral stem or increased offset stem will not generally change leg length.

37

Section 2 Chapter

3

Adult Elective Orthopaedics and Spine

Hip I Structured SBA Edward Holloway

HIP I STRUCTURED SBA QUESTIONS 1. An imaging report refers to a pathological avulsion at the insertion of the Iliopsoas tendon. What Gruen zone does this correspond to? A. Zone 1 B. Zone 2 C. Zone 4 D. Zone 6 E. Zone 7 2. A new lower limb arthroplasty consultant is choosing validated outcome measures to be used for his THA patients. Which of the following scores is completed by the clinician rather than the patient? A. Harris Hip Score B. Hip Disability and Osteoarthritis Outcome Score C. Oswestry Hip Score D. Oxford Hip Score E. Western Ontario and McMaster Universities Osteoarthritis Index 3. A physiotherapist refers a patient for an opinion on whether symptoms may be secondary to impingement between the ischium and lesser trochanter. Which test would be most useful to help detect this? A. Anterior Femoro-Acetabular Impingement test B. Dynamic External Rotation Impingement test C. Ischiofemoral Impingement test D. Patrick or FABER (Flexion, Abduction, External rotation) test E. Posterior Impingement test

4. A petite 73-year-old female underwent a THA 4 months ago. She presents to clinic at 6 months with pain and a snapping sensation in the groin on standing from a seated position. Initial examination does not reveal any concerns of infection and FBC/CRP/ESR are within normal values. Radiographs reveal a retroverted acetabular component. What is the most appropriate initial management for this patient? A. Radioisotope bone scan B. Revision of acetabular component C. Image-guided injection of iliopsoas tendon sheath D. Image-guided injection of hip joint E. Image-guided aspiration of hip joint 5. A woman who has recently given birth developed progressive, severe, non-traumatic hip and groin pain in the third trimester of her pregnancy. Hip radiographs show subchondral cortical loss, diffuse osteopaenia of the femoral head and neck and a preserved joint space. What is the most appropriate management? A. Arthroplasty B. Cannulated hip screws C. CT chest/abdo/pelvis D. Guided joint aspiration E. Protected weight bearing 6. A patient with painful osteoarthritis of their hip underwent femoral nailing for a middiaphyseal fracture 15 years previously. Their surgeon plans to perform hip replacement surgery while keeping the nail in place. What is the most suitable type of patient for this procedure? A. Large female over 65 years B. Large male over 65 years

27

Edward Holloway

C. Large male under 65 years D. Small female under 65 years E. Small male under 65 years 7. A nerve is damaged during the direct lateral hip approach when muscles are inadvertently split more than 5cm proximal to the greater trochanter. Asking the patient to perform which of the following movements is most likely to reveal a deficit? A. Dorsiflex ankle B. Extend great toe C. Extend hip D. Extend knee E. Stand on one leg 8. A patient received a hip arthroplasty typically reserved for younger patients with good femoral head bone stock which comprises approximately 3.5% of all hip arthroplasties. What follow-up is recommended for an asymptomatic patient with an ODEP 10 or 10A* rated implant? A. Annually, for as long as the device is implanted B. Annually for the first 5 years, two yearly to 10 years C. Annually for the first 5 years, two yearly to 10 years and three thereafter D. First year, once at 7 years, and once at 10 years E. First year, once at 7 years and three yearly thereafter

28

The nerve most commonly at risk during this approach crosses, in the majority of patients, the lateral border of which muscle? A. Rectus femoris B. Sartorius C. Tensor fascia lata D. Vastus intermedius E. Vastus lateralis 11. After inserting an uncemented cup, you carefully define quadrants by drawing a line from the ASIS to the centre of the cup and a second line perpendicular to this. You turn your back and your registrar inserts a screw in the anterosuperior quadrant. Which structure is most at risk? A. External iliac vessels B. Inferior gluteal nerve and vessels C. Internal pudendal nerve and vessels D. Obturator nerve and vessels E. Sciatic nerve 12. An audit of a department’s THA complications reveals a spike in intraoperative periprosthetic femoral fractures (IOPFF) equivalent to intraoperative Vancouver Type A2. Which factor is associated with the highest relative risk of this complication? A. Age 11–49 years B. Cementless stem C. Female sex D. Paediatric disease E. Previous trauma

9. You are consenting a 45-year old patient with a Garden II neck of femur fracture for surgery. What is the most common risk of this surgery? A. Femoral nerve injury B. Leg length discrepancy C. Avascular necrosis D. Sciatic nerve injury E. Trendelenburg gait

13. A 45-year-old welder with a history of steroid use presents with groin pain and an MRI that shows a 20% area of femoral head collapse. What is the most appropriate management? A. Bisphosphonate infusion B. Core decompression C. Proximal femoral osteotomy D. Total hip replacement E. Vascularised fibular graft

10. A patient has a hip replacement through an approach that is designed to be soft tissue preserving and is sometimes performed utilising a fracture table.

14. An 80-year-old patient listed for a THA has had previous lumbar spine surgery. Concerned about dislocation risk, you request sitting and standing lateral lumbar spine radiographs.

Hip I Structured SBA

The angle between a line from the centre of the S1 end plate and the centre of the segment between the two femoral heads (the bicoxofemoral axis), and the vertical is called what? A. APPt (anterior pelvic plane tilt) B. FPP (functional pelvic plane) C. PI (pelvic incidence) D. SPT (spinopelvic tilt) E. SS (sacral slope) 15. A 55-year-old patient with Parkinson’s disease presents with a Garden 4 neck of femur fracture. Radiographs show that the ratio of the inner canal diameter at the midportion of the lesser trochanter divided by the diameter 10cm distal is >0.75. What is the most appropriate management? A. Two-hole DHS B. Cemented dual mobility THA C. Cemented THA D. Uncemented dual mobility THA E. Uncemented THA 16. After thorough discussion, it is decided that the best option for a 20-year-old manual labourer with post-traumatic hip OA is a fusion. What is the most appropriate position of fusion? A. 0° external rotation, 0° adduction, 5° flexion B. 5° external rotation, 5° adduction, 25° flexion C. 10° external rotation, 15° abduction, 25° flexion D. 15° external rotation, 0° abduction, 5° flexion E. 15° external rotation, 15° abduction, 5° flexion 17. A patient underwent a THR for a NOF fracture while abroad on holiday. The patient was given strict rehabilitation instructions to avoid flexion beyond 90° and extreme internal rotation. What structure was most likely to have been injured during the patient’s surgery? A. Pudendal nerve B. Femoral nerve C. Superior gluteal nerve D. Lateral cutaneous nerve E. Sciatic nerve 18. A 77-year-old man presents with an insidious onset of hip and anterior thigh pain 15 years after a total hip arthroplasty. He denies fever or systemic upset,

and initial bloods show normal inflammatory markers. Anteroposterior (AP) radiograph left hip are shown in the radiograph (Figure 3.1). Figure 3.1 Anteroposterior (AP) radiograph left hip

You advise the patient that he should undergo revision surgery as he is at risk of what complication? A. Infection B. Dislocation C. Periprosthetic acetabular fracture D. Periprosthetic femur fracture E. Pseudotumour formation 19. You review a patient following hip arthroscopy and extensive debridement of a labral cyst. He has weakness of ankle dorsiflexion. Injury has most likely occurred as a result of what? A. Excessive traction at groin post B. Placement of anterior portal C. Placement of anterolateral portal D. Placement of distal anterolateral portal E. Placement of posterolateral portal 20. During intraoperative testing for stability of a THR you have implanted through a Modified Hardinge approach you see the hip is unstable in extension and external rotation. What change will increase the primary arc of hip motion? A. 20° versus 0° liner B. 36mm versus 32mm head C. High versus standard offset stem. D. Lateralised versus standard liner E. Skirted head

29

Edward Holloway

21. A 70-year-old woman presents with a red, swollen and painful area around the incision site of a THR performed 8 weeks earlier. She has a CRP of 78, WCC of 16 and temperature of 37.4° C. Her other observations are unremarkable. She did not attend 6-week follow-up and says the wound has not stopped leaking since she left hospital. What is the most appropriate management? A. Aspiration and culture-specific long-term antibiotic suppression B. Debridement, antibiotics, implant retention with exchange of modular components C. Debridement, antibiotics, implant retention with retention of modular components D. Empiric antibiotics E. Revision arthroplasty 22. A 53-year-old man with severe ulcerative colitis is referred for a THA. His pelvic x-ray shows Brooker grade 3 changes on the ipsilateral side replaced 4 years earlier. Which of the following prophylactic treatments is most appropriate? A. Diphosphonate 20mg/kg for 21 days B. Ibuprofen 400mg TDS for 4 weeks C. Indomethacin 25mg TDS for 6 weeks D. Single 2 500cGy radiotherapy dose E. Three 1 500cGy radiotherapy doses

A. B. C. D. E.

Alpha angle Lateral centre edge angle Sharp’s angle Tonnis angle Vertical centre edge angle

24. A young patient with cerebral palsy is referred with hip pain. You want to examine for contracture in a muscle with an origin at the inferior pubic symphysis and inferior pubic rami, which attaches to a point just posterior to the attachment of sartorius. What is the most appropriate test? A. Ely’s test B. Ober’s test C. Phelp’s test D. Thomas’ test E. Trendelenburg test 25. A 24-year-old woman falls while mountain biking and sustains the injury shown in this radiograph (Figure 3.3). In theatre, you struggle to reduce the fracture with in-line traction and rotation. Figure 3.3 Anteroposterior (AP) radiograph left hip

23. A 23-year-old woman has been referred by her physiotherapist with symptoms of hip impingement. A radiograph of her left hip is shown here (Figure 3.2). What is marked on her radiograph? Figure 3.2 Anteroposterior (AP) radiograph left hip

What is the most appropriate next step? A. Total hip arthroplasty B. Fixation in best position achieved C. Flexion to 90°, adduction, in-line traction, then internal rotation D. Extension, abduction, in-line traction, then external rotation E. Open reduction using a modified SmithPetersen approach

30

Hip I Structured SBA

26. A 30-year-old woman is referred for consideration of THA. Her radiographs have been classified as a Type B using the Hartofilakidis classification. What anatomical characteristic of the femur is most commonly associated with this? A. Excessive anteversion B. Excessive femoral bow C. Excessive retroversion D. Excessive valgus E. Excessive varus 27. The diagram shown here represents the scratch profile of a material used in THA femoral heads (Figure 3.4). Figure 3.4 Scratch profile

If such a component fractures, what bearing couple should be used during revision surgery? A. Ceramic on ceramic B. Ceramic on poly C. Metal on metal D. Metal on poly E. Oxinium on poly 28. A 45-year-old presents with symptoms of hip impingement and radiographs show the femoral head to be medial to Kohler’s line. What is the most likely diagnosis?

A. B. C. D. E.

Otto pelvis Coxa magna Coxa vara Coxa valga Acetabular retroversion

29. A 60kg woman develops progressive pain and numbness in the lateral calf with weakness of ankle dorsiflexion 48 hours after THA. Which of the following is the most appropriate action? A. Book for emergency surgery B. Nurse prone and review the following morning C. Nurse with knee in flexion and review the following morning D. Urgent MRI scan E. Withhold prophylactic LMWH 30. A surgeon admits a mistake was made during a THA. The patient complains that their operated leg feels short. What is most likely to have resulted in this? A. +8mm versus 0mm femoral head B. 125° versus 135° neck–shaft angle implant C. High versus standard offset stem D. Incomplete insertion of an uncemented stem E. Size 4 rather than size 3 implant inserted

31

Edward Holloway

HIP I STRUCTURED SBA ANSWERS 1. Answer E. Zone 7 Iliopsoas originates from the anterior and inferior aspects of the transverse processes of L1–L5 and the bodies and discs of T12–L5. It inserts into the lesser trochanters and is innervated by direct fibres from the lumbar plexus (L1/L2/L3). It acts as a flexor of the hip. Gruen zones are used to describe areas of loosening around a femoral stem. Zone 1 relates to the greater trochanter, zone 7 the lesser trochanter and zones 6 and 5, and zones 2 and 3 on the medial and lateral aspects of the femur, respectively. Zone 4 is at tip of the stem (Figure 3.5). Figure 3.5 Gruen zones

2. Answer A. Harris Hip Score All of the outcome measures apart from the Harris Hip Score are completed by the patient. The Harris Hip Score gives a score of 0–100, with 100 being the best possible outcome. The doctor-completed score consists of the four subscales of pain severity (1 item, 0–44), function (7 items, 0–47 points), absence of deformity (1 item, 0–4 points) and range of motion (2 items, 0–5 points).

32

3. Answer C. Ischiofemoral Impingement test Ischiofemoral impingement occurs due to an abnormal contact between the ischium and the lesser trochanter, most commonly following total hip arthroplasty or proximal femoral intertrochanteric osteotomy. This type of impingement can still develop in non-operative patients such as those with severe osteoarthrosis causing protrusio acetabuli or fracture malunion of the proximal femur. The impingement happens due to narrowed distance between the ischial tuberosity and the lesser trochanter, leading to compression of the quadratus femoris muscle, which becomes oedematous in the early stages but subsequently undergoes fatty replacement. Patients present with hip, groin and gluteal pain which may radiate to the lower leg due to sciatic nerve irritation. In certain cases, bursal inflammatory fluid formation may happen around the iliopsoas tendon at the lesser trochanter. The diagnosis of ischiofemoral impingement can be challenging, as clinical tests may be non-specific, and therefore imaging is crucial. The treatment of this condition varies from conservative measures such as CT-guided cortisone injection to surgical excision of the quadratus femoris. 4. Answer C. Image-guided injection of iliopsoas tendon sheath This case describes iliopsoas impingement syndrome. This is an underdiagnosed cause of groin pain following THR. The pain may be accompanied by limp, tenderness to palpation in the groin and, more rarely, a snapping sensation. Simple provocation tests are straight leg raising or resisted flexion in the seated position. Risk factors for iliopsoas impingement syndrome are acetabular component malposition (more likely in smaller patients), leg length discrepancy, excessively long acetabular screws and retained cement. An image-guided injection of the tendon sheath has diagnostic and therapeutic benefits. Definite options for management are revision of a malpositioned acetabular component, correction of leg length deformity and resection or

Hip I Structured SBA

tenotomy of the tendon depending upon the underlying cause. A radioisotope bone scan is unlikely to be useful 6 months post-surgery and the symptoms are not in keeping with loosening or infection. A revision may be indicated in the future but not as an initial management. Henderson RA, Lachiewicz PF. Groin pain after replacement of the hip: aetiology, evaluation and treatment. J Bone Joint Surg Br. 2012 Feb;94(2):145-151. 5. Answer E. Protected weight bearing Idiopathic transient osteoporosis of the hip (ITOH) is a atraumatic, progressive cause of hip and groin pain presenting more commonly in women (3:1 female:male) and in two distinct groups: (1) women in the third trimester of pregnancy and (2) middle aged men. ITOH is a diagnosis of exclusion with characteristic appearances on plain films and MRI. Radiographic features lag behind clinical symptoms by 4–6 weeks and include subchondral cortical loss, diffuse osteopaenia and joint effusion, but always a preserved joint space. MRI findings include T1: decreased signal and loss of fatty marrow, and T2: effusion and high signal and marrow oedema. Symptoms usually resolve spontaneously in 6–8 months, but weight bearing should be protected to reduce the risk of stress fractures. 6. Answer C. Large male under 65 years An important indication of hip resurfacing is patients with pre-existing metalwork in the medullary canal of the femur which precludes a metaphyseal stem. Advantages of a hip resurfacing over a THA include increased stability due to a larger head size, preservation of femoral head and neck bone stock, simpler femoral revision, lower mortality and the potential of a more normal gait pattern and participation in highdemand activities. After high-profile failures including certain implants the most suitable patient group to benefit from resurfacing is being re-defined, but it is generally considered to be most suitable for younger, larger male patients. Logishetty K, Muirhead-Allwood SK, Cobb JP. Hip resurfacing – what is its role in

modern orthopaedics? Bone & Joint 360. 2020 Feb;9(1):4-9. 7. Answer E. Stand on one leg The superior gluteal nerve is at risk during deep dissection in the direct lateral approach to the hip. The fibres of gluteus medius are split from the middle of the greater trochanter proximally. If the split is continued more than 3–5cm proximal to the trochanter there is risk of nerve damage. The superior gluteal nerve originates from the lumbosacral plexus with contribution from nerve roots L4–S1 and innervates gluteus medius, gluteus minimus, and tensor fascia lata. Weakness will manifest clinically during Trendelenburg test or hip abduction. 8. Answer E. First year, once at 7 years and three yearly thereafter ODEP 10A or 10A* rated hip resurfacing devices are the MatOrtho Adept Resurfacing Head (48–58mm), and Smith & Nephew Birmingham Hip Resurfacing Head (48–62mm). It is recommended that they be followed up during the first year, once at 7 years and three yearly thereafter. Patients at risk of adverse reaction to metal debris (ARMD) (female patients, males with femoral components smaller than 48mm and those with a DePuy ASR implant) should be reviewed annually for as long as the device is implanted. Those not at risk but symptomatic should also be seen annually. Other implants in asymptomatic patients not at risk should be seen annually for the first 5 years, two yearly to 10 years, and then three yearly thereafter. Logishetty K, Muirhead-Allwood SK, Cobb JP. Hip resurfacing – what is its role in modern orthopaedics? Bone & Joint 360. 2020 Feb;9(1):4-9. 9. Answer C. Avascular necrosis Garden classified femoral neck fractures into four types depending on the degree of displacement seen on an anteroposterior radiograph. Type I are incomplete fractures, type II are complete but non-displaced fractures, Type III are complete and partially displaced fractures and Type IV are complete and fully displaced. Young patients with Garden II neck of femur fractures would most commonly be managed

33

Edward Holloway

with closed reduction and fixation with either cannulated hip screws or an alternate device, unless the patient had significant comorbidities making an arthroplasty with a lower risk of reoperation a better option. Leg length discrepancy, sciatic nerve palsy injury and Trendelenburg gait would all be complications of THR. Parker MJ, Gurusamy KS. Internal fixation versus arthroplasty for intracapsular proximal femoral fractures in adults. Cochrane Database Syst Rev. 2006;(4):CD001708. doi:10.1002/ 14651858.CD001708. 10. Answer B. Sartorius The direct anterior approach (DAA) to the hip is performed with the aim of reducing muscle damage, length of stay, pain and complication rate. A fracture table with specific attachments is used by some to assist in femoral exposure. There is a risk of damage to the lateral cutaneous nerve of the thigh. This nerve arises from the lumbar plexus, or more rarely the femoral nerve itself, and travels through the pelvis on the iliacus muscle. It enters the thigh under the inguinal ligament at a point anywhere between the anterior superior iliac spine (ASIS) and the midinguinal point. The nerve then pierces the fascia lata medial and inferior to the ASIS. From here, the nerve takes a variable course but most commonly the medial border of sartorius. Meermans G, Konan S, Das R, Volpin A, Haddad FS. The direct anterior approach in total hip arthroplasty: a systematic review of the literature. Bone Joint J. 2017 Jun;99-B(6):732-740. 11. Answer A. External iliac vessels When placing screws to secure an uncemented acetabular component, quadrants are defined by drawing a line from the ASIS to the centre of the acetabulum and then a second line perpendicular to this. The posterosuperior quadrant is the safest for screw placement. The posteroinferior quadrant should be used with caution, as screws longer than approximately 20mm may put the sciatic nerve, internal pudendal nerve and vessels, and the inferior gluteal nerve and vessels at risk. The anteroinferior quadrant (zone of danger) puts the obturator nerve and vessels at risk. The

34

anteroinferior quadrant (zone of death) puts the external iliac vessels at risk. 12. Answer B. Cementless stem Intraoperative Vancouver A2 fractures are nondisplaced fractures of the proximal metaphysis. All answers are associated with an increased relative risk of calcar fracture, but cementless stems give the greatest relative risk (RR) (RR = 3.8). Age 11 – 49 RR = 1.5, female sex RR = 1.9, paediatric disease RR = 2.6, previous trauma RR = 3.6. Lamb JN, Matharu GS, Redmond A, Judge A, West RM, Pandit HG. Risk factors for intraoperative periprosthetic femoral fractures during primary total hip arthroplasty. An analysis from the National Joint Registry for England and Wales and the Isle of Man. J Arthroplasty. 2019;34(12):3065-3073.e1 13. Answer D. Total hip replacement Avascular necrosis of the hip is predominantly idiopathic but may be associated with alcohol abuse, steroid use, hypercoagulability, Caisson’s disease and sickle cell disease. Treatment options depend upon the clinical and radiographic stage (Ficat, Steinberg) of the disease and age/comorbidities of the patient. The relatively conservative options listed would be options in younger patients or those with earlier stages of disease. Once any significant amount of collapse has occurred in a patient over 40, the most likely option is an arthroplasty procedure. Petek D, Hannouche D, Suva D. Osteonecrosis of the femoral head: pathophysiology and current concepts of treatment. EFORT Open Rev. 2019;4(3):85-97. 14. Answer D. SPT (spinopelvic tilt) Increasingly, the relationship between the spine, hip and knee is assessed and considered in how it may affect the functionality and stability of a THR as the patient moves from lying to standing and from sitting to standing. All options can be measured radiographically to this end. APPt (anterior pelvic plane tilt) refers to the rotation of the pelvis in the sagittal plane as measured by the angle formed between the coronal plane and a line from the anterior superior iliac spine (ASIS) to pubic symphysis.

Hip I Structured SBA

The APP (anterior or anatomical pelvic plane) is defined by the pubic symphysis and the two anterior superior iliac spines. The FPP (functional pelvic plane) rotates this plane according to an individual’s pelvic tilt. It is considered a more accurate reference plane for assessing acetabular component position. PI (pelvic incidence) is the angle between two lines: one from the centre of the femoral head to the centre of the S1 end plate, and a second perpendicular to a line across the S1 end plate, intersecting the centre of the end plate. SPT (spinopelvic tilt) is the angle between a line from the centre of the S1 end plate and the centre of the segment between the two femoral heads (the bicoxofemoral axis), and the vertical. SS (sacral slope) is the angle between two lines: one parallel to the S1 end plate, and a second along a horizontal reference plane. Ike H, Dorr LD, Trasolini N, Stefl M, McKnight B, Heckmann N. Spine-pelvis-hip relationship in the functioning of a total hip replacement. J Bone Joint Surg Am. 2018 Sep 19;100(18):1606-1615. 15. Answer B. Cemented dual mobility THA The Dorr classification depends upon the ratio between the inner canal diameter at the level of the midpoint of the lesser trochanter and a point 10cm below that. Dorr C femurs as defined as a ratio >0.75 are most suitable for a cemented prosthesis. A patient with significant risk of dislocation and a grossly displaced NOF fracture would be a good candidate for a dual-mobility acetabular component. 16. Answer B. 5° external rotation, 5° adduction, 25° flexion This is the most appropriate position of hip fusion. 17. Answer E. Sciatic nerve The position of greatest risk of dislocation for a posterior approach THA is flexion and internal rotation. The structure most at risk during this approach is the sciatic nerve. 18. Answer D. Periprosthetic femur fracture The radiograph shows a loose Charnely femoral stem which is close to fracturing through the

posterior cortex of the femur (Figure 3.6). The THA may be infected but the fact that the acetabulum does not look loose so goes against this. Pseudotumour formation is associated with

Figure 3.6 Lateral radiograph of left hip

metal-on-metal bearing couples. 19. Answer E. Placement of posterolateral portal By far the most common direct nerve injury resulting from hip arthroscopy portal placement is of the lateral femoral cutaneous nerve when placing or working through the anterior portal. Sciatic nerve injury causing a foot drop is a rare complication of posterolateral portal placement, made more likely by placing the leg in external rotation. Papavasiliou AV, Bardakos NV. Complications of arthroscopic surgery of the hip. Bone Joint Res. 2012;1(7):131-144. 20. Answer B. 36mm versus 32mm head The primary arc of hip motion is the range that the hip can move before the neck impinges upon the acetabulum and is an important determinant of THA stability. A lipped or angled liner may improve stability by increasing coverage but actually decreases the primary arc of motion and, in this instance, may make anterior dislocation more likely by causing posterior impingement levering the hip out of joint. A high offset stem or lateralised liner will not alter the primary arc of movement but may improve stability by moving the femoral neck away from impinging soft tissues or osteophytes (potentially to the detriment of proper restoration of hip biomechanics). A skirted head (as found on the longer lengths) decreases the primary arc. 21. Answer E. Revision arthroplasty All the options are valid in managing the infected joint replacement. Patients not sufficiently fit to do well with further significant or potentially multiple procedures can be managed

35

Edward Holloway

with culture-specific, long-term suppressive antibiotics or an excision arthroplasty, if the infection is not able to be suppressed or the patient is unable to take long-term antibiotics. The success rates of DAIR (debridement, antibiotics, and implant retention) procedures vary enormously in the literature, from 11–100%. The success rate is correlated with type of organism (Streptococcus species tend to do better), exchange of modular components and time from procedure in early infection, or time from first presentation of symptoms in late infection. An older patient with long duration of symptoms is less likely to have successful infection eradication with a DAIR so may be best served with a one- or two-stage revision procedure depending upon the infecting organism and the surgeon’s preference. Grammatopoulos G, Kendrick B, McNally M, et al. Outcome following debridement, antibiotics, and implant retention in hip periprosthetic joint infection – an 18-year experience. J Arthroplasty. 2017 Jul;32(7):2248-2255. Kunutsor SK, Beswick AD, Whitehouse MR, Wylde V, Blom AW. Debridement, antibiotics and implant retention for periprosthetic joint infections: A systematic review and metaanalysis of treatment outcomes. J Infect. 2018 Dec;77(6):479-488. 22. Answer E. Three 1 500cGy radiotherapy doses Heterotopic ossification (HO) following THR occurs with an incidence of 5–90% and is associated with the risk factors of male sex, history of previous HO, older age, previous hip fusion, hypertrophic OA, ankylosing spondylitis, posttraumatic OA, Paget disease, osteonecrosis and rheumatoid arthritis. It is classified according to the Brooker classification according to the extent of heterotopic bone formation on an AP radiograph. Prophylaxis can be considered for patients at high risk of HO and the main forms are radiotherapy and non-steroidal anti-inflammatory drugs (NSAIDs). Diphosphonates have not been shown to be effective. NSAIDs would be contraindicated in a patient with severe inflammatory bowel disease. Multiple doses of radiation

36

therapy have been shown to be more effective than single doses. Board TN, Karva A, Board RE, Gambhir AK, Porter ML. The prophylaxis and treatment of heterotopic ossification following lower limb arthroplasty. J Bone Joint Surg Br. 2007 Apr;89 (4):434-440. 23. Answer D. Tonnis angle The following are measured during the radiographic assessment of patients with symptoms of femoroacetabular impingement: The Tonnis angle (also called the acetabular index or acetabular roof angle) is the angle between the horizontal and a tangential line from the medial to the lateral sourcil (weight bearing zone of the acetabulum). Alpha angle is measured on a Dunn view by drawing a line from the centre of the narrowest point of the femoral neck to the centre of the femoral head using a best-fit circle. The angle is between the line down the axis of the femoral neck and a line drawn to the location where the femoral head becomes ‘out of round’. The lateral centre edge angle (of Wiberg) is the angle between a line drawn from the centre of the femoral head to the lateral edge of the acetabulum and a second line that is parallel to the longitudinal pelvic axis. Sharp’s angle, or acetabular angle, is the angle between a horizontal line drawn through the teardrops and a line drawn from the teardrop to the lateral acetabular roof. The vertical centre edge angle is measured from a false profile radiograph and is between a vertical line through the centre of the femoral head and a line from the centre of the femoral head to the anterior most point on the acetabulum. Mannava S, Geeslin AG, Frangiamore SJ, et al. Comprehensive clinical evaluation of femoroacetabular impingement: part 2, plain radiography. Arthrosc Tech. 2017;6(5):e2003-e2009. 24. Answer C. Phelp’s test Phelp’s test assesses gracilis tightness. Ely’s test assesses rectus femoris tightness. Ober’s test assesses tightness of the tensor fascia lata. Thomas’ test assesses fixed flexion of the hip. Trendelenburg test assesses hip abductor dysfunction.

Hip I Structured SBA

25. Answer C. Flexion to 90°, adduction, in-line traction, then internal rotation Leadbetter described a technique in 1939 to aid in the reduction of intracapsular neck of femur fractures. The limb is flexed to relax the musculature around the hip and then internal rotation functions to relax the ligamentous structures. From the above position, further flexion and adduction open the fracture, allowing reduction to be achieved by maintaining traction while bringing the limb into extension with slight abduction and continued internal rotation.

28. Answer A. Otto pelvis Otto pelvis (arthrokatadysis or protrusio acetabuli) is a rare condition associated with hip osteoarthritis. Protrusio acetabuli is defined radiographically when the medial aspect of the femoral head projects beyond the ilioischial (Kohler’s) line. Though sometimes idiopathic, acetabular protrusion is more usually associated with inflammatory arthropathies, osteoporosis, osteomalacia and Paget disease. When performing THR, care must be taken to lateralise the medialised hip centre.

26. Answer A. Excessive anteversion The Hartofilakidis classification recognises three types of congenital hip disease in adults: type A – dysplasia, type B – low dislocation, and type C – high dislocation. In type A disease, the femoral head remains within the original acetabulum. In type B, it articulates with a false acetabulum which partially covers the original acetabulum. In type C, the femoral head has migrated superiorly and posteriorly to the hypoplastic original acetabulum. Adult hip dysplasia is most commonly associated with excessive femoral anteversion.

29. Answer A. Book for emergency surgery Nerve dysfunction following THA may be a result of intraoperative compression from retractors, traction injury secondary to manipulation or lengthening. If noted in the immediate postoperative period, tension can be taken off the sciatic nerve by nursing the patient with the knee in flexion. Progressive, painful sciatic nerve dysfunction should raise the suspicion of compression from haematoma. This is associated with patients of lower mass, and with anticoagulant use. Improved outcomes are associated with prompt exploration and evacuation of haematoma. Su EP. Post-operative neuropathy after total hip arthroplasty. Bone Joint J. 2017 Jan;99-B(1 Suppl):46-49. Butt AJ, McCarthy T, Kelly IP, Glynn T, McCoy G. Sciatic nerve palsy secondary to postoperative haematoma in primary total hip replacement. J Bone Joint Surg Br. 2005 Nov;87 (11):1465-1467.

27. Answer A. Ceramic on ceramic Figure 3.4 (see Questions section) represents the scratch profile of ceramic. Revision surgery for fractured or damaged ceramic components requires thorough debridement and removal of ceramic fragments. Revision with metal heads should be avoided and ceramic components used wherever possible. Trunnions that are damaged need to be replaced, but if only minimal damage exists; then a ceramic head with a titanium liner or trunnion adapter can be used. Rambani R, Kepecs DM, Mäkinen TJ, Safir OA, Gross AE, Kuzyk PR. Revision total hip arthroplasty for fractured ceramic bearings: a review of best practices for revision cases. J Arthroplasty. 2017 Jun;32(6):1959-1964.

30. Answer B. 125° versus 135° neck–shaft angle implant A longer femoral head length and incomplete insertion of an uncemented femoral stem will lengthen the patient. A larger sized femoral stem or increased offset stem will not generally change leg length.

37

Section 2 Chapter

4

Adult Elective Orthopaedics and Spine

Hip II Structured SBA James Gill and Majeed Shakokani

HIP II STRUCTURED SBA QUESTIONS 1. Metal hypersensitivity to orthopaedic implants is classed as what kind of sensitivity? A. Type I B. Type II C. Type III D. Type IV E. Type V 2. A 78-year-old female has been listed for total hip arthroplasty and noted to have protrusio on her anteroposterior (AP) radiograph. Which of the following conditions is not commonly associated with acetabular protrusio? A. Ankylosing spondylitis B. Marfan syndrome C. Neurofibromatosis D. Paget disease E. Rheumatoid arthritis 3. A 42-year-old male presents to the orthopaedic clinic with a 2-month history of left hip pain. His anteroposterior radiograph is shown here (Figure 4.1).

Figure 4.1 Anteroposterior (AP) radiograph hips

38

Which of the following is not associated with his condition? A. Glucocerebrosidase gene B. HbSS C. Protein S deficiency D. Scleroderma E. Simvastatin 4. A 24-year-old manual labourer presents with severe post-traumatic hip arthritis. Hip fusion has been discussed at the regional MDT. What is the optimal position for hip arthrodesis? A. 25° flexion, 5° adduction, 5° external rotation B. 10° flexion, 10° adduction, 10° external rotation C. 10° flexion, 5° adduction, 5° external rotation D. 25° flexion, 0° abduction, 10° internal rotation E. 25° flexion, 10° abduction, 10° external rotation 5. On counselling the man in question 5 in the outpatient clinic regarding the proposed hip fusion, you mention ipsilateral knee pain as a possible long-term complication of the procedure. What percentage of patients develop ipsilateral knee pain following hip arthrodesis? A. 20% B. 40% C. 60% D. 80% E. 100% 6. Which of the following results in increased abductor muscle force when performing a single leg stance after arthroplasty? A. Carrying a bag of shopping with the ipsilateral arm B. Medialisation of the acetabular cup C. Reducing offset D. Trunk lean to the ipsilateral side upon single leg stance E. Walking with a stick in the contralateral hand post

Hip II Structured SBA

7. A patient presents with a fractured metal stem of a total hip arthroplasty. On closer inspection of the previous anteroposterior radiographs of their hip taken 6 months ago, there was evidence of loosening. In which Gruen zones would loosening be expected in the pre-fracture radiographs in this scenario? A. 4, 5 B. 4, 5 and 6 C. 1, 2, 6 and 7 D. 7, 6, 4 and 5 E. 1, 2, 3, 4, 5, 6 and 7 8. Which of the following is the best mode of imaging to assess for a pseudotumour associated with a metal-on-metal hip resurfacing? A. Computed tomography B. MARS MRI C. SPECT D. Ultrasound E. White cell scan 9. A 60-year-old male falls off a camel while on holiday in Lanzarote and suffers a fracture to the ceramic head of his left total hip arthroplasty (THA). The THA was performed 2 years previously; components included an uncemented titanium cup with polyethylene liner and an uncemented stem with a ceramic head. Prior to the fall, he was completely happy with the hip. Which of the following is the most appropriate procedure? A. Revision of ceramic head with change of polyethylene liner B. Revise to a cobalt-chrome head with change of polyethylene liner C. Revise to a metal-on-metal bearing surface D. Revise to a metal-lined ceramic head with change of polyethylene liner E. Revise all implants 10. Which kind of lubrication predominates in a metal-on-polyethylene total hip arthroplasty? A. Boosted B. Boundary C. Elastohydrodynamic D. Squeeze film E. Weeping

11. One year after primary total hip arthroplasty performed using a Southern Moore approach a patient asks why his foot turns inwards when he walks? The most likely cause would be which of the following? A. Excessive acetabular cup anteversion B. Excessive retroversion of femoral stem C. Failure of the repair of the short external rotators D. Palsy of gluteus medius and minimus E. Weakness of gluteus maximus 12. When drilling a screw to augment fixation of an uncemented acetabular cup, profuse bleeding is encountered and the patient becomes hypotensive. Which vessel has been injured? A. External iliac B. Femoral C. Internal iliac D. Obturator E. Pudendal 13. Which of the following is the correct description of one of the quadrants described for acetabular screw placement? A. Anterior superior: Superior to a line drawn from the anterior superior iliac spine passing through the centre of the acetabulum and anterior to a line drawn perpendicular to this also passing through the centre of the acetabulum B. Posterior inferior: Inferior to a line drawn from the anterior superior iliac spine passing through the centre of the acetabulum and posterior to a line drawn perpendicular to this also passing through the centre of the acetabulum C. Posterior superior: Superior to a line drawn from the anterior superior iliac spine passing through the centre of the acetabulum and posterior to a line drawn perpendicular to this also passing through the centre of the acetabulum D. Posterior superior: Superior to a line drawn from the anterior superior iliac spine passing through the centre of the acetabulum and inferior to a line drawn perpendicular to this also passing through the centre of the acetabulum

39

James Gill and Majeed Shakokani

E. Posterior superior: Posterior to a line drawn from the anterior superior iliac spine passing through the centre of the acetabulum and superior to a line drawn perpendicular to this also passing through the centre of the acetabulum 14. A 78-year-old male is listed for total hip arthroplasty (THA). He is a Jehovah’s Witness and does not want any blood products to be given. What is the best way to reduce blood transfusion requirement in a Jehovah’s Witness undergoing THA? A. Autologous blood transfusion B. Cell salvage C. Erythropoietin D. Hypotensive anaesthesia E. Wound infiltration with local anaesthetic with adrenaline 15. You are beginning a total hip arthroplasty in theatre in a diabetic. A surgical house officer is assisting you and asks about the infection risk. Which factor has the biggest influence in reducing prosthetic joint infection? A. Antibiotic loaded bone cement B. Body exhaust suit C. Plastic isolator D. Systemic antibiotics E. Ultraclean air 16. What is the nerve supply to the superior gemelli? A. Nerve to gemelli B. Nerve to obturation internus C. Nerve to piriformis D. Nerve to quadratus femoris E. Obturator nerve 17. Which of the following structures exits the greater sciatic foramen and does not re-enter the lesser sciatic foramen? A. Nerve to obturator internus B. Nerve to quadratus femoris C. Obturator externus D. Obturator internus E. Pudendal nerve

40

18. Which anatomical structure does not enter the pelvis via the lesser sciatic foramen? A. Internal pudendal artery B. Internal pudendal vein C. Nerve to obturator internus D. Obturator internus E. Pudendal nerve 19. Which anatomical variation of the sciatic nerve exiting the greater sciatic foramen is most frequently observed? A. Common peroneal division exiting above piriformis and tibial nerve division exiting beneath piriformis B. Common peroneal division exiting above piriformis and tibial nerve exiting though piriformis C. Common peroneal division passing through piriformis and tibial nerve division exiting beneath piriformis D. Sciatic nerve exiting above piriformis E. Sciatic nerve exiting through piriformis 20. In the days following a total hip arthroplasty, a patient was found to have 0/5 power in ankle dorsiflexion and foot eversion but 5/5 power in ankle plantar flexion. Provided the injury was a neuropraxia at the level of the hip joint, which muscle would you expect to recover first on assessment with electromyography? A. Adductor magnus B. Peroneus longus C. Popliteus D. Short head of biceps femoris E. Tibialis posterior 21. A professional tennis player develops pain around the hip 2 months after metal-on-metal hip resurfacing. Which imaging modality can detect heterotopic ossification earliest? A. Bone scintigraphy B. Computed tomography C. Magnetic resonance imaging D. Radiographs E. Ultrasound scan

Hip II Structured SBA

22. What structures form the teardrop on an anteroposterior radiograph of the pelvis? A. Calcified ligamentum teres B. Cotyloid fossa, superior aspect of the obturator foramen and the cortical surface of the true pelvis C. Dense trabeculae of the origin of the superior public ramus D. Osteophyte within the cotyloid fossa E. True floor of the acetabulum and the superior pubic ramus 23. Which of the following conditions is associated with increased acetabular anteversion? A. Ankylosing spondylitis B. Legg–Calve–Perthes disease C. Pincer femoral acetabular impingement D. Rheumatoid arthritis E. Slipped upper femoral epiphysis 24. Acetabular protrusio can be defined using an anteroposterior radiograph as which of the following? A. Acetabular fossa medial to the ilioischial line B. Acetabular fossa medial to the iliopectineal line C. Centre-edge angle greater than 30 degrees D. Femoral head medial to ilioischial line E. Femoral head medial to iliopectineal line 25. A 37-year-old male who is training for the longdistance triathlon world championships, Ironman Hawaii, presents with a 6-week history of groin pain. Which of the following diagnoses requires urgent operative intervention? A. Cam lesion B. Iliotibial band syndrome C. Inferior medial femoral neck stress fracture involving 40% of neck width D. Piriformis syndrome E. Superior lateral femoral neck stress fracture involving 25% of neck width 26. What mechanical property predisposes ceramic to fracture? A. Failure at a point below the ultimate tensile strength secondary to repetitive loading B. Large area under the stress–strain curve C. Low modulus of elasticity

D. Material that exhibits linear stress stain relationship until the point of failure E. Progressive deformation in response to a constant force over a prolonged period 27. Sickle cell disease is associated with which complication following total hip arthroplasty? A. Early prosthetic loosening B. Higher risk for heterotopic ossification C. Increased blood loss D. Nerve palsy E. Periprosthetic infection 28. Which of the following ligaments is the strongest? A. Iliofemoral B. Ischiofemoral C. Ischiosacral D. Ligamentum teres E. Pubofemoral 29. Which of the following combinations of implant position is optimal for total hip arthroplasty? A. Cup inclination 30°, cup anteversion 20°, femoral stem anteversion 5° B. Cup inclination 40°, cup anteversion 15°, femoral stem anteversion 15° C. Cup inclination 40°, cup anteversion 20°, femoral stem anteversion 0° D. Cup inclination 50°, cup anteversion 30°, femoral stem anteversion 25° E. Cup inclination 50°, cup anteversion 40°, femoral stem anteversion 15o 30. When performing a total hip arthroplasty, what is the optimal combined version of the acetabular cup and femoral stem? A. 20° B. 35° C. 50° D. 5° E. 65° 31. A patient has suffered multiple dislocations of a total hip arthroplasty and has failed conservative measures. Which of the following is an indication for a constrained liner?

41

James Gill and Majeed Shakokani

A. B. C. D.

60° acetabulum inclination Femoral stem retroversion Gluteus medius and minimus deficiency Impingement of the femoral neck on a lipped liner E. Neutral version of the acetabulum 32. When performing the Hardinge approach to the hip, neurovascular structures are placed in jeopardy if the gluteus medius is split too far proximally. What is the maximum safe split above the tip of the greater trochanter that does not place the nerve at risk? A. 2cm B. 3cm C. 4cm D. 5cm E. 7cm 33. Which of the following describes a Ward’s triangle? A. A sign of osteoporosis B. Lateral to the primary compressive trabeculae and medial to the secondary compressive trabeculae C. Lateral to the secondary compressive trabeculae and superior to the primary trabeculae D. Medial to the primary compressive trabeculae and inferior to the primary tensile trabeculae E. Medial to the secondary tensile trabeculae and lateral to the secondary compressive trabeculae 34. Smith-Petersen first described a direct anterior approach to the hip. Which vessel is sacrificed as part of this approach to the hip? A. Ascending branch of the lateral femoral circumflex artery B. Descending branch of the lateral femoral circumflex artery C. Descending branch of the medial femoral circumflex artery D. Superficial external pudendal artery E. Transverse branch of the lateral femoral circumflex artery 35. Which size of particles are thought to be most biologically active?

42

A. B. C. D. E.

0.1µm 0.5µm 1.0µm 1.5µm 0.05µm

36. A 44-year-old male has deteriorating hip pain secondary to osteonecrosis. Symptoms have failed to improve despite conservative measures including protected weight bearing. A subchondral lucent line can be seen on the anteroposterior radiographs of the hip. What is the most appropriate management? A. Core decompression B. Free fibula graft C. Hip arthrodesis D. Rotational femoral osteotomy E. Total hip arthroplasty 37. Which of the following factors is not responsible for osteolysis around a total hip arthroplasty secondary to polyethylene wear debris? A. IL-1 B. IL-6 C. Osteoprotegerin D. PDGF E. Receptor activator of nuclear factor kappa-B ligand 38. With which femoral component would the most proximal bone loss be anticipated as a result of stress shielding? A. Cemented dual taper cobalt-chromium stem B. Cemented stainless steel composite beam stem C. Uncemented proximally porous coated cobalt-chromium stem D. Uncemented, fully porous coated cobaltchromium stem E. Uncemented, fully porous coated titanium stem 39. What is the intermuscular plane of the medial approach to the hip? A. Adductor brevis and adductor longus B. Adductor brevis and adductor magnus C. Adductor longus and gracilis D. Gracilis and adductor magnus E. Sartorius and adductor longus

Hip II Structured SBA

40. Which muscle is paired with its correct innervation? A. Adductor brevis – posterior division of obturator nerve B. Adductor longus – posterior division of obturator nerve C. Adductor magnus – tibial nerve and anterior division of obturator nerve D. Gracilis – posterior division of obturator nerve E. Pectineus – femoral nerve 41. The femoral circumflex vessels are named according to the relationship with which anatomical structure? A. Calcar femorale B. Iliopsoas C. Lesser trochanter D. Pectineus E. Profunda femoris artery 42. Taper slip cemented stem fixation is optimised by all of the following except? A. Cement mantle >2mm B. Flexible femoral stem C. Limited porosity cement D. Smooth femoral stem E. Stem centralisation 43. Which of the following is incorrect regarding the capsule of hip joint? A. Gluteus minimus and rectus femoris have direct attachments onto the capsule B. The capsule attaches more distally on the neck posteriorly compared to anteriorly C. The iliofemoral ligament shares an attachment with the direct head of the rectus femoris D. The ischiofemoral ligament is divided as part of the Southern–Moore approach to the hip E. The Y-ligament of Bigelow is divided as part of the Smith-Petersen approach to the hip 44. A 58-year-old male was seen in the outpatient clinic with nonspecific complaints of pain in the lumbar spine, buttock, lateral hip and thigh. Nerve impingement of which nerve can mimic hip joint pathology? A. Femoral nerve B. L2 C. Lateral cutaneous nerve of the thigh

D. Obturator nerve E. Posterior cutaneous nerve of the thigh 45. A 38-year-old female is diagnosed with pincertype femoral acetabular impingement. On MRI of the hip, what pattern of changes would be expected to be seen? A. Anterior intrasubstance labral tear and a posterior acetabular cartilage lesion B. Anterior intrasubstance labral tear and an anterior acetabular cartilage lesion C. Anterior labral avulsion and anterior acetabular cartilage delamination D. Posterior intrasubstance labral tear and posterior acetabular cartilage delamination E. Posterior labral tear and an anterior acetabular cartilage lesion 46. You are performing a cemented total hip arthroplasty for a 70-year-old retired farmer. A trial reduction is repeated with the definitive cup and stem cemented in place and a 32mm standard (neck length) head; however, stability is suboptimal. Stability is re-assessed with a 32mm ‘plus 4mm’ (neck length) head and stability is now satisfactory. The neck angle for the femoral implant is 125°. How much will a ‘plus 4mm’ head increase leg length and offset, respectively, compared to a standard head? A. 0mm, 4.0mm B. 2.3mm, 3.3mm C. 2.8mm, 2.8mm D. 3.3mm, 2.3mm E. 4.0mm, 0mm 47. A patient presents with pain 10 years following a total hip arthroplasty and radiographs show lucency in Gruen zones 4 and 5. Which mode of failure is the likely cause? A. Bending cantilever B. Calcar pivot C. Medial midstem pivot D. Pistoning: Cement within bone E. Pistoning: Stem within cement 48. You are reviewing the AP radiograph of a patient who has been referred with thigh pain 5 years following cemented total hip arthroplasty. A fracture is seen in the cement mantle just distal to the prosthesis.

43

James Gill and Majeed Shakokani

What mode of femoral stem loosening is this most likely to represent? A. Bending cantilever (fatigue) B. Calcar pivot C. Medial midstem pivot D. Pistoning: Stem within bone E. Pistoning: Stem within cement 49. Which of the following factors is not associated with increased risk of cemented stem fracture? A. Elongated femoral head B. Increased body mass C. Low neck cut D. Poor distal cement fixation E. Smaller stem size 50. Which factor does not predispose to an increased risk of prosthetic hip dislocation? A. Direct anterior approach B. Elevated BMI C. Parkinson’s disease D. Previous lumbar fusion E. Total hip arthroplasty for hip fracture 51. A 75-year-old patient develops degenerative hip arthritis 8 years following an intertrochanteric fracture treated with a dynamic hip screw. You are planning removal of all metalwork and cemented total hip arthroplasty. By how many femoral diameters should the stem bypass the most distal screw hole? A. 1.5 femoral diameters B. 2.5 femoral diameters C. 3.5 femoral diameters D. 4.5 femoral diameters E. 5.5 femoral diameters 52. Which of the following mechanical properties is not associated with the highly cross-linking of polyethylene? A. Increased fracture toughness B. Increased Young’s modulus C. Reduced linear wear D. Reduced ductility E. Smaller wear particles 53. Total hip arthroplasty is planned for a 40-yearold male with high developmental hip dysplasia (Crowe type IV).

44

What is the maximum amount of leg length correction that could be considered before proximal femoral osteotomy should be planned to reduce the risk of sciatic nerve palsy? A. 3cm B. 4cm C. 5cm D. 6cm E. 7cm 54. When assessing a patient 1 day following total hip arthroplasty, there is absence of sensation over the dorsum of the foot and an MRC grade 0 power of ankle dorsiflexion. Total hip arthroplasty was performed under spinal anaesthetic via a posterior approach and the nerve was not encountered at any stage during the operation. What is the most appropriate immediate management? A. Computed tomography to assess for haematoma or cement B. Foot drop splint and nerve conduction studies at 12 weeks if no clinical improvement C. Magnetic resonance imaging to assess for tethering of the nerve D. Plain film imaging E. Surgical exploration of the sciatic nerve 55. Which of the following factors increases primary arc range in total hip arthroplasty? A. Constrained liner B. Extended lipped liner C. Increased femoral neck length D. Increased offset E. Larger femoral head 56. Submicron particles generation results in osteolysis via the RANK ligand pathway. What type of wear is the most important process that generates submicron-sized particles in a metal-on-polyethylene total hip arthroplasty? A. Abrasive wear B. Adhesive wear C. Corrosive wear D. Mode 2 wear E. Mode 3 wear

Hip II Structured SBA

HIP II STRUCTURED SBA ANSWERS 1. Answer D. Type IV Type IV hypersensitivity is responsible for the hypersensitivity response to metallic orthopaedic implants. Type IV sensitivity is cell mediated opposed to antibody mediated, helper T cells activate cytotoxic cells and macrophages. Types I, II, III and V hypersensitivity are all antibody mediated. Currently, there are no guidelines for addressing suspected or known metal allergy preoperatively and there is no evidence-based support for either preoperative testing or routine use of hypoallergenic implants. Eftekhary N, Shepard N, Wiznia D, Iorio R, Long WJ, Vigdorchik J. Metal hypersensitivity in total joint arthroplasty. JBJS Rev. 2018 Dec;6 (12):e1. 2. Answer C. Neurofibromatosis Protrusio is not commonly associated with neurofibromatosis. Focal skeletal abnormalities associated with neurofibromatosis include scoliosis, congenital bowing of the tibia (anterior lateral) and forearm, pseudoarthrosis and limb hypertrophy. The other four systemic conditions are all associated with protrusio acetabuli. 3. Answer E. Simvastatin The radiograph demonstrates osteonecrosis (see Figure 4.1 in the Questions section). Simvastatin is not a cause of osteonecrosis and has been postulated to be protective against osteonecrosis. Glucocerebrosidase is the gene underlying Gaucher’s disease, which is a recognised cause of osteonecrosis. HbSS is the genotype of sickle cell disease, a recognised cause of osteonecrosis, and protein S deficiency is a clotting abnormality resulting in a hypercoagulable state. Immunological conditions such as SLE, Sjogren’s syndrome, dermatomyositis, rheumatoid arthritis and scleroderma have been associated with increased risk of osteonecrosis. These associations may be mediated by use of corticosteroids or other immunosuppressive drugs or may be related to the specific disease activity itself. Yang Z, Liu H, Li D, et al. The efficacy of statins in preventing glucocorticoid-related osteonecrosis in animal models a meta-analysis. Bone Jt Res. 2016;5:393-402.

4. Answer A. 25o flexion, 5o adduction, 5o external rotation Optimal position of flexion is a compromise between ease of standing and sitting. Any abduction of the hip results in coronal imbalance when standing, and therefore a few degrees of adduction is preferred to position the ankle joint beneath the centre of mass of the torso. External rotation is generally matched to the contralateral side. In a review article, Beaulé et al recommended flexion of 20–30°, adduction 5° and external rotation 5–10°. Beaulé PE, Matta JM, Mast JW. Hip arthrodesis: current indications and techniques. J Am Acad Orthop Surg. 2002;10:249-258. 5. Answer C. 60% In a retrospective study of 28 patients who had undergone hip arthrodesis, Callaghan et al showed that 60% of patients developed ipsilateral knee pain. The mean time to onset of knee pain was 23 years after hip arthrodesis. Back pain was equally common, whereas contralateral hip pain occurred in 25% of patients. Callaghan JJ, Brand RA, Pedersen DR. Hip arthrodesis. A long-term follow-up. J Bone Jt Surg. 1985;67:1328-1335. 6. Answer C. Reducing offset Reducing hip offset increases the force required by the abductors muscles to maintain a single leg stance. Medialisation of the cup reduces the moment arm of the body mass and therefore reduces the force required by the abductors to maintain a single leg stance. Walking with a stick in the contralateral hand reduces the abductor force. Trunk leaning to the ipsilateral side shifts the centre of mass over the hip joint which reduces the force required by the abductors to maintain single leg stance. Carrying a bag of shopping in the ipsilateral arm helps to balance the centre of mass and therefore reduces the force required by the abductors to maintain a single leg stance. 7. Answer C. 1, 2, 6 and 7 In a seminal orthopaedic paper, Gruen et al described both zones of failure of a cemented femoral total hip arthroplasty stem and five different modes of failure. Zone 1 describes the proximal lateral bone–cement–implant interface,

45

James Gill and Majeed Shakokani

zone 4 the tip and zone 7 the proximal medial aspect of the femur. The classical mechanism by which femoral stems fracture is due to loss of proximal support with a stem that it well fixed distally. The proximal medial and lateral cement breaks up, resulting in lucency in Gruen zones 1, 2, 6 and 7. This mode of failure is termed cantilever bending. Gruen TA, McNeice GM, Amstutz HC. Modes of failure of cemented stem-type femoral components: a radiographic analysis of loosening. Clin Orthop Relat Res. 1979;141:17–27. 8. Answer B. MARS MRI Metal artefact reduction sequence MRI or ultrasound are recommended by the Medicines and Healthcare products Regulatory Agency (MHRA) for imaging of metal-on-metal hip replacements. Ultrasonography is a good screening tool, is cheap and has no radiation hazard; however, the detection of small or deep lesions with ultrasound is difficult. Soft tissue inflammatory reactions to metal debris are a recognised complication of metal-on-metal resurfacing. These reactions are grouped under the umbrella term ‘adverse reactions to metal debris’ (ARMD). The spectrum of ARMD is extensive and ranges from small asymptomatic cysts to large soft tissue masses (pseudotumours). Inflammatory pseudotumour is the clinical term given to an aseptic mass in the periprosthetic tissues that is either solid or cystic and is associated with clinical, radiological or histopathological signs of inflammation. An aseptic-lymphocytic vasculitis-associated lesion (ALVAL) is a histological diagnosis that describes the unique cellular changes that occur periprosthetically in response to metal particles. Drummond J, Tran P, Fary C. Metal-onmetal hip arthroplasty: a review of adverse reactions and patient management. J Funct Biomater. 2015;6:486-499. 9. Answer D. Revise to a metal-lined ceramic head with change of polyethylene liner Ceramic head fracture is rare, especially with modern ceramics. The safest of the given options is to revise to a metal-lined ceramic head with a change of polyethylene. Metal-bearing surfaces should be avoided due to their reduced scratch resistance (hardness) compared to ceramicbearing surfaces. Revision of all implants is

46

unnecessary, provided all implants are in a satisfactory position and there has not been significant damage to the acetabular shell liner locking mechanism and the stem trunnion. Completed synovectomy is recommended during revision to remove as much of the ceramic debris as possible which can lead to accelerated third body wear. 10. Answer B. Boundary Boundary lubrication is the predominant mechanism by which lubrication occurs in prosthetic joints. Elastohydrodynamic, squeeze film, weeping and boosted lubrication occur in native synovial cartilaginous joints but not prosthetic joints. Boundary lubrication also occurs in native synovial cartilaginous joints. 11. Answer C. Failure of the repair of the short external rotators The short external rotators consist of piriformis, superior gemellus, obturator internus, inferior gemellus and quadratus femoris. The short external rotators are divided during the posterior approach to the hip. Repair of the short external rotators has been shown to reduce the risk of dislocation and so has become common practice. Failure of repair of the short external rotators can result in the foot turning inwards with walking. Excessive retroversion of the femoral stem may result in hip instability or an externally rotated lower leg. Weakness of the gluteus maximus results in weakness of hip extension. Excessive cup anteversion is most likely to result in anterior hip instability. Palsy of gluteus medius and minimus would result in weakness of hip abduction and a Trendelenburg gait. 12. Answer A. External iliac With profuse bleeding, it is likely that a major vessel has been injured. Of the major vessels (femoral, internal and external iliac), the most commonly injured is the external iliac artery. The anterior superior quadrant of the acetabulum is known as the ‘zone of death or danger zone’, as a screw or a drill that penetrates too far risks laceration of the external artery or vein. If a major vessel is injured during screw placement, the hip wound should be packed tightly. An anterior incision should be performed before closing the hip wound in order to gain proximal control of the bleeding artery.

Hip II Structured SBA

Shoenfeld NA, Stuchin SA, Pearl R, et al. The management of vascular injuries associated with total hip arthroplasty. J Vasc Surg. 1990;11:549-555.

unlike the anterior superior quadrant, the bone depth in the posterior superior quadrants allows screws 25mm or longer to be placed safely. Screws placed in the posterior inferior quadrant less than 20–25mm are safe; longer screws risk injury to the sciatic nerve, inferior gluteal nerve and vessels and internal pudendal nerve and vessels. Wasielewski RC, Cooperstein LA, Kruger MP, et al. Acetabular anatomy and the transacetabular fixation of screws in total hip arthroplasty. J Bone Joint Surg Am. 1990;72:501-508.

Figure 4.2 Acetabular screw position and risk of vascular injury

13. Answer E. Posterior superior: Posterior to a line drawn from the anterior superior iliac spine passing through the centre of the acetabulum and superior to a line drawn perpendicular to this also passing through the centre of the acetabulum The naming of the quadrants for acetabular screw placement is somewhat confusing (Figure 4.2). The quadrants are formed by drawing a line from the anterior superior iliac spine (ASIS) through the centre of the acetabulum, forming acetabular halves. The two halves anterior and posterior to this line are named accordingly and are used as the first word naming a quadrant. A second line is then drawn perpendicular to the first line at the midpoint of the acetabulum. The quadrants formed superior and inferior to this line denote the second word naming the quadrants. The anterior superior quadrant is anterior to a line drawn from the ASIS and superior to the line drawn perpendicular to this. The anterior quadrants (superior and inferior) should be avoided due to the lack of bone and proximity of the external iliac artery and vein and the obturator nerve, artery and vein. The posterior superior quadrant may be directed towards the sciatic nerve, superior gluteal nerve and vessels; however,

Figure 4.3 Safe acetabular quadrants for screw placement

14. Answer D. Hypotensive anaesthesia Hypotensive anaesthesia has been shown to reduce blood loss in primary and revision THA. In a case series of 100 patients who were Jehovah’s Witnesses undergoing THA, hypotensive anaesthesia was performed in 89 of them. Sixty-five patients underwent primary THA with an average of 450ml of blood loss; this was 43% less than a control group. Twentyfour patients who had had previous ipsilateral hip surgery underwent revision hip arthroplasty under hypotensive analgesia and sustained an average intraoperative blood loss of 680ml, which was 30% less than that of similar matched controls. The final 11 Jehovah’s Witnesses undergoing hip arthroplasty were not candidates for hypotensive anaesthesia. Other blood management techniques in patients who are Jehovah’s Witnesses include erythropoietin therapy, meticulous haemostasis, haemodilution and intraoperative blood salvage.

47

James Gill and Majeed Shakokani

Nelson CL, Bowen WS. Total hip arthroplasty in Jehovah’s Witnesses without blood transfusion. J Bone Joint Surg Am. 1986;68:350-353. Nelson CL, Stewart JG. Primary and revision total hip replacement in patients who are Jehovah’s Witnesses. Clin Orthop Relat Res. 1999;369:251-261. 15. Answer A. Antibiotic loaded cement The Medical Research Council (MRC) trial published by Lidwell et al showed that antibiotic loaded cemented was the greatest factor in reducing prosthetic joint infection. In decreasing order of effect, the other factors assessed were systemic antibiotics, ultraclean air, plastic isolators and body exhaust suits. Lidwell OM, Lowbury EJL, Whyte W, et al. Effect of ultraclean air in operating rooms on deep sepsis in the joint after total hip or knee replacement: a randomised study. Br Med J. 1982;285:10-14 16. Answer B. Nerve to obturator internus From superior to inferior, the short external rotators consist of piriformis supplied by the

nerve to piriformis, superior gemellus and obturator internus, which are both supplied by the nerve to obturator internus, inferior gemellus and quadratus femoris, which are both supplied by the nerve to quadratus femoris and obturator externus, which is supplied by the posterior branch of the obturator nerve (Figure 4.3). Particular attention should be paid to the nerve supply of the gemelli, as they are innervated by two different nerves, which are named after and also innervate the short external rotators immediately inferior to them. 17. Answer B. Nerve to quadratus femoris The nerve to quadratus femoris exits the greater sciatic foramen but does not re-enter the lesser sciatic foramen. Obturator internus exits the lesser sciatic foramen, not the greater sciatic foramen. The pudendal nerve and the nerve to obturator internus both exit the greater sciatic foramen and re-enter the lesser sciatic foramen. Obturator externus does not pass through either the lesser or greater sciatic foramen. Obturator externus originates from the ischiopubic ramus and the obturator membrane which spans the obturator foramen. 18. Answer D. Obturator internus Obturator internus exits rather than enters the pelvis via the lesser sciatic foramen. The pudendal nerve, internal pudendal vessels (artery and vein) and the nerve to obturator internus all enter the pelvis via the lesser sciatic foramen having first exited the pelvis via the greater sciatic foramen.

Figure 4.4 Anatomy of the short external rotators. GT = greater trochanter, PIRI = piriformis, SG = superior gemellus, OI = obturator internus, IG = inferior gemellus and QF = quadratus femoris

48

19. Answer C. Common peroneal division passing through piriformis and tibial nerve division exiting beneath piriformis Beaton and Anson performed a cadaveric study of the anatomical variants of the relationship of the sciatic nerve to the piriformis muscle. The most prevalent relationship is the sciatic nerve exiting the greater sciatic nerve beneath the piriformis muscle and occurs in about 84% of the population. The next most common variation is the sciatic nerve dividing in the pelvis with the common peroneal division passing through piriformis and the tibial division exiting beneath piriformis; this variant occurs in approximately 12% of the population (Figure 4.4).

Hip II Structured SBA

Figure 4.5 Sciatic nerve variations: a. Sciatic nerve exits beneath piriformis (84.2%) b. Peroneal nerve exits through piriformis and tibial nerve exits beneath piriformis (11.7%) c. Peroneal nerve exits above piriformis and tibial nerve exits beneath piriformis (3.3%) d. Sciatic nerve exits through piriformis (0.8%) e. Common peroneal exits above piriformis and tibial nerve exits through piriformis (hypothetical) f. Sciatic nerve exits above piriformis (hypothetical)

Beaton LE, Anson BJ. The relation of the sciatic nerve and of its subdivisions to the piriformis muscle. Anat Rec. 1937;70:1-5.

Peroneus longus is supplied by the superficial branch of the peroneal nerve, tibialis posterior and popliteus by the tibialis nerve.

20. Answer D. Short head of biceps femoris Weakness of ankle dorsiflexion and foot eversion represent a palsy of the superficial peroneal nerve. Intact ankle plantar flexion would suggest the tibial nerve is spared. The short head of biceps is the most proximal muscle to be supplied by the common peroneal nerve and therefore should be the first to be re-innervated. The long head of biceps is supplied by the tibial nerve. Two other muscles in the leg also have dual nerve supply: adductor magnus and pectineus. The adductor part of adductor magnus is supplied by the posterior division of the obturator nerve, whereas the hamstrings portion is supplied by the tibial nerve. The anterior fibres of the pectineus are supplied by the femoral nerve, whereas the posterior fibres are supplied by the anterior division of the obturator nerve.

21. Answer A. Bone scintigraphy Three-phase bone scintigraphy is the most sensitive imaging modality for early detection of heterotopic ossification. Plain film radiographs might not show heterotopic ossification until 1–4 weeks after it is visible on bone scintigraphy. Single-photon emission computed tomography (SPECT) improves the sensitivity and specificity of planar bone scintigraphy due to more accurate localisation of activity. Ghanem MA, Dannoon S, Elgazzar AH. The added value of SPECT-CT in the detection of heterotopic ossification on bone scintigraphy. Skeletal Radiol. 2020;49:291-298. 22. Answer B. Cotyloid fossa, superior aspect of the obturator foramen and the cortical surface of the true pelvis

49

James Gill and Majeed Shakokani

The pelvic teardrop is a radiographic landmark seen in the anteroposterior view of the pelvis. The teardrop is formed by a continuous U-shaped surface of bone, the lateral border of which is made up of the cortical surface of the cotyloid fossa (true floor of the acetabulum). This surface of bone continues through the acetabular notch and curves inferior medially beneath the transverse acetabular ligament through the superior margin of the obturator foramen. This forms the inferior bend of the U of the teardrop. The medial aspect of the teardrop is made of the cortical surface in the true pelvis. The bony structures that form the pelvic teardrop were confirmed in a radiographic study in which a strip of lead foil was applied to the true floor of the acetabulum, through the obturator foramen and onto the cortical surface of the true pelvis. The teardrop is a frequently used landmark in total hip arthroplasty. When templating for a THA, the inferior part of the cup should be level with the bottom of the teardrop. Vare VB. The anatomy of the pelvic tear figure. J Bone Joint Surg Am. 1952;34-A:167-169. 23. Answer A. Ankylosis spondylitis Ankylosis spondylitis is associated with increased acetabular anteversion. Slipped upper femoral epiphysis (SUFE), Legg–Calve–Perthes disease and pincer femoral acetabular impingement (FAI) are characteristically associated with acetabular retroversion. Rheumatoid arthritis is associated with acetabular protrusio. The increased acetabular anteversion associated with ankylosing spondylitis predisposes patients with ankylosing spondylitis who are undergoing total hip arthroplasty to anterior dislocation. Patients with ankylosing spondylitis compensate for fixed kyphosis of the spine with pelvic extension, which leads to increased acetabular anteversion. Pelvic extension and knee flexion allow a patient with fixed kyphosis of the spine (positive sagittal balance) to bring the head back over the pelvis. Addressing spinal deformity in patients with ankylosing spondylitis prior to total hip arthroplasty has been advocated. Direito-Santos B, França G, Nunes J, et al. Acetabular retroversion. EFORT Open Rev. 2018;3:595-603. 24. Answer D. Femoral head medial to ilioischial line

50

The definition of protrusio is the femoral head protruding medial to the ilioischial line. The iliopectineal line is medial to the ilioischial line, so if the femoral head protrudes medial to the iliopectineal line there will be acetabular protrusion. However, this is not the defining threshold for protrusio. An acetabular fossa medial to the ilioischial line defines coxa profunda (deep acetabular socket). Other definitions for acetabular protrusio have been proposed:  Centre-edge angle greater than 40°and medialisation of the medial wall of the acetabulum protruding past the ilioischial line  Acetabular fossa greater than 3mm beyond the ilioischial line in men and greater than 6mm in women. 25. Answer E. Superior lateral femoral neck stress fracture involving 25% of neck width All tension side (superior femoral neck) stress fractures require internal fixation. Compression (inferior femoral neck) stress fractures involving more than 50% width of the femoral neck also need internal fixation. The main stay of treatment for piriformis syndrome and iliotibial band syndrome is physical therapy. Symptomatic cam lesions may be amenable to surgical intervention when conservative measures have been exhausted. 26. Answer D. Material that exhibits linear stress stain relationship until the point of failure This describes a material that is brittle. Ceramic bearing surfaces fracture because they are brittle and have low toughness and poor fracture resistance. The area under the stress–strain curve describes a materials toughness. Ductile materials exhibit non-linear change in length (plastic deformation) beyond the elastic limit on a stress– strain curve. Failure at a point below the ultimate tensile strength secondary to repetitive loading is termed ‘fatigue failure’. Progressive deformation in response to a constant force over a prolonged period of time is the definition of ‘creep’. 27. Answer A. Early prosthetic loosening Early prosthetic loosening is associated with sickle cell disease; the mechanism is extended bone infarct disease. Psoriasis is associated with higher

Hip II Structured SBA

periprosthetic infection rate. Paget’s disease is associated with increased blood loss, and ankylosing spondylitis is associated with a higher risk of heterotopic ossification. 28. Answer A. Iliofemoral The iliofemoral ligament, also known as the Yligament of Bigelow, is the strongest ligament in the body. It originates from the anterior inferior iliac spine and then fans out in an inverted Y shape to attach along the anterior intertrochanteric line of the femur (Figure 4.5). The ligament is a static stabiliser and checks hip extension. Because the ligament limits hip extension, it allows maintenance of an upright posture with slight hip extension. With reduced need for muscle contractions, the ligament is therefore energy conserving. Figure 4.6 Iliofemoral ligament

29. Answer B. Cup inclination 40o, cup anteversion 15o, femoral stem anteversion 15o In a classic paper, Lewinnek et al proposed a relative ‘safety zone’ for the acetabular cup position of 40+/–10o inclination and 15+/–10o anteversion. The study has a number of limitations; it was a case series of 300 total hip arthroplasties but there were only 9 dislocations and there was no mention of femoral stem anteversion. Cup version cannot be considered in isolation; the relationship between cup and stem version is described by the term ‘combined anteversion’. While all surgeons accept the importance of implant orientation, there are numerous other factors that are important for hip stability, which

are commonly grouped into surgical factors (implant position, surgical approach, surgeon experience/volume), implant factors (head size, head–neck ratio, type of cup/acetabular liner) and patient factors (muscle weakness, soft tissue quality). Lewinnek GE, Lewis JL, Tarr R, Compere CL, Zimmerman JR. Dislocations after total hipreplacement arthroplasties. J Bone Jt Surg Am. 1978;60(2):217-220. 30. Answer B. 35o The combined anteversion is the sum total of acetabular version and femoral anteversion. Combined anteversion can be assessed on the operating table once components have been implanted and before the joint is closed by internally rotating the femur with the knee flexed until the cup and base of the femoral head are co-planar; the angle made between the lower leg and the floor is the combined anteversion. The optimal combined version is debated. Dorr et al believe there is a wide safe zone and recommended aiming for 25–45o with a mean of 35o. Dorr LD, Malik A, Dastane M, Zhinian W. Combined anteversion technique for total hip arthroplasty. Clin Orthop Rel Res. 2009 Jan;467 (1):119-127 31. Answer C. Gluteus medius and minimus deficiency A constrained polyethylene liner should be reserved for recurrent desolators with soft tissue dysfunction. A constrained liner encircles the femoral head and mechanically prevents the head from displacing out of the socket. Constrained liners increase the stress at the bone–implant interface, which increases the risk of early loosening. In all of the other options listed, suboptimal implant position or design could be corrected with revision of implants. 32. Answer D. 5cm Numerous sources report that the superior gluteal nerve may be damaged if the gluteus medius is split more than 5cm proximal to the greater trochanter. However, it may lie even closer. In a cadaveric study of 44 hips, the superior gluteal nerve was found to be a mean of 4.8cm (range

51

James Gill and Majeed Shakokani

2 to 9cm) from the greater trochanter. Ramesh et al reported 11% risk of superior gluteal nerve denervation following a series of Hardinge approaches in which the gluteus medius was not splint more than 4cm from the greater trochanter. Khan T, Knowles D. Damage to the superior gluteal nerve during the direct lateral approach to the hip. A Cadaveric Study. J Arthroplasty. 2007;22:1198-1200. Ramesh M, O’Byrne J, McCarthy N, Jarvis A, Mahalingham K, Cashman WF. Damage to the superior gluteal nerve after the Hardinge approach to the hip. J Bone Surg Br. 1996 Nov;78(6):903-906. 33. Answer B. Lateral to the primary compressive trabeculae and medial to the secondary compressive trabeculae Ward’s triangle is a space formed near the centre of the femoral neck by the intersection of three trabecular bundles, namely, the principal compressive, the secondary compressive and the tensile trabeculae (Figure 4.6). This central region, containing some thin and loosely arranged trabeculae, defines a neutral axis where tensile and compressive forces balance each other. The three boundaries of Ward’s triangle are medially the primary compressive trabeculae, laterally the secondary compressive trabeculae and superiorly the primary tensile trabeculae. Ward’s triangle itself is not a sign of osteoporosis; however, expansion of Ward’s triangle, which can be visualised on a plain film AP hip radiograph, is due to loss of trabeculae. 34. Answer A. Ascending branch of the lateral femoral circumflex artery The ascending branch of the lateral femoral circumflex artery crosses the interval proximally between sartorius (femoral nerve) and tensor fascia lata (superior gluteal nerve). The ascending branch of the lateral femoral circumflex artery should be identified and ligated to prevent excessive bleeding. In 1919 SmithPetersen first described a direct anterior approach (DAA) to the hip for reducing congenital hip dislocations. Smith-Petersen is also credited with the first DAA for hip arthroplasty in 1949. Over the subsequent decades, several

52

modifications to his technique have occurred, along with the development of new instruments to make it less invasive and easier to perform for THA. 35. Answer B. 0.5µm Particulates in the range of 0.1–1.0μm (submicron) are biologically active, with those in the range of 0.1–0.5μm thought to be the most biologically active and responsible for osteolysis. Macrophages phagocytose polyethylene particles, as they are a similar size to bacteria. Activated macrophages release cytokines, which stimulate osteoblasts to release RANK ligand. This in turn leads to activation of osteoclasts, which resorb bone. The cytokines released by activated macrophages are TNFα, IL-1, IL-6, PGE2 and PDGF. Activated macrophages also directly absorb bone via the release of matrix metalloproteinases (MMPs) and upregulate osteoclast differentiation via macrophage-colony stimulating factor (MCSF) (Figure 4.7). 36. Answer E. Total hip arthroplasty A subchondral lucent line describes the crescent sign. The crescent sign features in both the modified Ficat and Steinberg staging systems for adult hip osteonecrosis. Presence of the crescent sign indicates imminent femoral head collapse. Joint preserving procedures such as core decompression, osteotomy and vascularised bone grafting are limited to pre-collapse femoral heads, whereas joint replacement procedures are indicated in the presence of femoral head collapse. 37. Answer C. Osteoprotegerin Osteoprotegerin is a decoy receptor of the receptor activator of nuclear factor kappa-B ligand (RANKL). Osteoprotegerin binds RANKL to limit its activity. Platelet-derived growth factor, interleukin-1 and interleukin-6 are involved in signalling from macrophages to activate osteoclasts to resorb bone. Osteoclasts are activated indirectly via osteoblasts and the RANKL pathway. 38. Answer D. Uncemented, fully porous coated cobalt-chromium stem A number of factors contribute to stress shielding: stem fixation; cemented composite

Hip II Structured SBA

Figure 4.7 Ward’s triangle

Figure 4.8 A series of proinflammatory factors, including IL-1, IL-6, PGE2, TNF-α, can be produced by wear particle-activated macrophages. These cytokines can induce the expression of RANKL, which activates osteoclasts. Macrophages also release matrix metalloproteinases (MMPs) and downregulate production of tissue inhibitors of metal metalloproteinases (TIMPs).

beam versus taper slip, uncemented fully porous coated versus proximally porous coated, Young’s modulus of the cement stem and radius of the stem (Figure 4.8). With taper slip stem design,

viscoelastic properties of bone cement allow slip of the stem in the cement mantle as it is loaded axially. Axial forces are converted into hoop stresses throughout the length of the taper;

53

James Gill and Majeed Shakokani

therefore, provided the cement mantle is not excessively thick, bone is loaded along the length of the implant. Composite beam cemented stem designs are so named because the stem, cement and bone behave as one (a composite) construct. With composite beam stem designs, forces are concentrated at the distal tip of the stem opposed to throughout the proximal femur with taper slip designs. Composite beam designs therefore lead to more proximal bone loss than do taper slip designs. Calcar flanges were added to composite beam designs in an attempt to load the bony calcar and prevent proximal stress shielding. The loading of the femur is similar in uncemented stems to composite beam cemented stems. Proximal stress shielding is reduced in proximally porous coated stems opposed to fully porous coated stems. Increasing stem radius increases the stiffness and therefore increases stress shielding. The following commonly used metals for femoral stems are listed in decreasing order of stiffness: cobalt-chrome, stainless steel and titanium. Answer D is correct as it combines the greatest number of factors which contribute to stress shielding. Figure 4.9 Anteroposterior (AP) radiograph of right reverse hybrid THA, uncemented fully porous coated stem, with significant proximal femur bone loss secondary to stress shielding and osteolysis

39. Answer C. Adductor longus and gracilis There is no internervous plane for the medial approach to the hip. The superficial intermuscular plane is between adductor longus and gracilis, both of which are innervated by the anterior division of the obturator nerve. The deep intermuscular plane is between adductor brevis

54

supplied by the anterior division of the obturator nerve and adductor magnus, which has dual innervation. The adductor portion is supplied by the posterior division of the obturator nerve, and the hamstrings portion is supplied by the tibial portion of the sciatic nerve. 40. Answer A. Adductor brevis – posterior division of obturator nerve Gracilis is supplied by the anterior division of the obturator nerve. Two muscles around the hip and one muscle in the thigh have dual innervation: adductor magnus, pectineus and biceps femoris. Adductor magnus is supplied by the posterior division of the obturator nerve and the tibial nerve. Pectineus is supplied by the femoral and the obturator nerve. Biceps femoris is supplied by the tibial nerve (long head) and peroneal nerve (short head). Adductor longus is supplied by the anterior division of the obturator nerve. 41. Answer B. Iliopsoas The medial and lateral circumflex arteries are named according to their relationship to the iliopsoas tendon (Figure 4.9). The medial femoral circumflex artery (MFCA) is a branch of the profundal femoris; it winds around the medial side of the femur, passing between pectineus and iliopsoas (medial to it). The femoral head receives its blood supply primarily from the deep branch of the MFCA. The medial and lateral femoral circumflex arteries contribute to an extracapsular arterial ring at the base of the femoral neck. The extracapsular arterial ring gives rise to the retinacular vessels, which run superiorly along the femoral neck until they reach the cartilaginous border of the femoral head, at which point they penetrate the femoral head. The lateral femoral circumflex artery is also usually a branch of the profundal femoris artery and passes lateral to the iliopsoas tendon. The calcar femorale refers to the vertical plate of dense cancellous bone that develops in the posterior femoral neck separated from the lesser trochanter. It is not to be confused with the calcar, which is the lowermost and thickest point of the cortex constituting the medial wall of the femoral neck.

Hip II Structured SBA

Hammer A. The calcar femorale: a new perspective. J Orthop Surg (Hong Kong). 2019 MayAug;27(2):2309499019848778.

Figure 4.10 Femoral neck blood supply

42. Answer B. Flexible femoral stem Flexible femoral stems are not optimal for stem fixation, as they place increased stress on the cement mantle; hence, most femoral stems are made from stainless steel (316L) or cobaltchrome. A cement mantle >2mm reduces the risk of cement mantle fracture. Stem centralisation using a plastic tip centraliser aims to produce an even cement mantle around the femoral stem, which decreases stress on the cement mantle and improves fixation. The centraliser is hollow to allow the stem to subsidise by leaving a space for the tip of the stem to subsidise into. Prior to the use of hollow stem centralisers, ‘punch out’ fractures were observed in the distal portion of the cement mantle. Highly polished femoral stems reduce shear stresses at the cement–stem interface and allow subsidence, which converts axial load into radial hoop stresses. Matt finish taper slip stems are thought to have failed early due to abrasive wear at the cement–stem interface. Vacuum preparation of cement is a feature of third generation cementation and reduces the porosity of the cement, which reduces stress points in the cement and increases the strength of stem fixation. 43. Answer B. The capsule attaches more distally on the neck posteriorly compared to anteriorly

The capsule attaches more distally on the femoral neck anteriorly compared to posteriorly. Posteriorly, the capsule attaches more proximally. Hence, when performing arthroplasty for subcapital femoral neck fracture, there are often fragments of the head attached to the capsule posteriorly once the head has been removed. The gluteus minimus inserts into the anterior capsule and greater trochanter. The reflected head of the rectus femoris originates from the anterior hip joint capsule, whereas the direct head originates from the anterior inferior iliac spine. The ischiofemoral ligament forms the posterior hip joint capsule and is divided when performing the Southern–Moore (posterior) approach to the hip joint. The iliofemoral ligament forms the anterior hip joint capsule and is divided when entering the hip joint using the Smith-Petersen (anterior) approach. The iliofemoral ligament and the direct head of the rectus femoris tendon share an attachment to the ilium just superior to the acetabulum. 44. Answer B. L2 Radicular pain from nerve roots L1 and L2 may mimic referred hip pain in the groin. L2 radiculopathy may mimic referred hip pain in the thigh. Impingement of the lateral cutaneous nerve of the thigh causes numbness and burning pain over the lateral aspect of the thigh, the condition is known as meralgia paresthetica. Causes of compression include tight belts or irritation from seat belts worn during prolonged driving, pregnancy and obesity. The posterior cutaneous nerve of the thigh is rarely a cause of complaint due to impingement. It arises from the S1 to S3 nerve roots, so sensation may be diminished as part of cauda equina syndrome. The obturator nerve supplies sensation to the medial thigh and groin. The femoral nerve supplies sensation to the anterior medial thigh extending distal to the area supplied by the obturator nerve. 45. Answer A. Anterior intrasubstance labral tear and a posterior acetabular cartilage lesion A pincer-type lesion is due to overcoverage of the acetabulum and is common in middle-aged females. This results in abnormal contact between the acetabular rim and the femoral

55

James Gill and Majeed Shakokani

head–neck junction. The anterior superior femoral head–neck junction is levered against the acetabular rim and a contrecoup cartilage lesion may occur on the posterior inferior acetabulum. Protrusio and acetabular retroversion are causes of acetabular overcoverage. Acetabular retroversion may be seen on a plain film AP radiograph of the pelvis; the anterior wall may be seen crossing lateral to the posterior wall. This is called the crossover sign. The other common pattern of femoral acetabular impingement is termed cam impingement. Cam impingement is most common in young males and is caused by a non-spherical femoral head and decreased head–neck offset. In hip flexion the aspherical head engages with acetabular cartilage, causing a shearing force that results in delamination of the anterior acetabular cartilage and avulsion of the anterior labrum. The radiographic appearance of a non-spherical femoral head that may cause cam-type impingement is described as a ‘pistol grip deformity’. Imam S, Khanduja V. Current concepts in the diagnosis and management of femoroacetabular impingement. Int Orthop. 2011;35:14271435. 46. Answer B. 2.3mm, 3.3mm If the femoral neck angle is 125o this amounts to a 35o angle from the horizontal. Therefore, any further increase in the neck length will result in a smaller increase in the leg length compared to offset. When considering the addition of a ‘plus 4mm’ head, answer E (4.0mm, 0mm) is incorrect, as this change in leg length to offset could only be achieved with a neck angle of 180o. Likewise, answer A is incorrect (0mm, 4.0mm), as this change in leg length to offset could only be achieved with a neck angle of 90o. Answer D (3.3, 2.3mm) is incorrect, as this leg length to offset change would be the result a neck angle of 145o. Answer C (2.8, 2.8mm) is incorrect, as an equal increase in leg length and offset could only be achieve if a neck angle of 135o (45o to the horizontal) is used. The key to answering this question correctly is not a thorough understanding of trigonometry but rather a knowledge that if the neck angle is 135o, increasing neck length will increase leg length and offset in an equal ratio, and that if the neck angle is less than

56

135o, increasing neck length will increase leg length less relative to offset. 47. Answer B. Calcar pivot In a seminal orthopaedic paper, Gruen described both zones of failure of a cemented femoral total hip replacement stem and five different modes of failure. Zone 1 describes the proximal lateral bone–cement–implant interface, zone 4 the tip and zone 7 the proximal medial aspect of the femur. With calcar pivot mechanism of failure, the stem pivots on the calcar and toggles (windscreen wiper effect). Although Gruen’s original paper described loosening of cemented femoral stems, the calcar pivot mechanism is more common with uncemented stems with a large collar like the uncemented Austin Moore prosthesis. Gruen TA, McNeice GM, Amstutz HC. Modes of failure of cemented stem-type femoral components: a radiographic analysis of loosening. Clin Orthop Relat Res. 1979;141:17-27. 48. Answer E. Pistoning: Stem within cement A distal cement fracture and a radiolucent line between the stem and cement mantle in Gruen zones 1 and 2 are classical of pistoning of the stem within the cement (mode Ia). Mode Ib pistoning: stem within bone results in radiolucency in all Gruen zones. Gruen TA, McNeice GM, Amstutz HC. Modes of failure of cemented stem-type femoral components: a radiographic analysis of loosening. Clin Orthop Relat Res. 1979;141:17-27. 49. Answer D. Poor distal cement fixation Cemented femoral stem fracture is a rare complication. Poor proximal stem fixation or a low neck cut reduces the support for the stem proximally. If the stem remains well fixed distally, the increased moment arm due to loss of proximal medial support can result in catastrophic stem fatigue failure. This was described by Gruen as mode IV cantilever bending with lucency around the proximal zones (1, 2, 6 and 7). Increased body mass, elongated femoral heads and smaller stem sizes are all risk factors associated with stem fracture.

Hip II Structured SBA

Gruen TA, McNeice GM, Amstutz HC. Modes of failure of cemented stem-type femoral components: a radiographic analysis of loosening. Clin Orthop Relat Res. 1979;141:17-27. 50. Answer A. Direct anterior approach The direct anterior approach (DAA) has been shown to have a lower dislocation rate following total hip arthroplasty. Elevated BMI increases the risk of dislocation. Neurological disorders including Parkinson’s disease increase the risk of dislocation. There is an increased risk of dislocation following total hip arthroplasty for hip fracture. Proposed theories for this include predisposition to falls, capsular laxity compared to osteoarthritis and the technical challenge of restoring leg length and offset in the context of hip fracture. A history of lumbar spine fusion has been shown to increase the risk of hip dislocation. Kunutsor SK, Barrett MC, Beswick AD, et al. Risk factors for dislocation after primary total hip replacement: a systematic review and meta-analysis of 125 studies involving approximately five million hip replacements. Lancet Rheumatol. 2019;1:e111-121. Buckland AJ, Puvanesarajah V, Vigdorchik J, et al. Dislocation of a primary total hip arthroplasty is more common in patients with a lumbar spinal fusion. Bone Joint J. 2017;99B:585-591. 51. Answer B. Removal of all metalwork and cemented total hip arthroplasty bypassing the most distal screw hole by at least 2.5 femoral diameters The cemented stem must pass well below the distal screw hole to reduce the risk of periprosthetic fracture due to a stress riser. In a biomechanical study, Panjabi et al showed that normal stress patterns return at a distance of two times the cylinder (femur) diameter away from the stress riser. Panjabi MM, Trumble T, Hult JE, et al. Effect of femoral stem length on stress raisers associated with revision hip arthroplasty. J Orthop Res. 1985;3:447-455. 52. Answer A. Increased fracture toughness Highly cross-linked polyethylene (HXLPE) has a lower fracture toughness, decreased tensile

strength and decreased fatigue strength compared to standard cross-linked polyethylene. HXLPE has the theoretical advantage when used in total hip arthroplasty of increased wear resistance. In total knee arthroplasty, fracture and fatigue resistance are also important, and so the benefits of HXPLE are not thought to apply to total knee arthroplasty. 53. Answer B. 4cm Leg lengthening of more than 4cm is generally accepted to significantly increase the risk of sciatic nerve injury. Edwards et al were the first to report the increased risk of sciatic nerve injury associated with leg lengthening of more than 4cm. In a case series of 23 sciatic and peroneal nerve injury palsies, they showed sciatic nerve palsy occurred following mean lengthening of 4.4cm (range 4 – 5.1cm, n=3) and peroneal palsy occurred with a mean lengthening of 2.7cm (range 1.9 – 3.7cm, n=12). Edwards BN, Tullos HS, Noble PC. Contributory factors and etiology of sciatic nerve palsy in total hip arthroplasty. Clin Orthop Relat Res. 1987 May;218:136-141. 54. Answer D. Plain film imaging If the sciatic nerve has not been identified during the operation, then surgical exploration is mandated, as it could have been injured or accidentally tethered with a suture. Plain film imaging of the hip is sensible before surgical exploration to assess for leg lengthening, retained cement in the vicinity of the nerve or a prominent acetabular screw in the posterior inferior quadrant of the acetabulum. If the sciatic nerve was identified, protected throughout surgery and checked before closing the fascia lata, then the surgeon may wish to perform three-dimensional imaging to rule out a haematoma or cement compressing the sciatic nerve. If imaging is negative for occlusive pathology, one may wish still to explore the sciatic nerve or observe the palsy for improvement. 55. Answer E. Larger femoral head The primary arc range is the range of motion which components allow between the two extremes of impingement before the head begins to lever out of the cup. The range of motion allowed before the hip dislocates is termed the

57

James Gill and Majeed Shakokani

lever range. The jump distance or excursion distance is the distance the femoral head must translate out of the cup in order to dislocate. In a hemispherical cup, the excursion distance is approximately equal to the radius of the femoral head. Impingement occurs between the acetabular cup and the femoral neck and is dependent upon cup geometry, head size and neck size. The larger the head size is compared to the neck (head–neck ratio), the greater the primary arc range and stability will be. Increased offset and increased femoral neck length have no effect on the primary arc range. Extended lipped liners and constrained liners reduce primary arc range. When performing total hip arthroplasty, one should aim to centre the primary arc range within the patient’s functional hip range. 56. Answer B. Adhesive wear Adhesive bearing wear is the most important process that generates submicron-sized polyethylene (PE) particles. Adhesive wear occurs when the atomic forces between two opposing surfaces are stronger than the inherent strength of either material. In a metal-on-PE THA bearing surface, adhesive wear results in small portions of the PE surface adhering and transferring to the opposing metal femoral head. This leads to wear particle generation and the creation of pits and voids in the PE. Abrasive wear occurs when a soft material comes into contact with a hard material and the microscopic counter-face asperities of the harder material surface plough into the soft surface. Corrosive wear is defined as the unwanted dissolution of a metal in a solution. Four modes of wear have been described (Table 4.1). In mode 1, the two bearing surfaces are in contact with each other in the manner intended by the designer. In modes 2, 3 and 4,

58

Table 4.1 Mathematical analogues of the four wear modes

Surfaces in contact

Mathematical analogue

Mode number

Primary vs. primary

1 x 1 =1

1

Primary vs. secondary

1x2=2

2

Primary vs. primary vs. third bodies

1x1x3=3

3

Secondary vs. secondary

2x2=4

4

unintended surfaces are in contact with each other, representing malfunctioning of the prosthesis. In mode 2 wear, a bearing surface is wearing against a non-bearing surface, e.g. when the femoral head wears completely through the PE liner and contacts the metal shell or cement. In mode 3 wear, the primary bearing surfaces are still articulating with each other but with the addition of an interposed third body, e.g. cement, bone, metal or ceramic fragments. In mode 4 wear, two non-bearing surfaces are moving against each other, e.g. femoral neck and socket or ‘backside’ wear between an acetabular liner and shell or fretting between the metal stem and the surrounding bone or cement mantle. In Table 4.1, a primary bearing surface is assigned the number l; a secondary, non-bearing surface is assigned a 2, and third body particles are assigned a 3. Multiplication is the mathematical analogue for two or more surfaces in moving contact to cause wear. McKellop HA. The lexicon of polyethylene wear in artificial joints. Biomaterials 2007;28:5049-5057.

Section 2 Chapter

5

Adult Elective Orthopaedics and Spine

Knee Structured I SBA Kiran K. Singisetti

KNEE I STRUCTURED SBA QUESTIONS

D. If you are using posterior referencing, a size 4 femoral component may cause femoral notching E. If you are using anterior referencing, a size 5 femoral component may cause tightness of patellofemoral joint

1. A 65-year-old man presents to the clinic after initial improvement of knee pain with a knee replacement done 18 months ago. He now reports recent onset of patellar crepitus and jumping of patella. Radiographs show no change in position of the posterior stabilised knee replacement with patella resurfacing. This presentation could be related to which of the following conditions? A. Hypertrophic scar tissue at the inferior pole of the patella impinges on the femoral component during extension B. Hypertrophic scar tissue at the inferior pole of the patella impinges on the femoral component during flexion C. Hypertrophic scar tissue at the superior pole of the patella impinges on the femoral component during extension D. Hypertrophic scar tissue at the superior pole of the patella impinges on the femoral component during flexion E. Patellar component loosening

3. Component alignment and balancing form a key step in total knee arthroplasty. Which of the following is a correct guide to optimise tibial component rotation? A. Junction of the lateral and medial two-thirds of the tibial tubercle B. Achieving optimal posterior medial and anterior lateral cortical contact C. Extra-medullary alignment rod overlaying the lateral–middle third junctions of the tibial tubercle, the centre of the ankle mortise and second ray of a neutrally aligned foot D. Minimal anterior lateral tibial plateau uncoverage E. Trial reduction with absolute congruence of the femoral component and the anterior edge of PE tibial bearing surface

2. During a cruciate retaining knee replacement, the femoral size sagittal measurement comes up in between sizes 4 and 5. Which of the following statements is correct? A. If you are using anterior referencing, a size 5 femoral component may cause flexion instability B. If you are using posterior referencing, a size 5 femoral component may cause flexion instability C. If you are using anterior referencing, a size 4 femoral component may cause femoral notching

4. A patient attends the arthroplasty knee clinic 2 years following right total knee replacement. In the past year, she has complained of a sense of distrusting the knee without giving way, difficulty with stair descent, recurrent effusions and anterior knee pain. Possible causes for her symptoms could include which of the following? A. Inadequate restoration of the tibial slope B. Oversizing of the femoral component C. Over-resection of the distal femur D. Too little posterior femoral condylar resection E. Use of an anterior referencing knee system

59

Kiran K. Singisetti

5. A 27-year-old female felt a pop during a tackle while playing football. Which of the following ligament component deficiency causes a positive Pivot shift test? A. Anterolateral bundle of ACL B. Anteromedial bundle of ACL C. Posterolateral bundle of ACL D. Posteromedial bundle of ACL E. Posteromedial bundle of PCL 6. A patient with a previous ORIF of a lateral tibial plateau is seen in clinic and listed for a total knee replacement. Regarding the surgical approach for knee replacement, which of the following statements is correct? A. Avoid previous incision scars B. Dissect superficial to the deep fascia C. If longitudinal scars exist, choose the medialmost incision that affords appropriate exposure D. Respect the lateral-based vascular anatomy of the skin E. Transverse scars may be crossed with an incision in a perpendicular manner 7. A 70-year-old woman with tricompartmental knee osteoarthritis is scheduled to have a total knee replacement.

(a)

(b)

Figure 5.1 (a) Anteroposterior (AP) and (b) lateral radiographs knee

60

Which of the following intraoperative steps is not useful in improving patellofemoral tracking during total knee arthroplasty? A. External rotation of the femoral component B. External rotation of the tibial component C. Joint line preservation D. Lateralisation of the femoral component E. Lateralisation of the patellar component 8. A 56-year-old woman presents with a spontaneous onset of pain in the medial aspect of knee joint. Which of the following is a false statement regarding spontaneous osteonecrosis of the knee (SONK)? A. Bone scan has a low specificity and sensitivity in diagnosis of the condition B. Core decompression is a useful treatment following subchondral collapse C. Mainstay of initial treatment is non-operative D. It is more common in females E. MRI is useful in early diagnosis of the condition 9. A retired manual labourer presents with gradual worsening of knee pain with a previous history of high tibial osteotomy (HTO) performed 15 years earlier (Figure 5.1). Radiographs show progressive

Knee I Structured SBA

A. Lateral parapatellar approach is useful in fixed valgus deformity of knee B. Midvastus approach is relatively contraindicated in obese patients C. Midvastus approach may potentially achieve earlier rehab D. Minimally invasive surgical (MIS) approach has no long-term functional advantage E. Subvastus approach is an extensile approach

arthritis of the knee joint and you counsel patient about a total knee arthroplasty (TKA). Which of the following is a likely issue that you may encounter during TKA following previous HTO? A. Bipartite patella B. Lateral patella instability C. Patella baja D. Patella fracture E. Patella osteonecrosis 10. A patient with medial compartment knee arthritis attends your clinic. He is keen to consider a medial (partial) unicompartmental arthroplasty on the right knee. His radiographs are shown in Figure 5.2. Which of the following is not a contraindication for this procedure? A. ACL deficiency B. Anterior knee pain C. Fixed flexion deformity D. Inflammatory arthritis E. Uncorrectable varus deformity of more than 15 degrees 11. Which of the following statements is false regarding the surgical approach for primary total arthroplasty in a 62-year-old patient with knee arthritis?

(a)

12. Your colleague encourages you to use a knee replacement prosthesis that has the option of using an ‘all-polyethylene tibia component’. It would be prudent to look at the cost, advantage and disadvantages of the implant before considering a change of practice. Which of the following is false regarding an ‘allpolyethylene tibia component’? A. Better functional outcome B. Better stability C. Less modularity D. Less osteolysis E. More expensive 13. A patient reports continuing instability following a previous single bundle ACL reconstruction using hamstring graft. MRI suggests intact graft. Which of the following statements is correct regarding tunnel malposition?

(b)

Figure 5.2 (a) Anteroposterior (AP) and (b) lateral radiographs right knees

61

Kiran K. Singisetti

A. Too anterior femoral tunnel limits flexion of knee B. Too anterior femoral tunnel causes rotational instability C. Too anterior tibial tunnel limits flexion of knee D. Vertical femoral tunnel can risk femoral tunnel blowout E. Vertical inclination of femoral tunnel is associated with rotational instability 14. A 20-year-old elite football player sustains an anterior cruciate ligament (ACL) rupture reports instability symptoms despite a focussed exercise plan. He attends a specialist knee clinic. He wants to know the best graft option for a potential ACL reconstruction that would allow early return to sports. Which of the following graft options would you consider in this situation? A. Bone–patella–tendon–bone autograft B. Hamstring (four strand) autograft C. Quadriceps tendon autograft D. Synthetic graft E. Tendo-Achilles allograft 15. While performing a total knee arthroplasty, you are faced with a situation of normal extension gap but tight flexion gap. Which of the following solutions can be useful in this situation for sagittal balancing of the knee? A. Cut more proximal tibia B. Decrease the size of the femoral component C. Use distal femoral augmentation D. Use a thicker polyethylene insert E. Use a thinner polyethylene insert 16. A 24-year-old male walks into your clinic with a varus thrust at the right knee. He reports right knee pain and instability symptoms following a motorbike accident sustained 3 weeks ago. Dials test reveals increased external rotation of the right foot at 30 degrees flexion but not at 90 degrees flexion of knee. Which of the following is the most likely ligament injury? A. Isolated ACL injury B. Isolated MCL injury

62

C. Isolated posterolateral corner injury D. Isolated PCL injury E. Posterolateral corner and PCL injury 17. A 22-year-old female office worker had a previous arthroscopic total medial meniscectomy at a different hospital. Which of the following situations is a not a contraindication for meniscal transplantation? A. Asymptomatic meniscal loss B. Chondral changes C. Inflammatory arthropathy D. Instability E. Malalignment 18. A 21-year-old woman presents with recurrent instability of patella following an injury sustained 3 years ago. How is the isometric point for the femoral tunnel referenced from adductor tubercle when considering a medial patellofemoral ligament (MPFL) reconstruction? A. At the adductor tubercle B. Distal and anterior to the adductor tubercle C. Distal and posterior to the adductor tubercle D. Proximal and anterior to the adductor tubercle E. Proximal and posterior to the adductor tubercle 19. A 60-year-old male presents at the ED with a sudden onset of knee pain following a twisting incident at home. He underwent a medial unicompartmental knee replacement 5 years ago. His radiographs are shown in Figure 5.3. Which of the following is a true statement regarding liner dislocation with unicompartmental knee arthroplasty? A. Closed reduction of liner dislocation is usually successful B. Early liner dislocation can be due to impingement C. Fixed bearing has a higher risk of liner dislocation compared to mobile bearing D. Lateral unicompartmental knee replacement has less risk of liner dislocation compared to medial unicompartmental knee E. Residual cement debris is a common problem with newer uncemented prosthesis design

Knee I Structured SBA

(a)

(b)

Figure 5.3 (a) Anteroposterior (AP) and (b) lateral radiographs left knee

20. A 30-year-old presents with mechanical knee symptoms following an injury 3 months ago. MRI shows a medial meniscus tear involving the middle third. Which of the following meniscal tear patterns has the worst prognosis for meniscal repair? A. Bucket handle B. Horizontal C. Longitudinal D. Parrot beak E. Radial tear 21. A 20-year-old footballer presents with an unstable tear of the medial meniscus involving the body and anterior horn. You decide to repair the meniscus tear due its location in the red-red zone but find it difficult to access the anterior horn. Which of the following arthroscopic meniscal repair techniques is useful for repair of an anterior horn meniscal tear? A. All-inside B. All-outside C. Inside-out D. Outside-in E. Transtibial pull-out suture

22. A 13-year-old girl presents with a spontaneous onset of knee pain and mechanical symptoms in the last 6 months. MRI shows the following lesion on lateral femoral condyle (Figure 5.4). Which of the following statements is true related to this diagnosis? Figure 5.4 MRI scan knee

A. Condition is more common in females B. Lateral aspect of medial femoral condyle is the most common location C. Skeletally immature patients respond well to non-operative management D. This condition is more common in the elbow compared to the knee joint E. Unstable lesions can be treated by subchondral drilling

63

Kiran K. Singisetti

23. A 70-year-old woman with valgus knee arthritis is being considered for a total knee replacement. On examination, you find the medial collateral ligament to be intact. Which of the following statements is false? A. A constrained prosthesis is more likely to be used if there is a fixed valgus deformity. B. It is better to release the iliotibial band in knees too tight laterally in extension, but not in flexion. C. Patellar instability is a common problem with valgus knees D. Peroneal nerve palsy is more common in valgus knee compared to varus knee deformity correction E. The posterior condylar axis is more reliable than the transepicondylar axis for setting the correct femoral component rotation. 24. A 30-year-old amateur footballer attends clinic following a knee injury sustained a few months previously. Examination and imaging confirm a high-grade ACL injury. While counselling about ACL reconstruction, which of the following statements is not true about benefits of an ACL reconstruction? A. Better chance for return to sports B. Better chance of meniscal repair healing, if associated with ACL reconstruction C. Reduces instability symptoms of the knee D. Reduces risk of future knee arthritis E. Reduces risk of further meniscal injury 25. A 18-year-old gymnast presents with patellar instability where non-operative management (VMO exercises) has failed. CT scan of the knee is obtained to assess cross-sectional anatomy and measurements before considering a tibial tubercle osteotomy. Which of the following is a false statement? A. High Q angle can cause patellofemoral joint pain B. Lateral patellofemoral angle opens laterally C. Normal range for the Insall–Salvati ratio is 0.8 to 1.2 D. Sulcus angle of less than 140 degrees may suggest trochlear dysplasia E. TT–TG distance of more than 20mm is considered abnormal

64

26. A 21-year-old female presents with anterior knee pain, which has failed non-operative management. Imaging shows increased TT–TG distance with degeneration of the medial facet of the patella. Which of the following tibial tubercle osteotomies is recommended in this situation? A. Lateral and anterior displacement of tibial tubercle B. Lateral and posterior displacement of tibial tubercle C. Medial and anterior displacement of tibial tubercle D. Medial and distal displacement of tibial tubercle E. Medial displacement of tibial tubercle 27. During a revision knee arthroplasty procedure, assessment of remaining bone stock following removal of prosthesis shows some distal femoral bone loss. The surgeon decides to seat a femoral component on whatever remaining bone is present and implants a thicker polyethylene insert. At follow-up review, the patient reports anterior knee pain and flexion instability symptoms. What is the most likely reason for these symptoms? A. Lower joint line with decreased posterior condylar offset B. Lower joint line with increased posterior condylar offset C. Not resurfacing the patella D. Raised joint line with decreased posterior condylar offset E. Raised joint line with increased posterior condylar offset 28. A 25-year-old underwent ACL reconstruction using quadrupled hamstring graft 2 weeks previously. He asks for your advice on postoperative rehabilitation, as the physiotherapist is currently on leave. Which of the following exercises should be avoided in the first 4 to 6 weeks? A. Closed chain (e.g., squatting) knee exercises B. Isometric quadriceps exercises C. Neuromuscular training D. Open chain (e.g., seated leg extension) exercises E. Patella mobilisation

Knee I Structured SBA

29. A 25-year-old female attends clinic following a knee injury sustained while playing netball. Clinical examination reveals excessive anterior laxity with a soft end point on Lachman’s test. Which of the following statements is true regarding ACL reconstruction technique? A. Anterior knee pain is common with hamstring compared to BPTB autograft use B. Femoral tunnel expansion (windscreen wiper effect) is common with interference compared to suspensory fixation. C. The femoral tunnel is made before the tibial tunnel in a transtibial technique D. There is a higher rate of failure in allograft compared to autograft use in ACL reconstruction E. The outcome of single bundle ACL reconstruction is better with transtibial compared to anteromedial technique

30. A 12-year-old boy presents with a full-thickness ACL tear and instability symptoms. Tanner score is III. Parents are anxious about the long-term implications of the child’s ACL injury. Which of the following statements is true regarding ACL reconstruction in skeletally immature patients? A. There is a high risk of growth disturbance with transphyseal technique B. It is better to wait until skeletal maturity before considering ACL reconstruction C. Interference screw fixation is preferred in transphyseal technique D. Oblique femoral tunnel is preferred in transphyseal technique E. Soft tissue grafts are better for transphyseal technique

65

Kiran K. Singisetti

KNEE I STRUCTURED SBA ANSWERS 1. Answer C. Hypertrophic scar tissue at the superior pole of the patella impinges on the femoral component during extension Patellar clunk syndrome is related to the formation of a fibrous nodule on the undersurface of distal quadriceps, just above the patella. This is usually related to a posterior stabilised total knee replacement. Treatment is arthroscopic or open debridement of the fibrous nodule. Fukunaga K, Kobayashi A, Minoda Y, Iwaki H, Hashimoto Y, Takaoka K. The incidence of the patellar clunk syndrome in a recently designed mobile-bearing posteriorly stabilised total knee replacement. J Bone Joint Surg Br. 2009 Apr;91(4):463-468. Gopinathan P. Patello-femoral clunk syndromecurrent concepts. J Orthop. 2014 Jun 11;11(2): 55-57. 2. Answer D. If you are using posterior referencing, a size 4 femoral component may cause femoral notching The anterior referencing technique measures the size of the femur with the starting point from the anterior femoral cortex. This reduces the risk of notching of the anterior femoral cortex, but it is more difficult to control the posterior condylar offset. This carries the risk of flexion instability if the femoral component is undersized. With the posterior referencing technique, the measurement is referenced from the posterior condyles, the size of the flexion gap and posterior condylar offset can be better controlled, but there is a risk of anterior notching or overstuffing, if it is between sizes. Posterior referencing is considered generally more reliable in reducing the risk of flexion instability. If you are using a posterior referencing system, when faced with a femur in between sizes, there are some tips you could use if you decided to go with a smaller size femoral component. First, you could translate the cutting block 2mm anteriorly using an anti-notch guide. This may increase the flexion gap by 2mm but is usually well tolerated in most cases and avoids the risk of anterior notching. Second, a 3-degree flexion of the femoral component may help in reducing the

66

risk of anterior notching, though theoretically this may cause some loss of extension. Fokin AA, Heekin RD. Anterior referencing versus posterior referencing in total knee arthroplasty. J Knee Surg. 2014 Aug;27(4):303-308. Charette RS, Sheth NP, Boettner F, Scuderi GR, Melnic CM. Femoral component sizing during total knee arthroplasty: anterior versus posterior referencing. JBJS Rev. 2018 Jan;6(1):e4. 3. Answer E. Trial reduction with absolute congruence of the femoral component and the anterior edge of PE tibial bearing surface There are multiple checks that need to be performed to make sure the tibial base plate is appropriately aligned. The base plate needs to be aligned with the junction of the medial and middle thirds of the tibial tubercle. This should be achieved through rotation and not just simply translation in the coronal plane. This rotation should create minimal posterior–medial tibial plateau uncoverage. Additionally, this rotation should achieve optimal anterior medial and posterior lateral cortical contact in order to optimise load transfer and reduce the risk of subsidence. An additional check would be to use an extramedullary alignment rod clipped to the pinned tibial baseplate. The rod should overlay the medial–middle third junction of the tibial tubercle, the centre of the ankle mortise and point to the first or second ray of a neutrally aligned foot. Prosthetic clues for assessing tibial rotation include basing this relationship on a properly placed femoral component and the semicongruous PE bearing surface. With the knee fully extended, there should be absolute congruency between the femoral component and anterior edge of the semi-congruent tibial PE component. The knee should then be flexed up to 90° and again the PE tibial bearing should be absolutely congruent and without edge loading. 4. Answer E. Use of an anterior referencing knee system The patient is describing classic symptoms of flexion instability caused by an increased flexion gap compared to extension gap. Patients typically present with recurrent effusions, subjective instability (especially going downstairs), quadriceps weakness and diffuse peri-retinacular pain.

Knee I Structured SBA

Flexion instability is the result of a flexion space that is larger than the extension gap. It is caused by an inability to balance the flexion and extension space at the time of index arthroplasty or from gradual laxity of the posterior capsule or posterior cruciate ligament (PCL) in cruciateretaining (CR) designed implants. Technical factors that can lead to flexion instability include too little distal femoral resection in a pre-existing flexion contracture, overly aggressive posterior condylar resection with undersized femoral implants, excessive posterior slope on the tibia or over-release of the PCL in the CR knee. An appropriate extension gap can exist but over-resection of the posterior femur and/or undersizing or anteriorising of the femoral component will lead to a large flexion gap. This most commonly occurs when using an anterior referencing system to size the femoral component. As such, many surgeons prefer a posterior referencing system. 5. Answer C. Posterolateral bundle of ACL The anterior cruciate ligament (ACL) is composed of two bundles, anteromedial (AM) and posterolateral (PL). AM bundle of ACL is tight in flexion and loose in extension. PL bundle of ACL is tight in extension and loose in flexion. The Pivot shift test is performed with the patient’s knee starting in full extension. Maintaining internal rotation of the tibia, a valgus force is applied while the knee is slowly flexed to about 30 degrees. The examiner will feel for a subluxation of the lateral tibial plateau as it reduces to its normal position. The PL bundle of ACL is an important contributor to anteroposterior as well as rotational stability of the knee; deficiency of this component causes a positive Pivot shift test. Robinson J, Carrat L, Granchi C, Colombet P. Influence of anterior cruciate ligament bundles on knee kinematics: clinical assessment using computer-assisted navigation. Am J Sports Med. 2007 Dec;35(12):2006-2013. 6. Answer E. Transverse scars may be crossed with an incision in a perpendicular manner If previous longitudinal incisions exist, try to incorporate the most lateral incision that can

give adequate exposure. Due to the medially based blood supply, it is better to elevate a fullthickness medial flap rather than a lateral one. If you are unable to incorporate a lateral incision, then maintaining the widest possible skin bridge between incisions, without compromising exposure, is the best solution. When prior transverse incisions are present, it is safe to cross these incisions in a perpendicular manner. Respect the medially based vascular anatomy of the skin and incorporate previous incisions. Maintain full-thickness flaps, avoiding dissection superficial to the deep fascia. 7. Answer E. Lateralisation of the patellar component Patellofemoral tracking in total knee replacement is improved by the following steps: (1) external rotation of the femoral component, (2) avoidance of internal rotation of the tibial component, (3) joint line preservation, (4) medialisation of the patellar component, (5) avoidance of an oversized femoral component, (6) lateralisation of the femoral component and (7) secure repair of the medial retinaculum during closure. Lateralisation of the patellar component will increase the Q-angle and tendency to cause lateral maltracking of the patella. 8. Answer B. Core decompression is a useful treatment following subchondral collapse SONK was previously related to ischemia leading to necrosis; it is now considered due to a subchondral insufficiency fracture of the knee. It is seen more frequently in women (M:F 1:3) and typically affects those over the age of 55. Patients often report sudden onset of severe knee pain without significant trauma. This must be distinguished from secondary osteonecrosis of the knee. Bone scan may show a low uptake at the lesion but has a limited role in diagnosis due to its low specificity and sensitivity of diagnosis of the condition. Some association with meniscal root tears has been reported recently. The initial treatment is non-operative. Core decompression has a limited role in resistant cases prior to subchondral collapse. Arthroplasty (partial or total) is considered when there is a progressive degenerative change of the joint.

67

Kiran K. Singisetti

9. Answer C. Patella baja Patella baja is a common problem encountered in total knee replacement following previous high tibial osteotomy. This is more common in a closed wedge compared to an open wedge osteotomy. Posterior tibial slope should also be carefully considered, as a previous osteotomy has a tendency to alter this. Sang Jun Song, Dae Kyung Bae, Kang Il Kim, Chung Hwan Lee. Conversion total knee arthroplasty after failed high tibial osteotomy. Knee Surg Relat Res. 2016 Jun;28(2):89–98. 10. Answer B. Anterior knee pain Medial unicompartmental knee replacement is considered a suitable alternative to osteotomy for single compartment knee degenerative changes. The contraindications for this procedure include inflammatory arthritis, ACL deficiency, fixed varus deformity of more than 10 degrees and stiff knee. Patellofemoral degenerative changes were previously considered a relative contraindication, but more recent literature suggests the contrary. The Oxford Group report that anterior knee pain and early patellofemoral degenerative changes are not considered a contraindication for the medial unicompartmental arthroplasty. Severe patellofemoral chondral changes are still considered a contraindication for this procedure. Hamilton TW, Pandit HG, Maurer DG, et al. Anterior knee pain and evidence of osteoarthritis of the patellofemoral joint should not be considered contraindications to mobile-bearing unicompartmental knee arthroplasty: a 15-year follow-up. Bone Joint J. 2017 May;99-B(5):632639. 11. Answer E. Subvastus approach is an extensile approach Medial parapatellar approach is the commonest approach used for total knee replacement. Lateral parapatellar approach can be used in a valgus knee that is not correctable; the access to the lateral compartment is good but can occasionally cause difficulty in distal closure after deformity correction. Midvastus approach is advocated for an earlier rehab, as it avoids disruption of VMO insertion. Both midvastus and

68

subvastus approaches are less extensile and should not be attempted in obese patients and stiff knee and complex knee conditions. MIS knee replacement involves a smaller skin incision than does the traditional medial parapatellar approach; it may have less immediate postoperative pain but has not been shown to have better function in long term. 12. Answer E. More expensive The metal-backed tibial components are more commonly used across most arthroplasty registries, although, all-polyethylene tibial components have been reported to have better (or comparable) survival and lower rates of infection, instability, tibial component loosening and periprosthetic fracture. They are also cheaper compared to metal-backed tibial components. The disadvantage of all-polyethylene tibia is the lack of modularity. Houdek MT, Wagner ER, Wyles CC, Watts CD, Cass JR, Trousdale RT. All-polyethylene tibial components: an analysis of long-term outcomes and infection. J Arthroplasty. 2016 Jul;31 (7):1476-1482. Gudnason A, Hailer NP, W-Dahl A, Sundberg M, Robertsson O. All-polyethylene versus metal-backed tibial components-an analysis of 27,733 cruciate-retaining total knee replacements from the Swedish Knee Arthroplasty Register. J Bone Joint Surg Am. 2014 Jun;96(12):994-999. 13. Answer E. Vertical inclination of femoral tunnel is associated with rotational instability Too anterior femoral tunnel limits extension. Likewise, too anterior tibial tunnel causes roof impingement and limits extension. Too vertical femoral tunnel can lead to a non-anatomical femoral graft entry point, which has the potential to cause rotational instability. Pinczewski et al described the optimal tunnel position in their series of patients with good outcome. In the sagittal plane, the femoral tunnel was a mean of 86% posteriorly along Blumensaat’s line and the tibial tunnel was 48% along the tibial plateau. In the coronal plane, the tibial tunnel was 46% across the tibial plateau and the mean inclination of the graft was 19°.

Knee I Structured SBA

Pinczewski LA, Salmon LJ, Jackson WF, von Bormann RB, Haslam PG, Tashiro S. Radiological landmarks for placement of the tunnels in single-bundle reconstruction of the anterior cruciate ligament. J Bone Joint Surg Br. 2008 Feb;90(2):172-179.

translate the femoral component posteriorly or to use a thicker insert followed by addressing tight extension gap. Extension tight and flexion good can be addressed by either more distal femoral resection or posterior capsule release.

14. Answer A. Bone–patella–tendon–bone autograft An autograft is tissue obtained from the patient’s body. An allograft is tissue from a cadaver. The most common choices available are bone–patella–tendon–bone autograft, hamstring autograft, quadriceps tendon autograft and various allograft options. Synthetic graft options are now uncommon. The bone-to-bone healing with the patellar graft has an advantage for athletes who are interested in early return to sports, although this has the risk of residual anterior knee pain due to the morbidity associated with patellar tendon graft harvest. Traditionally, the bone–patella–tendon–bone graft was considered gold standard amongst graft options, though more recent studies have shown comparable results with hamstring autograft. Samuelsen BT, Webster KE, Johnson NR, Hewett TE, Krych AJ. Hamstring autograft versus patellar tendon autograft for ACL reconstruction: is there a difference in graft failure rate? A meta-analysis of 47,613 patients. Clin Orthop Relat Res. 2017 Oct;475(10):2459-2468. Gifstad T, Foss OA, Engebretsen L, et al. Lower risk of revision with patellar tendon autografts compared with hamstring autografts: a registry study based on 45,998 primary ACL reconstructions in Scandinavia. Am J Sports Med. 2014 Oct;42(10):2319-2328.

Extension good and tight in flexion can be addressed by decreasing the size of the femoral component. Other options are to recess PCL and address posterior slope of tibia if needed.

15. Answer B. Decrease the size of the femoral component Symmetric gap issues (such as tight or loose in both extension and flexion) are addressed with proximal tibia. Resecting more proximal tibia helps with tight extension and flexion. Using a thicker insert or tibial augmentation helps if loose in extension and flexion. Asymmetric gap issues: Extension good and loose in flexion can be addressed with an increase in the size of the femoral component; other options are to

Extension loose and flexion good can be addressed with distal femoral augmentation. Other options are to use a thicker insert followed by addressing tight flexion gap. 16. Answer C. Isolated posterolateral corner injury Posterolateral corner (PLC) injuries are generally associated with other ligament injuries but can occasionally present as isolated injuries. Dials test is a useful assessment for PLC injuries; asymmetry of external rotation of foot on the affected side at 30 degrees of knee flexion is seen only in isolated PLC injury. Asymmetry of external rotation of the foot on the affected side at both 30 and 90 degrees of knee flexion is suggestive of combined PLC and PCL injury. One of the causes of failure of an ACL reconstruction is a missed PLC injury. Ranawat A, Baker CL 3rd, Henry S, Harner CD. Posterolateral corner injury of the knee: evaluation and management. J Am Acad Orthop Surg. 2008 Sep;16(9):506-518. 17. Answer B. Chondral changes Chondral changes in the absence of osteophytes is not a contraindication for meniscal transplant. Osteophytes may interfere with the sitting of the meniscal graft. Instability and malalignment should be corrected before meniscal transplantation surgery. Most authors suggest that meniscal transplantation should be considered only in symptomatic meniscal loss, although some consider this in asymptomatic lateral meniscus loss. Inflammatory arthropathy, advanced arthritis, obesity and prior infection are also considered contraindications. Figueroa F, Figueroa D, Calvo R, Vaisman A, Espregueira-Mendes J. Meniscus allograft

69

Kiran K. Singisetti

transplantation: indications, techniques and outcomes. EFORT Open Rev. 2019 Apr;4(4):115120. International Meniscus Reconstruction Experts Forum (IMREF) 2015 Consensus Statement on the Practice of Meniscal Allograft Transplantation. Getgood A, LaPrade RF, Verdonk P, Gersoff W, Cole B, Spalding T; IMREF Group. Am J Sports Med. 2017 May;45(5):11951205. Epub 2016 Aug 25. 18. Answer B. Distal and anterior to the adductor tubercle In a cadaveric study Schottle et al described, the medial patellofemoral ligament (MPFL) was found to insert 1.9mm anterior and 3.8mm distal to the adductor tubercle. Schottle et al described the MPFL anatomical insertion on the femur as the isometric point for MPFL tunnel placement in reconstruction cases. In their study, they defined a radiographic point 1mm anterior to a line extending from the posterior cortex and 2.5mm distal to the posterior origin of the medial femoral condyle and proximal to the level of the posterior point of the Blumensaat line. 19. Answer B. Early liner dislocation can be due to impingement Lateral unicompartmental knee replacement is at a higher risk of liner dislocation. Hence, some authors recommend using a fixed bearing prosthesis on the lateral side. Bearing exchange alone should be carefully considered in selected patients having correctable causes such as impingement by remnant cement or bony spur, larger gap (thin bearing at the index operation), loss of entrapment by late bearing wear at the long-term follow-up or acute trauma. Sang-Gyun Kim S-G, Kim H-G, Lee S-Y, Lim H-C, Bae J-H. Redislocation after bearing exchange for the treatment of mobile bearing dislocation in medial unicompartmental knee arthroplasty. Knee Surg Relat Res. 2018 Sep;30 (3):234–240. van der List JP, Zuiderbaan HA, Pearle AD. Why do medial unicompartmental knee arthroplasties fail today? J Arthroplasty. 2016 May;31(5):1016-1021.

70

20. Answer E. Radial tear Radial meniscus tears lead to decreased hoop stresses of the meniscus and effectively a nonfunctional meniscus. Abram SGF, Beard DJ, Price AJ; BASK Meniscal Working Group. Arthroscopic meniscal surgery: a national society treatment guideline and consensus statement. Bone Joint J. 2019 Jun;101-B(6):652-659. 21. Answer D. Outside-in The three main techniques for meniscal repair are inside-out, outside-in, all-inside. All-inside meniscal repair is the most common technique and can be performed with a variety of suture anchor devices. Inside-out technique has conventionally been considered a gold standard, as it gives a strong repair though this is associated with surgical risks. In this technique, the tear is fixed by placement and fixation of the passing sutures from the intra-articular region with the use of special cannulae to the extracapsular area over the capsule with a posterolateral or posteromedial incision. Outside-in technique is useful for anterior third or horn of meniscus repair, where the suture ends are tied over capsule. Transtibial pull-out suture repair is useful for posterior root avulsions. All-outside repair doesn’t exist and is a misleading option. 22. Answer C. Skeletally immature patients respond well to non-operative management The MRI image in Figure 5.4 shows a presentation of osteochondritis dessicans (OCD) with an unstable lesion. It is more common in males. Non-operative management is better tolerated by the skeletally immature compared to young adults. The knee (distal femur) is the most common joint for OCD; other sites include the elbow (distal humerus) and ankle (talus). Though the image shows a lesion on the medial aspect of lateral condyle, it is described most commonly at the lateral aspect of the medial condyle (almost 80%). An unstable lesion is best managed by fixation. Subchondral drilling is an option for a stable but symptomatic lesion. The International Cartilage Repair Society (ICRS) scale of OCD lesions is based on the arthroscopic assessment:

Knee I Structured SBA

Type I: Stable lesion with a continuous but softened area covered by intact articular cartilage Type II: Lesion with partial articular cartilage discontinuity, stable when probed

protects against future development of knee arthritis. Repair of meniscal tear is more likely to be successful if an associated ACL deficiency is managed surgically.

Type III: Lesion with complete articular cartilage discontinuity, but no dislocation

25. Answer D. Sulcus angle of less than 140 degrees may suggest trochlear dysplasia TT-TG distance measures the distance between two perpendicular lines from the posterior cortex to the tibial tubercle and the trochlear groove; a value of greater than 20mm is usually considered abnormal. Sulcus angle is used to evaluate trochlear dysplasia; an angle of more than 140 degrees may indicate suspicion of dysplasia. The Insall– Salvati method helps to assess patellar height. Normal value is between 0.8 and 1.2. The lateral patellofemoral angle is a measurement of the patellar tilt; it is the angle between the line across femoral condyles and a second line along the lateral patellar facet. The Q angle is the angle between the line joining the anterior superior iliac spine and the centre of the patella and the second line joining the centre of the patella to the tibial tubercle. It can be measured both at flexion (15–20°) and extension; however, it may not be accurate in extension due to lateral patellar displacement. Traditionally measured with the patient supine and quadriceps relaxed, there has not yet been a standardisation of the position and state of muscle contraction while measuring the Q angle. It is an indicator of the net lateral force exerted on the patella by the quadriceps and the patellar tendon. Dejour H, Walch G, Nove-Josserand L, Guier C. Factors of patellar instability: an anatomic radiographic study. Knee Surg Sports Traumatol Arthrosc. 1994;2(1):19-26.

Type IV: Empty defect, or defect with a dislocated fragment or loose fragment within the bed 23. Answer E. The posterior condylar axis is more reliable than the transepicondylar axis for setting the correct femoral component rotation. Valgus knee deformity is defined by a tibiofemoral angle of greater than 10°. The posterior femoral condyle on the lateral side can be deficient in a valgus knee. Relying on the posterior condylar axis in lateral femoral condyle hypoplasia can result in internal rotation of the femoral component. The anteroposterior (AP) and the transepicondylar axis are more reliable reference lines to achieve appropriate femoral component rotation. Popliteus release can be considered in knees that are tight laterally in flexion but not in extension. Iliotibial band release can be considered in knees that are tight laterally in extension but not in flexion. Knees that are tight laterally in flexion and extension have popliteus tendon or LCL release (or both). Lange J, Haas SB. Correcting severe valgus deformity: taking out the knock. Bone Joint J. 2017 Jan;99-B(1 Supple A):60-64. Whiteside LA. Selective ligament release in total knee arthroplasty of the knee in valgus. Clin Orthop Relat Res. 1999 Oct;(367):130-140. 24. Answer D. Reduces risk of future knee arthritis There is some controversy about conservative versus surgical management of ACL reconstruction. While a structured rehabilitation programme may be suitable for some patients, ACL reconstruction has been shown to improve knee stability and thereby decrease the risk of further meniscal injuries. There is no significant evidence to suggest that an ACL reconstruction

26. Answer C. Medial and anterior displacement of tibial tubercle Distal patellar realignment procedures are used to help with patella compression syndrome. They can be classed as tibial tubercle anteriorisation (Maquet procedure), medialisation (Elmslie–Trillat procedure) or a

71

Kiran K. Singisetti

combination (anteriorisation and medialisation, as with Fulkerson procedure). Elmslie– Trillat is contraindicated with medial patella facet arthritis. 27. Answer D. Raised joint line with decreased posterior condylar offset The joint line is inadvertently raised due to proximal displacement of the femoral component if the distal femoral bone loss is not taken into consideration. This causes joint line elevation and potentially a smaller revision femoral component being used. A common surgical mistake is to use a thicker insert as the extension gap is too large, but this raises the joint line. Undersizing of the femoral component causes decreased posterior condylar offset, which is a cause of flexion instability. Posterior condylar offset is the maximum thickness of posterior condyles; some authors measure this as a ratio. Posterior condylar offset ratio is defined by Johal et al as the ‘maximal thickness of the posterior condyle projecting posteriorly to a straight line drawn as the extension of the posterior femoral shaft cortex, divided by the maximal thickness of the posterior condyle projecting posterior to a straight line drawn as the extension of the anterior femoral shaft cortex on a true lateral radiograph of the distal quarter of the femur’. Restoring joint line is another important consideration in revision knee arthroplasty. Some landmarks for joint line are previous meniscal scar, 10 to 15mm proximal to the tip of fibular styloid, 25mm distal to the sulcus of the medial epicondyle and 20mm from lateral epicondyle. Clement ND, MacDonald DJ, Hamilton DF, Burnett R. Posterior condylar offset is an independent predictor of functional outcome after revision total knee arthroplasty. Bone Joint Res. 2017 Mar;6(3):172-178. Johal P, Hassaballa MA, Eldridge JD, Porteous AJ. The Posterior Condylar Offset Ratio. Knee 2012 Dec;19(6):843-845. 28. Answer D. Open chain (e.g. seated leg extension) exercises Postoperative ACL rehabilitation is goal based rather than time based. Open chain and isokinetic exercises should be avoided in the first few

72

weeks for ACL postoperative rehabilitation. Closed chain knee exercises are those that are performed with the foot in contact with the ground or a machine. Closed chain exercises tend to cause compression of joints, which helps stabilise the joint. Open chain exercises tend to involve more shearing force across the joint; this may risk putting undue stretch on the graft. There are some differences in a hamstring (HS) versus bone–patella–tendon–bone (BPTB) graft on how open chain exercises can be delivered. For BPTB graft, open chain exercises can be started from 4 weeks postoperative in a restricted ROM of 90–45°, and extra resistance is allowed, for example, at a leg extension machine. For HS graft, open chain exercises can also be started from 4 weeks postoperative in a restricted ROM of 90–45°, but no extra weight should be added in the first 12 weeks to prevent graft elongation van Melick N, van Cingel RE, Brooijmans F, et al. Evidence-based clinical practice update: practice guidelines for anterior cruciate ligament rehabilitation based on a systematic review and multidisciplinary consensus. Br J Sports Med. 2016 Dec;50(24):1506-1515. 29. Answer D. There is higher rate of failure in allograft compared to autograft use in ACL reconstruction The transtibial technique involves femoral tunnel drilling through the tibial tunnel; hence the tibia is prepared first. The anteromedial technique of femoral tunnel placement is independent of the tibial tunnel. The anteromedial technique has been shown more reliable in appropriate femoral tunnel position compared to transtibial technique, thereby there is better functional outcome. There is no donor site morbidity with use of allograft but it has the disadvantage of higher failure rate. The incidence of anterior knee pain and difficulty in kneeling is more common with the patellar tendon (BPTB) autograft. 30. Answer E. Soft tissue grafts are better for transphyseal technique There is increasing literature reports about the risks of delaying surgery in symptomatic knees for the skeletally immature; this has the risk of meniscal and chondral injuries. Both

Knee I Structured SBA

transphyseal- and physeal-sparing techniques have been shown to have similar and low risk of growth disturbance in skeletally immature patients. The risk factors for growth disturbance involve high-speed drilling, use of

interference screws, overtension of graft, large tunnel size and oblique tunnel position. Soft tissue grafts with slightly more vertical tunnels in the transphyseal technique reduce the risk of growth disturbance.

73

Section 2 Chapter

6

Adult Elective Orthopaedics and Spine

Knee II Structured SBA Oliver Bailey

KNEE II STRUCTURED SBA QUESTIONS 1. While performing a posteromedial approach to the knee, which of the following structures helps you identify the correct plane? A. Saphenous nerve B. Saphenous vein C. Semitendinosus tendon D. Sural nerve E. Tibial artery 2. When performing a knee posterolateral corner reconstruction, which of the following structures has the most anterior femoral insertion point? A. Arcuate ligament B. Lateral collateral ligament C. Lateral head of the gastrocnemius D. Popliteofibular ligament E. Popliteus 3. All of the following are considered part of the posteromedial corner of the knee apart from which structure? A. Medial collateral ligament B. Oblique popliteal ligament C. Posterior oblique ligament D. Posteromedial joint capsule E. Semimembranosus tendon and its expansions 4. A 24-year-old male sustained a grade III PLC injury of his knee following a skiing injury 2 years previously. He is listed for surgery for chronic pain and instability. Which of the following is the most essential structure to identify while performing a posterolateral corner reconstruction? A. Common peroneal nerve B. ITB

74

C. LCL D. Popliteus E. Tibial nerve 5. When climbing stairs, roughly how does a patient’s body weight correlate with their joint reaction force of their patellofemoral joint? A. 0.5 times body weight B. 20 times body weight C. 3–4 times body weight D. 7–8 times body weight E. Unrelated to body weight 6. Which of the following is not a validated knee outcome measure? A. IKDC B. KOOS C. Lysholm Score D. Oxford Knee Score E. SF-30 7. You review a young adult who presents with a painless knee swelling and intermittent locking with no history of trauma. MRI is shown in Figure 6.1. This demonstrates a joint effusion with a mass like synovial proliferation with lobulated margins. What is the most likely diagnosis? Figure 6.1 MRI scan knee

Knee II Structured SBA

A. B. C. D. E.

Lipoma arborescens Pigmented villonodular synovitis Rheumatoid arthritis Synovial cell carcinoma Synovial chondromatosis

8. A colleague has performed a knee arthroscopy on a 45-year-old male 6 weeks ago. The operation note states the only abnormal finding was mild cartilage thinning medially (grade I). He continues to complain of knee pain. What is the most appropriate next step in management? A. Anteroposterior (AP) radiograph of the hip B. List for further knee arthroscopy C. MRI knee D. Physiotherapy E. Standing long leg alignment x-rays 9. You are supervising a trainee perform a knee arthroscopy on a 25-year-old female. You notice they have made their anterolateral arthroscopy portal quite inferior to where you had wanted. A complication occurs due to this portal placement. What piece of equipment will you ask for to deal with this complication? A. ACL repair kit B. All-inside meniscal repair kit C. Chondral fixation kit D. Microfracture kit E. Outside-in meniscal repair kit 10. A young female presents with snapping of her knee with episodes of locking. Sagittal MRI images of her lateral compartment show three 5mm-thick contiguous images of her meniscus from anterior to posterior horns with no tears obvious. What is your next management step? A. Inject with steroid B. List for arthroscopy C. List for arthroscopy + saucerisation D. List for arthroscopy + saucerisation +/– meniscocapsular repair E. Refer to physiotherapy 11. You review a young female with the results of her MRI after a twisting knee injury. You are

pleased to see she is now asymptomatic but note that within the lateral compartment of the knee her MRI demonstrates ‘a minimal meniscal width to maximal tibial width (on coronal slice) of 40%, and a ratio of the sum of the width of both lateral horns to the maximal meniscal diameter (on sagittal slice) of 80%’. What is your management plan? A. Discharge to physio B. Discharge with no follow-up C. List for arthroscopic meniscal repair D. List for arthroscopic saucerisation E. List for diagnostic arthroscopy 12. You review a 10-year-old boy with lateral knee pain. MRI shows five sagittal slices of 5mm-thick contiguous lateral meniscus from anterior to posterior horns. Which of the following x-ray findings is associated with the diagnosis? A. Hypoplastic patella B. Lateral tibial plateau fracture C. Narrowing lateral joint space D. Segond fracture E. Tibial eminence hypoplasia 13. You are performing an arthroscopic PCL reconstruction and utilise x-ray guidance while drilling the tibial tunnel. Which complication are you hoping to reduce with the use of x-ray fluoroscopy? A. Graft impingement B. Malplacement of the tunnel C. Overconstraint D. Popliteal artery injury E. Tibial fracture 14. During ACL surgery, you prematurely amputate the semitendinosus hamstring graft at a length of about 7cm. What is the likely intraoperative mistake? A. You had not appreciated the patient had a positive Dials test B. You had not fully released the extratendinous tethers to the medial head of gastrocnemius C. You had released the semitendinosus from the sartorial facia prior to using the tendon stripper

75

Oliver Bailey

D. You had used a closed loop tendon stripper rather than an open loop tendon stripper E. Your graft harvest wound was too small 15. At a 6-week postoperative review of one of your ACL reconstructions they complain of ongoing numbness over the medial border of their foot on the same side as their ACL reconstruction. What is the likely graft that this patient has had? A. Bone–patella–tendon–bone allograft B. Bone–patella–tendon–bone autograft C. Hamstring allograft D. Hamstring autograft E. Quadriceps 16. You have been asked to review one of your ACL reconstructions at 3 months postoperatively by the physiotherapist. They are concerned that the patient is unable to fully extend their knee. What is the likely intraoperative mistake? A. The entry point for the femoral tunnel is at the 12 o’clock position B. You tensioned the graft in 30 degrees of knee flexion rather than full extension C. You have a cortical ‘blowout’ while reaming the femoral tunnel; therefore, you secured the graft with a larger-than-normal femoral button D. The entry point for the tibial tunnel is 2mm anterior to the anterior horn of the lateral meniscus E. You prematurely amputate the hamstring graft during harvest, leading you to change your graft choice from quadrupled stranded hamstring graft to a bone–patella–tendon– bone graft 17. You are reviewing a 19-year-old female who plays netball nationally and who underwent an isolated ACL reconstruction 11 months ago. She was really happy with her rehabilitation, but on return to her first contact game at 10 months postoperatively her ACL graft failed. Lachman’s and Pivot shift tests are positive, Dials test is negative. On review of her preoperative MRI, you note an ACL rupture with the presence of a second fracture, but no other abnormality.

76

What is the most likely cause of their rerupture? A. The surgeon failed to address the anterolateral complex B. The surgeon failed to address the posterolateral complex C. The surgeon used hamstring autograft D. The surgeon used patella tendon autograft E. The tibial graft tunnel was too anterior 18. You are performing a total knee replacement using a measured resection technique with a PS design using anterior femoral referencing; however, on trialling the implants, the knee is ok in flexion but loose in extension. Which of the following is the best option to gain a stable knee? A. Resect more bone off the tibia and upsize the femur B. Anteriorise the femoral component and use distal augments C. Resect the PCL D. Resect more bone off the tibia and downsize the femur E. Downsize the femur and use a thicker insert 19. You are performing a total knee replacement using a measured resection technique with a PS design using anterior femoral referencing; however, on trialling the implants, you notice the knee is loose in flexion but ok in extension. Which of the following is the best option to gain a stable knee? A. Posteriorise the femoral component B. Proximalise the femur and use a thicker insert C. Resect more bone off the femur and upsize the femur D. Resect more bone off the tibia and upsize the femur E. Upsize the femur 20. You see an unhappy patient in clinic who underwent a total knee replacement 3 years ago. They have no history of wound problems, no history of trauma and no infective symptoms, but state they have never really been happy with their knee. Symptoms include anterior knee pain, clunking

Knee II Structured SBA

and difficulty walking. There is no evidence of osteolysis on x-ray. Which of the following investigations is most likely to demonstrate an abnormality? A. Bloods B. CT rotational profile of the leg C. Knee aspiration with a request for extended cultures D. Knee aspiration with a request for Gram stain E. Knee aspiration with a request for white cell count 21. You are performing a total knee replacement in a valgus knee and trying to balance the components. It is well balanced in extension but remains tight laterally in flexion. What is the next best appropriate intraoperative step? A. Downsize your femoral component with anterior referencing B. Release the popliteus C. Release the LCL D. Release the PCL E. Resect more of the tibia and distalise the femur with augments 22. A patient presents after a fall with a periprosthetic knee fracture extending proximally from the femoral component. What intraoperative decision has the original knee surgeon made that has increased the chances of this happening? A. In an attempt to improve patella tracking, they externally rotated the femoral component B. In an attempt to improve the extension gap, they performed a posterior release C. In an attempt to improve the flexion gap, they released the PCL and subsequently changed from a CR to a PS implant design D. On measuring the femoral component size using posterior referencing, it measured 4.5 and the decision was made to use a size 4 implant, as a size 5 may overstuff the PFJ E. They proximalised the femoral component due preoperative fixed flexion 23. One-year postoperatively from a total knee replacement (without patella resurfacing) a

patient presents to your clinic with severe worsening anterior knee pain and grinding over the PFJ. Skyline radiographs of the patella demonstrate fragmentation of the patella and accelerated arthritic changes. What decision intraoperatively has most likely led to this presentation? A. Denervation of the patella was performed with diathermy B. Due to poor patella tracking, a lateral retinacular release was performed C. The femoral component was anteriorised to increase the flexion gap D. The femoral component was externally rotated E. The patella was not resurfaced 24. You review a patient in clinic who is 4 years postoperative from a total knee replacement. They are complaining of increasing pain for the past 6 months. X-ray demonstrates an area of osteolysis behind the femoral component. Aspiration of the joint has the following results: Gram stain negative, WCC 4 000, PMN count 90%. Culture does not grow anything. Which is the most appropriate next step? A. One-stage revision B. Two-stage revision C. Arthroscopic washout of the knee D. DAIR procedure E. Nuclear medicine bone scan 25. A patient complains of numbness after undergoing a total knee replacement for an arthritic valgus knee. What is the likely area that has sensory disturbance? A. Dorsal aspect of the foot B. L4 dermatome C. L5 dermatome D. Medial aspect of the foot E. Sole of the foot 26. Which of the following radiographic findings makes a TKA more difficult? A. A Caton–Deschamps index of 0.7 B. A Dejour grade of C type C. An Insall–Salvati ratio of 0.7 D. Bipartite patella E. TT-TG of 12mm

77

Oliver Bailey

27. You are performing an Oxford knee replacement but note the ACL is torn. The patient has only been consented for a unicompartmental knee arthroplasty (UKA). You have not been trained in any other UKA. What is your management plan? A. Close the wound and wake the patient up B. Continue with the Oxford knee replacement C. Continue with the Oxford knee replacement and perform an ACL reconstruction D. Switch to a fixed bearing UKA E. Switch to a total knee replacement 28. You are examining a patient in the anaesthetic room just prior to surgery. The patient has been listed for an Oxford knee replacement. Which of the following examination findings are you most worried about? A. 0–95° range of movement B. 10° fixed flexion C. 11° correctable varus D. Age 68 E. Previous arthroscopy scars 29. A 45-year-old male presents with a chronic history of knee pain, swelling and locking. There is no history of trauma and they are systemically well. MRI shows multiple lobular cartilage lesions within the joint.

78

What is the next best management option? A. Anti-inflammatory medication + intraarticular steroid injection B. Arthroscopic synovectomy, removal of loose bodies and histopathological analysis C. Joint aspiration D. Neoadjuvant radiotherapy followed by wide local excision E. Open biopsy 30. A 67-year-old patient presents with an acutely swollen knee with no history of trauma. They are systemically well. X-ray of the knee is normal apart from an effusion. A joint aspirate has a negative Gram stain but demonstrates positively birefringent crystals. What is the next best management? A. Admit patient to await full culture and sensitivities B. Book on trauma list for emergency joint washout C. Reassure and discharge with antiinflammatory medication D. Reassure and discharge with antiinflammatory medication and ask GP to start allopurinol E. Refer for a knee MRI

Knee II Structured SBA

KNEE II STRUCTURED SBA ANSWERS 1. Answer C. Semitendinosus tendon To accurately perform the posteromedial incision, the semitendinosus tendon, flexion crease and medial head of the gastrocnemius are used for anatomical reference. In grossly swollen patients with distorted anatomy, finding the semitendinosus tendon is essential, as it is at risk of damage, and the fascial band attachments to the medial head of the gastrocnemius help you find the gastrocnemius muscle and confirm the correct plane of dissection. 2. Answer E. Popliteus The popliteus tendon is the most anterior femoral attachment of the posterolateral corner and on average is 18.5mm anterior to the femoral attachment of the LCL with the knee at 70 degrees flexion (Figure 6.2). Figure 6.2 Popliteus tendon

3. Answer A. Medial collateral ligament The superficial and deep portions of the MCL function in close association with the structures of the PMC; they are, however, not typically considered to be part of the PMC. The PMC is typically formed of 5 main components: the semimembranosus tendon and expansions, the oblique popliteal ligament, the posterior oblique ligament, the posteromedial joint capsule and the posterior horn of the medial meniscus.

4. Answer A. Common peroneal nerve One of the first structures you should identify when performing a PLC reconstruction is the common peroneal nerve, which is at risk of injury if not identified and protected throughout. The LCL and popliteus are useful to identify and repair but not essential when performing a reconstruction. The ITB is incised as part of the approach. The tibial nerve should not be seen during this approach. 5. Answer C. 3–4 times body weight The forces at the knee increase significantly with increasing degrees of flexion. At about 45 degrees of flexion (needed to climb the stairs), joint reaction force are about 3–4 times body weight. When walking on the flat, joint reaction forces equal roughly 0.5 times body weight. When squatting, joint reaction forces equal roughly 7–8 times body weight. When jumping, joint reaction forces can increase to over 20 times body weight. 6. Answer E. SF-30 The SF-36 is a popular general health outcome measure; the SF-30 is not. KOOS (Knee Injury and Osteoarthritis Outcomes Score) is useful in evaluating soft tissue knee injuries. IKDC (International Knee Documentation Committee) is useful in evaluating symptoms, function and activity. Lysholm Score was developed to evaluate knee ligament surgery. Oxford Knee Score was designed for patients undergoing knee replacement surgery. 7. Answer B. Pigmented villonodular synovitis Pigmented villonodular synovitis (PVNS) is a benign proliferative condition of the synovial membrane and tendon sheath. In the knee, patients can present with a painless effusion and symptoms of locking, catching and instability. It is characterised by synovial inflammation and haemosiderin deposits. 8. Answer A. Anteroposterior (AP) radiograph of the hip The knee arthroscopy only demonstrates mild wear in the medial compartment. This could

79

Oliver Bailey

produce ongoing knee pain; however, you should always examine the hip joint to ensure this is not the cause of symptoms. 9. Answer E. Outside-in meniscal repair kit Typically, the anterolateral portal is placed blind; therefore, to reduce iatrogenic injury to the anterior horn of the lateral meniscus, this portal should be placed high. Inferior placement risks creating a radial tear in the anterior horn of the lateral meniscus. Anterior horn tears are best approached from outside in, purely from a point of view of direction of instruments in relation to direction and area of tear. Using an all-inside technique is technically a lot more demanding for this type of tear. Although chondral and ligament injuries can occur with poor arthroscopic technique, these are not the main risk with inferior placement of the anterolateral portal. While bleeding and haematoma formation can occur following knee arthroscopy no major arterial vessel would likely to be encountered even with a poorly sited anterolateral arthroscopic portal placement. 10. Answer D. List for arthroscopy + saucerisation +/– meniscocapsular repair Three or more contiguous 5mm-thick sagittal images of the meniscus from anterior to posterior horns is suggestive of a discoid meniscus. ‘Snapping knee syndrome’ in the presence of a discoid meniscus is suggestive of defective posterior monoconidial attachments, thus allowing increased mobility of the meniscus. The treatment of choice of symptomatic patients is saucerisation and repair of the posterior meniscocapsular deficiency if needed. An asymptomatic discoid meniscus does not require treatment. 11. Answer B. Discharge with no follow-up The MRI is suggestive of a discoid lateral meniscus. Having a minimal meniscal width to maximal tibial width (on coronal slice) of >20% and having a ratio of the sum of the width of both lateral horns to the maximal meniscal diameter (on sagittal slice) >75% is highly suggestive of a discoid meniscus (sensitivity 95% and specificity

80

of 97%). Other MRI findings of a discoid meniscus is three or more 5mm-thick contiguous sagittal slices from anterior to posterior horns. Regardless of diagnosis, this is an incidental finding, and the asymptomatic patient can be reassured and discharged. 12. Answer E. Tibial eminence hypoplasia The MRI findings suggest a symptomatic lateral discoid meniscus. Plain x-ray findings associated with a lateral discoid meniscus include (1) widening of the lateral joint space, (2) squaring of the lateral femoral condyle, (3) cupping of the lateral tibial plateau and (4) tibial eminence hypoplasia. 13. Answer D. Popliteal artery injury Drilling under x-ray guidance reduces the risk of perforating too far into the popliteal fossa. Tibia tunnel placement can be improved using x-ray; however, it is mainly dictated by visualisation through a posteromedial portal, clearance of the PCL tibial footprint and usage of a PCL jig. 14. Answer B. You had not fully released the extratendinous tethers to the medial head of gastrocnemius The hamstring tendons typically have extratendinous tethers that require release. This is particularly prevalent for the semitendinosus, which quite reliably has a number of tethers between itself and the medial head of gastrocnemius up to 10cm proximal from its distal attachment. If these are not released prior to tendon stripping, then the tendon stripper can follow the attachment rather than the true tendon, thereby amputating the tendon prematurely. The Dials test is used to identify PCL and PLC injuries clinically. This should be done preoperatively, as patients with a PLC injury that has not been identified prior to ACL reconstruction have a higher rate of graft failure. Both a closed loop tendon stripper and an open loop stripper can be used to harvest the graft. A harvest wound that is small will make graft harvest more difficult and will make it more difficult to release the tethers; however, the wound length is not the primary cause of graft amputation.

Knee II Structured SBA

Releasing the semitendinosus from the sartorial fascia can be done before or after graft harvest. 15. Answer D. Hamstring autograft The medial side of the foot is supplied by the saphenous nerve. Saphenous nerve injury is a complication of hamstring graft harvest. 16. Answer D. The entry point for the tibial tunnel is 2mm anterior to the anterior horn of the lateral meniscus The ideal tibial tunnel entry point is about 2mm posterior to the anterior horn of the lateral meniscus. If the tibial tunnel is too anterior, then this can cause impingement in extension and an extension block. The entry point for the femoral tunnel is 2 or 10 o’clock (depending on the side). If the femoral tunnel is too vertical, then this can lead to rotational instability of the graft. Tensioning of the graft should ideally be made in about 30 degrees of knee flexion. Cortical blowouts occur during tunnel reaming and are dealt with by changing your fixation strategy; one such way is to use a larger suspensory button. There are a number of different techniques to deal with premature amputation of a hamstring graft. One such way is to use an alternative graft such as a bone–patella–tendon–bone graft. 17. Answer A. The surgeon failed to address the anterolateral complex The anterolateral ligament complex is now widely understood to be a secondary stabiliser for the ACL for rotational stability. Injury of it can be demonstrated radiographically by the presence of a Segond fracture; however, the lack of a Segond fracture does not mean the anterolateral complex is not injured. Sonnery-Cottet et al (2007) have shown a significantly reduced rate of graft rupture in young patients returning to pivoting sports when they have a combined ACL and ALL reconstruction as opposed to an isolated ACL reconstruction. This is further backed up by the results of the International ALC Consensus Group Meeting 2019. The posterolateral complex is an important structure to address if injured and would lead to

increased re-rupture rates if not addressed; however, a negative Dials test would suggest the PLC is intact. A tibial graft tunnel that is too anterior can lead to impingement in extension and graft failure; however, this is unlikely in this scenario as the patient states they were happy during their rehab. There is a lack of consensus as to the best type of graft to use; both hamstring autograft and patella tendon autograft are acceptable choices in elite athletes. Sonnery-Cottet B, Saithna A, Cavalier M, et al. Anterolateral ligament reconstruction is associated with significantly reduced ACL graft rupture rates at a minimum follow-up of 2 years: a prospective comparative study of 502 patients from the SANTI Study Group. Am J Sports Med. Jun;45(7):1547-1557. Getgood A, Brown C, Lording T, et al. The anterolateral complex of the knee: results from the International ALC Consensus Group Meeting. Knee Surg Sports Traumatol Arthrosc. 2019;27(1):166-176. 18. Answer E. Downsize the femur and use a thicker insert Downsizing the femur using anterior referencing increases the flexion gap, thereby making the knee loose in flexion and extension. This can then be corrected by inserting a thicker insert. With regards to the other options answer options, resecting more bone of the femur will increase the extension gap (upsizing the femur will only change the flexion gap or PFJ). Using distal femoral augments is an option; however, anteriorising the femoral component will overstuff the PFJ and cause a loose flexion gap. Resecting the PCL does not address the extension gap. Resecting more bone of the tibia will increase the flexion and extension gap. 19. Answer E. Upsize the femur If you have a loose knee in flexion, all of the following will reduce the flexion gap: Upsizing the femur using anterior referencing, using a thicker insert, posteriorising the femoral component. Proximalising the femur and using a thicker insert will balance the knee; however,

81

Oliver Bailey

proximalising the femur is a bigger undertaking than upsizing the femur. Posteriorising the femoral component will notch the femur if using anterior referencing and therefore is not ideal. 20. Answer B. CT rotational profile of the leg The history is not really pointing you towards a diagnosis of infection (although this should remain a potential diagnosis). Someone who has no history of infection or trauma and states they were ‘never really happy’ with their knee should point you towards malrotation of the components. Bell et al demonstrate that internal rotation of the tibial or femoral components increase the risk of ongoing knee pain. One reason for internal rotation leading to poor outcomes is that this interferes with patella tracking in a negative way, increasing lateral patella facet wear, clunking and anterior knee pain. Bell SW, Young P, Drury C, et al. Component rotational alighnment in unexplained painful primary total knee arthroplasty. Knee. 2014;21(1):272-277. 21. Answer B. Release the popliteus In a valgus knee, the following lateral compartment release maybe required: (1) osteophytes, (2) lateral capsule, (3) iliotibial band, (4) popliteus. The LCL can be released; however, if you are requiring to do this, you should be thinking about increased constraint. Release of the PCL, downsizing the femoral component and resecting more off the tibia will universally increase the flexion gap; however, in this case it is only tight laterally in flexion. 22. Answer D. On measuring the femoral component size using posterior referencing, it measured 4.5 and the decision was made to use a size 4 implant, as a size 5 may overstuff the PFJ When using posterior referencing, the femoral component size is measured referenced off the posterior femoral condyles. If downsizing (as in this case), then you risk notching the femur, which introduces a point of weak bone and increases the risk of periprosthetic knee fractures. When using posterior referencing and wanting to downsize the femoral component, then anteriorising the cutting jig will mitigate

82

the risk of notching; however, doing so carries the risk of overstuffing the PFJ. All other responses are valid intraoperative decisions. 23. Answer B. Due to poor patella tracking, a lateral retinacular release was performed The radiographic changes are consistent with osteonecrosis of the patella. Performing a lateral retinacular release can damage the lateral superior genicular artery, which supplies the patella (the other arterial supply being the medial superior genicular artery, which is incised during a medial parapatellar approach). Denervation of the patella has not been demonstrated to increase this complication. There is no good evidence for or against patella resurfacing; therefore, not resurfacing the patella is a valid surgical decision. Rotating the femur externally is an important step in total knee replacements and helps with patella tracking. Anteriorising the femoral component can overstuff the PFJ and lead to anterior knee pain, but is unlikely to lead to such aggressive presentation as above. 24. Answer B. Two-stage revision A WCC over 2 500 and a PMN count of over 70% is indicative of infection. This, coupled with the increasing pain and osteolysis, leads to the most likely diagnosis being prosthetic joint infection. Arthroscopic washout and DAIR procedures are not applicable in chronic infections. A bone scan can be a useful investigation; however, it is likely to be positive in the presence of the above and therefore unlikely to be helpful. As culture did not grow anything, there is an argument to repeat the aspiration; however, this is not in the answer stem. A one-stage revision for infection should only be performed when the organism is known and is an easily treated organism; as we do not know the organism, the most appropriate management is offering a twostage revision. 25. Answer A. Dorsal aspect of the foot During correction of a valgus knee to neutral mechanical alignment, the common peroneal nerve can be stretched. This would lead to

Knee II Structured SBA

sensory disturbance over the dorsal aspect of the foot and foot drop. 26. Answer C. An Insall–Salvati ratio of 0.7 Insall–Salvati (normal 0.8–1.2) and Caton– Deschamps (normal 0.6–1.3) are assessments of patella height. An Insall–Salvati ratio of 0.7 demonstrates patella baja, which can make a TKA more difficult. Dejour grade C represents a degree of trochlear dysplasia; this on its own does not increase TKA difficulty. A bipartite patella is a normal variant. TT–TG is a measurement of the distance from the tibial tubercle to the trochlear groove in the axial plane. It is a useful measurement in patella instability. A TT–TG of 0–15mm is normal, 15–20 borderline, >20 abnormal. 27. Answer A. Close the wound and wake the patient up ACL insufficiency is a contraindication of a mobile bearing UKA such as the Oxford UKA. A Zuk is a fixed bearing UKA and can be used in the presence of ACL insufficiency; however, you should not be using an implant with which you are unfamiliar. The patient has only been consented for a UKA; therefore, you cannot legally change to a different operation. This patient should have been consented for a UKA +/– TKA. 28. Answer A. 0–95° range of movement You need at least 110° of knee flexion to perform an Oxford knee replacement. This will allow you good access to the femur during preparation. Previous scars can make surgery more challenging; however, arthroscopy scars are fine. You can perform an Oxford knee on knees with up to 15 degrees of fixed flexion or correctable varus. Age is not a contraindication. 29. Answer B. Arthroscopic synovectomy, removal of loose bodies and histopathological analysis

This presentation is consistent with synovial chondromatosis. This is a benign proliferative disease of the synovium characterised by multiple intra-articular loose bodies of cartilage at various stages of calcification. This is most prevalent in the fourth or fifth decade, with males being more commonly affected. The knee is by far the most commonly affected joint. Malignant change into synovial chondrosarcoma is rare. Synovial chondromatosis is best managed with arthroscopic removal of loose bodies and synovectomy. The chance of local recurrence is reasonably common, with studies quoting between 3% and 23%. 30. Answer C. Reassure and discharge with antiinflammatory medication Pseudogout (calcium pyrophosphate dihydrate deposition) is characterised by effusion and weakly positive birefringent crystals on joint microscopy. It is more common in the elderly population. X-ray findings can demonstrate chondrocalcinosis and erosions, but in the early stages can be normal. Causes can be idiopathic, hereditary (AD pattern) or secondary to another systemic disease such as haemochromatosis, hyperparathyroidism, hypothyroidism, SLE or renal disease. Treatment of pseudogout includes anti-inflammatory medication, steroid joint injection and management of the causative condition if present. If there were signs consistent with septic arthritis, then joint washout or admitting awaiting full culture results would be appropriate. An MRI would be appropriate if there was a history of trauma with a normal knee x-ray. Gout is negatively birefringent on joint microscopy, and management includes starting anti-inflammatory medication for an acute flareup with allopurinol started in the community once the acute flare-up has improved.

83

Section 2 Chapter

7

Adult Elective Orthopaedics and Spine

Foot and Ankle Structured SBA Gavin Heyes and Lyndon Mason

FOOT AND ANKLE STRUCTURED SBA QUESTIONS Anatomy and Biomechanics 1. From the options listed below, please choose the most appropriate description of the anatomy of the spring ligament. A. Originates on the lateral malleolus and inserts onto the lateral aspect of the talus B. Originates on the medial malleolus and inserts onto the medial wall of the calcaneum C. Originates on the medial malleolus and inserts onto the medial aspect of the talus D. Originates on the navicular and inserts onto the medial cuneiform E. Originates on the sustentaculum tali and inserts onto the navicular 2. From the options listed below, please choose the most appropriate anatomical structure to occur in the third layer of the foot. A. Abductor digiti minimi B. Adductor hallucis C. Peroneus longus D. Plantar interossei E. Quadratus plantae 3. From the options listed below, please choose the most appropriate muscle that plantar flexes the 1st metatarsal. A. Flexor hallucis brevis B. Flexor hallucis longus C. Peroneus longus D. Tibialis anterior E. Tibialis posterior

84

4. From the options listed below, please choose the most appropriate description of the anatomy of the Lisfranc ligament. A. Dorsal ligament between to medial cuneiform and the 2nd metatarsal B. Interosseous ligament between the medial cuneiform and the 2nd metatarsal C. Plantar ligament between the 2nd metatarsal and 5th metatarsal D. Plantar ligament between the intermediate cuneiform and the 2nd metatarsal base E. Plantar ligament between the medial cuneiform and the 2nd metatarsal 5. What structure attaches to the plantar surface of the hallucal sesamoids? A. Abductor hallucis B. Adductor hallucis C. Flexor hallucis brevis D. Lateral sesamoid ligament E Medial sesamoid ligament 6. What is the main blood supply of the talar body? A. Anterior tibia artery B. Artery of the sinus tarsi C. Perforator artery D. Peroneal artery E. Posterior tibial artery 7. Which muscle contracts eccentrically during the heel strike phase of the gait cycle? A. Extensor hallucis longus B. Lateral head of gastrocnemius C. Medial head of gastrocnemius D. Tibialis anterior E. Tibialis posterior

Foot and Ankle Structured SBA

8. What nerve supplies adductor hallucis? A. Baxter’s nerve B. Deep peroneal nerve C. Lateral plantar nerve D. Medial plantar nerve E. Superficial peroneal nerve

Achilles and Heel 9. What is the most common aetiological factor for plantar fasciopathy? A. Calf tightness B. Depression C. Obesity D. Smoking E. Job requiring standing 10. Which of the following best describes the reactive phase of Achilles tendinopathy? A. Increased production of large proteoglycans, which bind with large amounts of water B. Cell death, minimal fibrillar collagen C. Collagen fibre dysrepair, increased chondrocytic cellularity D. Production of type III collagen E. Ingrowth of neovessels 11. What is the most likely diagnosis with pain occurring on calcaneal squeeze test? A. Calcaneal fracture B. Flexor hallucis longus tendinopathy C. Plantar fasciopathy D. Radiculopathy E. Tarsal tunnel syndrome 12. What is the most common diagnosis when pain occurs on palpation of the medial plantar aspect of the calcaneum? A. Insertional Achilles tendinopathy B. Insertional plantar fasciopathy C. Medial calcaneal nerve neuropraxia D. Non-insertional Achilles tendinopathy E. Tarsal tunnel syndrome 13. What is the first line treatment of plantar fasciopathy? A. Laser therapy B. Physiotherapy C. Mechanical overload reduction

D. Shockwave therapy E. Ultrasound-guided pulse radiofrequency 14. What is the most effective treatment for noninsertional Achilles tendinopathy in the literature? A. Autologous blood injection B. Exercise C. GTN patches D. Orthotic E. Polidocanol injection 15. What statement is true in regard to Achilles tendon ruptures? A. Functional rehabilitation is equal to surgical treatment regarding the incidence of rerupture B. Immobilisation increases load to failure C. In the acute phase, collagen type I is the first collagen that is layered down D. Use extrinsic healing with passive motion E. Use of heel wedges in functional rehabilitation works by shortening tendon

Midfoot 16. Which statement is true in regard to Lisfranc injuries? A. A tightrope controls axial movement B. Fusion improves functional outcome as compared to fixation C. Quality of anatomical reduction is the best predictor of functional outcomes D. MRI is indicated when radiographs show diastasis E. Transarticular screws reduce movement at the tarsometatarsal joint as compared to bridge plate 17. ‘Too many toes’ sign would most likely be seen in which case? A. Charcot–Marie–Tooth disease B. Fibular hemimelia C. Stage IIA tibialis posterior tendon dysfunction D. Stage IIB tibialis posterior tendon dysfunction E. Iselin’s disease

85

Gavin Heyes and Lyndon Mason

18. At the level of the navicular, when harvesting flexor digitorum longus for tibialis posterior tendon reconstruction, what structure lies immediately dorsal or deep to it? A. Adductor hallucis B. Baxter’s nerve C. Flexor hallucis longus D. Lumbricals E. Plantar fascia 19. A patient attends your clinic after transfer of care. They attend with an in-shoe orthotic in a bag. The orthotic is built up on the lateral side of the heel and contains a depressed section at the medial aspect of the forefoot. What condition would the likely pathology be associated with? A. Charcot–Marie–Tooth disease with foot drop B. Diastematomyelia with grade 5 power in all muscle groups C. Rheumatoid arthritis D. Tarsal coalition E. Varus malunion of os calcis with posttraumatic osteoarthritis subtalar joint 20. A 53-year-old is currently being operated on for a fixed flat foot deformity. The surgeon has just completed preparation of the hindfoot for fusion, reduced it and held it with guidewires. The surgeon notes a forefoot deformity that will require intervention. What is the deformity likely to be? A. Abduction and supination deformity B. Adduction deformity C. Pronation deformity D. Valgus deformity E. Varus deformity 21. A patient attends clinic for the results of an excision biopsy performed from a discrete firm lesion in the foot. The histopathology report included no cell atypia, myofibroblast proliferation and collagen proliferation of type III collagen more so than type I collagen. What is the likely diagnosis? A. Fibromyxoma B. Gardner’s disease C. Ledderhose disease D. Lipoma E. Synovial sarcoma

86

Forefoot 22. A 42-year-old female attends with a painful bunion she has had for the past 2 years. She has failed conservative measures. On examination, there is no first ray instability or pain on grind test. Her radiographs reveal a hallux valgus angle of 25°and intermetatarsal angle of 12°. What is the most appropriate surgical intervention? A. Basal osteotomy B. Distal chevron osteotomy C. Lapidus fusion D. Moberg osteotomy E. Proximal phalanx osteotomy 23. A 42-year-female with a bunion presents with pain on shod weight bearing originating at the 2nd and 3rd metatarsal heads. Which of the following is the most likely diagnosis? A. Claw toe B. Freiberg’s disease C. Plantar plate rupture D. Morton’s neuroma E. MTP joint synovitis 24. A 23-year-old professional footballer sustains an undisplaced diaphyseal proximal 5th metatarsal fracture during training after a few weeks of grumbling about foot pain. What is the most appropriate treatment? A. Cast immobilisation and non-weight bearing B. Internal fixation C. Metatarsal strapping and full weight bearing D. Metatarsal strapping and non-weight bearing E. Stiff shoe 25. What is the most frequent iatrogenic complication of excision of both tibial and fibular sesamoids of the hallux? A. Flexor hallucis longus tendonitis B. Hallux valgus C. Hallux varus D. Intractable keratosis E. Weakness on tip toe stance 26. Floating toe is an iatrogenic complication most frequently associated with which procedure? A. Keller’s resection arthroplasty

Foot and Ankle Structured SBA

B. C. D. E.

Kidner’s procedure Morton’s neuroma excision Stainsby procedure Weil metatarsal osteotomy

27. What force on the proximal phalanx is created by the abductor hallucis in a hallux valgus deformity? A. Adduction B. Adduction and supination C. Dorsiflexion and pronation D. Plantar flexion E. Plantar flexion and pronation 28. A fit and healthy 75-year-old presents to clinic with pain, stiffness, swelling, erythema over her left 1st metatarsal phalangeal joint (MTPJ) following a silastic 1st MTPJ arthroplasty performed 12 years ago. Radiographs demonstrate osteolysis around the component and a valgus deformity. Blood tests are normal with regard to full blood picture, erythrocyte sedimentation rate and C-reactive protein. What is the appropriate treatment? A. First stage of two-stage 1st MTPJ fusion B. Revision to 1st MTPJ fusion with inlay bone graft C. Revision to excision arthroplasty D. Revision to silastic arthroplasty E. Steroid injection 29. An 18-year-old young woman presents with forefoot pain and stiffness of the 2nd metatarsal phalangeal joint. Pain worsened on axial loading of the joint. Radiographs demonstrate arthrosis and flattening of the metatarsal head. What is the likely diagnosis? A. Kohler’s disease B. Sever’s disease C. Freiberg’s disease D. Mueller–Weiss syndrome E. Turf toe 30. A 1-year-old female infant is brought into your clinic. Her mother is concerned about her 4th toes bilaterally. They appear to be shortened and overlapping the 5th toes. Radiographs demonstrate disruption of Maestro’s parabola and premature closure of the 4th metatarsal physis. You

also notice the child to be smaller than average, with a short, webbed neck. What condition could this child have? A. Klippel–Feil syndrome B. Poland syndrome C. Sprengel deformity D. Trisomy 21 E. Turner’s syndrome 31. An 83-year-old nursing home patient presents with pain over 2nd toe, inability to fit into shoes, chronic paronychia and an ulcer over the 2nd toe proximal interphalangeal joint (PIPJ). The PIPJ is fixed in flexion, the metatarsal phalangeal joint (MTPJ) is slightly extended and the distal interphalangeal joint (DIPJ) is hyperextended and flexible. What is the most appropriate treatment? A. Amputation through proximal one-third of proximal phalanx B. MTPJ release + DIPJ fusion C. MTPJ release + flexor to extensor transfer + temporary K-wire fixation D. MTPJ release + MTPJ replacement E. MTPJ release + PIPJ fusion + temporary Kwire fixation 32. A 46-year-old man presents with pain in the right 1st metatarsal phalangeal joint (MTPJ). He recalls a rugby injury to the joint 20 years ago. On examination he has moderate pain on end range of motion, reduced range of motion (10° dorsiflexion, 50° plantar flexion) and pain on axial loading of the 1st MTPJ. Radiographs demonstrate around 40% joint space narrowing with dorsal osteophyte on the metatarsal and phalanx. He has failed conservative treatment and still wishes to play rugby. What is the most appropriate treatment? A. Dorsal cheilectomy B. Keller’s procedure C. MTPJ arthrodesis D. MTPJ arthroplasty E. Synovectomy 33. When performing a scarf osteotomy in an otherwise normally aligned foot, to avoid shortening or lengthening the 1st metatarsal, what landmark or reference point should be used for your distal transverse cut?

87

Gavin Heyes and Lyndon Mason

A. Parallel to proximal phalanx base joint surface B. Parallel with the 5th metatarsal phalangeal (MTP) joint C. Perpendicular to 1st metatarsal D. Perpendicular to 2nd metatarsal E. Perpendicular to cut surface after removing medial eminence 34. An 18-month-old presents to your clinic with bilateral deformities of their 4th and 5th toes. On examination, there is a flexion and varus deformity to all toes. What is the likely cause of the deformities? A. Absence of extensor digitorum B. Central nervous system lesion C. Congenital bands around toes D. Contracture of flexor digitorum longus and/ or brevis. E. Delta phalanx

Ankle 35. A 28-year-old man is seen in a nurse-led dressings clinic 2 weeks after arthroscopic ankle surgery. He is complaining of persistent numbness over the dorsum of his foot but not in the first web space. What is the most likely cause of this complication? A. Anterocentral portal placement B. Anterolateral portal placement C. Anteromedial portal placement D. Posterolateral portal placement E. Posteromedial portal placement 36. A normally fit and well 32-year-old- woman is taken to theatre for examination under anaesthesia for chronic ankle instability despite several courses of physiotherapy. On the lateral image, 10mm of forward shift is demonstrated during an anterior drawer test compared to the unaffected ankle. Which ligament is most likely damaged? A. Anterior inferior tibiofibular ligament B. Anterior talar fibular ligament C. Calcaneofibular ligament D. Posterior inferior tibiofibular ligament E. Posterior talar fibular ligament 37. A 43-year-old man sustains as pronation external rotation injury to his left ankle while playing

88

football. Initial radiographs show increased tibiofibular clear space. Which of these is the correct group of ligaments which form the structure that is injured, resulting in the increased tibiofibular clear space? A. AITFL, ATFL, PITFL B. AITFL, IOL, CFL C. AITFL, IOL, PITFL D. PITFL, CFL, ATFL E. PITFL, IOL, ATFL 38. A patient is brought into the emergency department after sustaining an ankle injury while playing basketball. The ankle is grossly swollen and tender. Radiographs show a pronation external rotation injury pattern. Which answer best describes the sequence of injury? A. ATFL disruption, oblique fibula fracture at the level of the syndesmosis, PITFL disruption or posterior malleolus fracture, transverse medial malleolus fracture or deltoid ligament injury B. Medial malleolus transverse fracture or deltoid disruption, ATFL disruption, lateral short oblique fracture or spiral fracture of the fibula above the syndesmosis, PITFL avulsion or posterior malleolus fracture C. Medial malleolus transverse fracture or deltoid disruption, ATFL disruption, lateral short oblique/ spiral fracture of the fibula below the syndesmosis, PITFL avulsion or posterior malleolus fracture D. Medial malleolus transverse fracture or deltoid disruption, ATFL disruption, transverse or comminuted fibula fracture above the level of the syndesmosis. E. Vertical medial malleolus fracture, ATFL disruption or fibula fracture below the joint line 39. A 30-year-old keen cross country runner presents to a foot and ankle clinic with non-specific ankle pain and occasional swelling. They describe a couple of episodes of mild ankle sprains over the past couple of years. An osteochondral defect of the talus is suspected. What is the most common location for these injuries on the talus? A. Anterior lateral talar dome B. Central anterior talar dome

Foot and Ankle Structured SBA

C. Central lateral talar dome D. Central medial talar dome E. Posterior medial talar dome 40. A 70-year-old man presents to an elective foot and ankle clinic with pain in his right ankle. He had an ankle injury 30 years ago, which he was told could not be operated on at the time and was managed in a cast. Weight bearing ankle radiographs show end stage ankle arthritis. The patient has decided on an ankle arthrodesis for treatment. During ankle arthrodesis surgery, what is the ideal position of the ankle? A. 10° dorsiflexion, 10° external rotation, 5° hindfoot valgus B. 10° dorsiflexion, neutral external rotation, 5° hindfoot valgus C. Neutral dorsiflexion, 10 ° external rotation, 5° hindfoot varus D. Neutral dorsiflexion, 10° external rotation, 5° hindfoot valgus E. Neutral dorsiflexion, neutral external rotation, 5° hindfoot valgus 41. What is the risk of talus avascular necrosis following a talar neck fracture associated with subtalar dislocation? A. 0–5% B. 5–10% C. 15–20% D. 55–60% E. 95–100% 42. What ankle position at the time sustaining a Pilon fracture results in the worst outcomes? A. Dorsiflexion B. Neutral C. Plantar flexion D. Valgus E. Varus 43. What is the most common malignancy of the foot? A. Ewing’s sarcoma B. Liposarcoma C. Metastasis D. Osteosarcoma E. Synovial sarcoma

Diabetes 44. A 33-year-old male with a history of ataxia, diminished proprioception, contralateral pes cavus and clawing suffers an acute unilateral painless midfoot collapse with associated erythema and swelling. Which of the following aetiological factors would not typically be associated with the likely underlying process? A. Diabetes mellitus B. Hansen’s disease C. Still’s disease D. Syphilis E. Syringomyelia 45. A patient with diabetes type 2 develops a midfoot collapse with ulceration. A tissue viability nurse describes the ulcer as grade 2. The radiographs are reported as a ‘rocker bottom deformity with osseous fragmentation’. What is the most appropriate management? A. Exostectomy B. Midfoot plantar flexion osteotomy and fixation with a mega construct C. Negative pressure dressing and an ankle–foot orthosis D. Range of motion walker E. Total contact casting 46. A 49-year-old male patient with a history of type 2 diabetes, neuropathy and retinopathy presents to the diabetic foot and ankle MDT clinic with a chronic foot ulcer. Which of the following is a negative predictor for diabetic ulcer wound healing? A. Arterial brachial pressure index ratio of 0.5 B. Inability to feel a 5.07 Semmes–Weinstein monofilament around the ulcerated area. C. Serum albumin 2.9 g/dL D. Total lymphocyte count 2.8 (x109/L) E. Transcutaneous oxygen pressure 41mm Hg 47. Which of the following is the weakest predictor of associated osteomyelitis in diabetic foot ulcers? A. MRI with Gadolinium demonstrating contrast enhancement B. Positive organism and sensitivity following ulcer wound swab C. Radiographs demonstrating cortical erosions D. Ulcer that probes to bone E. White blood cell scan

89

Gavin Heyes and Lyndon Mason

FOOT AND ANKLE STRUCTURED SBA ANSWERS Anatomy and Biomechanics

1. Answer E. Originates on the sustentaculum tali and inserts onto the navicular Otherwise known as the plantar calcaneonavicular ligamentous complex, it is a broad and thick band with three constituent ligaments that connect the anterior margin of the sustentaculum tali of the calcaneus to the plantar surface of the navicular. Its individual components are the superomedial, intermedial (medioplantar) and lateral (inferoplantar) ligaments, which fan out and attach to the navicular bone at three separate locations. The ligament combines with the superficial and deep deltoid to provide the ligamentous support to the proximal aspect of the medial longitudinal arch. The distal aspect of the medial longitudinal arch is supported by the navicularcuneiform ligament. Campbell KJ, Michalski MP, Wilson KJ, et al. The ligament anatomy of the deltoid complex of the ankle: a qualitative and quantitative anatomical study. J Bone Joint Surg Am. 2014 Apr 16;96 (8):e62. Taniguchi A, Tanaka Y, Takakura Y. Anatomy of the spring ligament. J Bone Joint Surg Am. 2003;85(11):2174-2178. Swanton E, Fisher L, Fisher A, Molloy A, Mason L. An anatomic study of the naviculocuneiform ligament and its possible role maintaining the medial longitudinal arch. Foot Ankle Int. 2019 Mar;40(3):352-355. 2. Answer B. Adductor hallucis  Layer 1 – 3 muscles – abductor hallucis, abductor digiti minimi, flexor digitorum brevis  Layer 2 – 2 muscle groups, 2 tendon groups – flexor hallucis longus, flexor digitorum longus lumbricals, quadratus plantae  Layer 3 – 3 muscles – adductor hallucis, flexor hallucis brevis, flexor digiti minimi brevis  Layer 4 – 2 muscles groups, 2 tendons – dorsal interossei, plantar interossei, peroneus longus, tibialis posterior

90

3. Answer C. Peroneus longus Peroneus longus – The peroneus longus inserts onto the 1st metatarsal base and to a lesser extent the medial cuneiform, traversing the plantar aspect of the foot from lateral to medial. The action is to plantar flex the 1st metatarsal, and participate in the eversion of the foot. In nonbipedal apes, the peroneus longus lateralised the first ray, which was used for grasping. Flexor hallucis longus – This muscle inserts onto the base of the distal phalanx of the hallux. It acts to flex all joints of the hallux and, to a lesser extent, plantar flex the ankle joint. Flexor hallucis brevis – This muscle inserts on the medial and lateral sides of the proximal phalanx, comprising a component of the hallucal sesamoid complex. It flexes the hallux along with the flexor hallucis longus muscle. Tibialis anterior – This muscle inserts onto the dorsal and medial aspect of the medial cuneiform and 1st metatarsal. Its function is to dorsiflex and invert the foot. Tibialis posterior – This muscle inserts onto the navicular and medial cuneiform, and in a variable nature onto the other cuneiforms. It acts to invert the foot and plantar flex the foot at the ankle. 4. Answer B. Interosseous ligament between the medial cuneiform and the 2nd metatarsal The ligaments at the 2nd metatarsal base have a unique arrangement in that there is no intermetatarsal ligament between the 1st and 2nd metatarsals. Instead, in addition to the dorsal ligaments, there are two ligaments between the medial cuneiform and 2nd metatarsal base. These two large ligaments maintain the relationship of the 2nd metatarsal base to the medial cuneiform. The interosseous ligament, also called the Lisfranc ligament, attaches to the lateral aspect of the medial cuneiform and the medial aspect of the 2nd metatarsal base. The plantar ligament attaches to the lateral aspect of the medial cuneiform and the plantar aspect of the base of the 2nd and 3rd metatarsals. There are three dorsal ligaments attached to the 2nd metatarsal base, one from each of the first three cuneiforms.

Foot and Ankle Structured SBA

Solan et al1 showed that on biomechanical testing the plantar and Lisfranc ligaments were significantly stiffer and stronger than the dorsal ligament, and the Lisfranc ligament was significantly stronger and stiffer than the plantar ligament. Panchbhavi VK, Molina D 4th, Villarreal J, Curry MC, Andersen CR. Three-dimensional, digital, and gross anatomy of the Lisfranc ligament. Foot Ankle Int. 2013 Jun;34(6):876-880

artery, providing branches to the superior surface of the body and neck of the talus.2 Gelberman and Mortensen’s work3 changed the consensus to the posterior tibial artery as the main blood supply, along with its anastomotic network to the sinus tarsi artery, allowing only retrograde blood flow to the talar body from the talar neck. Miller et al4 used a gadolinium-enhanced MRI study and found the posterior tibial artery to be the main contributor, like Gelberman and Mortensen’s initial findings. However, they found an entry point not only at the talar neck but also an antegrade flow entry point at the posterior tubercle.

5. Answer C. Flexor hallucis brevis The sesamoids are closely connected with the fibrous layer of the joint capsule as well as with the medial and lateral sesamoid ligaments that are blended with the capsule. Sharpey’s fibres from the sesamoid ligaments penetrate the sesamoids on their capsular side. Anterior to the medial and lateral sesamoid ligaments are the collateral ligaments that fan out distally and plantarward, connecting to the base of the proximal phalanx. The dense, fibrous plantar pad enshrouds the plantar aspect of the sesamoids and anchors the sesamoid complex to the base of the proximal phalanx. The tendons of the flexor hallucis brevis are attached to the plantar surface of the sesamoids. However, the tendons of the adductor and abductor hallucis mainly bypass the sesamoids. The intersesamoid ligament connects the sesamoids. The flexor hallucis longus nestles on the plantar aspect between the sesamoids with the inter-sesamoid ligament, blending with its synovial tendon. 6. Answer E. Posterior tibial artery The talus is 60% covered by articular cartilage. Blood vessels enter the talus via capsular and ligamentous attachments, limiting the arterial entry sites to the talar neck, the medial surface of the body below the medial malleolus, the sinus tarsi and the posterior tubercle. Earlier studies seemed to confirm that the main blood supply was from the anterior tibial

7. Answer D. Tibialis anterior The tibialis anterior contracts eccentrically during heel strike; thus, it acts to help control the lowering of the foot onto the ground. Injury to the tibialis anterior or to the motor nerve supply (deep peroneal nerve) will mean that the foot will slap onto the ground during initial contact. The tibialis anterior will contract concentrically during the initial and midswing phases of the gait cycle. If the tibialis anterior is weak during these phases, then clearance of the foot becomes a problem, which can lead to a compensatory high stepping gait. Bland DC, Prosser LA, Bellini LA, Alter KE, Damiano DL. Tibialis anterior architecture, strength, and gait in individuals with cerebral palsy. Muscle Nerve. 2011 Oct;44(4):509-517. Brunner R, Rutz E. Biomechanics and muscle function during gait. J Child Orthop. 2013 Sep 15;7(5):367-371 8. Answer C. Lateral plantar nerve Five main nerves innervate the foot (Table 7.1).

2

3

1

Solan MC, Moorman CT 3rd, Miyamoto RG, Jasper LE, Belkoff SM. Ligamentous restraints of the second tarsometatarsal joint: a biomechanical evaluation. Foot Ankle Int. 2001 Aug;22(8):637-641.

4

Phemister DB. Changes in bones and joints resulting from interruption of circulation: I. General considerations and changes resulting from injuries. Arch Surg. 1940;41(2):436-472. Gelberman RH, Mortensen WW. The arterial anatomy of the talus. Foot Ankle. 1983;4(2):64-72. Miller AN, Prasarn ML, Dyke JP, Helfet DL, Lorich DG. Quantitative assessment of the vascularity of the talus with gadolinium-enhanced magnetic resonance imaging. J Bone Joint Surg Am. 2011;93:1116-1121

91

Gavin Heyes and Lyndon Mason

Table 7.1 Nerve anatomy of the foot

Nerve

Motor function in the foot

Sensory function

Lateral plantar nerve

Adductor hallucis Quadratus plantae Lumbricals 4–5 Interossei

Plantar foot + lateral 1½ digits

Medial plantar nerve

Flexor hallucis brevis Flexor digitorum brevis Lumbricals 2–3 Abductor hallucis

Plantar foot + medial 3½ digits

Baxter’s nerve (first branch of lateral plantar)

Abductor digiti minimus

Deep peroneal nerve

Extensor hallucis brevis Extensor digitorum brevis

1st web space (dorsal)

Superficial peroneal nerve

-

Medial branch – dorsomedial hallux Lateral branch – dorsum foot

Achilles and Heel

9. Answer A. Calf tightness Individuals with calf tightness, defined as less than 0 degrees of ankle dorsiflexion, are 23 times more likely to have plantar heel pain. Obesity (BMI greater than 30) are 2.9 times more likely to have plantar heel pain. Those with jobs involving standing are 3.6 times more likely to have plantar heel pain if they stand for long periods throughout day. This is because peak forces for walking and running occur at 60% of stance, with the plantar fascia taking 1.8 BW during walking and 3.7 BW during running. Obesity does not only affect the plantar fascia due to weight. Tenocyte inhibition, weaker collagen and cytokines occur with obesity. Hypercholesterolaemia is also associated with Achilles tendon ruptures. For every 1 unit increase in depression, anxiety or stress (in the DASS subscales), the odds ratios for having plantar heel pain are increased by 1.3.

10. Answer A. Increased production of large proteoglycans, which bind with large amounts of water It is important to understand the pathophysiology and ‘patient specific’ risk factors to individualise tendinopathy treatment and develop sound treatment theory where evidence is lacking. The continuum model proposed by Cook and Purdam is well established. They proposed that the cell response is the initial trigger in tendinopathy

92

and not collagen breakdown. The continuum contains 3 phases: reactive, dysrepair and degeneration. Tenocytes manufacture components of extracellular matrix. The extracellular matrix’s main component is tightly packed collagen. Proteoglycans in tendons are typically small. Phases

Reactive – The tenocyte responds to acute overload by increasing the production of large proteoglycans, which bind with large amounts of water. The tendon therefore swells homogenously (fusiform), which serves to increase cross-sectional area. This increase in cross-sectional area reduces strain. The response takes minutes to a few days. No collagen damage or neovessels occur. The goal of treatment in this phase is to reduce tenocyte activation/response. So, the swelling associated with the reactive phase is not inflammation. Dysrepair – The ongoing reaction to inappropriate loading leads eventually to collagen fibre dysrepair, increased chondrocytic cellularity and proteins with the production of weak type III collagen. Disorganisation leads to early ingrowth of neovessels. Clinically, it can be difficult to distinguish this stage, but it can be picked up on imaging. Changes remain reversible. Degeneration – This is typified by areas of cell death, minimal fibrillar collagen and the ingrowth of neovessels. In tendon matrix

Foot and Ankle Structured SBA

heterogeneity, the tendon becomes knobbly due to fibrotic thickening. Irreversible changes occur within those areas, but this does not necessarily mean clinical benefit cannot be achieved. Treatment requires stimulation of cell response. Cook JL, Purdam CR. Is tendon pathology a continuum? A pathology model to explain the clinical presentation of load-induced tendinopathy. Br J Sports Med. 2009 Jun;43(6):409-416. 11. Answer A. Calcaneal fracture The following tests are described for the diagnosis of the cause of heel pain: Heel squeeze test – Calcaneal stress fracture is typically painful by squeezing the calcaneum from both sides. Silfverskiold’s test – Passive ankle dorsiflexion with the knee flexed to 90 degrees. Abnormal if when going from extension to flexion, there is an increase in passive dorsiflexion of the ankle. This diagnoses a tight gastrocnemius complex. Passive toe dorsiflexion (plantar fasciopathy) – Tightens the windlass mechanism and exacerbates pain. Dorsiflexion-eversion test (tarsal tunnel syndrome) – Tibial nerve is compressed

Table 7.2 Effectiveness of treatment options for plantar fasciopathy compared to placebo

Intervention

Improvement as compared to placebo

Improvement VAS clinically better than placebo

Ultrasoundguided pulsed radiofrequency

High

No

Low-level laser therapy

Moderate

No

Dry needling

Moderate

No

Calcaneal taping

Moderate

No

Shockwave therapy

Low

No

Orthotic

Low

No

Calf muscle stretching

No

No

Plantar fascia stretching

No

No

Low-dye taping

No

No

Pulsed radiofrequency electromagnetic field

No

No

12. Answer B. Insertional plantar fasciopathy This is pathagnomic of plantar fasciopathy. Other common pathological sites are shown in Figure 7.1. 13. Answer C. Mechanical overload reduction In the majority of cases, plantar fasciopathy is a self-limiting problem. If you do nothing, then 90% will resolve by 10 months. First line treatment usually involves reducing mechanical overload such as losing weight, reducing calf tightness and changing activity. All other conservative measures have been investigated quite extensively with level 1 evidence. A summary of the conclusions from a level 1 meta-analysis by Savioli et al5 showed no intervention had a clinical improvement compared to placebo on the visual analogue scale (VAS) (Table 7.2). 5

Salvioli S, Guidi M, Marcotulli G. The effectiveness of conservative, non-pharmacological treatment, of plantar heel pain: a systematic review with meta-analysis. Foot. 2017;33:57-67.

Figure 7.1 Pathological sites of pain

93

Gavin Heyes and Lyndon Mason

14. Answer B. Exercise Wilson et al6 completed a level 1 meta-analysis on exercise splint and orthotics for noninsertional Achilles tendinopathy. In regard to exercise, they included 22 studies (1 137 participants). The strength of evidence for all studies was ‘moderate’ for reducing pain (VAS), improving function (assessed by VISA-A) and improving quality of life (SF-36). The current balance of evidence does not support a clear recommendation of one type of exercise protocol over another for Achilles tendinopathy. In regard to orthotics, there were two RCTs showing no improvement in pain or function as compared to placebo. A level 1 meta-analysis by Lin et al7 on autologous blood-derived products (including PRP) found that in seven RCTs there was no difference to placebo in functional outcomes. One RCT by Ebbesen et al8 on polidocanol injection versus lidocaine showed no significant difference in any outcome. There are two RCTs on GTN patches versus placebo that showed in the short-term there were no significant differences detected in any of the outcomes. In the treatment group, 20% had to discontinue patch application due to headaches. 15. Answer A. Functional rehabilitation is equal to surgical treatment regarding the incidence of re-rupture In the initial phase of tendon healing, collagen type III is the initial collagen layered down. After 3 days, type I collagen production increases from 15- to 22-fold. After 2 weeks, a fibrous bridge consisting of fibroblasts and collagen fibres fuses the tendon. Between 3 and 4 weeks, the collagen 6

7

8

94

Wilson F, Walshe M, O’Dwyer T, Bennett K, Mockler D, Bleakley C. Exercise, orthoses and splinting for treating Achilles tendinopathy: a systematic review with metaanalysis. Br J Sports Med. 2018 Dec 1;52(24):1564-1574. Lin MT, Chiang CF, Wu CH, Hsu HH, Tu YK. Metaanalysis comparing autologous blood-derived products (including platelet-rich plasma) injection versus placebo in patients with Achilles tendinopathy. Arthroscopy. 2018 1;34(6):1966-1975. Ebbesen BH, Mølgaard CM, Olesen JL, Gregersen HE, Simonsen O. No beneficial effect of Polidocanol treatment in Achilles tendinopathy: a randomised controlled trial. Knee Surg Sports Traumatol Arthrosc. 2018;26(7):2038-2044.

fibres begin to organise longitudinally, a process that continues for a number of months. Collagen fibril crosslinking improves with applied stress. Healing tendons undergoing passive motion will undergo intrinsic healing from tendon cells from the epitenon. If immobilised, the tendon heals predominately by granulation tissue from the endotenon. Prolonged immobilisation causes decreased fibrillogenesis, and collagen and elastic fibres are less organised. Protective passive mobilisation has been shown to increase load to failure significantly when compared to immobilisation. The most recent meta-analysis by Zhang et al9 included a total of nine meta-analyses. When functional rehabilitation was used, conservative intervention was equal to surgical treatment regarding the incidence of re-rupture, range of motion, calf circumference and functional outcomes while reducing the incidence of other complications. Where functional rehabilitation was not performed, conservative intervention could significantly increase re-rupture rate. Critical to using a functional walking orthosis for an Achilles tendon rupture is ensuring the orthosis achieves the required equinus. Ellison et al10showed that using heel wedges causes the foot to flex at the midfoot and the Achilles does not shorten.

Midfoot

16. Answer B. Fusion improves functional outcome as compared to fixation Most studies agree that the quality of anatomical reduction is the best predictor of functional outcomes. Smith et al11 performed a meta-analysis 9

10

11

Zhang H, Tang H, He Q, et al. Surgical versus conservative intervention for acute Achilles tendon rupture: a PRISMA-compliant systematic review of overlapping meta-analyses. Medicine (Baltimore). 2015;94(45):e1951. Ellison P, Molloy A, Mason LW. Early protected weightbearing for acute ruptures of the Achilles tendon: do commonly used orthoses produce the required equinus?. J Foot Ankle Surg. 2017 Sep 1;56(5):960-963. Smith N, Stone C, Furey A. Does open reduction and internal fixation versus primary arthrodesis improve patient outcomes for Lisfranc trauma? A systematic review and meta-analysis. Clin Orthop Relat Res. 2016;474(6):1445-1452.

Foot and Ankle Structured SBA

on fixation vs fusion, where three trials met criteria for inclusion. There was no difference in PROMs or alignment; the only difference was hardware removal. In a biomechanical study, transarticular screws and dorsal plates showed similar ability to reduce the first and second TMT joints after TMT and Lisfranc ligament transection and to resist TMT joint displacement with weight bearing load.12 In regard to the use of tightrope, biomechanical studies have shown this can control translation, but there is no axial control; therefore, this is often combined with bridge plating.13 17. Answer D. Stage IIB tibialis posterior tendon dysfunction This sign is indicative of forefoot abduction. This may be seen following clinical examination and would most likely be seen in stage IIB tibialis posterior tendon dysfunction. This sign is caused by forefoot abduction; it would be seen in association with talonavicular uncoverage >40%. Stage IIA does not have significant talonavicular uncoverage and thus no ‘too many toes’ sign (Table 7.3). Fibular hemimelia is associated with a ball and socket ankle and tarsal coalition. In more severe cases, the lateral rays are also deficient, and therefore will not have ‘too many toes’. Charcot–Marie–Tooth disease would be associated with pes cavus and forefoot adductus. Iselin’s disease is an apophysitis of the base of the 5th metatarsal. Johnson KA, Strom DE. Tibialis posterior tendon dysfunction. Clin Orthop Relat Res. 1989;239:196-206. Myerson MS. Adult acquired flatfoot deformity: treatment of dysfunction of the posterior tibial tendon. Instr Course Lect. 1997;46:393-405.

12

13

Alberta FG, Aronow MS, Barrero M, Diaz-Doran V, Sullivan RJ, Adams DJ. Ligamentous Lisfranc joint injuries: a biomechanical comparison of dorsal plate and transarticular screw fixation. Foot Ankle Int. 2005 Jun;26 (6):462-473. Ahmed S, Bolt B, McBryde A. Comparison of standard screw fixation versus suture button fixation in Lisfranc ligament injuries. Foot Ankle Int. 2010 Oct;31(10):892896.

Table 7.3 Myerson modification of Johnson and Strom classification of adult acquired flat foot deformity

Stage

Description

I

Mild medial pain and swelling with no deformity, can perform heel-rise test but demonstrates weakness on repetition, tenosynovitis on pathology with normal tendon length

II

Moderate pain with or without lateral pain, flexible deformity, unable to perform heel-rise test, elongated tendon with longitudinal tears

IIA

30% talar head uncoverage

III

Severe pain, fixed deformity, unable to perform heel-rise test, visible tears on pathology

IV

Lateral talar tilt

IVA

Flexible ankle valgus without severe arthritis

IVB

Fixed ankle valgus with or without arthritis

18. Answer C. Flexor hallucis longus Flexor hallucis longus (FHL) runs with flexor digitorum longus (FDL) and intersects at the knot of Henry. Distal to the knot of Henry there are often multiple other connections between the two tendons, allowing the harvesting of the tendon with minimal functional loss. Baxter’s nerve, the first branch of the lateral plantar nerve, turns medially around the calcaneus to travel laterally, sending the branch to the abductor digiti quinti (ADQ). The plantar fascia is plantar and superficial to FDL. Adductor hallucis is deeper and more laterally located and does not become accessible until further distal. Lumbricals are further deep and dorsal. 19. Answer B. Diastematomyelia with grade 5 power in all muscle groups What is described is a corrective orthotic for pes cavus. Rheumatoid arthritis and tarsal coalitions are most likely associated with pes planus; in addition, tarsal coalitions may be bony and therefore unlikely to respond to a corrective orthotic. Osteoarthritis of the subtalar joint may not respond well to a corrective orthotic

95

Gavin Heyes and Lyndon Mason

due to the stiffness in movement of the joint. Diastematomyelia (a congenital disorder in which a part of the spinal cord is split) and Charcot–Marie–Tooth disease are both associated with pes cavus; however, in the presence of a foot drop that is unlikely to be the orthotic prescribed. 20. Answer E. Varus deformity The deformity most commonly encountered is a varus deformity. It developed as part of an adaptive mechanism to re-establish the columns of the foot while in pes planus. The forefoot abnormalities will require derotation through the Chopart joints. It may also require a Cotton osteotomy to plantar flex the first ray. Prior to reduction of the hindfoot, the forefoot would be expected to show an abduction and supination deformity. With coverage of the talar head, the abduction deformity should be corrected; however, the rotation may not have been fully appreciated. This would typically manifest itself as a persistent varus deformity. 21. Answer C. Ledderhose disease Ledderhose disease, or plantar fibromatosis, is the correct diagnosis. The disease is named after Dr Georg Ledderhose, a German surgeon who described the condition in 1894. It is a similar disease to Dupuytren’s disease. As in most forms of fibromatosis, it is usually benign. The nodules are typically slow growing and most often found in the central and medial portions of the plantar fascia. Options for intervention include radiation therapy, cryosurgery, treatment with collagenase clostridium histolyticum or surgical removal only if discomfort hinders walking. In synovial sarcoma, histopathology would show a high histological grade, including cell atypia and the presence of poorly differentiated epithelial and spindle cells. For a lipoma, histological analysis demonstrates acellular stroma and lack of atypia. Fibromyxomas typically occur in the sub or periungual region, and histological analysis would show poor margins, spindle- and stellate-shaped cells, eosinophilic cytoplasm, fibrous and myxoid stroma and infrequent mitotic and

96

minimal atypia. Gardner’s syndrome is a variant of familial adenomatous polyposis and is associated with multiple tumours of soft tissue, skin and osteomas.

Forefoot

22. Answer B. Distal chevron osteotomy The patient has acquired hallux valgus deformity, and on radiographs this has been described as mild (Table 7.4). Therefore, a basal osteotomy would be too powerful for this correction. The examination shows no instability of the first ray, ruling out the Lapidus fusion (fusion of the 1st tarsometatarsal joint), and no arthritis (negative grind test), ruling out the 1st metatarsal phalangeal joint fusion. The Moberg osteotomy is a dorsiflexion osteotomy usually preserved for arthritis to allow range of motion. The proximal phalanx osteotomy is primarily used to correct a hallux interphalangeus deformity (delta phalanx) or to supplement a 1st metatarsal osteotomy. The distal chevron osteotomy is the most appropriate treatment. Table 7.4 Classification of severity of hallux valgus based on the hallux valgus and intermetatarsal angles

Hallux valgus angle

Intermetatarsal angle

Mild

20°

23. Answer E. MTP joint synovitis With the presence of a bunion deformity, the metatarsal head is often elevated. This can frequently lead to transfer metatarsalgia to the lesser metatarsals. Commonly, this leads to synovitis and pain of the 2nd and possibly 3rd MTP joint. Certainly, this is far more common than other causes of 2nd and 3rd MTP joint pain, Freiberg’s disease, plantar plate rupture and Morton’s neuroma. Morton’s neuroma is common in middleaged (average 50 years old) women, and the incidence is at least 4 to 15 times higher in females. Clinically, there may be tenderness and a dorsal bulging may be found. It may also be

Foot and Ankle Structured SBA

present as an enlargement of the interdigital space. When pressure is applied axially to the intermetatarsal space, acute pain is induced. The pressure can be exerted while tightening the metatarsals with the other hand, and this may be associated with a painful and palpable clicking sensation (Mulder’s sign). This test demonstrates a 94–98% sensitivity. It most commonly occurs in the third intermetatarsal space, which it is theorised is due to the anastomosis of the medial and lateral plantar nerves, making it stiffer and more prone to injury. 24. Answer B. Internal fixation Jones described a fracture in the proximal threequarter segment of the shaft distal to the styloid.14 The Jones fracture was later defined by Stewart as a transverse fracture at the junction of the diaphysis and metaphysis without extension into the 4th and 5th intermetatarsal articulation.15 Dameron16 defined the proximal 5th metatarsal as having three zones: zone 1 – styloid process; zone 2 – meta-diaphyseal area; zone 3 – proximal diaphysis. Ekrol17 further differentiated fractures of zone 1 and explained their differences due to muscle insertion at the base of the 5th metatarsal. For treatment, zone 1 injuries have been shown to be easily treated with symptomatic measures, with this no different to cast immobilisation at 4 weeks and 3 months.18 A boot, however, is preferred to a sandal, as it results in

significantly lower peak pressure at the 5th metatarsal during walking.19 In zone 2 and 3 injuries, biomechanical testing has shown fracture gapping during weight bearing.20 Zone 2 and 3 fractures are also at a watershed area, with the retrograde nutrient artery being damaged during fracture. What is described in this clinical scenario is a zone 3 fracture, probably secondary to a stress fracture. It has been shown in an operative vs non-operative study that the operative treatment group demonstrated a reduced time to return to sport and faster clinical union by almost 50% compared to the non-operative group.21 In this patient, therefore, surgery would be offered. In the non-athletic group, conservative management is commonly offered first (non-weight bearing cast); however, there is still a 30% non-union risk and a 30% refracture risk.22 25. Answer E. Weakness on tip toe stance Excision of both sesamoids should be avoided if at all possible. Excision of both sesamoids relatively lengthens the flexor hallucis brevis and decreases its moment arm at the metatarsal phalangeal joint, essentially defunctioning it. It has been shown biomechanically that decreases in the effective tendon moment arm of the flexor hallucis longus tendon occurred with resection of both the medial and lateral sesamoids.23 One of the largest series of sesamoidectomy showed a

19

14

15

16

17

18

Jones R. I. Fracture of the base of the fifth metatarsal bone by indirect violence. Ann Surg. 1902 Jun;35(6):697. Stewart IM. Jones’s fracture: fracture of base of fifth metatarsal. Clin Orthop. 1960;16:190-198. Dameron JT. Fractures and anatomical variations of the proximal portion of the fifth metatarsal. J Bone Joint Surg Am. 1975 57(6):788-792. Ekrol I, Court-Brown CM. Fractures of the base of the 5th metatarsal. Foot. 2004 Jun 1;14(2):96-98. Akimau PI, Cawthron KL, Dakin WM, Chadwick C, Blundell CM, Davies MB. Symptomatic treatment or cast immobilisation for avulsion fractures of the base of the fifth metatarsal: a prospective, randomised, singleblinded non-inferiority controlled trial. J Bone Joint Surg Br. 2016;98(6):806-811.

20

21

22

23

Hunt KJ, Goeb Y, Esparza R, Malone M, Shultz R, Matheson G. Site-specific loading at the fifth metatarsal base in rehabilitative devices: implications for Jones fracture treatment. PM&R. 2014 Nov 1;6(11):1022-1029. Morris PM, Francois AG, Marcus RE, Farrow LD. The effect of peroneus brevis tendon anatomy on the stability of fractures at the fifth metatarsal base. Foot Ankle Int. 2015;36(5):579-584 Mologne TS, Lundeen JM, Clapper MF, O’Brien TJ. Early screw fixation versus casting in the treatment of acute Jones fractures. Am J Sports Med. 2005;33(7):970975. Quill Jr GE. Fractures of the proximal fifth metatarsal. Orthop Clin North Am. 1995;26(2):353-362. Aper RL, Saltzman CL, Brown TD. The effect of hallux sesamoid excision on the flexor hallucis longus moment arm. Clin Orthop Relat Res. 1996;325:209-217.

97

Gavin Heyes and Lyndon Mason

30% extreme difficulty or an inability to stand on tip toe.24 The most recent systematic review on sesamoidectomy showed that most did well, with 94.4% of patients returning to sports, with 90.0% returning to their previous level, at a mean of 11.8 +/– 1.8 weeks. The overall complication rate, however, was 22.5% and the revision rate was 3.0%. The complications varied but included hallux valgus (medial sesamoidectomy), hallux varus (lateral sesamoidectomy), loss of range of motion of the 1st metatarsophalangeal joint, weakness of plantar flexion strength and transfer metatarsalgia. It was difficult to clearly divide the complications; however, the alteration in mechanical loading after sesamoidectomy appears to cause complications that include transfer metatarsalgia, neuroma, stress fracture of the metatarsals and lateral sesamoiditis.25 26. Answer E. Weil metatarsal osteotomy Weil metatarsal osteotomy involves a nearhorizontal osteotomy at the metatarsal head and neck. The most frequent complication reported is a floating toe. Morton’s neuroma excision complications do not typically include mechanical toe deformities. Complications may include stump neuroma and painful or sensitive scar. Stainsby procedure is used in toe clawing to reduce the plantar plate to its correct position under the metatarsal head. It is performed by excising the proximal one-third to one-half of the proximal phalanx and suturing the proximal end of the extensors to the flexors. Loss of function of the toe is the most important problem. Keller’s excision arthroplasty is performed on the hallux with complications including weakness on push off and cock-up deformity. Kidner’s procedure involves resection of a symptomatic accessory navicular and reconstruction of the tibialis posterior.

24

25

98

Lee S, James WC, Cohen BE, Davis WH, Anderson RB. Evaluation of hallux alignment and functional outcome after isolated tibial sesamoidectomy. Foot Ankle Int. 2005 Oct;26(10):803-809. Shimozono Y, Hurley ET, Brown AJ, Kennedy JG. Sesamoidectomy for hallux sesamoid disorders: a systematic review. J Foot Ankle Surg. 2018;57(6):11861190.

Trnka H, Schuh R. Strategies for managing complications of osteotomies of the lesser metatarsals. In Alexander I, Blumen E, Greisberg J, eds. Advanced Reconstruction Foot and Ankle 2. Rosemont, IL: American Academy of Orthopaedic Surgeons; 2015. Migues A, Slullitel G, Bilbao F, Carrasco M, Solari G. Floating-toe deformity as a complication of the Weil osteotomy. Foot Ankle Int. 2004 Sep;25(9):609-613. 27. Answer E. Plantar flexion and pronation Hallux valgus is a rotational and lateralising deformity of the hallux. The phalanx deviates into valgus and the metatarsal into varus. As the metatarsal migrates medially, the flexor hallucis brevis (sesamoids contained within) remains in place, becoming a valgus force to the phalanx as well as a flexor. This initiates rotation; the abductor hallucis becomes a plantar flexor and pronator of the hallucis as it moves plantarward and medially relative to its usual line of pull. 28. Answer B. Revision to 1st MTPJ fusion with inlay bone graft Improvements in AOFAS score to 74.9 can be achieved with fusion, although one should be aware that they unite slower, have a significantly higher reoperation rate and have lower AOFAS scores than primary fusions.26,27 There is no strong evidence for revision to a further silastic implant. Given there is osteolysis around the primary component, revision is likely to be technically challenging and will have a high probability of instability. 29. Answer C. Freiberg’s disease The history is typical for a presentation of Freiberg’s disease. The radiograph description is indicative of avascular necrosis of part of the metatarsal head. The condition was first described by Dr Alfred H. Freiberg in 1914. It is an uncommon condition, occurring most 26

27

Gross CE, Hsu AR, Holmes GB, Lee S. Revision MTP arthrodesis for failed MTP arthroplasty. Foot Ankle Spec. 2013;6(6):471-478. Baumhauer JF, DiGiovanni BF. Salvage of first metatarsalphalangeal joint arthroplasty complications. Foot Ankle Clin N Am. 2003; 8:37-48.

Foot and Ankle Structured SBA

often in young women, athletes and those with abnormally long metatarsals. Kohler’s disease is osteochondritis of the navicular, typically affecting children between 5 and 10 years and affects boys more often than girls. Sever’s disease is calcaneal apophysitis. Mueller–Weiss syndrome is adult onset avascular necrosis of the navicular. Turf toe is a sprain of varying grades affecting the big toe. 30. Answer E. Turner’s syndrome Turner’s syndrome is characterised by brachymetatarsia (described in the question), downslanting eyes, droopy eyelids, short, webbed neck, teeth crowding, broad (Shield) chest, short carrying angle and short fingers and occurs in females. Klippel–Feil syndrome is a congenital condition characterised by fusion of at least two cervical vertebrae, a short neck, low hairline, scoliosis and Sprengel deformity and may also have cardiac and pulmonary abnormalities. Poland syndrome is a congenital disorder characterised by underdeveloped chest muscle and short, webbed fingers on one side. Trisomy 21 (Down’s syndrome) is a congenital

disorder characterised by mild to moderate developmental delay, poor immune function, heart defects, epilepsy, thyroid disorder, micrognathia, slanted eyes, macroglossia, short neck, large space between 1st and 2nd toes and shortened metatarsals 3–5. Sprengel deformity is characterised by malposition and hypoplasia of the scapula. 31. Answer A. Amputation through proximal onethird of proximal phalanx In a low-demand nursing home patient with chronic infection and ulceration, an amputation would have the least complications. In the absence of infection and in a higher-demand patient, option E would address all the involved joints. 32. Answer A. Dorsal cheilectomy This describes a grade 2 hallux rigidis (Coughlin and Shurnas Classification, Table 7.5). A Dorsal cheilectomy is the accepted procedure for a grade 2 as it will allow a greater range of motion. Synovectomy alone would not provide sustainable pain relief or range of motion required for this gentleman. Keller’s

Table 7.5 Coughlin and Shurnas classification of hallux rigidus28

28

Grade

Dorsiflexion

Radiographic findings

Clinical findings

0

40°–60° +/– or 10–20% loss compared to other side

Normal

Stiffness

1

30°–40° +/– or 20–50% loss compared to other side

Dorsal osteophyte. Minimal joint space narrowing, flattening of metatarsal head and periarticular sclerosis

Stiffness and mild pain on end range of motion

2

10°–30° +/– or 50–70% loss compared to other side

Dorsal, lateral +/– medial osteophyte. Flattening metatarsal head with >1/4 dorsal joint involvement. Mild to moderate joint space narrowing

Stiffness. Moderate to severe pain, occurring just before end range of motion

3

35 degrees, perilunate fracture dislocation, dorsal intercalated segment instability (DISI), malalignment and fracture of the proximal pole. Despite 1mm of displacement being associated with instability, early surgical fixation is generally reserved for displacement >2mm, which represents a highly unstable fracture pattern. The Scaphoid Waist Internal Fixation for Fractures Trial (SWIFFT) is a multi-centre randomised controlled trial comparing surgical fixation (open or percutaneous headless compression screw fixation) and below elbow cast immobilization (6–10 weeks) for acute undisplaced or minimally displaced (2mm or less) bicortical scaphoid waist fractures (Dias et al, 2020). At one year, there was no statistically significant difference in days off work, pain and function (PRWE scores), and rates of union (98% versus 96%). More participants in the surgery group (14%) had a potentially serious complication than in the cast immobilisation group (1%); however, fewer participants in the surgery group had a cast-related complication (2% versus 18%). The study recommended early CT scanning (to assess for crossing trabeculae) if plain radiographs suggest non-union at 6–12 weeks, with urgent surgical fixation advised for confirmed cases of non-union. Of note, the cost of surgery to the NHS was significantly higher per patient when compared to cast immobilisation alone (£2350 versus £727). Dias JJ, Singh HP. Displaced fracture of the waist of the scaphoid. J Bone Joint Surg Br. 2011;93(11), 1433-1439. Clay NR, Dias JJ, Costigan PS, Gregg PJ, Barton NJ. Need the thumb be immobilised in scaphoid fractures? A randomised prospective trial. J Bone Joint Surg Br. 1991;73:828-832. Dias J, Brealey S, Choudhary S, et al. Surgery compared with cast immobilization for adults with a bi-cortical fracture of the scaphoid waist (SWIFFT): a multicentre, pragmatic, open-

label, parallel-group, two-arm randomised clinical trial. Lancet, 2020;396, 390-401. 23. Answer A. Clenched fist anteroposterior (AP) Plain radiographs remain the standard initial imaging technique for suspected scaphoid fractures. The following four plain radiographic scaphoid views should be obtained (Rubin & Dalinka, 2005):  Posteroanterior (PA)  PA with ulnar deviation  Semi-pronated oblique  Lateral If early plain radiographic views appear normal despite the clinical suspicion of a scaphoid fracture, a repeat clinical and radiographic assessment should be performed at 10–14 days post-injury. If repeat views remain inconclusive, magnetic resonance (MR) imaging is the gold standard diagnostic investigation for equivocal cases (Smith et al, 2010). Despite guidance from the National Institute for Health and Care Excellence (NICE) recommending that clinicians should “consider MRI for firstline imaging in people with suspected scaphoid fractures following a thorough clinical assessment” (NICE, 2016), plain radiographs remain the universally accepted first line imaging modality. Scaphoid fracture displacement (and non-union) is best assessed, and quantified, using computed tomography (CT) imaging (Smith et al, 2010). The clenched fist view is reserved for assessment of dynamic scapholunate instability, as seen with complete tears of the scapholunate ligament. Traumatic dorsal wrist pain with tenderness over the scapholunate interval (just distal to Lister’s tubercle) would be an indication for this view if the PA wrist radiograph is inconclusive. Rubin DA, Dalinka RH. Expert Panel on Musculoskeletal Imaging. Acute Hand and Wrist Trauma [online publication]. Reston, VA: American College of Radiology (ACR); 2005:8. Smith M, Bain GI, Turner PC, Watts AC. Review of imaging of scaphoid fractures. ANZ J Surg. 2010;80(1-2), 82-90. National Institute for Health and Care Excellence (2016). Fractures (non-complex): NICE guideline [NG38]. NICE; 2016. www.nice.org.uk › guidance › documents › fractures-full-guideline2.

275

Matthew Brown and David Yeoh

24. Answer A. Broad-spectrum antibiotics and consideration of tetanus prophylaxis Although this patient requires urgent irrigation and debridement, the question asks for the first step and this has to be option A. High-power injections are potentially devastating injuries, with all except the injection of air and water requiring urgent open irrigation and debridement. For toxic substances, including grease, paint and solvents, immediate decompression with irrigation and removal of foreign material are indicated. Injected material travels along planes of least resistance (including neurovascular planes) up into the forearm, and the debridement of foreign material and soft tissue necrosis is essential. Amputation rates are high, and debridement within 6 hours is advised (Amsdell & Hammert, 2013). Those managed non-operatively should be observed for signs of compartment syndrome. Amsdell SL, Hammert WC. High-pressure injection injuries in the hand: current treatment concepts. Plast Reconstr Surg. 2013;132(4):586e591e. 25. Answer B. Lateral band Dupuytren’s disease is a benign fibroproliferative disease of the palmar and digital fascia that is characterised by subcutaneous cords and nodules and progressive flexion deformities. The ring finger is most often affected. The lateral bands are not implicated in Dupuytren’s disease and are part of the extensor expansion (the conjoined lateral bands insert into the distal phalanx as the terminal extensor tendon). Normal fascial bands in the hand are termed cords when affected by Dupuytren’s disease. MCPJ flexion is caused by the central cord, which is composed of the distal pretendinous cord. PIPJ flexion is caused by the spiral cord, which is composed of four separate fascial structures (from proximal to distal): pretendinous band, spiral band, lateral digital sheet and Grayson’s ligament. The spiral cord moves the digital neurovascular bundle superficially and towards the midline and should be considered during fasciectomy to correct a PIPJ contracture. DIPJ contracture is caused by the retrovascular cord. 26. Answer D The anterior oblique (beak) ligament is often spared

276

A partial intra-articular fracture of the thumb metacarpal base is described. Such fractures are commonly referred to as Bennett’s fractures and they result from axial loading of the thumb while in flexion. The fracture should be distinguished from extra-articular (metaphyseal) fractures, complete articular (three-part) Rolando fractures and comminuted intra-articular fractures. Fracture subluxation occurs, with the primary deforming forces being abductor pollicis longus (APL) and adductor pollicis (AP). APL inserts onto the metacarpal base and pulls the metacarpal both proximal and dorsal. AP inserts onto the ulnar base of the proximal phalanx and results in metacarpal adduction (narrowing the first web space) and supination. Successful reduction therefore requires longitudinal traction, pronation and dorsal thumb base pressure. Anatomical restoration of the articular surface is required (a step greater than 1mm should not be accepted) and intraoperative radiographs should be completed. An undisplaced anteromedial fragment, of variable size, is held in position by the anterior oblique (‘volar beak’) ligament, which remains intact (including for Rolando fractures). The volar beak ligament connects the tubercle of the trapezium and the volar thumb metacarpal base. Most thumb base fractures are unstable (Liverneaux et al, 2015). Stable patterns that maintain reduction after manipulation may be treated conservatively with a thumb spica. The majority of cases will require percutaneous Kwire transfixion to the neighbouring trapezium and/or index finger metacarpal (two wires are often used). Care should be taken to avoid iatrogenic injury to the terminal branches of the superficial radial nerve. Open reduction and internal fixation is reserved for cases that cannot be reduced closed. Adduction malunion can be partially compensated for with thumb hyperextension; however, incongruence of the thumb base will result in secondary osteoarthritis and stiffness. Due to fracture complexity, Rolando fractures are not usually amenable to closed reduction techniques, with open reduction and internal fixation often indicated. Liverneaux PA, Ichihara S, Hendriks S, Facca S, Bodin F. Fractures and dislocations of the base of the thumb metacarpal. J Hand Sur Eur. 2015;40(1):42-45.

Hand II Structured SBA

27. Answer E. Ulnar deviation of the carpus This patient presents with rheumatoid arthritis, a chronic autoimmune disease characterised by synovial joint destruction and multi-system involvement. A hallmark feature is synovial joint pannus formation, a fibrovascular tissue characterised histologically by prominent intimal hyperplasia. The classical appearance of the rheumatoid hand includes any combination of digital swan neck and Boutonnière deformities, triggering, rheumatoid nodules, ulnar deviation of the digits at the MCPJs, radial deviation of the carpus and Z-deformity (or Boutonnière) of the thumb (classified according to the Nalebuff classification). Advanced disease may be further complicated by attritional rupture of the flexor pollicis longus tendon (Mannerfelt syndrome) and/or sequential rupture of the extensor tendons, from ulnar to radial, starting with extensor digiti minimi (Vaughan-Jackson syndrome). Splenomegaly and neutropaenia are features of Felty’s syndrome, a rare complication of rheumatoid disease. Anti-cyclic citrullinated peptide (anti-CCP) antibody testing is useful in the diagnosis of rheumatoid arthritis. It is present early in the disease process, has a high specificity (90%) and can help identify patients most at risk of severe disease with irreversible damage (Lee & Schur, 2003). However, it has low sensitivity (66%) and only 25–50% of rheumatoid patients test positive. Rheumatoid factor (RF) is an IgM antibody that targets native IgG, forming an immune complex that is deposited in the tissues. RF has a sensitivity and specificity of 71% and 80%, respectively. Lee DM, Schur PH. Clinical utility of the anti-CCP assay in patients with rheumatic diseases. Ann Rheum Dis. 2003; 62(9), 870-874. 28. Answer B. Full-thickness skin graft This tissue defect requires coverage. Splitthickness skin grafts are avoided on the palmar surface of the hand due to high rates of contraction and low sensibility. Full-thickness skin grafts contract less due to the presence of dermis. They also contain more epidermal appendages (including hair follicles, sweat glands and mechanoreceptors) and provide superior

sensibility. Hence, they are preferred on the palmar aspect of the hand (Matsui et al, 2014). The glabrous skin (devoid of hair follicles) of the volar or medial forearm, antecubital fossa, groin crease and abdomen are the preferred donor sites. The reconstructive ladder for tissue defects (from the simplest to the most complex) include: - Primary wound healing - Healing by secondary intention - Skin graft - Flap (local, distant or free) Full-thickness grafts must be meticulously defatted (to permit capillary ingrowth) and be completely immobilised and undisturbed for 10–14 days. For skin grafts of all varieties, the recipient bed must be uncontaminated, free from infection and not have exposed tendon, bone or nerve present. Skin flaps include a vascular attachment and are used to cover exposed nerves and areas of bone without periosteum and tendon without peritenon (composed of the paratenon and epitenon). Split-thickness skin grafts are preferred on the dorsum of the hand. They can be meshed to increase the surface area and to permit the drainage of blood and serum which could otherwise compromise graft survival. Negative pressure (or vacuum-assisted) wound therapy facilitates granulation and healing through the promotion of angiogenesis and cellular proliferation, wound contraction, oedema reduction and toxic exudate removal (Matsui et al, 2014). It can be used to expedite secondary wound healing and/or act as a bridge for staged reconstructive procedures. The preferred definitive treatment for the featured case would be a full-thickness skin graft. Matsui J, Piper S, Boyer MI. Nonmicrosurgical options for soft tissue reconstruction of the hand. Curr Rev Musculoskelet Med. 2014;7(1):68-75. 29. Answer D. Midcarpal instability Unlike radiocarpal instability, midcarpal instability may not be associated with a history of wrist trauma, and hypermobile patients are at particular risk. Dissociative carpal instability represents instability within or between bones of the same carpal row (e.g. DISI, VISI).

277

Matthew Brown and David Yeoh

Midcarpal instability is a type of non-dissociative carpal instability. The midcarpal shift (or catch-up clunk) test assesses for midcarpal instability (Feinstein et al, 1999). In midcarpal instability secondary to hypermobility, ulnar deviation causes the capitate to sublux dorsally over the scapholunate socket. Reactive contraction of the wrist extensors causes the distal carpal row to relocate abruptly with a clunk. Anterior and posterior drawer testing of the midcarpal joint can also assess for midcarpal instability. Surgery should be avoided when managing midcarpal instability secondary to hypermobility. Conservative treatment consists of analgesia, temporary splintage, activity modification and physiotherapy concentrating on proprioceptive control. Traumatic midcarpal instability in those without hypermobility is most commonly managed non-operatively; however, dorsal capsular reefing or limited midcarpal arthrodesis may be used for severe or refractory cases. Feinstein WK, Lichtman DM, Noble PC, Alexander JW, Hipp JA. Quantitative assessment of the midcarpal shift test. J Hand Surg Am. 1999;24:977-983. 30. Answer A. 2.5mg/kg of bupivacaine 0.5% with adrenaline (1:200 000) Knowledge of maximum doses for commonly used local anaesthetics is essential. Adrenaline (1:200 000) may be used for carpal tunnel decompression, but caution should be maintained if used more distally. Adrenaline 1:10 000 is incorrect and is the intravenous concentration used for cardiac arrest. It is important to commit to memory that a 1% solution (e.g. lidocaine) contains 10mg of solute (the active ingredient) per millilitre of solution (10mg/ml). Therefore, a 0.5% solution (e.g. bupivacaine) contains 5mg of solute per millilitre of solution (5mg/ml). The maximum safe dose of commonly used local anaesthetics: Bupivacaine 0.5% 2mg/kg Bupivacaine 0.5% with adrenaline 2.5mg/kg Lidocaine 1% 3mg/kg Lidocaine 1% with adrenaline 7mg/kg

278

Plain lidocaine and bupivacaine provide 2–4 hours and 4–12 hours of anaesthesia, respectively 31. Answer D. V-Y advancement flap The volar V-Y advancement technique is especially useful for transverse or oblique distal digital tissue loss when bone is exposed (Atasoy et al, 1970). It preserves digital length and sensibility. There is often insufficient skin for its use with more volar tissue loss. A full-thickness triangular flap is elevated while maintaining its vascular and neural supply. The base of the triangle is formed by the distal traumatic cut edge, and the apex is designed to sit at the DIPJ flexion crease. Once mobilised and satisfactorily contoured, the base of the triangle is sutured to the intact dorsal distal tissue (i.e. the nailbed). The volar ‘V’ defect is closed without tension to form a ‘Y’. Split-thickness skin grafting may be used when covering exposed pulp in the absence of exposed bone; however, it is also reasonable to manage such cases with healing by secondary intention, especially in children. Z-plasty is a technique used for scar or contracture lengthening and it is not an appropriate option for this injury. Candidates should practice drawing a Z-plasty and remember that the commonly used 60-degree Z-plasty will result in 75% lengthening along its central limb. The Moberg flap is a bipedicled volar advancement flap used for covering thumb tip amputations. It is raised through parallel incisions just dorsal to the neurovascular pedicles. It can be advanced by a maximum of 1.5cm. Its principal advantages in the thumb include restoration of sensibility, length preservation and the provision of durable and well-padded phalangeal coverage. Atasoy E, Ioakimidis E, Kasdan ML, Kutz JE, Kleinert HE. Reconstruction of the amputated fingertip with a triangular volar flap: a new surgical procedure. J Bone Joint Surg Am. 1970;52:921-926. 32. Answer D. Sagittal band repair or reconstruction The radial and ulnar sagittal bands encircle each MCPJ to stabilise and centralise the extensor tendon during digital motion. Sagittal band rupture leads to subluxation or dislocation of the extensor tendon away from the injured band

Hand II Structured SBA

upon MCPJ flexion. Also termed the ‘boxer’s knuckle’, sagittal band rupture may be mistaken for trigger finger due to the extensor digitorum communis (EDC) tendon snapping back over the metacarpal head upon digital extension (also termed pseudotriggering). The radial sagittal band is more frequently compromised following trauma or as a result of inflammatory arthropathy such as rheumatoid arthritis. Closed traumatic sagittal band ruptures usually involve the radial sagittal band of the middle or ring finger. They are more common than open injuries resulting from lacerations. It is important to have a working knowledge of the extensor apparatus in the digits. The sagittal bands stabilise the EDC tendon centrally as it passes over the MCPJ. The EDC does not directly insert into the proximal phalanx. MCPJ extension is therefore mediated through the pull of the sagittal bands, which surround the base of the proximal phalanx en route to their insertion onto the MCPJ volar plate (they also insert into the intermetacarpal ligaments). The EDC trifurcates over the proximal phalanx: the central slip inserts into the base of the middle phalanx and the lateral components unite with the lateral bands. Acute sagittal band injuries with no subluxation are treated with buddy strapping for 4 weeks (Rayan & Murray, 1994). Acute injuries associated with subluxation may be treated with MCPJ hyperextension splintage for 8 weeks. Direct surgical repair or reconstruction is reserved for those presenting late or when splintage has failed. Rayan GM, Murray D. Classification and treatment of closed sagittal band injuries. J Hand Surg Am. 1994;19:590-594. 33. Answer A. EIP to EPL tendon transfer Up to 5% of undisplaced distal radius fractures managed non-operatively may be complicated by delayed extensor pollicis longus (EPL) tendon rupture (Roth et al, 2012). The mechanism remains unclear; however, it has been suggested that extravasation of blood and fracture debris into the third extensor compartment causes relative tendon ischaemia and attritional rupture. Extensor indicis proprius (EIP) to EPL tendon transfer is the most common treatment

strategy. The three-skin incision technique requires the EIP tendon to be identified through a transverse incision just proximal to the index finger MCPJ (EIP is ulnar to EDC). It is incised and retracted subcutaneously through a second transverse incision at the wrist. The freed EIP tendon is passed subcutaneously and attached to the EPL through a longitudinal incision at the level of the thumb metacarpal. It is commonplace to tension and secure the tendons using a Pulvertaft weave technique with non-absorbable monofilament sutures. Roth KM, Blazar PE, Earp BE, Han R, Leung A. Incidence of extensor pollicis longus tendon rupture after nondisplaced distal radius fractures. J Hand Surg Am. 2012;37(5):942-947. 34. Answer C. The condition is often self-limiting The featured patient is most likely presenting with tenosynovitis of the first extensor compartment, also known as de Quervain’s tenosynovitis. This is a clinical diagnosis. Dorsoradial tenderness and local swelling 1–2cm proximal to the radial styloid combined with pain upon ulnar deviation of the wrist with the thumb clasped in the palm aids diagnosis (Finkelstein’s test). Irritation of the abductor pollicis longus (APL) and extensor pollicis brevis (EPB) tendons as they pass through their short osseoligamentous tunnel (approximately 2cm) is common in the fifth and sixth decades of life and is up to six times more common in females. It must be distinguished from other common causes of radialsided wrist pain in this age group, including first CMC and scaphotrapezial-trapezoid (STT) joint osteoarthritis. Therefore, the most useful investigation at first presentation is plain radiography to exclude alternative causes of pain. The intersection syndrome between the tendons of the second and third extensor compartments is far less common, with pain (and occasionally crepitus) experienced more proximally. Management should begin with activity modification, analgesia (oral and/or topical) and wrist splintage. Cortisone injection is an effective second-line treatment option (with or without ultrasound-guidance), especially in those with acute presentations and nondiabetics, with 50–80% success after one or two

279

Matthew Brown and David Yeoh

injections (Weiss et al, 1994). Patients should be advised of the risks of injection, which include infection, steroid flare, fat necrosis and skin depigmentation. Anatomical variation is a common cause of treatment failure and ‘normal’ anatomy consisting of two tendons and one tendon sheath may occur in less than 20% of cases (Jackson et al, 1986). The APL tendon is often composed of several tendon slips, and a separate EPB tendon subsheath may be seen. Surgical decompression is reserved for refractory cases. The tendons of the first extensor compartment should be identified and fully decompressed: inspect for a separate EPB subsheath and excise the septa. Identify and carefully retract the terminal branches of the superficial radial nerve to avoid iatrogenic injury. The radial artery is also at risk. Incise the sheath on its dorsal margin to prevent later painful palmar subluxation of the tendons. Weiss AP, Akelman E, Tabatabai M. Treatment of de Quervain’s disease. J Hand Surg Am. 1994;19:595-598. Jackson WT, Viegas SF, Coon TM, Stimpson KD, Frogameni AD, Simpson JM. Anatomical variations in the first extensor compartment of the wrist. A clinical and anatomical study. J Bone Joint Surg Am. 1986;68:923-926. 35. Answer C. Radial shortening osteotomy The patient is likely to be presenting with avascular necrosis of the lunate, also known as Kienbock’s disease. Kienbock’s disease has a multi-factorial aetiology (Lichtman et al, 2010). Ulnar negative variance is considered a risk factor, especially when compared to ulnar positive variance, with increased radiolunate contact stresses described. The relationship between the lunate’s coronal shape and ulnar length suggests that type I (trapezoid) lunates (which coexist with ulnar negative variance) have the weakest configuration and the greatest potential for fatigue compared to the rectangular or pentagonal morphology of type II and III lunates, respectively (Antuna Zapico, 1966). The vascular supply of the lunate is an additional risk factor. Around 92% have a dual vascular supply, with the remaining 8% reliant on a single palmar supply (Gelberman et al, 1980). Three

280

anastomotic variants are described: Y pattern (59%), I pattern (31%) and X pattern (10%). The I pattern represents a single vessel to the lunate and is considered at highest risk. In summary, type I lunates (trapezoid) and I pattern vascularity are risk factors for the development of Kienbock’s disease. Repetitive trauma and vascular disease are also risk factors. The Lichtman classification describes four stages of Kienbock’s disease and may help guide management (Lichtman et al, 2010). It is graded according to plain radiographic and magnetic resonance imaging appearances. The featured case has stage I disease, which is characterised by normal plain radiographs; however, MR imaging will demonstrate decreased signal intensity on both T1- and T2-weighted images. Radionuclide bone scanning will also be positive. Such cases are treated with a period of immobilisation. Stage II will show sclerosis but no lunate collapse, although multiple fracture lines may be visible. Such cases are often managed with a joint-levelling procedure (e.g. radial shortening osteotomy). Ulnar lengthening is less commonly performed, as it requires two osteotomy sites and bone graft. The aim of such procedures is to offload the lunate to achieve neutral or slight ulnar negative variance. In stage III, lunate flattening or collapse is seen, with IIIA and IIIB subtypes representing maintained carpal alignment and height or carpal rotation and proximal migration of the capitate, respectively. Scaphoid flexion will produce a DISI carpal instability. Stage III disease is treated with partial fusions of the scaphocapitate or scaphotrapezialtrapezoid (STT) joints or proximal row carpectomy. Stage IV is characterised by perilunate arthritis. Treatment options include proximal row carpectomy, total wrist replacement, wrist arthrodesis or pyrocarbon replacement. Vascularised bone grafting of the lunate or distal radius core decompression may be used in stages I, II and IIIA. The featured patient has failed non-operative management and option C is the next most appropriate treatment option. Ulnar impaction syndrome (also called ulnocarpal abutment), seen with ulnar-positive variance, is an important differential diagnosis. MR imaging will demonstrate low signal changes (sclerosis) in the proximal ulnar aspect of the

Hand II Structured SBA

lunate and commonly attenuation of the TFCC. Pain on ulnar deviation would be expected. Lichtman DM, Lesley NE, Simmons SP. The classification and treatment of Kienböck’s disease: the state of the art and a look at the future. J Hand Sur Eur. 2010;35(7):549-554. Antuna Zapico JM. Malacia del Semilunar [tesis doctoral]. Industrias y Editorial Sever Cuesta. Valladoid, Spain: Universidad de Valladoid; 1966. Gelberman RH, Baumann TD, Menon J and Akeson WH. The vascularity of the lunate bone and Kienbock’s disease. J Hand Surg Am. 1980;5:272-278. 36. Answer E. Ulnar-shortening osteotomy Ulnar impaction syndrome (also called ulnocarpal abutment) is seen with ulnar-positive variance. It is a cause of ulnar-sided wrist pain. Common acquired causes include distal radius or Essex-Lopresti radial head fractures that heal short. The ballottement (or piano key) test assesses for pain and/or increased distal ulna displacement in the sagittal plane relative to the radius. A positive test suggests distal radioulnar joint (DRUJ) instability and/or ulnocarpal impaction. The power grip can increase ulnar variance by 2mm, especially during pronation (Adams, 2011), and pronated and/or power grip plain radiographs may identify a dynamic increase in ulnar variance. In cadaveric studies, a 2.5mm increase in ulnar variance is associated with a 42% increase in ulnocarpal loading (Palmer & Werner, 1988). In cases of ulnar impaction syndrome, the proximal ulnar aspect of the lunate will often demonstrate subchondral sclerosis on magnetic resonance (MR) imaging, with decreased signal intensity on both T1- and T2-weighted images. On occasions, high signal on T2-weighted images may be seen, representing bone oedema. TFCC degeneration is a frequent association on MR imaging and at wrist arthroscopy. An ulnar-shortening osteotomy preserves the articular cartilage and is suitable for most cases of ulnar positive variance. Corrective osteotomy of the malunited distal radius is a valid alternative. In cases of distal radius malunion with up to 20 degrees of dorsal angulation, joint levelling with an isolated ulnar-shortening osteotomy is a

quicker and technically less demanding alternative with equivalent outcomes (Srinivasan et al, 2013). It is achieved using an oblique segmental resection osteotomy and compression plating. The dorsal sensory branch of the ulnar nerve is at risk during the approach. The wafer procedure is an alternative, which resects 2–4mm of distal ulna from beneath the TFCC. However, the wafer procedure removes the ulnar articular cartilage and can destabilise the TFCC and DRUJ. Option A is incorrect, as stabilisation is not indicated. Additionally, arthroscopic debridement is unlikely to provide long-term relief. The Sauvé-Kapandji procedure, ulnar head resection (Darrach procedure) and ulnar head replacement are generally reserved for cases of DRUJ arthritis. The Darrach procedure is most often reserved for low-demand, elderly patients due to the risk of stump instability and impingement. Ulnar head replacement would fail early in a manual labourer. Adams BD. Distal radioulnar joint instability. In Wolfe SW, Hotchkiss RN, Pederson WC, Kozin SH, eds. Green’s Operative Hand Surgery. 6th ed. New York, NY: Elsevier Churchill Livingstone; 2011:465-522. Palmer AK, Werner FW. Biomechanics of the distal radioulnar joint. Clin Orthop Relat Res. 1984;187:26-35. Srinivasan RC, Jain D, Richard MJ, Leversedge FJ, Mithani SK, Ruch DS. Isolated ulnar shortening osteotomy for the treatment of extra-articular distal radius malunion. J Hand Surg Am. 2013;38:1106-1110. 37. Answer C. Intravenous antibiotics and elevation Hand infections can result in severe disability and they should be taken seriously. Intravenous broad-spectrum antibiotics should be commenced without delay. Plain radiographs will help exclude a retained foreign body or fracture. Pyogenic flexor tenosynovitis is an infection of the flexor tendon sheath, which usually results from penetrating trauma. The flexor sheaths are closed spaces, although there is potential for communication between the thumb and little finger flexor sheaths via the space of Parona. Increased pressure creates tendon adhesions and obstruction of the vincula arterial system

281

Matthew Brown and David Yeoh

will eventually lead to tendon necrosis. Although Staphylococcus aureus remains the most common organism (up to 80%), animal (dog and cat) and human bites are often polymicrobial, with Pasteurella species and Eikenella corrodens proving common isolates, respectively (Flevas et al, 2019). Although not all signs may be present in the early stages, Kanavel (1921, 1939) described four cardinal features of pyogenic flexor tenosynovitis:  Semi-flexed digit  Fusiform digital swelling  Tenderness along the flexor tendon sheath  Pain on passive digital extension (diffuse and not localised to a particular joint) Acute, mild presentations in the absence of a clear history of penetrating trauma may be managed in the first 24 hours with a trial of intravenous antibiotics and strict elevation in a Bradfordtype sling. Dorsal-blocking splintage may be considered. There should be a very low threshold for operative intervention and frequent serial examinations are essential. If conservative treatment does not realise clinical improvement within 12–24 hours, or if the presentation is severe or delayed, prompt surgical irrigation using the two-incision technique will prevent tendon necrosis and long-term disability (Flevas et al, 2019). The featured case will require incision and drainage (as animal bites are considered contaminated wounds); however, the initial management should prioritise intravenous broad-spectrum antibiotics and elevation. The two-incision technique utilises an incision just proximal to the A1 pulley and a second incision proximal to the DIPJ flexor crease, which permits access to the tendon sheath so that catheter irrigation can occur from proximal to distal (Carter et al, 1969). An alternative single midlateral incision technique to expose the entire tendon sheath is usually reserved for severe infection with tendon necrosis. Differential diagnoses include cellulitis, septic arthritis, local abscess (e.g. felon), inflammatory arthropathy (e.g. rheumatoid arthritis, gout) or aseptic tenosynovitis. Flevas DA, Syngouna S, Fandridis E, Tsiodras S and Mavrogenis AF. Infections of

282

the hand: an overview. EFFORT Open Rev, 2019;4:183-193. Kanavel A. Infections of the Hand. 4th ed. Philadelphia, PA: Lee & Febiger; 1921. Kanavel A. Infections of the Hand. 7th ed. Philadelphia, PA: Lee & Febiger; 1939. Carter SJ, Burman SO, Mersheimer WL. Treatment of digital tenosynovitis by irrigation with peroxide and oxytetracycline: review of nine cases. Ann Surg. 1966;163:645-650. 38. Answer B. Flexor tendon tenodesis (FDS sling) Swan neck deformity is described in this clinical scenario. It is a common feature of rheumatoid arthritis affecting the hand and may coexist with Boutonnière deformity in adjacent digits. Although inflammatory arthropathy is the commonest cause, swan neck deformity may result from intrinsic tightness secondary to MCPJ disease, FDS rupture, volar plate insufficiency, mallet deformity or extrinsic spasticity (McKeon & Lee, 2015; Dickson & Harrison, 2017). Both swan neck and Boutonnière deformities represent an imbalance between flexor and extensor forces. In rheumatoid arthritis, swan neck deformity can originate at either the DIPJ (mallet deformity) or at the PIPJ (synovitis and volar plate/capsule attenuation and/or FDS rupture). Flexor tendon tenodesis using the FDS tendon (the ‘sublimis sling’) can be used to provide a soft tissue checkrein to correct PIPJ hyperextension (Wei & Terrono, 2015). Other surgical options include oblique retinacular ligament reconstruction, lateral band mobilisation and, for end-stage disease (with pain and stiffness), PIPJ arthrodesis (Dickson & Harrison, 2017). Cases driven by a mallet deformity may be treated with DIPJ arthrodesis alone or combined with a PIPJ procedure. Passive PIPJ correction with an Oval-8 (or Silver Ring) splint is an effective conservative treatment in patients with a correctable swan neck deformity; however, the featured case states that conservative measures have failed. Extensor tenotomy over the middle phalanx is a treatment for DIPJ hyperextension seen in Boutonnière deformity, not swan neck deformity. McKeon KE, Lee DH. Posttraumatic Boutonnière and swan neck deformities. J Am Acad Orthop Surg. 2015;23:623-632.

Hand II Structured SBA

Dickson DR, Harrison JWK. Hand oral core topics. In Banaszkiewicz PA, Kader DF, eds. Postgraduate Orthopaedics. 3rd ed. Cambridge: Cambridge University Press; 2017:421-494. Wei DH, Terrono AL. Superficialis sling (flexor digitorum superficialis tenodesis) for swan neck reconstruction. J Hand Surg Am. 2015;40(10):2068-74. 39. Answer C. Clinical diagnosis, confirmed with nerve conduction studies, is the gold standard Cubital tunnel syndrome is a common compressive neuropathy in the upper limb, surpassed only by carpal tunnel syndrome. This patient presents with clawing, presumably of the ring and little fingers, which is an advanced feature of ulnar neuropathy. Diagnosis is a clinical one, although neurophysiological assessment may be performed to confirm the diagnosis (Mackinnon & Novak, 2011). The ligament of Struthers is found on the lateral aspect of the elbow and associated with a supracondylar humeral process. It is a potential site for median nerve compression, and it should not be confused with the arcade of Struthers, which is a potential site of ulnar nerve compression at the elbow. The Martin-Gruber anastomosis relates to a normal variant motor branch of the median nerve joining the ulnar nerve, most commonly in the forearm, and its presence will not affect the outcome of surgical decompression. Iatrogenic injury to the branches of the medial cutaneous nerve of the forearm at the time of decompression surgery is possible. It may lead to altered medial forearm sensation and painful hyperalgesia and neuroma formation (Mackinnon & Novak, 2007). Mackinnon SE, Novak CB. Compression neuropathies. In Wolfe SW, Hotchkiss RN, Pederson WC, Kozin SH, eds. Green’s Operative Hand Surgery. 6th ed. New York, NY: Elsevier Churchill Livingstone; 2011:465-522. Mackinnon SE, Novak CB. Operative findings in reoperation of patients with cubital tunnel syndrome. Hand. 2007;2(3):137-143. 40. Answer C. Pronator syndrome A neuropathy of the median nerve is described. In the hand, the median nerve innervates the palmar skin of the radial three and a half digits,

radial palmar skin (via the palmar cutaneous branch), radial two lumbricals and the three thenar muscles: opponens pollicis, abductor pollicis brevis and flexor pollicis brevis (via the recurrent motor branch). LOAF is a useful mnemonic when recalling these intrinsic muscles. A median nerve lesion proximal to the carpal tunnel is most likely, given that sensation over the thenar eminence is reduced. The palmar cutaneous branch of the median nerve (purely sensory) arises 5cm proximal to the proximal wrist crease. It travels between the median nerve and flexor carpi radialis (FCR) tendon and crosses the wrist superficial to the transverse carpal ligament. Neurophysiological testing is often normal in pronator syndrome and the diagnosis is usually one of exclusion. It is far less common than carpal tunnel syndrome, and diurnal variation is not a feature. Altered sensation in the median nerve distribution of the hand (including the thenar skin) is classically described. Provocative tests include resisted forearm pronation with the elbow extended, resisted middle finger FDS flexion, and deep palpation (or a Tinel response) along the leading edge of pronator teres with the forearm held in maximal supination (Mackinnon & Novak, 2011). Well-developed forearm musculature is a risk factor. Multiple areas of median nerve entrapment are described. Elbow radiographs to exclude a rare supracondylar humeral process (and the associated ligament of Struthers) is essential. If splintage and activity modification fail, surgical decompression of the median nerve should be considered: release of the bicipital aponeurosis, the interval between the ulnar and humeral heads of pronator teres and the aponeurotic arch of flexor digitorum superficialis (FDS). Although a median nerve schwannoma could account for the sensory disturbance described, normal MR imaging and no mention of a mass lesion make this diagnosis less likely. Wartenberg’s syndrome (known as cheiralgia paraesthetica) is a compressive neuropathy of the superficial radial nerve between the tendons of brachioradialis and extensor carpi radialis longus (ECRL) in the midforearm during pronation. Intersection syndrome is tenosynovitis between the first and overlying second extensor

283

Matthew Brown and David Yeoh

compartments that is most often associated with repetitive wrist extension, for example in rowers. Mackinnon SE, Novak CB. Compression neuropathies. In Wolfe SW, Hotchkiss RN, Pederson WC, Kozin SH, eds. Green’s operative hand surgery. 6th ed. New York, NY: Elsevier Churchill Livingstone; 2011:465-522. 41. Answer C. The Jahss reduction manoeuvre is useful Metacarpal neck fractures (known as boxer’s fractures) result from axial loading, for example, when landing a punch. The little finger is most commonly affected. There is little consensus regarding the accepted degree of angulation in these patients. The flexion angle should be measured on the lateral radiograph (the oblique view often overestimates the deformity). Cadaveric studies demonstrate that fracture angles up to 30 degrees result in near normal mechanics (Beredjiklian, 2009). Functional studies have variously identified acceptable malunion flexion deformities ranging anywhere from 20–60 degrees. Westbrook et al (2008) concluded that angulation up to 50 degrees (in the absence of rotational deformity) was not associated with deleterious outcomes at 2 years, including validated questionnaire assessment, grip strength and little finger strength. Reduced knuckle contour and an extensor lag are seen in those with persistent angular deformity. Moulded ulnar gutter casts (in the intrinsic plus, or Edinburgh, position of safety) with later buddy strapping is a reasonable conservative treatment option. Intramedullary Kwire fixation (using the bouquet technique) for transverses fractures avoids impairment of tendon gliding over dorsal metalwork. Entry at the dorsoulnar aspect of the metacarpal base is preferred for the little finger. After drilling the cortex, pre-contoured 1 or 1.25mm K-wires are advanced antegrade up the metacarpal shaft. Fracture reduction using the Jahss technique is well described: flex the MCPJ and PIPJ to 90 degrees and apply dorsal pressure through the proximal phalanx while stabilising the metacarpal. When reduced, the intramedullary wires are advanced across the fracture into the metacarpal head. The wires may be rotated to assist reduction. Wong et al

284

(2006) found that percutaneous K-wire transfixion to the neighbouring metacarpals is an accepted alternative with equivalent outcomes described. Beredjiklian PK. Small finger metacarpal neck fractures. J Hand Surg Am. 2009;34(8), 1524-1526. Westbrook AP, Davis TRC, Armstrong D, Burke FD. The clinical significance of malunion of fractures of the neck and shaft of the little finger metacarpal. J Hand Surg Eur. 2008;33 (6):732-739. Wong TC, Ip FK, Yeung SH. Comparison between percutaneous transverse fixation and intramedullary K-wires in treating closed fractures of the metacarpal neck of the little finger. J Hand Surg Br. 2006;31(1):61-65. 42. Answer D. Open reduction and internal fixation Closed multiple metacarpal fractures are usually associated with high-energy trauma. They are considered highly unstable and frequently present with gross swelling and significant metacarpal shortening and/or rotation. Open reduction and internal fixation is indicated to restore metacarpal length and alignment and to permit early digital mobilisation and rehabilitation (Souer & Mudgal, 2008). Longitudinal dorsal incisions between the fractured metacarpals permit fixation of up to two metacarpals. Transverse or oblique fractures will suffice with plate fixation and screws spanning four cortices both proximal and distal to the fracture. Spiral fractures may be treated with lag screws, ideally two, with patterns permitting only a single lag screw requiring supplementary plate fixation. Intramedullary fixation using the bouquet technique should be reserved for transverse metacarpal shaft or neck fractures. Periosteal closure around the metalwork and early digital mobilisation will help to minimise extensor tendon adhesions and stiffness. Solitary metacarpal shaft fractures that are minimally displaced may be treated nonoperatively. Unacceptable displacement includes shortening greater than 5mm and/or rotational malalignment and/or flexion deformities greater than 10–20 degrees in the index and middle

Hand II Structured SBA

fingers or greater than 20–30 degrees in the ring and little fingers (AO Trauma). Fractures of the ring and little finger metacarpals tolerate greater flexion (when compared to the middle and index fingers) as there CMCJs have greater sagittal plane movement (assess this in your own hand). Souer JS, Mudgal CS. Plate fixation in closed ipsilateral multiple metacarpal fractures. J Hand Surg Eur. 2008;33(6):740-744. AO Trauma Reference. Metacarpals. www2 .aofoundation.org 43. Answer E Open reduction and internal fixation The radiograph demonstrates a three-part intraarticular fracture of the thumb metacarpal base, commonly termed a Rolando fracture. The three classical fragments include the metacarpal shaft, a palmar fragment and a dorsal fragment. Successful closed reduction is often impossible

and open reduction is advised. The surgical approach is the same for both Rolando and Bennett fractures, with the palmar Wagner approach affording good visualisation of the articular surface. Internal fixation is with a Tor L-plate (Jupiter, 2003). For severely comminuted thumb base fractures, skeletal traction through intermetacarpal pinning or external fixation are options (Liverneaux et al, 2015). CMCJ arthrodesis is reserved for painful malunion with articular incongruence. Jupiter J. Thumb metacarpal fractures. In Browner BD, ed. Skeletal Trauma: Basic Science, Management, And Reconstruction. 3rd ed. Philadelphia, PA: Saunders; 2003:1202. Liverneaux PA, Ichihara S, Hendriks S, Facca S, Bodin F. Fractures and dislocations of the base of the thumb metacarpal. J Hand Sur Eur. 2015;40(1):42-45.

285

Section 4 Chapter

16

Children’s Orthopaedics / Hand and Upper Limb

Children’s Orthopaedics I Structured SBA Sattar Alshryda and Paul Banaszkiewicz

CHILDREN’S ORTHOPAEDICS I STRUCTURED SBA QUESTIONS 1. While you are templating for a proximal femoral varus osteotomy (PFVO), the student nurse asks you how the surgery works for Perthes disease. PFVO for Perthes disease works by all the following factors EXCEPT which? A. Altering hip biomechanics B. Altering the rate of femoral head healing C. Forced rest and activity modification dictated by surgery D. Increased blood flow to the area E. The load-relieving effect of varus angulation 2. While you are performing a proximal femoral varus osteotomy (PFVO) for Perthes disease, the year 2 Core Surgical Trainee assisting you in theatre asks you to explain some of the technical issues involved with the procedure. Concerning PFVO in Perthes disease, which of the following is correct? A. A significant correlation exists between the amount of varus angulation obtained at surgery and the Stulberg outcome at maturity B. A significant number of patients obtain Stulberg class I or II hips following proximal femoral varus osteotomy C. A varus angulation of 20–25° is recommended D. For lateral pillar group C, a higher postoperative neck–shaft angle (less varus angulation) is associated with a greater probability of obtaining a Stulberg class I or II outcome E. Surgery may lead to a significant limb length discrepancy

286

3. A 15-year-old boy presents to the ED with severe right knee pain after falling off his bicycle. His radiograph is shown in Figure 16.1. Regarding distal femoral physeal fractures in children, all the following statements are true EXCEPT which? A. Account for less than 1% of fractures in children B. Physeal arrest occurs in approximately 40% of cases C. Interposed periosteum is an indication for open reduction D. Metaphyseal screw fixation is the preferred method of treatment E. Account for between 6% and 9% of physeal fractures Figure 16.1 Anteroposterior (AP) radiograph right knee

4. A newborn with CTEV is being treated with the Ponseti technique and regularly attending the paediatric orthopaedic clinic for serial cast changes Concerning CTEV, which of the following is correct?

Children’s Orthopaedics I Structured SBA

A. Adduction of the foot is required to stretch the medial soft tissues B. Children managed with boots and bars had an increased recurrence of deformity than did those managed with ankle–foot orthosis C. The Denis Browne boots and bar (DBB) is very well tolerated D. The Pirani score quantifies the severity of clubfoot and is a reliable prognostic of outcome E. With the Ponseti technique, about 8% of idiopathic clubfeet require further surgical treatment after a percutaneous tenotomy.

Questions 5–8 A young girl presents to the ED with localised swelling, ecchymosis and tenderness over the lateral aspect of the left elbow. Her radiographs are shown in Figure 16.2. You suspect a lateral condylar fracture.

Figure 16.2 AP and lateral radiographs of left elbow

5. All the following concerning lateral humeral condylar fractures in children are true EXCEPT which? A. The pull-off theory suggests that lateral humeral condylar fractures are avulsion fractures. B. The push-off theory postulates that these fractures are the result of a force directed upwards and outwards along the radius

C. They are the most common paediatric elbow fracture D. They represent 12–20% of paediatric elbow fractures. E. They typically occur in children aged approximately 6 years 6. The girl’s radiographs are discussed at the morning trauma meeting. The ST3 volunteers to use the Milch classification system, but this generates a lot of confusion amongst the audience. Concerning lateral humeral condylar fractures in children, which of the following is true? A. A Milch type I fracture is a Salter–Harris type IV fracture B. A Milch type II fracture is characterised by a fracture line that courses lateral to the trochlea and into the capitulotrochlear groove C. Lateral condylar fractures present clinically with severe deformity and neurovascular compromise D. Milch is of little use in determining fracture management and is largely of historical interest E. Milch type I fractures extend into the apex of the trochlea 7. Following the trauma meeting, she has been listed for surgery. Concerning management of lateral humeral condylar fractures in children, which of the following is correct? A. A varus force on the elbow may assist in visualising the fracture fragment B. In general, three pins should be used for fixation to reduce the chance of pin failure and fracture re-displacement C. Lateral approach is generally preferred D. Surgical management usually consists of open reduction and smooth pin fixation E. With open reduction, all dissection should be posterior to avoid damaging the blood supply to the distal fragment and risking osteonecrosis 8. The parents of the young girl have asked to see you in recovery following surgery. They want to know if everything will be fine following fracture fixation.

287

Sattar Alshryda and Paul Banaszkiewicz

Concerning complications of lateral humeral condylar fractures, all the following are true EXCEPT which? A. Cubitus valgus deformity is caused by lateral physeal arrest B. Cubitus valgus is a much less common complication than a cubitus varus deformity C. Cubitus varus deformity is most common in nondisplaced and minimally displaced fractures D. Non-union is most common in patients treated non-surgically E. Ulnar nerve palsies typically present early 9. A 14-year-old male returns to the fracture clinic 6 months following ORIF of both forearm bones. His fractures have healed, and his radiographs are shown in Figure 16.3. His parents ask if the plates should be removed. Regarding retention of forearm plates in children, which of the following is correct? A. Complications occur in nearly 1 in 6 patients B. Female gender and older age are isolated predictors of a complication C. Implant related fractures are less likely to occur when using a DCP D. The most common complications are pain or irritation from the plate E. The risk of implant-related fracture is higher with ulnar plating 10. All the following concerning unicameral bone cyst are true EXCEPT which?

Figure 16.3 Radiographs demonstrating plating both forearm bones

288

A. Aetiological theories include altered haemodynamics with venous obstruction causing increased interosseous pressure and cyst formation B. Follow-up x-rays demonstrate that the growth plate moves away from the cyst as the child grows C. Multiple steroid injections are usually required to achieve healing D. Recurrence or persistence of the cyst after surgical curettage and bone grafting occurs in approximately 30–45% of cases E. When fractures do occur, they generally involve the growth plate 11. The following statements concerning the Baumann procedure to correct equinus gait in children with spastic diplegic CP are all FALSE EXCEPT which? A. Allows for selective correction of the contracted gastrocnemius and soleus B. Disrupts the muscle architecture C. Has a high rate of over-lengthening D. Has a high recurrence rate E. Its use for correction of a mild fixed equinus deformity in children has been abandoned 12. The following concerning high energy open tibial fractures in children are all false EXCEPT which? A. Clear established guidelines for management exist B. Gustilo grade III open fractures in children have been shown to have a better outcome than adults C. Monolateral external fixation is currently the most commonly used method of stabilisation for these injuries D. Studies have shown good remodelling potential following tibial malunion E. The use of a programmable circular external fixator has a high rate of complications 13. A 8-year-old boy presents to clinic after a coincidental radiograph of his right proximal femur demonstrates a bone lesion (Figure 16.4). He is asymptomatic.

Children’s Orthopaedics I Structured SBA

Figure 16.4 Anteroposterior (AP) radiograph right femur

B. Girls are affected twice as often as boys C. Most authors recommend accepting any displacement in Salter–Harris type I or II injuries more than 48 hours old. D. Physeal fractures account for up to 70% of all paediatric fractures E. Salter–Harris type I accounts for 20% of all physeal injuries

Questions 16 & 17 A 10 year old boy attends the ED after falling 15 feet onto his left leg. AP pelvis radiograph is shown in Figure 16.5.

Concerning the management options of this bone lesion, which option is the most appropriate? A. The lesion can be managed with observation and serial radiographs B. Any surgery undertaken should involve using autograft bone C. It is necessary to remove structural bone from the outer cyst wall D. Internal fixation lessens the risk of additional surgery E. When performing curettage plus bone grafting, it is often necessary to use adjunctive materials, such as phenol or liquid nitrogen to prevent recurrence 14. Concerning total hip arthroplasty in patients with CP, which of the following is correct? A. Dislocation rate is reported as 12% B. In general, THA should be avoided because of concerns with dislocation, infection and early prosthetic failure C. Heterotopic ossification occurs in 20% of hips D. Revision rates of 30% have been reported at 5 years E. Most common complication of THA is periprosthetic hip fracture 15. Regarding physeal injuries in children, which is correct? A. As a general rule, bones remodel better close to the knee and far from the elbow

Figure 16.5 Anteroposterior (AP) radiograph pelvis

16. Concerning hip fractures in children, which of the following is correct? A. They account for 5% of all paediatric fractures B. Cervicotrochanteric fractures (type III also known as basicervical fractures) are the most common fracture in children C. Delbet classification describes three types of fracture based on the anatomical location of the fracture line D. They are frequently associated with lifethreatening injuries E. Transcervical fractures (type II) constitute the least common of the hip fractures

289

Sattar Alshryda and Paul Banaszkiewicz

17. The patient is taken to theatre for fracture fixation. Concerning the management of hip fractures in children, which is correct? A. Coxa vara, defined as a femoral neck–shaft angle of 14 years B. A distal femoral valgus deformity is almost universally present C. There is decreased internal tibial torsion D. There is increased femoral anteversion E. Guided growth using 8 plate tension band is best utilised for recurrent deformity 21. An 8-week-old baby girl with DDH is seen in clinic 2 weeks after starting treatment with a Pavlik harness. Parents are concerned as she hasn’t been kicking her leg out straight for the past 3 days. Concerning transient femoral nerve palsy following treatment in a Pavlik harness, which is correct? A. Following development of femoral nerve palsy the harness should be abandoned B. Is a common complication following use of a Pavlik harness C. Is diagnosed late D. It is strongly predictive of failure of treatment E. Smaller patients are at increased risk of femoral nerve palsy 22. In the orthopaedic paediatric clinic, you are testing the hips of a 2-week-old baby girl referred to the clinic by the paediatric doctors because they think she has clicking hips and possibly a dislocated hip. Concerning DDH, which is correct? A. Asymmetric skinfolds are a common finding with unilateral hip dislocation B. Hip clicks indicate significant hip pathology C. With each hip examination, a one-time-only Barlow and Ortolani test should be performed to minimise the risk of cartilage damage. D. With the Ortolani test, the examiner attempts to dislocate the hip E. With the Barlow test, the examiner attempts to reduce a dislocated hip

Children’s Orthopaedics I Structured SBA

23. You are reviewing the radiographs of a 6-monthold child in clinic who is being followed up for DDH. Radiographic measurement in DDH include which of the following? A. Hilgenreiners line is parallel to Perkin’s line at the lateral edge of the acetabulum B. MRI is routinely used for the diagnosis and evaluation of DDH particularly after open reduction C. Perkin’s line is drawn through the triradiate cartilages D. The acetabular index which in normal newborns averages 40° E. The ossific nucleus of the femoral head should be located in the lower medial quadrant 24. A 5-year-old boy with spastic diplegic CP attends for regular orthopaedic follow-up. Concerning the hip in cerebral palsy, which is correct? A. A large number of children who progress to dislocation develop a chronically painful hip by early adulthood. B. Hips with a migration index (MI) >50% can still reduce spontaneously with physiotherapy C. The majority of hips subluxate in an anterosuperior direction D. The spasticity or contracture usually involves the abductor and iliopsoas muscles E. There is little evidence to support improved outcome in hip surveillance programmes 25. Concerning radiographic evaluation of CP, all the following are true except which? A. Both the migration index and acetabular index should be measured B. Femoral anteversion is more conveniently measured using ultrasound C. Preoperative arthrography is indicated in spastic hip dysplasia D. The AI steadily increases as the MI increases E. The migration index (MI) is calculated by dividing the width of the uncovered femoral head (AI) by the total width of the femoral head

26. With CP management, which is correct? A. A high migration index (MI) is the most powerful predictor of hip dislocation B. Abduction bracing prevents windblown hips and hyperabduction deformity developing. C. Cerebral palsy patients aged 2 through 8 years should have an orthopaedic examination once a year D. Soft tissue lengthening should be done as soon as progressive hip subluxation is recognised E. The iliopsoas is relatively easy to inject with botulinum A 27. Concerning the surgical management of CP hips, which is correct? A. Age at the time of surgery has a significant predictive effect of success for soft tissue lengthening. B. If the MI is >60%, soft tissue lengthening has only a 20% success rate C. Patient age and severity of subluxation are the two most important factors to consider in hip reconstruction D. Redirection of force is the most important component of the varus derotation osteotomy of the femur (VDRO) E. The most effective treatment for the severely subluxated or dislocated hip is a comprehensive multi-stage approach. 28. Concerning Down’s syndrome and hip disease, which is true? A. Patients with Down’s syndrome who have hip dislocation typically have unstable hips before walking B. The natural history of hip dislocation is divided into three distinct phases C. Surgical management of the subluxing/dislocating hip may include Salter innominate osteotomy D. The acetabulum has increased acetabular anteversion E. There is a 10% incidence of hip instability in Down’s syndrome

291

Sattar Alshryda and Paul Banaszkiewicz

29. A 10-year-old boy presents to the ED with rightsided groin and knee pain. His radiographs are shown in Figure 16.6. Predisposing factors for presentation include all the following EXCEPT which?

C. Over-reduction of the slip D. Pin placement through the anterior cortex of the femoral neck E. Unstable SUFE 31. Regarding the surgical management of SUFE, which is correct? A. Fixation with a second screw leads to a 33% increase in resistance to shear forces, but increases the complication rate 2-fold B. Pin removal is advised C. The entry point should ideally be absolutely central in the femoral neck D. The superolateral corner of the proximal femur should be avoided in order to avoid damage to the artery of Bancovich with resultant ON E. Transient pin penetration does not cause chondrolysis 32. An 11-year-old boy returns to the orthopaedic clinic 6 months following pinning of SUFE. He complains of pain in the groin and thigh, and on clinical examination has decreased hip range of motion, particularly internal rotation. The most likely diagnosis is which of the following? A. Chondrolysis B. FAI C. ON D. Osteoarthritis E. Septic arthritis

Figures 16.6 (a) Anteroposterior (AP) pelvis, (b) frog lateral, (c) anteroposterior AP and (d) lateral radiographs right knee

A. B. C. D. E.

Growth hormone deficiencies Hyperthyroidism Obesity Pituitary deficiency Renal rickets

30. The risk of ON following SUFE is increased with all the following EXCEPT which? A. Joint effusion B. Manipulation

292

33. Which of the following describes the arithmetic method for determining LLD? A. Assumes the distal femoral physes grow at 5mm each year B. Is a complex method of predicting LLD C. Is based on the assumption that boys and girls stop growing at the ages of 14 years and 12 years, respectively D. Is based on the assumption the proximal tibial physes grow at 3mm each year E. Uses chronological rather than bone age

Children’s Orthopaedics I Structured SBA

34. In spina bifida, all the following are TRUE except which? A. An individual patient’s lower-limb deformities cannot be predicted purely on the neurosegmental level and observed muscle imbalance or paralysis B. Lower limb deformities in spina bifida are often severe, rigid and technically difficult to correct, and have high rates of complications and risk of recurrence C. The Ilizarov technique has been used widely in dealing specifically with the correction of lower limb deformities in patients with spina bifida D. The primary neurological lesion is in the lower motor neurone E. There is variable combinations of flaccid paralysis and sensory loss

35. A 9-year-old boy has a plating performed of a distal radius fracture. The fracture is now fully healed and his parents are asking if the plate needs to be removed. Implant removal in children is recommended in all the following cases EXCEPT which? A. All children undertaking sports activities B. If it may complicate another orthopaedic procedure in the future C. If there is a potential for implant migration D. When significant remodelling potential of the bone exists E. When there is significant stress shielding of the bone

293

Sattar Alshryda and Paul Banaszkiewicz

CHILDREN’S ORTHOPAEDICS I STRUCTURED SBA ANSWERS 1. Answer B. Altering the rate of femoral head healing Proximal varus osteotomy does not appear to alter the rate of femoral head healing in Perthes disease. It is possible that varus osteotomy works by a combination of factors, including alternation of hip biomechanics, increased blood flow to the region and forced rest and activity modification induced by the surgery. Although the load-relieving effect of varus angulation is relatively limited based on mathematic modelling and hip force calculation studies, this is still likely to have an influence. Kim HKW, da Cunha AM, Browne R, et al. How much varus is optimal with proximal femoral osteotomy to preserve the femoral head in Legg-Calve-Perthes disease? J Bone Joint Surg Am. 2011;93:341-347. 2. Answer E. Surgery may lead to a significant limb length discrepancy Limb length discrepancy (LLD) is a recognised complication of the procedure along with limited hip abduction, trochanteric bulging and persistent limping. To avoid these complications, it is recommended to keep the neck–shaft angle 105°. A neck shaft angle (NSA) of less than 105° should be avoided, as progressive varus may occur. Kim et al found no significant correlation between the postoperative neck–shaft angle at maturity and Stulberg outcome or between the amount of varus change and Stulberg outcome. They recommended 10° to 15° of varus correction when performing proximal femoral varus osteotomy on hips that are in the early stages of Legg–Calve-Perthes disease. Preoperative planning should involve templating for a medial closing wedge osteotomy and stabilization with a fixed angle device Herring JA, Kim HT, Browne R. Legg-CalvePerthes disease. Part II: Prospective multicenter study of the effect of treatment on outcome. J Bone J Surg Am 2004;86-A(10):2121-2134.

294

Wiig O, Terjesen T, Svenningsen S. Prognostic factors and outcome of treatment in Perthes’ disease: a prospective study of 368 patients with five-year follow-up. J Bone Joint Surg Br. 2008;90(10):1364-1371. 3. Answer C. Interposed periosteum is an indication for open reduction Interposed periosteum may cause a block to reduction, but is not necessarily an indication for open reduction, as it has been shown to resorb and not cause physeal arrest. These injuries have a high incidence of physeal arrest, ranging from 27.3% to 90% with a mean of 40% of cases. This may result in leg length discrepancy and/or angular deformity. Metaphyseal screw fixation sparing the physis is probably the ideal in Salter–Harris type II fractures if the metaphyseal fragment is large enough to afford screw purchase. Closed reduction and cast immobilisation has a high risk of re-displacement. Garrett B, Hoffman E, Carrara H, et al. The effect of percutaneous pin fixation in the treatment of distal femoral physeal fractures. J Bone Joint Surg Br. 2011;93(5):689. 4. Answer E. With the Ponseti technique, about 8% of idiopathic clubfeet require further surgical treatment after a percutaneous tenotomy While the Pirani and Dimeglio scores quantify the severity of the clubfeet, they have not been shown as reliably prognostic of the outcome. Janicki et al reported that children with CTEV managed with Denis Browne boots and bars had far fewer recurrences than those managed with ankle–foot orthoses. The AFO was believed to be as effective as DBB and more convenient for child and family; and had been regarded as the preferred method of bracing. It is abduction that stretches the medial soft tissues. An AFO is unable to control this, which was postulated as one reason for the lower success rate when using them. Janicki et al found that AFO are ineffective in preventing recurrences following correction of idiopathic clubfeet by the Ponseti method. The DBB design of the foot abduction orthosis is not universally tolerated and newer braces such as the Mitchell Brace and the Dobbs

Children’s Orthopaedics I Structured SBA

articulated abduction orthosis have been introduced to improve tolerance and retention of the foot in the braces. The Ponseti method of management of idiopathic clubfeet has been adopted widely with high rates of success. It been shown to be consistently successful in achieving correction of deformity without the need for extensive surgery. Using the Ponseti technique, Janicki et al reported only 5 (3%) of 171 idiopathic clubfeet required any surgical treatment beyond the percutaneous tenotomy. Janicki J, Wright JG, Weir S, et al. A comparison of ankle foot orthoses with foot abduction orthoses to prevent recurrence following correction of idiopathic clubfoot by the Ponseti method. J Bone Joint Surg Br. 2011;93(5):7. 5. Answer C. They are the most common paediatric elbow fracture Lateral humeral condylar fractures are the second most common paediatric elbow fracture after supracondylar fractures, accounting for between 12% and 20% of all children’s elbow fractures. They are easily missed and can also displace, leading to malunion and deformity. Push-off and pull-off theories also have been proposed to account for these fractures. The most likely cause of lateral humeral condylar fracture is a combination of the push-off and pull-off mechanisms. Milch type II fractures,

which extend into the apex of the trochlea, are probably caused by avulsion forces acting on the condyle while the olecranon directs the force into the trochlea. Milch type I fractures are probably caused by impaction of the radial head against the capitellum with a fall onto an outstretched, extended arm. Tejwani N, Phillips D, Goldstein RY. Management of lateral humeral condylar fracture in children. J Am Acad Orthop Surg. June 2011;19:350-358. 6. Answer D. Milch is of little use in determining fracture management and is largely of historical interest More important in determining management is the degree of displacement. If the degree of maximal fracture displacement is greater than 2mm, then the fracture should be managed operatively. Management of fractures less than 2mm displaced is more controversial. Milch classification of lateral condylar fractures was classic ‘catch a less-prepared candidate out’ viva material a few years ago (Figure 16.7). The classification system involves differentiating between Salter–Harris II and IV and which Milch fracture has the worst prognosis. Miltch II, despite being SH II is the worst- the catch is that in general a SH IV fracture has a poorer prognosis than a SH II fracture. Classification systems are now generally avoided in the exam unless they are first volunteered by a candidate.

Figure 16.7 Milch classification lateral condylar fractures

295

Sattar Alshryda and Paul Banaszkiewicz

Figure 16.8 (a–c) Jakob classification 1–3

Miltch type I is a Salter–Harris IV fracture. A Miltch type II is a Salter–Harris type II fracture (a way to remember is two = two). Minimal deformity is seen at the elbow following a lateral condylar fracture. Milch described two lateral condylar fracture patterns. Type I is a simple fracture of the lateral condyle that results from a force directed along the radius with impaction of the head of the radius in the capitulotrochlear sulcus. These fractures are characterised by a fracture line that courses lateral to the trochlea and into the capitulotrochlear groove, resulting in a Salter–Harris type IV fracture. The elbow remains stable because the trochlea is intact. Milch type II fracture is a fracture-dislocation of the lateral condyle resulting from a force that is directed upwards and outwards along the ulna, impacting the olecranocoronoid ridge against the trochlear groove. Type II fractures are characterised by a fracture line that extends into the apex of the trochlea, resulting in a Salter–Harris type II fracture. The elbow is unstable because the trochlea is disrupted. Jakob classified paediatric lateral humeral condylar fracture based on fracture fragment displacement. Type I fracture is nondisplaced, with an intact articular surface. The fracture line does not completely traverse the cartilaginous epiphysis (Figure 16.8(a)).

296

Type II fracture is complete and extends through the articular surface (Figure 16.8(b)). The fracture may be moderately displaced. Type III fracture involves complete displacement and rotation of the fracture, resulting in elbow instability and loss of the normal anatomical relationship between the capitellum and the proximal radius (Figure 16.8(c)). Tejwani N, Phillips D, Goldstein RY. Management of lateral humeral condylar fracture in children. J Am Acad Orthop Surg. June 2011;19:350-358. Jakob R, Fowles JV, Rang M, Kassab MT. Observations concerning fractures of the lateral humeral condyle in children. J Bone Joint Surg Br. 1975;57(4):430-436. 7. Answer D. Surgical management usually consists of open reduction and smooth pin fixation With open reduction, pins are usually placed outside the surgical incision. Two pins are usually recommended. Three pins are occasionally necessary with significant fracture displacement or comminution, but this may result in an increased incidence of loss of motion and lateral spur formation. A valgus rather than varus force on the elbow can recreate the initial deformity and may assist in visualising the fracture fragment. All dissection should be done anterior to avoid damaging the blood supply to the distal fragment

Children’s Orthopaedics I Structured SBA

and subsequently placing the patient at risk of osteonecrosis. A posterolateral or lateral or posterior approach can be used to approach the fracture. A posterolateral approach is generally preferred, being considered a safe approach through the interneural interval (Kocher interval) between anconeus and the exterior carpi ulnaris, thus protecting the posterior interosseous nerve. The lateral approach is riskier for damage to the PIN. Many authors have suggested that all lateral humeral condylar fractures should be managed surgically because of the risk of displacement and the high rate of non-union. Some surgeons believe that very few truly nondisplaced lateral humeral condylar fractures exist. Some surgeons, however, believe that if the degree of maximal fracture displacement is less than 2mm in all radiographs, then the fracture is stable and not at risk for displacement and can be treated non-operatively. Cubital varus is thought to occur due to traumatic inflammation occurring around the fracture site leading to overgrowth. Tejwani N, Phillips D, Goldstein RY. Management of lateral humeral condylar fracture in children. J Am Acad Orthop Surg. June 2011;19:350-358. 8. Answer E. Ulnar nerve palsies typically present early Tardy ulnar nerve palsy involves slow, progressive paralysis of the ulnar nerve. It is caused by stretching of the nerve, as is seen with cubitus valgus. Although the time to presentation varies widely, this complication typically presents late (average, 22 years post-injury) that traumatic inflammation about the fracture leads to overgrowth at the fracture site. Tejwani N, Phillips D, Goldstein RY. Management of lateral humeral condylar fracture in children. J Am Acad Orthop Surg. June 2011;19:350-358. 9. Answer A. Complications occur in nearly 1 in 6 patients The most common complication of a retained plate is an implant-related fracture, occurring in about 7% of patients. The second most common complication is pain and irritation from the plate. Less fractures are seen when using a one–third tubular plate.

The risk of implant-related fracture is increased with radial plates and with the use of a DCP when compared with ulnar plates and one-third tubular plates, respectively. The increased stiffness characteristics of the DCP in contrast to a tubular plate probably results in a greater stress-shielding effect on the bone. Male gender and younger age were isolated predictors of a complication. Clement ND, Yousif F, Duckworth AD, Teoh KH, Porter DE. Retention of forearm plates: risks and benefits in a paediatric population. J Bone Joint Surg Br. 2012;94:134-137. 10. Answer E. When fractures do become evident, they generally involve the growth plate itself Fractures very rarely involve the growth plate. Other aetiological theories include a defect in endochondral bone formation. Option B explains why two-thirds of these lesions are not in contact with the growth plate when discovered in children older than 10 years. In option C, some recent studies cast doubt on the efficacy of steroid injections for simply bone cysts. In option D, this relatively high local recurrence rate may be due to the fact that surgically treated cysts are more active, more aggressive, and likely to be recurrent. Alshryda S, Wright J. Chapter 44: Evidence based treatment for simple bone Cyst. In Alshryda H, Banaszkiewicz, eds. Paediatric Orthopaedics: An Evidence-Based Approach to Clinical Questions. Springer; 2016. pp. 51-75. 11. Answer A. Allows for selective correction of the contracted gastrocnemius and soleus In CP, a fixed contracture of the calf muscles disrupts the normal heel, ankle and forefoot rockers and impairs the function of the foot. This equinus deformity decreases the stability in stance, causes a loss of the smooth translation of the body over the foot and often leads to an inadequate clearance of the foot during the swing phase of gait. Baumann described intramuscular lengthening of the gastrocnemius and soleus by multiple incisions over the muscle bellies in the anterior aponeurosis of the gastrocnemius and the adjacent fascia of soleus. This procedure allows selective

297

Sattar Alshryda and Paul Banaszkiewicz

fractional lengthening of both muscles. It is suggested as the procedure of choice for correction of a mild fixed equinus deformity in CP children. The procedure does not disrupt the muscle architecture with a low rate of over-lengthening and low recurrence rate. Svehlík M, Kraus T, Steinwender G, Zwick EB, Saraph V, Linhart WE. The Baumann procedure to correct equinus gait in children with diplegic cerebral palsy: long-term results. J Bone Joint Surg Br. 2012;94:1143-1147. 12. Answer C. Monolateral external fixation is currently the most commonly used method of stabilisation for these injuries Unlike adults, no clear guidelines for the management of these fractures in the skeletally immature patient exist. These injuries are associated with similar rates of complications and outcomes to adults, particularly if the child is aged > 10 years. The magnitude of remodelling depends on a child’s age and gender. Studies have shown in girls aged 9–12 years and boys aged 11–12 years, only approximately 50% of tibial deformity is corrected, and no more than 25% is corrected in children aged > 13 years. Monsell et al reported on the use of a circular external frame in children. They concluded that it is a well-tolerated and reliable method of treatment with rapid time to union and few complications in comparison with other treatment methods. Where appropriate expertise exists, they recommended its use in the management of high-energy open tibial fractures in children. Monsell FP, Howells NR, Lawniczak D, Jeffcote B, Mitchell SR. High-energy open tibial fractures in children: treatment with a programmable circular external fixator. J Bone Joint Surg Br. 2012;94:989-993. 13. Answer D. Internal fixation lessens the risk of additional surgery Wilke et al reported a multi-institutional, retrospective review to evaluate their experience with proximal femoral unicameral bone cysts. The number of patients requiring additional surgeries was increased among those who did not undergo internal fixation. There was no difference in time to radiographic healing.

298

However, time to return to normal activities was reduced if patients had received internal fixation. They concluded that internal fixation should be considered in the treatment of proximal femoral unicameral bone cyst. Most UBCs in the lower extremity with a low risk of pathological fracture are treated with observation and serial radiographs. Surgical treatment is indicated when UBCs are symptomatic, rapidly enlarging and/or are at risk for pathological fracture, such as in the proximal femur. Curettage with or without bone grafting shows modest healing rates and high recurrence rates. Other treatment options include aspiration and injection of the lesion with corticosteroids, bone marrow aspirates or demineralised bone matrix, all with questionable superiority over other surgical options. In locations at risk for pathological fracture, prophylactic internal fixation should be considered (dynamic hip screw). Unicameral bone cysts are benign, fluidfilled cavities that tend to expand and weaken the local area. They are generally seen in the metaphyseal areas of long bones in skeletally immature persons. They usually present with a pathological fracture. Such fractures occur through thin, weakened bone and are generally not grossly displaced, nor are they difficult to treat. The most common site is the proximal humerus, followed by the femur. While growth plate injury can occur if the cyst is immediately adjacent to or involves the epiphyseal growth plate, recurrence is the most common complication. AP femoral radiograph is suggestive of a high probability of a fracture resulting from the unicameral bone cyst and surgical treatment would be recommended. Allograft bone chips have proved effective in the treatment of these cysts. The choice of autologous bone graft or a substitute is dependent on the orthopaedic surgeon’s preference. In choice C, the opposite is true. It is not necessary to remove structural bone from the outer cyst wall. In choice E, the opposite is true. There has been a move away from using these materials as they may interfere with graft and bone healing. Wilkins RM. Unicameral bone cysts. J Am Acad Orthop Surg. 2000;8(4):217–224.

Children’s Orthopaedics I Structured SBA

Wilke B, Houdek M, Rao RR, Caird MS, Larson AN, Milbrandt T. Treatment of unicameral bone cysts of the proximal femur with internal fixation lessens the risk of additional surgery. Orthopedics. 2017 Oct 10;40(5):e862–867. 14. Answer A. Dislocation rate is reported as 12% THA provides clinically important pain relief and functional improvement for patients with CP. A number of recent studies have reported similar outcomes in terms of implant survival and postoperative complications compared with patients with a diagnosis of primary osteoarthritis. Beware, however, that these procedures were being performed in tertiary centres by experienced high-volume hip surgeons. Patients should be able to walk independently or with the use of gait aids. The procedure is probably best avoided in non-ambulators. Pain relief in CP following THA was reported in three studies to be around 87–94% but most (80–83%) of these patients were independently mobile and hence less severely affected with CP. The HO figure is higher; it is reported between 58% and 80%, but this isn’t usually clinically significant. Revision rates vary between reported series between 5% and 15%. The most common complication was recurrent prosthetic hip dislocation. Periprosthetic hip fracture is the second most common complication. The latter primarily occurred intraoperatively or within 6 months of the index THA. Other complications reported included aseptic loosening, periprosthetic joint infections (PJI), heterotopic ossification, wound dehiscence and deep vein thrombosis (DVT). Weber M, Cabanela ME. Total hip arthroplasty in patients with cerebral palsy. Orthopedics. 1999;22:425-427. Adams CT, Lakra A. Clinical and functional outcomes of total hip arthroplasty in patients with cerebral palsy: a systematic review. J Orthop. 2020 Jan 25;21:19-24. Buly RL, Huo M, Root L, Binzer T, Wilson PD Jr. Total hip arthroplasty in cerebral palsy: long-term follow-up results. Clin Orthop. 1993;296:148–153.

15. Answer A. As a general rule, bones remodel better close to the knee and far from the elbow Physeal fractures account for up to 30% of all paediatric fractures. Boys are affected twice as often as girls. Salter–Harris type I accounts for 6% of all physeal injuries. Most authors recommend accepting any displacement in Salter–Harris type I or II injuries more than 7–10 days old. 16. Answer D. They are frequently associated with life-threatening injuries Hip fractures in children most often result from high-energy mechanisms and are frequently associated with life-threatening injuries. Primary evaluation focuses on airway management, cardiovascular stabilisation, detection or exclusion of potentially life-threatening injuries of the head, neck, thorax, abdomen and pelvis. They are rare in children, accounting for 99% efficiency, with less than 10 colony-forming units per cubic metre at the centre of the operating theatre. Theatres can employ laminar-flow systems, which may be either vertical or horizontal. Screening for MRSA infection does reduce infection rates, as positive patients can be given eradication therapy preoperatively; this includes topical mupirocin and chlorhexidine baths 5 days preoperatively. They can also be segregated on the wards – ring fencing elective wards can help reduce infection in arthroplasty patients and is recommended. Intravenous glycopeptides are the treatment of choice for MRSA infection. 37. Answer C. Elevated serum WCC The working group on the Diagnosis of Periprosthetic Joint Infection at the Proceedings of the International Consensus Meeting on Periprosthetic Joint Infection in 2013 defined periprosthetic joint infection as two positive periprosthetic cultures with identical organisms, OR a sinus tract communicating with the joint, OR three of the following minor criteria: Elevated CRP and ESR Elevated synovial fluid WCC OR ++ change on leukocyte esterase test strip Elevated synovial fluid PMN% Positive histological analysis of periprosthetic tissue Single positive culture A raised serum white cell count is not included in the definition.

384

www.efort.org/wp-content/uploads/2013/10/ Philadelphia_Consensus.pdf Parvizi J, Gehrke T, Chen AF. Proceedings of the International Consensus on Periprosthetic Joint Infection. Bone Joint J. 2013 Nov;95-B (11):1450-1452. 38. Answer D. Powdered antibiotics should be used and added before mixing Powdered, not liquid, antibiotic should be used and added to the cement before mixing. Typically, high-dose vancomycin and gentamicin are used, but this should be adapted according to known organism sensitivities. Only heat-stable antibiotics can be used, due to the exothermic reaction while cement polymerises. Commercially available antibiotic cement has lower doses used for prophylaxis, not for treatment of infection. Antibiotic elution is highest for the first 24–72 hours following implantation. The addition of antibiotics can weaken cement, and so should only be used in the recommended doses. 39. Answer C. Non-articulating cement spacers should be used in patients with bone loss or lack of soft tissue constraint Cement spacers can be articulating or nonarticulating. While there are no absolute contraindications to their use, in patients with bone loss or a lack of soft tissue integrity, nonarticulating spacers should be used. There is no reported difference between handmade or industry-made cement spacers on the rate of infection eradication. Synovial PMN% >90% is positive for acute infection, and a target of five intraoperative tissue samples should be obtained, according to consensus meeting definitions. 40. Answer E. Two-week history of increasing pain in the hip, with a raised CRP and ESR; radiographs are satisfactory, culture results show S. aureus Patients presenting within 3 weeks of surgery, or the onset of their symptoms, with stable implants and a known organism are suitable to undergo a DAIR procedure. If the history is longer than this, or the organism is not known, then a one- or two-stage revision should be performed. In cases where there is loosening (i.e. the acetabulum in DeLee and Charnley zone 3),

Pathology I Structured SBA

revision is required, rather than just exchange of the femoral head. In the presence of a normal CRP and ESR, with negative culture, and aseptic loosening, a revision procedure would be required rather than DAIR. 41. Answer E. This patient will demonstrate limited response to ACTH testing The adrenal gland consists of two parts: the cortex, which produces mineralocorticoids, glucocorticoids and androgens, and the medulla, which produces catecholamines. Chronic steroid use (such as in rheumatoid arthritis, COPD and polymyalgia rheumatica) can suppress the usual hypothalamic-pituitary-adrenal axis and render a patient unable to respond appropriately to trauma and stress. Patients require additional intravenous steroid in the perioperative period to prevent an adrenal crisis. Symptoms of adrenal insufficiency include hypotension, confusion, hypoglycaemia, sweating, abdominal pain and lethargy. In addition to fluids and glucose, intravenous steroid (typically hydrocortisone) is required. Patients with adrenal insufficiency will not respond to ACTH testing. 42. Answer D. Stop etanercept 2 weeks before surgery, continue methotrexate Guidelines have now been published describing the perioperative management of antirheumatoid medication in patients undergoing hip and knee surgery. Disease-modifying drugs including methotrexate, sulfasalazine, hydroxychloroquine and leflunomide can be continued throughout the peri-operative period. Biological agents should be stopped, and the timing depends on the dosing-cycle, which relates to the half-life of the drug. They should be resumed a minimum of 14 days after surgery, when the wound has healed and there is no concern about infection. www.rheumatology.org/Portals/0/Files/ACRAAHKS-Perioperative-Management-Guideline .pdf 43. Answer E. The risk of transmission from patient to surgeon with a needlestick injury is 1:300 If a surgeon sustains a needlestick injury, this should be reported straight away to the

occupational health department, and an incident form completed. The area affected should be washed immediately with water and encouraged to bleed. Occupational health should be contacted in-hours, or the emergency department out-ofhours. Baseline blood-tests from both donor and recipient should be obtained for HIV, HepB and HepC. In high-risk patients (including multiple sexual partners, multiple tattoos or piercings, blood transfusions abroad, recreational drug user, known blood-borne viruses), then post-exposure prophylaxis should be started immediately, in conjunction with occupational health or infectious disease departments. The risk of transmission from a needlestick injury of HIV is 0.3% (1:300), hepatitis C (3%), and hepatitis B (30%). The risk is less with exposure to a contaminated mucous membrane (~0.09%). Hollow, rather than solid, needles increase the risk of transmission. 44. Answer A. They are at increased risk of developing a DVT Factor V Leiden mutation is an inherited disorder of factor V resulting in thrombophilia. Patients are at increased risk of developing DVT and PE and require prophylaxis/treatment for this. 45. Answer E. Variable-rate insulin infusion should be stopped before the administration of subcutaneous insulin postoperatively Guidelines regarding perioperative diabetic management suggest that if a patient will miss less than one meal, then a variable-rate insulin infusion is not required if they have good diabetic control (HbA1C less than 69mmol/L). In elective cases, diabetic control can be optimised, as indicated by changes in HbA1c. If patients have poor diabetic control, or are likely to miss more than one meal, then a variable-rate insulin infusion should be started, and continued until the patient is eating and drinking and has had their usual diabetic medication. They should be prioritised on elective operating lists. Dietcontrolled diabetics do not need list-order prioritisation. www.diabetes.org.uk/resources-s3/2017-09/ Surgical%20guidelines%202015%20-%20full% 20FINAL%20amended%20Mar%202016_0.pdf

385

Rory Morrison and Rajesh Kakwani

https://anaesthetists.org/Portals/0/PDFs/ Guidelines%20PDFs/Guideline_perioperative_ management_surgical_patient_diabetes_2015_ final.pdf?ver=2018-07-11-163756-413&ver= 2018–07-11–163756-413 https://bnf.nice.org .uk/treatment-summary/diabetes-surgery-andmedical-illness.html Sudhakaran S, Surani SR. Guidelines for perioperative management of the diabetic patient. Surg Res Pract. 2015;2015:284063. 46. Answer B. Early operative intervention for acetabular fracture fixation The formation of heterotopic ossification is associated with spinal cord injury, traumatic brain injury, burns, high injury severity score, total hip arthroplasty, blast injuries, amputation through the zone of injury, prolonged need for mechanical ventilation and decubitus ulcers. Surgical approaches to the acetabulum in particular have been associated with HO, the extended iliofemoral having the greatest association. A delay in surgical intervention is associated with an increased risk of developing HO, so early surgical fixation is recommended. 47. Answer C. High doses of NSAIDs reduce the risk of developing heterotrophic bone formation after hip surgery. Prophylaxis against HO includes the use of the non-steroidal anti-inflammatory indomethacin, a non-selective COX-1 and COX-2 inhibitor which inhibits the effect of prostaglandin E2 on the differentiation of progenitor cells. Due to the cost and potential to induce malignancy with radiotherapy treatment, indomethacin is often the first line of treatment, although this has been associated with the risk of non-union of fractures. When radiotherapy is used, a single dose of ~700cGy within 72 hours following surgery has been shown to be of benefit. Bisphosphonates are no longer recommended to help prevent HO formation. Fransen M, Anderson C, Douglas J, et al. Safety and efficacy of routine postoperative ibuprofen for pain and disability related to ectopic bone formation after hip replacement surgery (HIPAID): randomised controlled trial. BMJ. 2006;333:519.

386

Neal B, Rodgers A, Dunn L, Fransen M. Non-steroidal anti-inflammatory drugs for preventing heterotopic bone formation after hip arthroplasty. Cochrane Database Syst Rev. 2000; (3):CD001160. 48. Answer B. Indomethacin can prevent further HO formation Prophylaxis against HO includes the use of the non-steroidal anti-inflammatory indomethacin, a non-selective COX-1 and COX-2 inhibitor which inhibits the effect of prostaglandin E2 on the differentiation of progenitor cells. Surgery to excise HO should be delayed until mature bone is noted, and the suggested delay is 6 months for traumatic HO, 12 months in spinal cord injury and 18 months for traumatic brain injury. The Brooker classification describes HO around the hip, and comprises 4 classes, with IV being the worst (ankylosis of the hip). Radiotherapy as a single-dose of 700–800cGy can be used up to 72 hours following surgery. Heterotopic ossification (HO) is the formation of lamellar bone in extra-skeletal tissue, typically between the joint capsule and muscle. It can be classified into three subtypes; neurogenic, traumatic, and fibrodysplasia ossificans progressiva (caused by a mutation in the ACVR1 gene). Factors associated with formation of HO include spinal cord injury, traumatic brain injury, burns, high injury severity score, total hip arthroplasty, blast injuries, amputation through the zone of injury and decubitus ulcers. Surgical approaches to the acetabulum in particular have been associated with HO, the extended iliofemoral having the greatest association. The hip and elbow are the joints most commonly affected, with restricted movement of the joint noted. The Brooker classification has been developed to classify HO around the hip (Table 20.2), and the Hastings and Graham classification for HO at the elbow (Table 20.3). Plain radiographs can show HO, with bony cortex, sharp demarcation and trabecular bone pattern indicating mature bone. Bone scans can give an earlier diagnosis, although they are a more expensive test. Ultrasound and CT scans are also of use to aid diagnosis. Prophylaxis against HO includes the use of the non-steroidal anti-inflammatory indomethacin, a

Pathology I Structured SBA

Table 20.2 Brooker classification HO hip

Grade

Description

1

Islands of bone in soft tissue

2

Bone spurs from either femur or pelvis; >1cm gap between spurs

3

Bone spurs from either femur or pelvis; 5μm 28. Chronic renal impairment can lead to tertiary hyperparathyroidism What is the mechanism by which this develops? A. Loss of renal function leads to a reduction in the 1α-hydroxylation of vitamin D, which lowers the serum calcium, stimulating parathyroid hormone; eventually this leads to hyperplasia of the chief cells and independent PTH production

Pathology III Structured SBA

B. Loss of renal function leads to a reduction in the 25-hydroxylation of vitamin D, which lowers the serum calcium, stimulating parathyroid hormone production. C. Loss of renal function leads to a reduction in the 25-hydroxylation of vitamin D, which lowers the serum calcium, stimulating parathyroid hormone; eventually this leads to hyperplasia of the parafollicular C cells and independent PTH production D. Loss of renal function leads to a reduction in the retention of renal calcium, stimulating parathyroid production E. Loss of renal function leads to an increased renal loss of calcium and PTH, which lowers the serum calcium, stimulating more parathyroid hormone production; eventually this leads to hyperplasia of the C cells and independent PTH production 29. Paget disease of the bone can be a difficult problem to treat, but also causes problems in the implementation of orthopaedic procedures. What is the pathophysiology of the disease? A. Decreased bone resorption, causing expansion and disorder of the bone architecture B. Decreased bone resorption coupled with increased bone formation, causing disordered bone architecture C. Increased bone formation in an uncontrolled manner, leading to a smaller compensatory increase in bone resorption D. Increased bone formation in an uncontrolled manner with a paradoxical decrease in bone resorption E. Increased bone resorption and a compensatory increase in bone formation at a disordered rate

30. In the peripheral nervous system, neurones communicate via specialised synapses where neurotransmitters cross the synaptic cleft, initiating an action potential. Which of the following is the best description of the ion concentration changes occurring in an action potential? A. The resting cell membrane potential is – 70mV; depolarisation occurs as potassium rushes out of the cells through voltage-gated ion channels; sodium then moves into the cell to restore the resting potential B. The resting cell membrane potential is – 70mV; depolarisation occurs as sodium rushes into the cells through voltage-gated ion channels; potassium then moves out of the cell to balance the potential; the resting potential is restored via the Na/K ATP pump C. The resting cell membrane potential is – 70mV; depolarisation occurs as sodium and potassium rush out of the cell through voltage-gated ion channels; the resting potential is then restored by Na/K ATP pumps D. The resting cell membrane potential is – 70mV; depolarisation occurs as calcium rushes into the cell via voltage-gated ion channels; the resting potential is restored by sodium moving into the cell E. The resting cell membrane potential is – 70mV; depolarisation occurs as sodium moves out of the cell and calcium moves in through voltage-gated channels; the resting potential is restored by ATP pumps reversing this process.

401

Tom Marjoram and Majeed Shakokani

PATHOLOGY III STRUCTURED SBA ANSWERS 1. Answer E. Physis Achondroplasia is due to a mutation in FGFR3 and is inherited in an autosomal dominant pattern (although there is a high rate of spontaneous mutation). It affects the proliferation of chondrocytes in the proliferative zone of the physis. 2. Answer D. Hyaline cartilage As the name suggests, an enchondroma is a lesion of cartilaginous origin. Histology shows chondroid matrix and it is very difficult histologically to differentiate from an osteosarcoma. 3. Answer C. Small myelinated, unmyelinated, large myelinated Local anaesthetic works by inhibiting sodium influx by blocking sodium ion channels in the cell membrane of the nerve. They work more effectively on certain nerve fibres depending on the level of myelination and the diameter of the axon (thin axon more affected, myelinated most affected). The phrase coined for this effect is ‘differential blockade’. In order of most to least affected: type B (autonomic), type C (pain), type A (temperature, proprioception and motor). 4. Answer D. Inhibiting the formation of the brush border The nitrogen-containing bisphosphonates inhibit the mevalonic acid pathway, which produces proteins essential for the formation of the bush (or ruffled) border, the site of attachment to the bone – thus resorption is inhibited. Nonnitrogen-containing bisphosphonates work by forming an analogue of ATP, which in turn leads to apoptosis. 5. Answer E. Staphylococcus aureus E. coli is gram negative; Pseudomonas are gramnegative rods, S. aureus is the only gram-positive cocci and is also the most common cause of bacterial infection. Clostridium are grampositive, rod-shaped bacilli. H. influenzae are gram-negative cocci.

402

6. Answer A. Binding to the 30s ribosomal subunit Gentamycin has been well documented to cause hearing loss. A much better-known side effect is nephrotoxicity. Gentamycin works well for gram-negative infections and therefore is the correct answer. It works by attaching to the 30S ribosomal subunit. 7. Answer A. Affecting the adjacent hydrogen nuclei, therefore increasing the contrast Gadolinium works by reducing the relaxation time of adjacent hydrogen ions, therefore producing a measurable MRI signal differential indicating the regions in which the contrast agent is present. 8. Answer C. Acid-fast bacilli on Ziehl–Neelsen stain TB are acid-fast bacilli, and the Ziehl–Neelsen stain is used to determine this. The initial stain is applied, and the tissue washed with an acid – retention of the initial stain indicates that the cells are ‘acid fast’. 9. Answer D. 200% Elastin has been shown to withstand up to 200% strain. The ligamentum flavum is not pure elastin; therefore, its strain resistance will be lower. Strain is defined as change in length as a proportion of original length. 10. Answer E. Stress–strain Young’s modulus of elasticity is defined as stress– strain. Stress is force per unit area and strain is change in length over original length. Therefore, a combination of force per unit area and change in length gives a measure of elasticity. 11. Answer A. A decrease in force per unit area when a constant deformation is maintained. Stress relaxation is a reduction in the force required to maintain a constant deformation. It is often confused with creep, which is increasing deformation with a constant force. 12. Answer A. Water decreases, chondrocyte number decreases but size increases, keratin increases and chondroitin decreases

Pathology III Structured SBA

Aging and degeneration are separate processes, and it is important to appreciate this. Aging results in a decrease in water content, a reduction in chondrocyte number and an increase in keratin with a reduction in chondroitin. 13. Answer A. Dorsal column afferent proprioception decussates in brainstem, corticospinal tract efferent motor function decussates in brainstem, spinothalamic tract afferent temperature decussates near level of exit The dorsal columns are afferent and decussate in the brainstem. The corticospinal tracts are efferent motor tracts also decussating in the brainstem. The spinothalamic tract is unique in that this afferent pathway decussates within two levels of its exit – this is essential knowledge in the understanding of the Brown-Séquard syndrome. 14. Answer D. The amplitude of a motor response indicates the number of working motor units (rather than nerve size) The latency not only relies on nerve conduction speed but also neuromuscular transmission and depolarisation of the muscle membrane. Conduction speed is not faster in a distal portion of a nerve. The amplitude of a motor response relies on the size of the muscular contraction rather than the size of the nerve, and this is therefore correct. In general, upper limb conduction is faster than lower limb. Temperature is well known to have an effect on the rate of conduction of both myelinated and unmyelinated nerve fibres. 15. Answer A. The A band contains both actin and myosin The H band contains only myosin, The Z disc is between two adjacent actin units. The M line represents the connection between two myosin fibres. The A band does indeed contain both fibres. The I band contains only actin.

17. Answer E. Strength is high at implantation; this decreases at 3–6 months and returns to full strength at 1–2 years Cortical grafts are used for their initial structural support. This initial strength reduces as the process of creeping substitution takes hold at 3–6 months, but by 1 year the strength has returned. 18. Answer D. Fresh – No structural degradation; Frozen – Reduced immunogenicity; Freezedried – Easily stored and transported Fresh bone shows no structural degeneration but contains cellular material, increasing the risk of immunogenicity and infection. Frozen bone has a reduced immunogenicity, but frozen bone is more expensive to transport. Freeze-dried does not require specialist transport and has reduced immunogenicity but incorporates less well than the others. 19. Answer E. 3rd class has the load at the opposite end of the lever from the fulcrum with the applied force between A 1st class lever has a central fulcrum with opposing forces at either end (Figure 22.1). A 2nd class lever has a fulcrum at one end with both the forces acting on the same side; the load is closer to the fulcrum than the counter force. A 3rd class lever has a fulcrum at one end with both forces acting on the same side of the fulcrum, with the load farther from the fulcrum than the counteracting force; therefore, the counterforce is at a disadvantage. Figure 22.1 Levers

16. Answer C. Calcium moves into the cell, binding to troponin C, which displaces troponin I, and allowing engagement of the myosin head Calcium is vital for muscle function. It binds with troponin C, displacing the inhibitory troponin I, and this allows the myosin head to engage, allowing contraction.

403

Tom Marjoram and Majeed Shakokani

20. Answer B. Haemophilia A – X-linked recessive affecting factor VIII; Haemophilia B – Xlinked recessive affecting factor IX Haemophilia A is an X linked recessive disorder affecting factor VIII and haemophilia B is Xlinked recessive affecting factor IX. 21. Answer C. Type A cells – Macrophage origin, located in superficial layer, perform phagocytosis; Type B cells – Fibroblast origin, located in deep layer, synthesise synovial fluid Synovial lining type A cells are of macrophage origin and are responsible for the removal of debris and infection found in the superficial layer. The Type B cells are fibroblast-like and are found in the deep layer. They aid the synthesis of key proteins in the synovial fluid. 22. Answer E. The fluid behaves differently depending on the rate of strain A non-Newtonian fluid can behave differently under low strain environments to high strain environments. This aids the function of the synovial fluid under high loads within the joint. 23. Answer E. Annulus – Type I collagen with low proteoglycan content and low water content; Nucleus – Type II collagen with high proteoglycan and high water content The normal disc has type I collagen in the annulus with type II in the nucleus. The annulus has a relatively low water content and the nucleus, due to the presence of proteoglycans, has a high water content. This reduces in the degenerate nucleus. 24. Answer A. Core diameter While much emphasis is placed on the design of implants, it is the quality of the bone that has the most effect. When considering the design of a screw, however, it is the core diameter that has the most impact on screw pull-out. 25. Answer B. Moving the centre of rotation inferior and medial to improve the fulcrum for the deltoid The reversal of the ball and socket moves the centre of rotation medially and inferior. This

404

increases the lever arm for the deltoid. This operation is often used in the absence of the rotator cuff muscles. 26. Answer B. It binds to antithrombin III, increasing its effectiveness at inhibiting factor Xa LMWH is an antithrombin III activator. Antithrombin III binds to factor Xa, inhibiting its function and thus reducing the effectiveness of the clotting cascade. 27. Answer D. They filter the air with high efficiency, removing 99% of particles larger than 0.5μm HEPA filters have high efficiency (99%) and remove particles larger than 0.5μm. This reduces the particles in the air on which bacteria are transported. It does not filter out bacteria themselves. 28. Answer A. Loss of renal function leads to a reduction in the 1α-hydroxylation of vitamin D, which lowers the serum calcium, stimulating parathyroid hormone; eventually this leads to hyperplasia of the chief cells and independent PTH production Tertiary hyperparathyroidism results from loss of renal function, which leads to a reduction in the 1α-hydroxylation of vitamin D, which, in turn, lowers the serum calcium, stimulating parathyroid hormone. Eventually this leads to hyperplasia of the chief cells and independent PTH production. It is this final step that transforms secondary into tertiary. 29. Answer E. Increased bone resorption and a compensatory increase in bone formation at a disordered rate Paget disease of the bone results from an abnormal increase in bone resorption and this is coupled with an increase in bone formation but at a rapid and disorganised rate. 30. Answer B. The resting cell membrane potential is –70mV; depolarisation occurs as sodium rushes into the cells through voltage-gated ion channels; potassium then moves out of the cell

Pathology III Structured SBA

to balance the potential; the resting potential is restored via the Na/K ATP pump The resting potential is –70mV; as an action potential spreads, voltage-gated sodium channels are stimulated and open, allowing an influx of sodium and depolarising the membrane.

The potassium channels then open, allowing potassium to leave the cell and ending the action potential. The resting state relies on the Na/K ATP pump to remove sodium from the cell, repolarising the membrane.

405

Section 5 Chapter

23

Applied Basic Sciences

Orthopaedic Oncology Structured SBA Heledd Havard, Walid A. Elnahal and Craig H. Gerrand

ORTHOPAEDIC ONCOLOGY STRUCTURED SBA QUESTIONS 1. A 52-year-old female presents with a 2-year history of sternal pain with associated diffuse rib pain and generalised muscle weakness. X-rays demonstrate widespread osteopaenia with multiple old rib fractures and a recent Dotatate scan revealed an area of increased uptake within the sternum. Further investigations demonstrated raised FGF23 levels and high urinary phosphate. What is the most likely diagnosis? A. Hypoparathyroidism B. Metastatic bone disease C. Paget disease D. Phosphaturic mesenchymal tumour E. Pseudohypoparathyroidism 2. A 10-year-old female presents with an enlarging painless mass in the left buttock extending over the greater trochanter, resulting in some restriction of hip movement. The overlying skin is tense and at risk of ulceration. Investigations demonstrate hyperphosphataemia and a periarticular well-circumscribed cystic mass with fluid–fluid levels suggestive of a sedimentation sign. What is the most likely diagnosis? A. Calcifying epithelioma of Malherbe B. Gout C. Infection D. Synovial osteochondromatosis E. Tumoural calcinosis 3. A 14-year-old male presents to the ED after sustaining an ankle injury playing football. X-rays performed demonstrate a Weber A fracture of the lateral malleolus. Note is made of a welldefined lesion with a geographic margin based in the anterior cortex of the tibial diaphysis with

406

thinning of the cortex and associated sclerosis. There is no evidence of periosteal reaction and the patient denies any symptoms in the area. What is the most appropriate management? A. Above knee cast with period of non-weight bearing B. Management of fracture and observation of tibial lesion until skeletal maturity C. Management of fracture with no mention of cortical lesion D. Surgical fixation of fracture with curettage and grafting of tibial lesion E. Urgent biopsy 4. Radiographs of a young boy demonstrate an exophytic lesion arising from the cortical surface of the proximal phalanx of his left ring finger. There does not appear to be continuity with the medullary cavity. The lesion is excised, as it is causing impingement to flexion at the PIPJ. Histology demonstrates a ‘bizarre’ proliferation of bone, cartilage and fibrous tissue. Which of the following is the most likely diagnosis? A. Nora lesion B. Ollier disease C. Osteochondroma D. Parosteal osteosarcoma E. Periostitis ossificans 5. Bone tumours are often associated with classical appearances on radiological investigation and are commonly located in specific anatomical locations. Which of the following statements is incorrect? A. Adamantinoma usually occurs in the diaphysis of the tibia B. Bone cysts such as UBC or ABC usually affect the metaphyseal region

Orthopaedic Oncology Structured SBA

C. Chondroblastoma and clear cell chondrosarcoma typically occur in the epiphyseal region D. Ewing’s sarcoma in long bones most often occurs in the diaphyseal/metaphyseal portions E. Giant cell tumours of bone are almost always located in the diaphyseal region 6. Which of the following statements about osteochondromas is true? A. A cartilage cap 0.5cm thick is concerning for malignant transformation B. Malignant transformation is common, and higher in those with osteochondromatosis (hereditary multiple exostoses). C. Multiple osteochondromas are characteristic of Ollier disease and Maffucci syndrome D. Osteochondromas usually arise in the diaphyseal region of bone E. There is usually continuity of the medullary cavity between the osteochondroma and the underlying bone 7. The following statements regarding cartilaginous tumours are all true except which? A. An enchondroma is a benign cartilage tumour that occurs only in bones which develop from endochondral ossification B. Enchondromas are most commonly seen in the hands and feet but can occur elsewhere C. Enchondromas typically do not increase in size after skeletal maturity; however, growth or pain raises suspicion of malignant transformation D. In the setting of multiple lesions, malignant transformation is less frequent than in osteochondromatosis (hereditary multiple exostoses) E. Larger lesions may require curettage and grafting due to pain or impending fracture risk 8. A 7-year-old girl presents to ED after a minor fall on the playground at school. Radiographs demonstrate a large lytic lesion in the proximal humerus with an associated pathological fracture which is minimally displaced. The lesion has a geographic margin, no periosteal reaction and a ‘fallen leaf’ fragment in it. What is the most appropriate next step?

A. Conservative management in collar and cuff with serial x-rays B. Manipulation under anaesthetic with elastic nailing C. Oncology referral to exclude cancer D. Paediatric referral for metabolic bone workup and formal assessment of nonaccidental injury E. Urgent CT and/or MRI scan 9. A 12-year-old boy presents to ED with a short history of a swelling in the left shoulder. He sustained an injury playing rugby a few weeks beforehand; however, because of increasing swelling and pain he attended for an x-ray. This demonstrated an eccentric metaphyseal expansile lytic lesion in the proximal humerus with cortical erosion. CT and MRI confirmed fluid–fluid levels and no evidence of soft tissue mass or periosteal reaction. What is the likely diagnosis? A. Aneurysmal bone cyst B. Chondroblastoma C. Ewing’s sarcoma D. Giant cell tumour of bone E. Simple bone cyst with swelling secondary to trauma 10. A 17-year-old male presents with a 6-month history of pain in the right proximal thigh. He denies any significant trauma. He complains of night pain and denies any other systemic symptoms. He finds anti-inflammatory medication very helpful in relieving pain. The following statements are all true except which? A. His night pain is likely to be secondary to production of prostaglandin B. Histology would demonstrate osteoid trabeculae separated by vascular fibrous connective tissue and sharply demarcated from the surrounding bone C. Imaging is likely to identify a cortically based osteolytic lesion within the metaphysis or diaphysis with a sclerotic rim D. Treatment almost always requires surgical excision E. This usually occurs in adolescents and young adults with a male predilection

407

Heledd Havard, Walid A. Elnahal and Craig H. Gerrand

11. A 13-year-old male presents with a history of acute onset right thigh pain following a short prodromal fever. He has a mildly elevated ESR and his remaining laboratory results are normal. X-rays demonstrate a lesion in the mid-diaphysis of the femur, and MRI confirms no evidence of any soft tissue component. Biopsy demonstrates a histiocyte rich lesion. Which of the following statements is true? A. Biopsy should always be sent for histology only B. Disseminated forms include Letterer–Siwe disease and Hand–Schuller–Christian disease C. Eosinophilic granuloma always requires surgical treatment D. Histiocytes are characteristic of Ewing’s sarcoma of bone E. Radiological appearances are pathognomonic 12. A 30-year-old female physiotherapist presents with right wrist pain and swelling with no history of trauma. X-ray demonstrates an eccentric, lytic lesion in the distal radius (Figure 23.1), and MRI demonstrates a breach in the cortex with a small soft tissue component. Biopsy reveals a tumour

comprising multinucleated giant cells and stromal cells. The following statements are all true except which? A. Denosumab is a recognised adjuvant treatment in selected cases B. It can be graded radiologically using the Campanacci system C. Patients who develop lung metastases have a poor outcome D. Recurrence rates can be high following curettage E. This tumour rarely affects skeletally immature patients 13. A 20-year-old male presents with a 3-month history of progressive right knee pain, swelling and restricted movement. X-ray demonstrates a small radiolucent epiphyseal lesion within the tibial plateau with no cortical involvement and MRI confirms extensive surrounding oedema with no soft tissue component. Biopsy demonstrated hyaline cartilage with ‘chicken-wire calcification’. What is the most likely diagnosis? A. Chondroblastoma B. Clear cell chondrosarcoma C. Giant cell tumour of bone D. Osteoblastoma E. Osteoid osteoma 14. A 50-year-old male presents following a fall from scaffolding with injuries to his left arm and left knee. Radiographs demonstrate a pathological fracture of the midshaft of the humerus through a well-defined radiolucent area. X-rays of the femur and tibia also demonstrate expansile radiolucent lesions with a thin sclerotic rim and a ground-glass matrix. He has a previous history of two benign soft tissue excisions from his left buttock and medial thigh. What is the most likely diagnosis? A. Brown tumour of hyperparathyroidism B. Mazabraud syndrome C. McCune–Albright syndrome D. Metastatic bone disease E. Paget disease

Figure 23.1 AP radiograph wrist

408

15. A 60-year-old female presents with a 10-year history of an enlarging painless mass in the right thigh. An

Orthopaedic Oncology Structured SBA

MRI scan demonstrates a 10cm x 5cm wellencapsulated homogenous lesion superficial to the adductor compartment with post-contrast fat-saturated T1 imaging confirming complete suppression of fat signal intensity within the mass. All of the following statements are true except which? A. Following excision, most lesions require adjuvant radiotherapy to prevent recurrence B. Lesions deep to the fascia with thick septations and heterogeneity raise the possibility of a sarcoma C. MDM2 amplification may help differentiate benign from atypical lesions D. Multiple subcutaneous lesions can be associated with Dercum’s disease E. Marginal resection is acceptable 16. A 30-year-old female presents with a recurrent painful mass in the left infraclavicular fossa. She also complains of intermittent paraesthesia in the left arm with some mild weakness. The mass lies deep to a surgical scar from a previous excision of a benign fibrous lesion. She has a history of familial adenomatous polyposis. Biopsy showed bland fibroblasts and myofibroblasts. All of the following statements are true except which? A. Histology typically demonstrates clonal fibroblastic proliferation with abundant collagen B. Radiotherapy can be used in recurrent cases or where surgical excision is not possible C. There are systemic treatment options, including anti-oestrogens D. This condition can be associated with Gardner’s syndrome E. Treatment is usually with primary excision and local recurrence is uncommon 17. A 28-year-old male presents with pain and swelling in the left knee. He describes recurrent episodes of swelling following minor trauma and has previously had aspiration of the joint in the ED in which blood-stained fluid was removed. A recent MRI demonstrates diffuse synovitic changes with a mass in the intercondylar notch, low signal on T1 because of haemosiderin deposition and early destructive changes in the articular cartilage.

What is the most likely diagnosis? A. Gout B. Haemophillia C. Pseudogout D. Rheumatoid arthritis E. Tenosynovial giant cell tumour 18. A 40-year-old male presents with a painful mass in the thigh. It is particularly painful if knocked. He complains of intermittent tingling and a shooting pain into the foot. MRI demonstrates a mass of low T1 signal and high T2 with diffuse enhancement with gadolinium and a ‘sweet potato’ sign (Figure 23.2 shows coronal T2 sequence). All of the following statements are true except which? Figure 23.2 MRI coronal T2 sequence

A. Histology is likely to demonstrate Antoni A and B cells B. Immunohistochemistry demonstrates strongly uniform S100 antibody staining C. Lesions only affect sensory nerves and not motor nerves D. They can be associated with mutations affecting the NF2 gene E. Verocay bodies are pathognomonic 19. A 56-year-old female with neurofibromatosis type 1 is referred with an enlarging painful mass in her right thigh. Recent PET scan has demonstrated increased avidity with a rise in the SUV of the lesion compared with previous imaging. All of the following statements are true except which?

409

Heledd Havard, Walid A. Elnahal and Craig H. Gerrand

A. A change in symptoms or imaging characteristics of a long-standing neurofibroma requires urgent investigation and biopsy B. Neurofibromas are composed of Schwann cells, making them difficult to distinguish from schwannomas C. Neurofibromas may present as superficial lesions characteristically seen in the lower limbs D. Plexiform neurofibromas can be very large and occupy entire compartments E. Solitary neurofibromas are usually benign lesions; however, malignant transformation can occur in patients with neurofibromatosis 20. A 40-year-old male presents with a 5-year history of pain and swelling in the left knee, which is worse with activity. He complains of intermittent stiffness and locking and is no longer able to cycle. A radiograph of the knee demonstrates multiple small, loose bodies with a well-preserved joint space. What is the most likely diagnosis? A. Chondrosarcoma B. Rheumatoid arthritis C. Synovial chondromatosis D. Tenosynovial giant cell tumour E. Tophaceous gout 21. A 50-year-old male presents with a painful lesion on the medial plantar aspect of his right foot. He has been aware of the lesion for some years; however, it is now exquisitely painful. Imaging has demonstrated a superficial, small both legs swing at same time => walker in front C. Walker in front => stronger leg follows through => weaker leg D. Walker in front => weaker leg follows through => stronger leg E. Weaker leg in front => walker follows through => stronger leg 11. A 70-year-old male recently underwent a belowknee amputation and was fitted in a prosthesis

several months afterwards. He presented to clinic complaining of swelling of his thigh just above the prosthesis. The skin was indurated, and a diagnosis of choke syndrome was made. This most likely could have been due to which of the following? A. Loose proximal and loose distal socket B. Loose proximal and normal-fitting distal socket C. Loose proximal and tight distal socket D. Tight proximal and loose distal socket E. Tight proximal and tight distal socket 12. Which deformity would result in a patient undergoing a Chopart amputation without tendon balancing? A. Equinus deformity B. Hindfoot valgus C. Hindfoot varus D. Planus deformity E. Supination deformity 13. A 65-year-old diabetic with severe peripheral arterial disease presents with a spontaneous rupture of his tibialis anterior tendon, causing difficulty in dorsiflexion of his ankle. He has a 3cm gap on ultrasound examination. Which one of the following would be the best option to manage this? A. Darco shoe with hindfoot rocker B. Palmaris longus autograft to bridge the gap C. Posterior leaf spring ankle–foot orthosis (AFO) D. Posterior tibialis tendon transfer onto the talar neck E. Solid ankle cushioned heel (SACH) foot 14. A 32-year-old male presents with a cavovarus deformity. Examination reveals a positive Coleman block test. Which of the following orthotics will be most suitable? A. Recessed first ray with lateral hindfoot post B. Elevated first ray with lateral hindfoot post C. Elevated first ray with medial hindfoot post D. Recessed first ray with medial hindfoot post E. Recessed second to fourth rays with lateral hindfoot post

429

Emerson Budhoo

15. Which of the following best explains the mechanics in utilising a cane for ipsilateral hip arthritis? A. A cane held in the contralateral hand reduces joint reaction forces through the affected hip up to 50% by reducing abductor muscle pull B. A cane held in the contralateral hand reduces joint reaction forces through the affected hip up to 25% by reducing abductor muscle pull C. The cane improves the abductor muscle pull on the opposite side of the hip arthritis D. The cane reduces the abductor muscle pull on the opposite side of the hip arthritis E. The cane reduces the joint reaction force when it is held on the same side as the hip arthritis 16. A 34-year-old male undergoes a below-knee amputation for a mangled limb. He wishes to continue running on a regular basis. Which of the following terminal devices would be best suited? A. Articulated dynamic response foot with lengthened keel B. Articulated dynamic response foot with sagittal split keel C. Articulated dynamic response foot with shortened keel D. Elastic keel foot E. Solid ankle cushioned heel (SACH) foot 17. Which one of the following amputations involves preservation of the calcaneus, which is then fused to the distal tibia? A. Boyd B. Chopart C. Gritti–Stokes D. Symes E. Transmetatarsal 18. A 68-year-old male underwent a total knee arthroplasty utilising a posterior stabilised (PS) design prosthesis. Four months afterwards, he felt his knee ‘dislocate’ and went to the ED. Radiographs revealed a cam jump. Which of the following surgical errors could have led to this? A. Loose extension gap B. Loose flexion gap

430

C. Patella baja D. Tight extension gap E. Tight flexion gap 19. Which one of the following issues may arise in a patient with a below-knee amputation in which the heel of the foot prosthesis is too rigid? A. Excessive knee extension B. Excessive knee flexion C. Excessive valgus knee stress D. Reduced knee flexion E. Reduced knee rotation 20. Which of the following best describes the appropriate position of the feet in the Denis Browne bar brace? A. 60–70° external rotation (ER) on the affected side, 30° ER on the unaffected side, 10–15° dorsiflexion bilaterally B. 60–70° ER on the affected side, 0° (neutral) rotation on the unaffected side, 10–15° dorsiflexion bilaterally C. 30° ER of the affected side, 0° (neutral) rotation on the unaffected side, 10–15° dorsiflexion bilaterally D. 60–70° ER on both affected and unaffected sides, 10–15° dorsiflexion bilaterally E. 30° ER bilaterally, 10–15° dorsiflexion bilaterally 21. Which of the following correctly describes the Kleinert protocol following flexor tendon repairs? A. Active finger extension, active finger flexion B. Active finger extension, dynamic splintassisted passive finger flexion C. Active finger flexion, dynamic splint-assisted passive finger extension D. Passive finger extension, active finger flexion E. Passive finger flexion, dynamic splint-assisted passive finger extension 22. Which of the following correctly describes the principle involved in application of a Pavlik harness in an infant with developmental dysplasia of both hips? A. Anterior straps flex the hip to 90–100° degrees, posterior straps externally rotate the hip to 20° degrees

Prosthetics and Orthotics Structured SBA

B. Anterior straps prevent flexion of the hip, posterior straps abduct the hip to 45° C. Anterior straps flex the hip to 40–50°, posterior straps abduc the hip to 20° D. Anterior straps flex the hip to 90–100°, posterior straps prevent adduction of the hip E. Anterior straps flex the hip to 90–100°, posterior straps prevent internal rotation of the hip 23. Which one of the braces can be utilised in a child with scoliosis in which the apex of the curve is above T7 vertebra? A. Boston-style brace B. Charleston bending brace C. Milwaukee brace D. TLSO E. Wilmington brace 24. What type of prosthetic knee design is best suited in a young active female below-knee amputee who desires a prosthesis with variable cadence? A. Constant friction knee B. Fluid-controlled hydraulic knee C. Manual locking knee D. Stance phase control knee E. Variable-friction knee 25. Which of the following statements is correct regarding halo traction for cervical spine injuries? A. It can be used to treat type II odontoid fractures in younger patients B. Dural puncture is the most common complication C. It is more effective in treating subaxial spine injuries D. More pins are required for adults than paediatrics E. Supraorbital nerve palsy is the most common nerve injury due to halo placement 26. A 64-year-old male suffers a laceration to his wrist, severing his ulna nerve. Repair is performed, but 1 year afterwards he develops a claw hand deformity. His joints are fully mobile and he refuses any further surgical intervention. Which of the following splints is indicated?

A. Dorsal splint to flex the metacarpophalangeal joint (MCP) joint of the ring and middle fingers to 35–45° B. Dorsal splint to flex the MCP joint of the ring and middle fingers to 90° and extend the proximal interphalangeal (IP) joint to neutral C. Dorsal splint to neutralise the MCP joint of the ring and middle fingers and flex the IP joints to 45° D. Volar wrist splint with the wrist in 20° flexion, MCP in 90° flexion E. Volar wrist splint with the wrist in 45° flexion, MCP in 10° flexion 27. Following an altercation, a 54-year-old male noticed his middle finger was swollen. The resident on call made a clinical diagnosis of a central slip rupture of his extensor tendon apparatus. There were no bony injuries on x-ray. Which one of the following splints may be useful in the acute setting? A. Buddy strapping B. Capener splint C. Extension block splinting D. Mallet splint E. Sugar tong splint 28. A 45-year-old male sustained multiple trauma involving closed midshaft femur fracture, hemopneumothorax and intracerebral haemorrhage secondary to basal skull fracture. He was placed in ICU for optimisation as he was haemodynamically and physiologically unstable. Which one of the following traction devices would be best suited to temporarily splint his femur fracture? A. Bryant’s traction B. Bucks traction C. Dunlop traction D. Hamilton Russell traction E. Thomas splint 29. A 60-year-old poorly controlled diabetic presents with an Eichenholtz stage III midfoot collapse. Which of the following orthotics would be most appropriate? A. Charcot restraint orthotic walker (CROW) boot B. Double rocker sole modification footwear

431

Emerson Budhoo

C. Heel-to-toe rocker sole modification footwear D. High arch medial support insoles with hindfoot valgus post E. Total contact casting 30. Which of the following is true regarding an individual with a vaulting gait? A. It can occur in a unilateral transfemoral amputee in which the prosthesis is too short B. It occurs during the initial and middle parts of the prosthetic swing phase C. It occurs during the prosthetic side midstance phase D. It usually occurs when the prosthesis is too anteriorly aligned E. The primary compensatory mechanism is engagement of the hip abductors on the non-amputee side 31. Which of the following is a cause of a medial whip in an above knee amputee with a prosthesis? A. Decreased abduction of hip at toe off B. Increased adduction of hip at toe off C. Internal rotation of hip at toe off D. Knee axis of the prosthesis is in excessive external rotation E. Knee axis of the prosthesis is in excessive internal rotation 32. Which one of the following conditions would a UCBL (University of California at Berkeley Laboratory) insert be best suited for? A. Patient with chronic Achilles tendinopathy B. Pes cavus with rigid hindfoot varus C. Stage IIb posterior tibial tendon dysfunction (PTTD) D. Stage III PTTD E. Stage IV PTTD 33. Which one of the following correctly describes the proper positioning of the trim lines in an

432

ankle–foot orthosis in a 6-year-old patient with forefoot abduction secondary to cerebral palsy? A. At the ankle, the trim line should fully cover the lateral malleolus to improve donning B. At the forefoot, the trim line must be at the level of the 5th metatarsal head C. At the forefoot, the trim line must be distal to the 5th metatarsal head D. At the forefoot, the trim line must be proximal to the 5th metatarsal head E. At the top, the trim line must be 2cm proximal to the fibula head 34. Which of the following statements is true? A. A long transradial amputation allows up to 60° pronation and supination and maintains strong elbow flexion B. A long transradial amputation preserves 40–75% of forearm length C. A medium transradial amputation is beneficial due to preservation of pronation and supination D. A medium transradial amputation preserves 55–85% of forearm length E. A short transradial amputation preserves 20–40% of forearm length 35. A 48-year-old female started to complain of severe paraesthesia in her left ring and little fingers for a duration of 6 months. Clinical examination revealed cubital tunnel syndrome. Which of the following bracing positions will assist in relief of symptoms? A. Night splinting with elbow in 20° flexion and forearm in supination B. Night splinting with elbow in 45° flexion and forearm neutral C. Night splinting with elbow in 45° flexion and forearm pronated D. Night splinting with elbow in 45° flexion and forearm supinated E. Night splinting with elbow in 60° flexion and forearm pronated

Prosthetics and Orthotics Structured SBA

PROSTHETICS AND ORTHOTICS STRUCTURED SBA ANSWERS 1. Answer A. Constant friction knee The constant friction knee utilises a hinge with a rubber pad to apply friction to the knee to decrease knee swing. This only allows a single speed of walking. It is durable and cheap. Stancephase control knee mechanism is particularly beneficial in the elderly population. Polycentric knees are ideal for the active adult, but they are very costly and heavy. 2. Answer D. The affected knee is maintained in slight flexion with ipsilateral toe walking Minimal knee flexion is the typical sign seen in knee arthritis with ipsilateral toe walking. Both these measures are quadriceps-avoidance mechanisms to reduce the tension on the knee capsule. In ankle arthritis, the ipsilateral step length would be increased to increase the swing phase and reduce the time of weight bearing on the

affected ankle. Patients lean their trunk laterally over the affected side in hip arthritis. This decreases the moment arm of body weight and reduces the joint reaction force on the hip joint. The swing phase of gait is increased on the ipsilateral side in hip arthritis. 3. Answer A. The metabolic demand is 100% above normal in transfemoral (TF) amputations related to peripheral vascular disease The metabolic demand is 100% above normal in TF amputations in patients with peripheral arterial disease. This puts a strain on the heart and also leads to them being easily fatigued. Vascular versus traumatic BKA results in increased energy expenditure of 40% and 25%, respectively. The metabolic demand in TF amputation is approximately 68%. 4. Answer E. Socket pain medial proximally and lateral distally As seen in the diagram in Figure 24.1, a foot that is placed too far inset will cause the prosthesis to

Figure 24.1 Foot placed too far inset

433

Emerson Budhoo

tilt into varus. This will create a lever arm effect, with the proximal medial leg acting as the fulcrum. As a result, the lateral distal part of the prosthesis will also cause compression. Choke syndrome is associated with a prosthesis that is too snug. Increased knee extension occurs when the foot component is placed too anterior. 5. Answer D. Swing-through gait Swing-through gait describes the fastest gait pattern. Both crutches are placed forward, followed by both feet swinging ahead of the crutches. This requires a high consumption of energy. It is even faster than normal gait. Four-point gait is a very slow walking speed and three points (left crutch, right foot, right crutch, left foot repeat) are in contact with the ground at all times. Three-point gait involves moving both crutches and the weaker limb while bearing all the weight on the crutches, followed by the stronger limb moving forward. Swing-to gait occurs with bilateral lower limb weakness in which both the crutches move forward, followed by both limbs swinging just behind the crutches. 6. Answer B. Body-powered prosthesis (terminal device activates with shoulder abduction and flexion) Shoulder abduction and flexion are necessary to clear the upper limb away from the body, and this position would best serve to activate the terminal device in heavy labourers. Myoelectric prosthesis requires a lot more maintenance and is not suitable for heavy labour. It is more cosmetically appealing and has two main types: a 2site, 2-function device in which there are separate electrodes for flexion/extension and a 1-site, 2-function device in which there is one electrode for flexion/extension. 7. Answer B. The piston mechanism in the hydraulic knee allows variable cadence The piston mechanism in the hydraulic knee allows variable cadence when the knee flexes, depending on the walking speed. The hydraulics (liquid) adjust to control the speed at which the pylon flexes and extends. The polycentric knee has a variable centre of rotation. When the COR is posterior to the line of weight bearing, it allows control in the stance phase and when it

434

is anterior, flexion is improved. The polycentric knee is best suited for an active individual. 8. Answer D. Insole with medial hindfoot post A medial hindfoot post can be used to correct hindfoot valgus, which occurs in posterior tibial tendon dysfunction. Knee–ankle–foot orthosis will not properly correct her weight bearing axis, as the pathology is at the level of the ankle. Medial arch support orthotic is frequently required in posterior tibial tendon dysfunction. 9. Answer C. Elbow movement is controlled by shoulder extension and depression Elbow movement in an upper limb prosthesis is activated when the shoulder extends and depresses while the terminal device is activated by shoulder abduction and flexion. 10. Answer D. Walker in front => weaker leg follows through => stronger leg The person places the walker in front of them, then steps forward with the weaker leg. The stronger leg follows through last. 11. Answer D. Tight proximal and loose distal socket Choke syndrome is caused by obstructed venous outflow due to a socket that is too tight proximally and loose distally. Blood flows distally via the arterial system into the limb, but venous return is impaired by a tight proximal socket. This leads to swelling and fluid retention in the stump, causing an ‘orange peel’ appearance. Treatment is aimed at increasing the internal diameter of the socket to accommodate venous return 12. Answer A. Equinus deformity Chopart amputation involves disarticulation of the calcaneocuboid and talonavicular joints. A common issue that occurs is an equinus deformity due to unopposed action of the Achilles tendon. The tibialis anterior tendon should be reinserted into the talar neck to avoid this deformity. 13. Answer C. Posterior leaf spring ankle–foot orthosis (AFO) Surgery would result in many complications including infection and wound breakdown in

Prosthetics and Orthotics Structured SBA

this gentleman with significant circulatory problems. Palmaris autograft can be considered in a younger patient with no circulatory issues or peripheral neuropathy. There is normal power in the tibialis anterior so a posterior tibial tendon transfer will not be indicated. SACH is a foot prosthesis for patients with amputations in which a terminal device is indicated. An AFO will keep the foot perpendicular to the long axis of the tibia. In this way, the foot will clear the ground during the swing phase of gait and aid in stability during stance. 14. Answer A. Recessed first ray with lateral hindfoot post A common cause of a cavovarus deformity at this age is Charcot–Marie–Tooth disease. Patients present with a hindfoot varus, plantarflexed first ray and possible claw toes. A positive Coleman block test is indicative of a forefoot-driven deformity (plantar-flexed first ray). A semi-rigid insole can help correct the deformity with a recess for the 1st metatarsal head and a lateral post to correct the heel varus. 15. Answer A. A cane held in the contralateral hand reduces joint reaction forces through the affected hip up to 50% by reducing abductor muscle pull The cane held in the contralateral hand exerts a force whose direction of moment is the same as that of the affected hip abductors. This therefore reduces the force by 50% required by these abductors during gait. 16. Answer A. Articulated dynamic response foot with lengthened keel A lengthened keel in a dynamic response foot prosthesis is indicated for very high demand athletes. An articulated dynamic response foot with a sagittal split keel allows for eversion and inversion. This is ideal when walking on uneven ground. A shortened keel is indicated for individuals who engage in a fair amount of walking. Structurally, foot prostheses can be divided into two types: articulated and non-articulated. SACH and elastic keel configurations are the two simplest types of non-articulated foot prostheses. They are inexpensive and durable. They do not store and return energy.

17. Answer A. Boyd Children with severe fibula deficiency involving the foot may require amputation followed by a prosthesis foot. Symes amputation involves an ankle disarticulation with preservation of the heel pad, while the Boyd amputation involves a modified ankle disarticulation where the calcaneus along with the heel pad are preserved. The retained calcaneus is fused to the distal tibia. A Chopart amputation involves amputation at the level of the calcaneocuboid and talonavicular joints. A Gritti–Stokes amputation is through the knee. 18. Answer B. Loose flexion gap This usually happens when the knee with a loose flexion gap goes into hyperextension. The cam rotates over the tibial post and dislocates. This should be addressed immediately via closed reduction. Afterwards, the loose flexion gap must be addressed soon, in which the posterior femur needs to be augmented. 19. Answer B. Excessive knee flexion A prosthetic heel that is too rigid results in excessive knee flexion and lateral rotation of the toes, while a very soft heel leads to excessive knee extension. 20. Answer A. 60–70° external rotation (ER) on the affected side, 30° ER on the unaffected side, 1015° dorsiflexion bilaterally After serial casting via the Ponseti method, an Achilles tenotomy is frequently performed. Following this, the affected side is placed in 60–70° ER and the unaffected limb is placed in 30° ER. Dorsiflexion is required to prevent the hindfoot from going into an equinus deformity. ER is necessary to prevent recurrence of the cavovarus deformity. 21. Answer B. Active finger extension, dynamic splint-assisted passive finger flexion Both the Duran and Kleinert protocols are lowforce and low-excursion methods in dealing with flexor tendon injuries. Both methods involve active finger flexion and passive finger extension movements. The Kleinert utilises dynamic splinting, while the Duran involves static splinting.

435

Emerson Budhoo

22. Answer D. Anterior straps flex the hip to 90–100°, posterior straps prevent adduction of the hip Positioning of the Pavlik orthotic harness is very important in treating developmental dysplasia of the hips. Excessive flexion can lead to a transient femoral nerve palsy. Excessive hip abduction (usually beyond 60°) can lead to avascular necrosis of the femoral head secondary to impingement of the posterior superior retinacular branch of the medial circumflex femoral artery. The posterior straps prevent hip adduction. 23. Answer C. Milwaukee brace The Milwaukee brace is a type of cervical-thoracic-lumbar-sacral orthosis (CTLSO) invented in the 1940s. It is very bulky and no longer frequently used. However, in high thoracic curves it may still be indicated. TLSO, Boston-style brace and Wilmington brace are thoracolumbosacral orthosis braces and are indicated for curves where the apex is below T7. The Wilmington brace is a custom-made brace specific to the patient, whereas the Boston style brace is prefabricated. The Charleston brace is a nighttime brace, custom designed to fit the patient while they are lying down but not standing up. 24. Answer B. Fluid-controlled hydraulic knee Cadence is also known as stride rate, i.e. the number of steps a runner takes per minute. Fluid-controlled knees are of two types: pneumatic (air) and hydraulic (fluid) controlled. Pistons move through the medium as the knee flexes and extends, leading to various resistances throughout the gait. This allows the user to walk comfortably at various walking speeds. The main disadvantage is its heaviness and cost. The variable-friction knee also provides variable resistance and hence cadence when the knee flexes from an extended position. However, it requires frequent adjustment and replacement of parts, making it less advantageous than the fluid-controlled hydraulic knee. The stance phase control knee is commonly used in prosthetics, especially those with above knee amputations. The knee needs to be unloaded to initiate swing or for one to sit. The constant friction knee only allows a single speed of walking. The

436

manual locking knee is the most stable knee used in prosthetics. The knee is locked during gait and then unlocked when the patient wants to sit down. 25. Answer A. It can be used to treat type II odontoid fractures in younger patients Halo traction is useful in the paediatric population to treat upper cervical spine injuries including type II odontoid fractures, atlanto-occipital dislocations, C1–C2 dislocations and atlas fractures. Halo orthosis connects the skull to the upper torso via an external frame. It is used for upper cervical spine injuries. It has good control over rotational movements. It is inadequate for lower cervical segment injury, and a ‘snaking phenomenon’ can occur in which there is focal kyphosis in the mid cervical spine. Adults require a total of 4 pins in the skull as compared to the paediatric population (6–8 pins). Abducens nerve palsy is the most common nerve palsy and is due to excessive traction. Dural puncture is uncommon and accounts for less than 1% of complications associated with skull pin placement. 26. Answer A. Dorsal splint to flex the metacarpophalangeal (MCP) joint of the ring and middle fingers to 35–45° A low-level ulna claw hand results in hyperextension of the MCP joints of the ring and middle fingers due to intrinsic weakness and unopposed action of the extensors. This results in loss of function of the hand. The function of the splint is to: (1) passive flex the MCP joint; (2) prevent shortening of the MCP collateral ligament; and (3) allow active IP flexion. The dynamic splint prevents extension at the MCP joint. 27. Answer B. Capener splint This injury would result in a Boutonnière deformity. This is best treated with a Capener splint, which splints the PIP joint in full extension. Active DIP flexion and extension exercises are encouraged to prevent contracture of the oblique retinacular ligament. The mallet splint is used for extension tendon injuries at the distal phalanx. These injuries result in the DIP joint being flexed. The splint keeps the DIP joint in extension to encourage healing of the tendon

Prosthetics and Orthotics Structured SBA

back to its insertion. Buddy strapping is usually used for undisplaced phalangeal injuries. Extension block splinting is used for stable PIP joint dislocations after reduction is achieved. The extension block splint is applied with the PIP joint at 45° flexion. Sugar tong splinting is used in the acute management of distal radial and ulna fractures. 28. Answer D. Hamilton Russell traction The Hamilton Russell traction combines suspension of the distal fragment as well as traction to the femur. The sling prevents posterior angulation of the distal fragment. Skin traction is applied to the leg or a Bucks traction splint may be utilised. This form of traction is used for femur shaft traction when the duration of splinting anticipated is greater than 24 hours, as in this case. Bucks traction is sometimes used preoperatively for femur shaft fracture using tape or a premade boot. It is not used to maintain or hold reduction. It is used to prevent contractures of the hip or knee and no more than 10lb can be attached. Bryant’s traction combines Bucks traction and Gallows traction, although used rarely in femur shaft fractures in infant or small children. Thomas splint is used when transporting patients with injuries to the thigh and hip. The Thomas halfring splints consist of a padded half-circle of steel that is strapped to the hip. A U-shaped rod is attached to the ring and extends along both sides of the leg. An ankle strap may be fashioned from cloth and tied or twisted to apply traction force. 29. Answer B. Double rocker sole modification footwear The management of a Charcot foot is dependent on the stage and location of the disease. Eichenholtz stages I and II are initially managed via total contact casting and a CROW walker, respectively. Stage III disease requires accommodative footwear. Midfoot collapse is managed with a double rocker sole modification footwear. This decreases pressure in the plantar midfoot. A heel-to-toe rocker sole is utilised when the hindfoot is rigid/fused and one requires increased propulsion at toe off.

30. Answer B. It occurs during the initial and middle of the prosthetic swing phase Vaulting occurs for several reasons: (1) prosthesis is too long (2), prosthesis is too posteriorly aligned, (3) patient is fearful of not being able to clear the ground during gait. Vaulting involves increased plantar flexion of the contralateral ankle in order to elevate the hip on the prosthetic side. This aids in clearance during the initial and middle parts of the prosthetic swing phase. It inevitably places excessive stress on the metatarsal heads and ankle joint of the contralateral limb. 31. Answer D. Knee axis of the prosthesis is in excessive external rotation A medial whip is defined as movement of the heel medially during toe off. Causes can be broadly classified into prosthetic cause and amputee cause. Prosthetic causes include excessive external rotation of the knee, shank or foot. Amputee causes include a habitual gait in which the hip externally rotates at toe off or a socket that is donned in external rotation. 32. Answer C. Stage IIb posterior tibial tendon dysfunction (PTTD) The UCBL brace is a rigid brace designed to correct flexible hindfoot deformities such as valgus secondary to PTTD. It limits subtalar motion and, controls calcaneal eversion in the frontal plane and forefoot abduction in the transverse plane. It will therefore only work if the deformity is flexible. Stages III and IV PTTD result in a rigid subtalar joint and hindfoot valgus. This orthotic will therefore not work. Stage IIb PTTD deformity is flexible and the subtalar joint is correctable. This brace will therefore be ideal in correcting the deformity. 33. Answer C. At the forefoot, the trim line must be distal to the 5th metatarsal head The forefoot trim line must be distal to the metatarsal head to avoid metatarsus adductus. In forefoot adduction, the trim line must be proximal to the 5th metatarsal head. The trim line must also be 2cm under the fibula head to prevent compression/irritation of the common peroneal nerve. In forefoot abduction, there

437

Emerson Budhoo

must be decreased coverage at the level of the lateral malleoli to ease donning. The trim line is the border of the trimmed section of the orthotic that is important in determining stiffness of an ankle–foot orthotic. 34. Answer A. A long transradial amputation allows up to 60° pronation and supination and maintains strong elbow flexion There are 3 main levels of transradial amputation: (1) Short: 0–35% length preservation (2) Medium: 35–55% length preservation/ pronation and supination are lost (3) Long: 55–90% length preservation/ pronation and supination up to 60° and maintenance of strong elbow flexion

438

35. Answer B. Night splinting with elbow in 45° flexion and forearm neutral Night bracing may be effective in early cases of cubital tunnel syndrome in which there are no signs of muscle wasting and a functioning flexor carpi ulnaris. It is effective in approximately half of all patients. The ulna nerve is under least tension when the elbow is flexed between 40° and 50° in a neutral position. Surgery will be necessary if symptoms persist. The ulna nerve can be compressed in the elbow at the arcade of Struthers, between Osborne’s ligament and MCl, two heads of the FCU and medial intermuscular septum.

Section 5 Chapter

25

Applied Basic Sciences

Biomechanics Structured SBA Edward Jeans and Richard Hutchinson

BIOMECHANICS STRUCTURED SBA QUESTIONS 1. When stabilising a middiaphyseal femoral fracture using an intramedullary nail, locking bolts are passed proximal and distal to the fracture. The main function of these locking bolts is which of the following? A. Allow for easier nail extraction B. Increase the bending stiffness of the construct C. Increase the torsional stiffness of the construct D. Increase the working length of the construct E. Reduce the risk of proximal/distal nail migration 2. Which of the following forms of synovial joint lubrication is most important in prolonged stance? A. Boosted lubrication B. Boundary lubrication C. Micro-elastic lubrication D. Squeeze-film lubrication E. Weeping lubrication 3. A surgeon implants a cemented stainless steel femoral stem during a total hip arthroplasty. In error, the surgeon then applies a cobalt-chrome head to the trunnion before reducing and closing the hip. What mechanism of corrosion could have been avoided if a stainless steel head had been used? A. Crevice corrosion B. Fretting corrosion C. Galvanic corrosion D. Pitting corrosion E. Stress corrosion 4. A cobalt-chrome head and a ceramic femoral head are being tested in a lab. During one test, a

sharp object is run along each material’s surface. The size and depth of the resultant scratch is assessed. What material property is being tested for in this scenario? A. Hardness B. Stiffness C. Strength D. Toughness E. Elasticity 5. During the manufacturing process of crosslinked UHMWPE, vitamin E is added to the polymer. What is the primary role of vitamin E in crosslinked UHMWPE? A. Acts as a bactericidal agent B. Increases cross-linking C. Reduces risk of delamination D. Reduces surface roughness E. Works as an antioxidant agent 6. An implant company are designing a new uncemented femoral stem. They decided to use a titanium alloy, as its Young’s modulus is significantly lower than a stainless steel option. A lower Young’s modulus is advantageous in this case because of which of the following? A. Allows for better osseointegration B. Allows for greater stress relaxation C. Increases implant stiffness D. Reduces stress shielding E. Reduces the risk of implant corrosion 7. Which of the following statements best describes the ‘yield point’ of a material? A. The point at which elastic deformation starts to occur B. The point at which necking starts to occur C. The point at which plastic deformation starts to occur

439

Edward Jeans and Richard Hutchinson

D. The point at which work-hardening starts to occur E. The point of failure 8. A surgeon doing a total hip replacement decides to use a larger diameter femoral head to reduce the risk of future dislocation. How does increasing head diameter most improve hip stability? A. Increases jump distance B. Moves the centre of rotation medially C. Reduces the head–neck ratio D. Reduces the range of the primary arc E. Tightens surrounding soft tissue restraints 9. A surgeon inserting a femoral broach into a tight femoral canal decides to pause between each strike of the hammer for 10 seconds in order to reduce the risk of femoral fracture. Which of the following viscoelastic features best explains the surgeon’s logic? A. Stress relaxation B. Creep C. Hysteresis D. Strain-rate dependence E. Strain relaxation 10. A surgeon wishes to maximise the stability of a total hip replacement by increasing the primary arc range of motion, and thus reducing the risk of impingement. Which of the following does NOT increase the primary arc? A. Increasing femoral head size B. Removing acetabular osteophytes C. Reducing femoral neck diameter D. Using a liner with a 10° posterior lip E. Increasing head–neck ratio 11. What is the most important mechanism of wear leading to osteolysis in a metal-on-polyethylene total hip joint bearing surface? A. Abrasive wear B. Adhesive wear C. Corrosive wear D. Erosive wear E. Third body wear 12. A metal is being tested for its material properties in a laboratory by undergoing cyclical loading,

440

with increasing loads on each cycle. It is noted once loads are applied that exceed the material’s yield point, the stress required to cause further plastic deformity goes up with each loading cycle. Which of the following best describes the cause for this phenomenon? A. Hooke’s law B. Increasing grain size C. Necking D. Stress-shielding E. Work-hardening 13. In Ponseti casting, a moulded plaster applies a constant stress to the soft tissues of the foot and ankle. Over a period of time, this constant stress results in a gradual correction of the deformity. Which viscoelastic material property is being demonstrated in this treatment? A. Creep B. Hysteresis C. Strain-rate dependence D. Strain-relaxation E. Stress-relaxation 14. Which of the following best describes the primary role of the patella? A. Increases both flexion and extension moment arms B. Increases the extension moment arm C. Increases the flexion moment arm D. Reduces the extension moment arm E. Reduces the flexion moment arm 15. The primary role of the femoral and tibial stems in a hinged knee replacement is to do which of the following? A. Increase implant stiffness B. Increase the surface area of the implant–bone interface C. Maintain joint stability D. Prevent periprosthetic fracture E. Reduce wear at the bearing surface 16. An uncemented total hip replacement is being revised for aseptic loosening. On extraction it is noted there is significant backside wear of the polyethylene. Which of McKellop’s modes of wear is best described in this case? A. Mode 1 B. Mode 2

Biomechanics Structured SBA

C. Mode 3 D. Mode 4 E. Mode 5 17. Cross-linking in polymers is known to increase material strength and stiffness. Which type of bond is formed between long monomer chains during cross-linking? A. Covalent B. Hydrogen C. Ionic D. Peptide E. Van der Waals 18. Polymethylmethacrylate (PMMA) cement powder contains a polymer and which other substance? A. Decelerator B. Stabiliser C. Accelerator D. Activator E. Initiator 19. Zirconium dioxide is used as an additive in PMMA cement. What is its primary function? A. Anti-microbial B. Catalyst C. Initiator D. Radio-opacity E. Stabiliser

C. Congruent D. Equatorial E. Polar 22. An implant engineer is trying to design a hardon-hard bearing surface for a THR that achieves hydrodynamic fluid-film lubrication. Which of the following will most help increase the probability of them achieving this? A. Increasing acetabular cup outer diameter B. Increasing femoral head diameter C. Reducing liner diameter D. Using a congruent bearing design E. Using highly cross-linked polyethylene 23. A patient with cuff arthropathy is offered a reverse-polarity shoulder replacement. Which of the following statements is NOT true in reverse shoulder replacements (RSA)? A. RSA allows some of the fibres from the anterior and posterior deltoid to act as shoulder abductors B. RSA increases deltoid tension through lowering the centre of rotation C. RSA increases the deltoid lever arm D. RSA lateralises the centre of rotation, improving cuff function E. RSA reduces torque at the implant–bone interface, compared to anatomical designs 24. Figure 25.1 shows the stress–strain curves for three different materials.

20. A block of PMMA cement is being tested for its response to different loads. With regard to the mechanical properties of PMMA cement, which of the following statements is correct? A. It does not undergo creep B. It is strongest against shear C. It is strongest against tension D. It is strongest against torsion E. It is strongest against compression 21. Fluid-film lubrication can significantly reduce wear in prosthetic joints. Which type of total hip bearing design is most likely to allow fluid-film lubrication? A. Bipolar B. Captive cup

Figure 25.1 Stress–strain curve

441

Edward Jeans and Richard Hutchinson

Regarding this chart, which of the following statements is true? A. Material B has the highest Young’s modulus B. Material B is the most ductile C. Material C is the stiffest material D. Material C is the toughest material E. Material A has the greatest yield strength 25. Figure 25.2 shows a schematic diagram of fracture fixed with a plate. Which letter corresponds to the working length of the plate?

Figure 25.2 Plate fixation fracture

A. B. C. D. E.

A B C D E

C. Cutting both nail ends flush with the bone D. Using two nails of the same diameter E. Using two nails of the same material 28. A surgeon wants to achieve primary bone healing of a fracture. Which of the following methods of plate fixation will NOT aid primary bone healing? A. Bridge plating B. Buttress plating C. Compression plating D. Neutralisation plating E. Tension-band plating 29. A surgeon fixes a Weber B lateral malleolar fracture using a lag screw and a one-third tubular plate in neutralisation mode. What is the primary role of a neutralisation plate? A. Protects against bending forces B. Protects against compressive forces C. Protects against shear forces D. Protects against tension forces E. Protects against torsional forces 30. Figure 25.3 demonstrates an S–N curve for a metal. What does point A represent?

26. Which of the following increases the working length of a screw fixed into a bone? A. Ensuring the screw is bicortical B. Increasing the number of screw threads C. Increasing the screw core diameter D. Increasing the screw thread diameter E. Reducing the distance between two adjacent screws 27. A 9-year-old boy sustains a transverse femoral fracture, which is being managed with elastic nails. All of the following are important principles in maintaining a balanced construct in elastic stable intramedullary nailing (ESIN) EXCEPT which? A. Avoiding ESIN in highly comminuted fracture patterns B. Bending both nails at the same level

442

Figure 25.3 S–N curve

Biomechanics Structured SBA

A. B. C. D. E.

Endurance limit Fatigue life Fatigue strength Ultimate tensile strength Yield strength

31. Which of the following will NOT affect the bony integration of hydroxyapatite (HA) coated implants for total hip replacement? A. Decreased surface roughness B. Early mobilisation C. HA coating thickness of